Pharmacology > TEST BANK > Pharmacology Clear and Simple- A Guide to Drug Classifications and Dosage Calculations 3rd Edition W (All)

Pharmacology Clear and Simple- A Guide to Drug Classifications and Dosage Calculations 3rd Edition Watkins Test Bank,100% CORRECT

Document Content and Description Below

Pharmacology Clear and Simple- A Guide to Drug Classifications and Dosage Calculations 3rd Edition Watkins Test Bank Chapter 1. History of Pharmacology 1. A nurse working in radiology administ... ers iodine to a patient who is having a computed tomography (CT) scan. The nurse working on the oncology unit administers chemotherapy to patients who have cancer. At the Public Health Department, a nurse administers a measles-mumps-rubella (MMR) vaccine to a 14-month-old child as a routine immunization. Which branch of pharmacology best describes the actions of all three nurses? A) Pharmacoeconomics B) Pharmacotherapeutics C) Pharmacodynamics D) Pharmacokinetics Ans: B Feedback: Pharmacology is the study of the biologic effects of chemicals. Nurses are involved with clinical pharmacology or pharmacotherapeutics, which is a branch of pharmacology that deals with the uses of drugs to treat, prevent, and diagnose disease. The radiology nurse is administering a drug to help diagnose a disease. The oncology nurse is administering a drug to help treat a disease. Pharmacoeconomics includes any costs involved in drug therapy. Pharmacodynamics involves how a drug affects the body and pharmacokinetics is how the body acts on the body. 2. A physician has ordered intramuscular (IM) injections of morphine, a narcotic, every 4 hours as needed for pain in a motor vehicle accident victim. The nurse is aware this drug has a high abuse potential. Under what category would morphine be classified? A) Schedule I B) Schedule II C) Schedule III D) Schedule IV Ans: B Feedback: Narcotics with a high abuse potential are classified as Schedule II drugs because of severe dependence liability. Schedule I drugs have high abuse potential and no accepted medical use. Schedule III drugs have a lesser abuse potential than II and an accepted medical use. Schedule IV drugs have low abuse potential and limited dependence liability. 3. When involved in phase III drug evaluation studies, what responsibilities would the nurse have? A) Working with animals who are given experimental drugs B) Choosing appropriate patients to be involved in the drug study C) Monitoring and observing patients closely for adverse effects D) Conducting research to determine effectiveness of the drug Ans: C Feedback: Phase III studies involve use of a drug in a vast clinical population in which patients are asked to record any symptoms they experience while taking the drugs. Nurses may be responsible for helping collect and analyze the information to be shared with the Food and Drug Administration (FDA) but would not conduct research independently because nurses do not prescribe medications. Use of animals in drug testing is done in the preclinical trials. Select patients who are involved in phase II studies to participate in studies where the participants have the disease the drug is intended to treat. These patients are monitored closely for drug action and adverse effects. Phase I studies involve healthy human volunteers who are usually paid for their participation. Nurses may observe for adverse effects and toxicity. 4. What concept is considered when generic drugs are substituted for brand name drugs? A) Bioavailability B) Critical concentration C) Distribution D) Half-life Ans: A Feedback: Bioavailability is the portion of a dose of a drug that reaches the systemic circulation and is available to act on body cells. Binders used in a generic drug may not be the same as those used in the brand name drug. Therefore, the way the body breaks down and uses the drug may differ, which may eliminate a generic drug substitution. Critical concentration is the amount of a drug that is needed to cause a therapeutic effect and should not differ between generic and brand name medications. Distribution is the phase of pharmacokinetics, which involves the movement of a drug to the bodys tissues and is the same in generic and brand name drugs. A drugs half-life is the time it takes for the amount of drug to decrease to half the peak level, which should not change when substituting a generic medication. 5. A nurse is assessing the patients home medication use. After listening to the patient list current medications, the nurse asks what priority question? A) Do you take any generic medications? B) Are any of these medications orphan drugs? C) Are these medications safe to take during pregnancy? D) Do you take any over-the-counter medications? Ans: D Feedback: It is important for the nurse to specifically question use of over-the-counter medications because patients may not consider them important. The patient is unlikely to know the meaning of orphan drugs unless they too are health care providers. Safety during pregnancy, use of a generic medication, or classification of orphan drugs are things the patient would be unable to answer but could be found in reference books if the nurse wishes to research them. 6. After completing a course on pharmacology for nurses, what will the nurse know? A) Everything necessary for safe and effective medication administration B) Current pharmacologic therapy; the nurse will not require ongoing education for 5 years. C) General drug information; the nurse can consult a drug guide for specific drug information. D) The drug actions that are associated with each classification of medication Ans: C Feedback: After completing a pharmacology course nurses will have general drug information needed for safe and effective medication administration but will need to consult a drug guide for specific drug information before administering any medication. Pharmacology is constantly changing, with new drugs entering the market and new uses for existing drugs identified. Continuing education in pharmacology is essential to safe practice. Nurses tend to become familiar with the medications they administer most often, but there will always be a need to research new drugs and also those the nurse is not familiar with because no nurse knows all medications. 7. A nurse is instructing a pregnant patient concerning the potential risk to her fetus from a Pregnancy Category B drug. What would the nurse inform the patient? A) Adequate studies in pregnant women have demonstrated there is no risk to the fetus. B) Animal studies have not demonstrated a risk to the fetus, but there have been no adequate studies in pregnant women. C) Animal studies have shown an adverse effect on the fetus, but there are no adequate studies in pregnant women. D) There is evidence of human fetal risk, but the potential benefits from use of the drug may be acceptable despite potential risks. Ans: B Feedback: Category B indicates that animal studies have not demonstrated a risk to the fetus. However, there have not been adequate studies in pregnant women to demonstrate risk to a fetus during the first trimester of pregnancy and no evidence of risk in later trimesters. Category A indicates that adequate studies in pregnant women have not demonstrated a risk to the fetus in the first trimester or in later trimesters. Category C indicates that animal studies have shown an adverse effect on the fetus, but no adequate studies in humans. Category D reveals evidence of human fetal risk, but the potential benefits from the use of the drugs in pregnant women may outweigh potential risks. 8. Discharge planning for patients leaving the hospital should include instructions on the use of over-the-counter (OTC) drugs. Which comment by the patient would demonstrate a good understanding of OTC drugs? A) OTC drugs are safe and do not cause adverse effects if taken properly. B) OTC drugs have been around for years and have not been tested by the Food and Drug Administration (FDA). C) OTC drugs are different from any drugs available by prescription and cost less. D) OTC drugs could cause serious harm if not taken according to directions. Ans: D Feedback: It is important to follow package directions because OTCs are medications that can cause serious harm if not taken properly. OTCs are drugs that have been determined to be safe when taken as directed; however, all drugs can produce adverse effects even when taken properly. They may have originally been prescription drugs that were tested by the FDA or they may have been grandfathered in when the FDA laws changed. OTC education should always be included as a part of the hospital discharge instructions. 9. What would be the best source of drug information for a nurse? A) Drug Facts and Comparisons B) A nurses drug guide C) A drug package insert D) The Physicians Drug Reference (PDR) Ans: B Feedback: A nurses drug guide provides nursing implications and patient teaching points that are most useful to nurses in addition to need-to-know drug information in a very user friendly organizational style.Lippincotts Nursing Drug Guide (LNDG) has drug monographs organized alphabetically and includes nursing implications and patient teaching points. Numerous other drug handbooks are also on the market and readily available for nurses to use. Although other drug reference books such as Drug Facts and Comparisons, PDR, and drug package inserts can all provide essential drug information, they will not contain nursing implications and teaching points and can be more difficult to use than nurses drug guides. 10. The nurse is preparing to administer a medication from a multidose bottle. The label is torn and soiled but the name of the medication is still readable. What is the nurses priority action? A) Discard the entire bottle and contents and obtain a new bottle. B) Find the drug information and create a new label for the bottle. C) Ask another nurse to verify the contents of the bottle. D) Administer the medication if the name of the drug can be clearly read. Ans: A Feedback: When the drug label is soiled obscuring some information the safest action by the nurse is to discard the bottle and contents because drug labels contain a great deal of important information, far more than just the name of the drug. Concentration of the drug, expiration date, administration directions, and precautions may be missing from the label and so put the patient at risk. Looking up drug information in a drug handbook or consulting with another nurse will not supply the expiration date or concentration of medication. Be safe and discard the bottle and its contents. 11. What aspect of pharmacology does a nurse study? (Select all that apply.) A) Chemical pharmacology B) Molecular pharmacology C) Impact of drugs on the body D) The bodys response to a drug E) Adverse and anticipated drug effects Ans: C, D, E Feedback: Nurses study pharmacology from a pharmacotherapeutic level, which includes the effect of drugs on the body, the bodys response to drugs, and both expected and unexpected drug effects. Chemical and molecular pharmacology (Options A and B) are not included in nursing pharmacology courses. 12. The nurse, providing patient teaching about home medication use to an older adult, explains that even when drugs are taken properly they can produce negative or unexpected effects. What are these negative or unexpected effects called? A) Teratogenic effects B) Toxic effects C) Adverse effects D) Therapeutic effects Ans: C Feedback: Negative or unexpected effects are known as adverse or side effects. Teratogenic effects are adverse effects on the fetus and not a likely concern for an older adult. Toxic effects occur when medication is taken in larger than recommended dosages caused by an increase in serum drug levels. Therapeutic effects are the desired actions for which the medication is prescribed. 13. After administering a medication, for what would the nurse assess the patient? A) Drug effects B) Allergies C) Pregnancy D) Preexisting conditions Ans: A Feedback: After the medication is administered, the nurse assesses the patient for drug affects, both therapeutic and adverse. The nurse would assess the patient for allergies, preexisting conditions, and pregnancy before administering a medication. 14. The nurse receives an order to administer an unfamiliar medication and obtains a nurses drug guide published four years earlier. What is the nurses most prudent action? A) Find a more recent reference source. B) Use the guide if the drug is listed. C) Ask another nurse for drug information. D) Verify the information in the guide with the pharmacist. Ans: A Feedback: The nurse is responsible for all medications administered and must find a recent reference source to ensure the information learned about the medication is correct and current. Using an older drug guide could be dangerous because it would not contain the most up-to-date information. Asking another nurse or the pharmacist does not guarantee accurate information will be obtained and could harm the patient if the information is wrong. 15. What would the nurse provide when preparing a patient for discharge and home medication self-administration? A) Personal contact information to use if the patient has questions B) Thorough medication teaching about drugs and the drug regimen C) Over-the-counter medications to use to treat potential adverse effects D) A sample size package of medication to take home until prescription is filled Ans: B Feedback: The nurse is responsible for providing thorough medication teaching about drugs and the drug regimen to ensure the patient knows how to take the medication and when to notify the provider. The nurse never provides personal contact information to a patient. If adverse effects arise, the patient is taught to call the health care provider and should not self-medicate with over-the-counter drugs, which could mask serious symptoms. The nurse never dispenses medication because it must be properly labeled for home use; this is done by the pharmacy. 16. In response to the patients question about how to know whether drugs are safe, the nurse explains that all medications undergo rigorous scientific testing controlled by what organization? A) Food and Drug Administration (FDA) B) Drug Enforcement Agency (DEA) C) Centers for Disease Control and Prevention (CDC) D) Joint Commission on Accreditation of Healthcare Organizations (JCAHO) Ans: A Feedback: The FDA is responsible for controlling and regulating the development and sale of drugs in the United States, allowing new drugs to enter the market only after being subjected to rigorous scientific testing. The DEA regulates and controls the use of controlled substances. The CDC monitors and responds to infectious diseases. The JCAHO is an accrediting body that inspects acute care facilities to ensure minimum standards are met. 17. The nurse, assisting with Phase I drug studies, is talking with a woman who asks, Why cant I participate in this study? What would be the nurses best response? A) Drugs pose a greater risk to women of reproductive age. B) Drugs are only tested on men because they are stronger. C) Women are more prone to adverse effects from medications. D) Drugs affect women differently than they affect women. Ans: A Feedback: Phase I drug trials usually involve healthy male volunteers because chemicals may exert an unknown and harmful effect on ova in women which could result in fetal damage when the woman becomes pregnant. Drugs are tested on both men and women, but women must be fully informed of risks and sign a consent stating they understand the potential for birth defects. Women are not more prone to adverse effects of medications. Although some drugs may affect women differently than men, this is a rationale for why drugs need to be tested on women, not an explanation of why women are not included in a phase I study. 18. The patient tells the nurse about a new drug being tested to treat the disease she was diagnosed with and asks the nurse whether the doctor can prescribe a medication still in the preclinical phase of testing. What is the nurses best response? A) The doctor would have to complete a great deal of paperwork to get approval to prescribe that drug. B) Sometimes pharmaceutical companies are looking for volunteers to test a new drug and the doctor could give them your name. C) Drugs in the preclinical phase of testing are only tested on animals and so would not be available to you. D) Drugs in the preclinical phase of testing are given only to healthy young men and so would not be available to you. Ans: C Feedback: During the preclinical phase of testing drugs are tested on animals and are not available to patients. In phase I, the drug is tested on volunteers who are usually healthy young men. It is only in phase III studies that the drug is made available to prescribers who agree to closely monitor patients getting the medication. 19. The nurse is caring for a patient who had a severe, acute, previously unseen adverse effect of a drug in Phase III testing. The patient asks, After all the testing done on this drug, didnt they know this adverse effect could occur? What is the nurses best response? (Select all that apply.) A) Pharmaceutical companies sometimes underreport problems to make more money. B) Your response to this medication will be reported to the drug company and the Food and Drug Administration (FDA). C) When a drug begins to be used by a large clinical market, new adverse effects may be found. D) The pharmaceutical company weighs the benefits of the drug with the severity of adverse effects. E) After a drug reaches phase III testing it is considered an accepted drug and will not be recalled. Ans: B, C Feedback: When a new and unexpected adverse effect occurs, especially one of a serious nature, it is reported to the drug company who reports it to the FDA immediately. When a large number of people begin using the drug in phase III studies, it is not unusual to identify adverse effects not previously noted. It would be both unprofessional and inaccurate to imply that pharmaceutical companies put profit ahead of patient concern because lawsuits would remove any potential profit if a drug proves harmful. The FDA is responsible for weighing risk versus benefit in deciding whether to allow the drug to move to the next phase of testing. Drugs found to have serious adverse effects can be removed from the market at any time. 20. The telephone triage nurse receives a call from a patient asking for a prescription for a narcotic to manage his surgical pain. The nurse explains that narcotic prescriptions must be written and cannot be called in to the pharmacy. The patient says, Why are narcotics so difficult to get a prescription for? What is the nurses best response? A) The Drug Enforcement Agency (DEA) determines the risk for addiction and the Food and Drug Administration (FDA) enforces their control. B) The increase in the number of drug addicts has made the rules stronger. C) The Centers for Disease Control and Prevention (CDC) regulates use of controlled substances to reduce the risk of injury. D) Controlled substances like narcotics are controlled by the FDA and the DEA. Ans: D Feedback: Controlled substances are controlled by the FDA and the DEA: the DEA enforces control while the FDA determines abuse potential. Regulations related to controlled substances have remained strict and specific and have not been significantly impacted by substance abusers. The CDC is not involved in control of narcotics and other controlled substances. 21. The nurse explains the Drug Enforcement Agencys (DEAs) schedule of controlled substances to the nursing assistant who asks, Do you ever get a prescription for Schedule I medications? What is the nurses best response? A) Schedule I medications have no medical use so they are not prescribed. B) Schedule I medications have the lowest risk for abuse and do not require a prescription. C) Schedule I medications are only prescribed in monitored units for patient safety. D) Schedule I medications are found in antitussives and antidiarrheals sold over the counter. Ans: A Feedback: Schedule I medications have no medical use and are never prescribed. Schedule V medications have the lowest risk for abuse and are found mostly in antitussives and antidiarrheals but they are not sold over the counter. 22. The nurse, working on the maternity unit, receives a call from a pregnant woman asking how she can know whether a medication is safe to take while pregnant. What is the nurses best response? A) You can take any drug indicated as a Category A. B) No medications should be taken during pregnancy. C) Never take medication until you receive approval from your health care provider. D) Most medications are safe but you need to weigh benefit against risk. Ans: C Feedback: The best response to a pregnant woman asking about medication usage is to talk with her obstetric practitioner because the best advice will come from someone who knows their health and pregnancy history. While Category A drugs have no known risk, they may be contraindicated by the womans health condition or pregnancy issues and many pregnant women would not know what it means to be a Category A drug. Medications can be helpful during pregnancy if taken safely and appropriately. Although risk benefit needs to be weighed, it should occur with advice from the obstetric practitioner. 23. A patient asks the nurse, What is a Drug Enforcement Agency (DEA) number? What is the nurses best response? A) DEA Numbers are given to physicians and pharmacists when they register with the DEA to prescribe and dispense controlled substances. B) Physicians must have a DEA number in order to prescribe any type of medication for patients. C) DEA numbers are case numbers given when someone breaks the law involving a controlled substance. D) DEA numbers are contact numbers to talk with someone at the DEA when questions arise about controlled substances. Ans: A Feedback: All pharmacists and physicians must register with the DEA. They are given numbers that are required before they can dispense or prescribe controlled substances. DEA numbers are only needed when prescribing controlled substances. A DEA number is neither a case number nor a phone number. 24. When moving to another state, what is the nurse responsible for becoming familiar with? A) Local policies and procedures for controlled substance administration B) Local providers Drug Enforcement Agency (DEA) number for prescribing controlled substances C) The agency monitoring controlled substances in the new state D) Board of Nursing regulations of controlled substances in the new state Ans: A Feedback: The nurse needs to learn local policies and procedures for controlled substance administration because they can vary with some local governments more rigorous than others. Nurses do not memorize a providers DEA numbers. The DEA is a federal agency that monitors controlled substances in all states. State boards of nursing do not regulate controlled substances but may regulate how controlled substances are administered by nurses. 25. The patient looks at the prescription provided by the doctor and asks the nurse whether he can request a generic substitution. The nurse answers No when noting what on the prescription? A) No refills B) DAW C) Brand name used on prescription D) Patient older than 65 years of age Ans: B Feedback: DAW stands for dispense as written and means that the doctor does not want a generic substituted for the prescribed medication. Requesting no refills does not preclude the substitution of a generic medication. Even when the brand name is ordered, the pharmacist can substitute a generic equivalent so long as the prescriber does not write DAW. Generic substitutions are not impacted by the patients age. 26. The patient asks the nurse why generic drugs would be used and voices concerns that only the brand name product will be safe. What is the nurses best response? A) Generic drugs are often less expensive. B) Some quality control problems have been found with generic drugs. C) Most generic drugs are very safe and can be cost effective as well. D) Although initial cost is higher for a brand name it may cost less in the long run. Ans: C Feedback: Most generic medications are completely safe and may be identical to the brand name drug except generic medications are often less expensive, but this does not address the patients concern about safety. Although some quality control issues have occurred in the past, this does not address the patients concerns regarding safety or explain why generic drugs are prescribed and used. Although some doctors believe initial cost is higher but will cost less over time, this response also does not address the patients concerns. 27. While studying for the test, the nursing student encounters the following drug: papaverine (Pavabid). What does the nursing student identify the name Pavabid as? A) The generic name B) The chemical name C) The brand name D) The chemical and generic name Ans: C Feedback: Several clues indicate the brand name including capitalization of the first letter in the name and in parentheses. Generic names are not capitalized; chemical names are descriptions of the chemistry of the medication resulting in complicated names. 28. The patient is prescribed a medication that was just placed in Phase IV study. The patient tells the nurse, This medication is too expensive. Could the doctor order a generic form of this medication? What is the nurses most accurate response? A) Medications are not produced in generic form until the patent expires, which normally takes several years. B) You can request the generic form but the binder used may make the drug less effective for this medication. C) The generic form of the medication would not be any less expensive because this is a relatively new medication. D) Generic medications are lower quality drugs and that would mean you would not be getting the best treatment available. Ans: A Feedback: When a new drug enters the market, it is given a time-limited patent; generic forms of the medication cannot be produced until the patent expires. Because no generic version of this drug will exist because it is so new, it is impossible to predict what binder will be used or what the cost would be. 29. The nurse learns that a drug needed by the patient is classified as an orphan drug and recognizes what as a reason for this classification? (Select all that apply.) A) The drug is rarely prescribed. B) The drug has dangerous adverse effects. C) The drug treats a rare disease. D) The patent on the medication is still effective. E) Production by a company that only manufactures drugs. Ans: A, B, C Feedback: Drugs are classified as orphan drugs when they are not financially viable for a drug company to produce either because of risk for lawsuits about adverse effects or because the drug is not prescribed, which is often seen in rare diagnoses. Generic drugs are not produced until the patent expires, but this has no impact on classifying a particular drug as an orphan drug. Generic drugs are often produced by companies that only manufacture drugs without conducting research, but this has no bearing on the classification of orphan drugs. 30. While collecting a medication history, the patient admits to doubling the recommended dosage of an over-the-counter (OTC) medication, saying Its harmless or they would require a prescription. What is the nurses best response? A) OTC drugs are serious medications and carry serious risks if not taken as directed. B) Taking medications like that is careless and you could kill yourself doing it. C) Sometimes you need to take more than the package directs to treat the symptoms. D) Did you notify your doctor of the increased dosage you were taking? Ans: A Feedback: OTC drugs are no less a medication than prescription drugs and carry the same types of risks for overdosage and toxicity if directions are not followed. Although increasing the dosage is careless and dangerous, it is important to use the information as a teaching opportunity rather than scolding the patient. Agreeing with the patient or asking her if she talked to the doctor misses the teaching opportunity, which could be harmful for the patient. 31. The patient asks the nurse, Is it safe to take over-the-counter (OTC) medications with prescription medications? What is the nurses best response? (Select all that apply.) A) OTC medications can interact with prescription medications. B) It is important to tell your doctor all medications you take, including OTC. C) OTC medications could mask or hide signs and symptoms of a disease. D) You should avoid taking any OTC medication when taking prescription drugs. E) Taking OTC medications can make your prescription medication more effective. Ans: A, B, C Feedback: OTC medications can interact with prescription medications or other OTC so it is always important to consult your pharmacist and provider for advice. To provide the most accurate instruction, the health care provider must know all medications taken including dietary supplements, OTC, and prescription. OTC medications could mask or hide symptoms of a disease so it is always important to consult a physician if symptoms persist. OTC medications are not prohibited when taking prescription drugs as long as no drug interaction occurs. How an OTC will impact a prescription medication varies depending on the medications involved, so it is incorrect to say it will make the prescription drug more effective. 32. Before administering a prescription medication, what information does the nurse find on the drug label? (Select all that apply.) A) Brand name B) Generic name C) Drug concentration D) Expiration date E) Adverse effects Ans: A, B, C, D Feedback: Prescription drug labels will contain the brand name, generic name, drug concentration, and expiration date. Adverse effects will not be listed on drug labels. 33. The nurse is preparing a medication that is new to the market and cannot be found in the nurses drug guide. Where can the nurse get the most reliable information about this medication? A) Package insert B) Another nurse C) Drug manufacturer D) Physician Ans: A Feedback: The most reliable information about the drug can be found on the package insert supplied by the manufacturer because it was prepared according to strict Food and Drug Administration (FDA) regulations. Asking another nurse or the physician is not reliable and cannot be verified as accurate. It would not be realistic to call the drug manufacturer for information. 34. The nurse explains that what drug resource book is compiled from package inserts? A) Nurses Drug Guide B) Physiciass Desk Reference (PDR) C) Drug Facts and Comparisons D) AMA Drug Evaluations Ans: B Feedback: The PDR is a compilation of information found on package inserts. The Nurses Drug Guide uses more easily understood language and incorporates nursing considerations and patient teaching points. Drug Facts and Comparisons includes cost comparison, often not found in other drug resource guides. The AMA Drug Evaluations is far less biased than the PDR and includes drugs still in the research stage of development. Chapter 2. Basics of Pharmacology 1. Drugs do not metabolize the same way in all people. For what patient would a nurse expect to assess for an alteration in drug metabolism? A) A 35-year-old woman with cervical cancer B) A 41-year-old man with kidney stones C) A 50-year-old man with cirrhosis of the liver D) A 62-year-old woman in acute renal failure Ans: C Feedback: The liver is the most important site of drug metabolism. If the liver is not functioning effectively, as in patients with cirrhosis, drugs will not metabolize normally so that toxic levels could develop unless dosage is reduced. A patient with cervical cancer or kidney stones would not be expected to have altered ability to metabolize drugs so long as no liver damage existed. The patient with renal failure would have altered excretion of the drugs through the renal system but metabolism would not be impacted. 2. A patient presents to the emergency department with a drug level of 50 units/mL. The half-life of this drug is 1 hour. With this drug, concentrations above 25 units/mL are considered toxic and no more drug is given. How long will it take for the blood level to reach the non-toxic range? A) 30 minutes B) 1 hour C) 2 hours D) 3 hours Ans: B Feedback: Half-life is the time required for the serum concentration of a drug to decrease by 50%. After 1 hour, the serum concentration would be 25 units/ mL (50/2) if the body can properly metabolize and excrete the drug. After 2 hours, the serum concentration would be 12.5 units/mL (25/2) and reach the nontoxic range. In 30 minutes the drug level would be 37.5 units/mL, whereas in 3 hours the drug level would be 6.25. 3. A patient has recently moved from Vermont to Southern Florida. The patient presents to the clinic complaining of dizzy spells and weakness. While conducting the admission assessment, the patient tells the nurse that he have been on the same antihypertensive drug for 6 years and had stable blood pressures and no adverse effects. Since his move, he has been having problems and he feels that the drug is no longer effective. The clinic nurse knows that one possible reason for the change in the effectiveness of the drug could be what? A) The impact of the placebo effect on the patients response. B) The accumulative effect of the drug if it has been taken for many years. C) The impact of the warmer environment on the patients physical status. D) Problems with patient compliance with the drug regimen while on vacation. Ans: C Feedback: Antihypertensive drugs work to decrease the blood pressure. When a patient goes to a climate that is much warmer than usual, blood vessels dilate and the blood pressure falls. If a patient is taking an antihypertensive drug and moves to a warmer climate, there is a chance that the patients blood pressure will drop too low, resulting in dizziness and feelings of weakness. Even mild dehydration could exacerbate these effects. Most antihypertensives are metabolized and excreted and do not accumulate in the body. Patients must be very compliant with their drug regimen on vacation. After several years on an antihypertensive drug, the effects of that drug are known; therefore, the placebo effect should not be an issue. 4. An important concept taught by the nurse when providing medication teaching is the need to provide a complete list of medications taken to health care providers to avoid what? A) Spending large amounts of money on medications B) Allergic reactions to medications C) Drugdrug interactions D) Critical concentrations of medications in the body Ans: C Feedback: It is important that all health care providers have a complete list of the patients medications to avoid drugdrug interactions caused by one provider ordering a medication, unaware of another medication the patient is taking that could interact with the new prescription. Using the same pharmacist for all prescriptions will also help to prevent this from happening. Informing the provider of all medications taken will not reduce costs of medications, which is best accomplished by requesting generic medications. Allergies should be disclosed to all health care providers as well, but this is not why it is important to provide a complete list of medications taken. Critical concentrations are desirable because that is the amount of drug needed to cause a therapeutic effect, or, in other words, to have the effect the drug is prescribed for. 5. A pharmacology student asks the instructor what an accurate description of a drug agonist is. What is the instructors best response? A) A drug that reacts with a receptor site on a cell preventing a reaction with another chemical on a different receptor site B) A drug that interferes with the enzyme systems that act as catalyst for different chemical reactions C) A drug that interacts directly with receptor sites to cause the same activity that a natural chemical would cause at that site D) A drug that reacts with receptor sites to block normal stimulation, producing no effect Ans: C Feedback: Agonists are drugs that produce effects similar to those produced by naturally occurring neurotransmitters, hormones, or other substances found in the body. Noncompetitive antagonists are drugs that react with some receptor sites preventing the reaction of another chemical with a different receptor site. Drugenzyme interactions interfere with the enzyme systems that stimulate various chemical reactions. 6. A nurse is caring for a patient who has been receiving a drug by the intramuscular route but will receive the drug orally after discharge. How does the nurse explain the increased dosage prescribed for the oral dose? A) Passive diffusion B) Active transport C) Glomerular filtration D) First-pass effect Ans: D Feedback: The first-pass effect involves drugs that are absorbed from the small intestine directly into the portal venous system, which delivers the drug molecules to the liver. After reaching the liver, enzymes break the drug into metabolites, which may become active or may be deactivated and readily excreted from the body. A large percentage of the oral dose is usually destroyed and never reaches tissues. Oral dosages account for the phenomenon to ensure an appropriate amount of the drug in the body to produce a therapeutic action. Passive diffusion is the major process through which drugs are absorbed into the body. Active transport is a process that uses energy to actively move a molecule across a cell membrane and is often involved in drug excretion in the kidney. Glomerular filtration is the passage of water and water-soluble components from the plasma into the renal tubule. 7. A nurse is working as a member of a research team involved in exploring the unique response to drugs each individual displays based on genetic make-up. What is this area of study is called? A) Pharmacotherapeutics B) Pharmacodynamics C) Pharmacoeconomics D) Pharmacogenomics Ans: D Feedback: Pharmacogenomics is the area of study that includes mapping of the human genome. In the future, medical care and drug regimens may be personally designed based on a patients unique genetic make-up. Pharmacotherapeutics is the branch of pharmacology that deals with the uses of drugs to treat, prevent, and diagnose disease. Pharmacodynamics involves how a drug affects the body. Pharmacoeconomics includes the costs involved in drug therapy. 8. The nurse uses what term to describe the drug level required to have a therapeutic effect? A) Critical concentration B) Dynamic equilibrium C) Selective toxicity D) Active transport Ans: A Feedback: A critical concentration of a drug must be present before a reaction occurs within the cells to bring about the desired therapeutic effect. A dynamic equilibrium is obtained from absorption of a drug from the site of drug entry, distribution to the active site, metabolism in the liver, and excretion from the body to have a critical concentration. Selective toxicity is the ability of a drug to attach only to those systems found in foreign cells. Active transport is the process that uses energy to actively move a molecule across a cell membrane and is often involved in drug excretion in the kidney. 9. A nurse is caring for a patient who is supposed to receive two drugs at the same time. What is the nurses priority action? A) Wash her hands before handling the medications. B) Consult a drug guide for compatibility. C) Question the patient concerning drug allergies. D) Identify the patient by checking the armband and asking the patient to state his name. Ans: B Feedback: A nurse should first consult a drug guide for compatibility when two or more drugs are being given at the same time. After compatibility is determined the medication can be administered. The nurse will perform hand hygiene, check for patient allergies, and ensure the right patient receives the medication by using two identifiers. 10. The nurse is talking with a group of nursing students who are doing clinical hours on the unit. A student asks if all intramuscular (IM) drugs are absorbed the same. What factor would the floor nurse tell the students to affect absorption of the IM administration of drugs? A) Perfusion of blood to the subcutaneous tissue B) Integrity of the mucous membranes C) Environmental temperature D) Blood flow to the gastrointestinal tract Ans: C Feedback: A cold environmental temperature can cause blood vessels to vasoconstrict and decreases absorption or in a hot environment vasodilate and increase absorption of IM medications. Blood flow to the subcutaneous tissues interferes with subcutaneous injection and blood flow to the gastrointestinal (GI) tract causes alterations in absorption for oral medications. The condition of mucous membranes can interfere with sublingual (under the tongue) and buccal (in the cheek) administration of drugs. 11. The patient is taking a drug that affects the body by increasing cellular activity. Where does this drug work on the cell? A) Receptor sites B) Cell membrane C) Golgi body D) Endoplasmic reticulum Ans: A Feedback: Many drugs are thought to act at specific areas on cell membranes called receptor sites. After the receptor site is activated, this in turn activates the enzyme systems to produce certain effects, such as increased or decreased cellular activity, changes in cell membrane permeability, or alterations in cellular metabolism. Receptor sites are generally located on the outside of cells and allow the drug to bypass the cell membrane. The Golgi body and endoplasmic reticulum are not involved in this process. 12. Several processes enable a drug to reach a specific concentration in the body. Together they are called dynamic equilibrium. What are these processes? (Select all that apply.) A) Distribution to the active site B) Biotransformation C) Absorption from the muscle D) Excretion E) Interaction with other drugs Ans: A, B, D Feedback: The actual concentration that a drug reaches in the body results from a dynamic equilibrium involving several processes: Absorption from the site of entry (can be from the muscle, the gastrointestinal (GI) tract if taken orally, of the subcutaneous tissue if given by that route); Distribution to the active site; biotransformation (metabolism) in the liver; excretion from the body. Interaction with other drugs is not part of the dynamic equilibrium. 13. A nurse is administering digoxin to a patient. To administer medications so that the drug is as effective as possible, the nurse needs to consider what? A) Pharmacotherapeutics B) Pharmacokinetics C) Pharmacoeconomics D) Pharmacogenomics Ans: B Feedback: When administering a drug, the nurse needs to consider the phases of pharmacokinetics so that the drug regimen can be made as effective as possible. Pharmacogenomics is the area of study that includes mapping of the human genome. Pharmacotherapeutics is the branch of pharmacology that deals with the uses of drugs to treat, prevent, and diagnose disease. Pharmacoeconomics includes all costs involved in drug therapy. 14. The nurse is explaining how medications work to a group of peers and explains that disruption of a single step in any enzyme system disrupts what? A) Cell life B) Cell membrane C) Cell receptor sites D) Cell function Ans: D Feedback: If a single step in one of the many enzyme systems is blocked, normal cell function is disrupted. Cell life and cell membrane may be impacted by disruption of some enzymes but not all enzymes. Receptor sites would not be disrupted by disruption in a single step in the enzyme system. 15. The processes involved in dynamic equilibrium are key elements in the nurses ability to determine what? A) Dosage scheduling B) Amount of solution for mixing parenteral drugs C) Timing of other drugs the patient is taking D) How long the patient has to take the drug Ans: A Feedback: These processes are key elements in determining the amount of drug (dose) and the frequency of dose repetition (scheduling) required to achieve the critical concentration for the desired length of time. The processes in dynamic equilibrium are not key elements in determining the amount of diluents for intramuscular (IM) drugs; they do not aid in the timing of the other drugs the patient is taking or how long the patient has to take the drug. 16. What factor influences drug absorption? A) Kidney function B) Route of administration C) Liver function D) Cardiovascular function Ans: B Feedback: Drug absorption is influenced by the route of administration. IV administration is the fastest method; drug absorption is slower when given orally. Kidney function impacts excretion, liver function impacts metabolism, and cardiovascular function impacts distribution. 17. What does the lipid solubility of the drug influence? A) Absorption of the drug B) Metabolism of the drug C) Excretion of the drug D) Distribution of the drug Ans: D Feedback: Factors that can affect distribution include the drugs lipid solubility and ionization and the perfusion of the reactive tissue. The lipid solubility of a drug does not influence absorption, metabolism, or excretion. 18. The nursing students are learning about the half-life of drugs. A student asks the instructor to explain half-life. What is the instructors best response? A) Half-life of a drug is the time it takes for the amount of drug in the body to decrease to half of the peak level it previously achieved. B) Half-life is the amount of time it takes for the drug to be metabolized by the body. C) Half-life is the amount of time it takes for half of the drug to reach peak level in the body. D) Half-life of a drug is the time it takes for the drug to reach half its potential peak level in the body. Ans: A Feedback: The half-life of a drug is the time it takes for the amount of drug in the body to decrease to half the peak level it previously achieved. Therefore Options B, C, and D are not correct. 19. The patient is taking a 2-mg dose of ropinerol XR. The drug has a half-life of 12 hours. How long will it be before only 0.25 mg of this drug remains in the patients system? A) 24 hours B) 36 hours C) 48 hours D) 60 hours Ans: B Feedback: The half-life of a drug is the time it takes for the amount of drug in the body to decrease to half of the peak level it previously achieved. At 12 hours there will be 1 mg of the drug available to the body. At 24 hours there will be 0.5 mg; at 36 hours there will be 0.25 mg; at 48 hours there will be 0.125 mg, and at 60 hours there will be 0.0625 mg. 20. The patient has a diagnosis of multiple sclerosis and is taking the drug interferon beta-1a (Rebif). The patient takes this drug by subcutaneous injection three times a week. The dosage is 44 mcg per injection. If the patient takes an injection on Monday, how much of the drug would still be in the patients system when she takes her next injection on Wednesday, assuming the half-life of the drug is 24 hours? A) 22 mcg B) 16.5 mcg C) 11 mcg D) 5.5 mcg Ans: C Feedback: The half-life of a drug is the time it takes for the amount of drug in the body to decrease to 1 half the peak level it previously achieved. On Tuesday, there would be 22 mcg remaining in the body, so option A is incorrect. On Wednesday 11 mcg would remain, so option C is the correct answer. At 12 hours before taking the next dose on Wednesday, there would be 16.5 mcg remaining. If the injection were not taken on Wednesday, 12 hours after the dose was due, there would be 5.5 mcg remaining. 21. The patient is a 6-year-old child who is taking 125 mg of amoxicillin every 6 hours. Assuming that the half-life of Amoxicillin is 3 hours, how much Amoxicillin would be in the childs body at the time of the next administration of the drug? A) 62.5 mg B) 46.875 mg C) 31.25 mg D) 15.625 mg Ans: C Feedback: The half-life of a drug is the time it takes for the amount of drug in the body to decrease to 1 half the peak level it previously achieved. Option A would occur at 3 hours after the original dose of amoxicillin. Option B would occur 4 1/2 hours after the original dose. Option C would occur at 6 hours after the original dose. Option D would occur at 7 1/2 hours after the original dose. 22. A drug with a half-life of 4 hours is administered at a dosage of 100 mg. How much of the drug will be in the patients system 8 hours after administration? A) 75 mg B) 50 mg C) 37.5 mg D) 25 mg Ans: D Feedback: The half-life of a drug is the time it takes for the amount of drug in the body to decrease to 1 half the peak level it previously achieved. Option A would occur 2 hours after administration of the drug. Option B would occur at 4 hours. Option C would occur at 6 hours. Option D would occur at 8 hours after the original administration of the drug. 23. The nurse administers amoxicillin 500 mg. The half-life of this drug is approximately 1 hour. At what point would the drug level in the body be 62.5 mg if the drug was not administered again? A) 1 hours after the original dose B) 2 hours after the original dose C) 3 hours after the original dose D) 4 hours after the original dose Ans: C Feedback: The half-life of a drug is the time it takes for the amount of drug in the body to decrease to one-half of the peak level it previously achieved. At a dose of 500 mg the drug level would be 250 mg in 1 hour, 125 mg in 2 hours, 62.5 mg in 3 hours, and 31.25 mg in 4 hours so the correct answer is 3 hours. 24. The nurse is caring for a patient who is receiving gentamicin, 250 mg and fluconazole (Diflucan), 500 mg at the same time. The nurse knows that if these two drugs competed with each other for protein-binding sites, what would this do? A) Make the patient gentamicin deficient B) Make the patient fluconazole deficient C) Counteract any positive benefit the drugs would have D) Alter the effectiveness of both drugs Ans: D Feedback: Some drugs compete with each other for protein-binding sites, altering effectiveness or causing toxicity when the two drugs are given together. Nothing in the scenario would indicate that the patient would be either Gentamicin or Diflucan deficient, nor does it indicate that these drugs cannot be given together because they would counteract each other. 25. The student nurse asks the instructor why a patient with a central nervous system infection is receiving antibiotics that will not cross the bloodbrain barrier. What is the instructors most correct response? A) A severe infection alters the bloodbrain barrier to allow the drug to cross. B) A medication that is water soluble is more likely to cross the blood-brain barrier. C) Antibiotics are the exception to the bloodbrain barrier and cross easily. D) An infection that spreads outside the central nervous system helps drugs cross the barrier. Ans: A Feedback: Effective antibiotic treatment can occur only when the infection is severe enough to alter the bloodbrain barrier and allow antibiotics to cross. Lipid- soluble, not water-soluble, medications cross the bloodbrain barrier more easily and most antibiotics are lipid soluble, so they are not the exception. No matter where the infection originates, drugs must cross the bloodbrain barrier to treat central nervous system infections. 26. The patient is taking low dose aspirin daily for his heart. The nurse knows only a portion of the medication taken actually reaches the tissue due to what process? A) Distribution B) First-pass effect C) Reduced absorption D) Gastrointestinal circulation Ans: B Feedback: Drugs that are taken orally are usually absorbed from the small intestine directly into the portal venous system and then delivers these absorbed molecules into the liver, which immediately break the drug into metabolites, some of which are active and cause effects in the body, and some of which are deactivated and can be readily excreted from the body. As a result, a large percentage of the oral dose is destroyed at this point and never reaches the tissues. This process is not caused by distribution, absorption, or gastrointestinal circulation. 27. What needs to happen to the proteindrug complex for the drugs to reach the cells where the drug can act? A) The proteindrug complex must break itself into smaller pieces to enter the capillaries. B) The binding site on the protein picks up a chemical to make it soluble in the serum. C) The drug must break away from the protein-binding site and float freely. D) The drug must be dissolved in the plasma so it can enter the capillaries and then the tissues. Ans: C Feedback: Most drugs are bound, to some extent, to proteins in the blood to be carried into circulation. The proteindrug complex is relatively large and cannot enter into capillaries and then into tissues to react. The drug must be freed from the proteins binding site at the tissues. This occurs without the introduction of another chemical or by dissolving in it plasma. 28. The nurse is reviewing the results of the patients laboratory tests. What must the nurse keep in mind when reviewing these results related to medication administration? A) The patients emotional response to the disease process B) The timing of the last dose of medication relative to when blood was drawn C) The possibility of a druglaboratory test interaction D) A change in the bodys responses or actions related to the drug Ans: C Feedback: The body works through a series of chemical reactions. Because of this, administration of a particular drug may alter results of tests that are done on various chemical levels or reactions as part of a diagnostic study. This druglaboratory test interaction is caused by the drug being given and not necessarily by a change in the bodys responses or actions. The patients emotional response or timing of the last dose is not important in drug- laboratory interactions. 29. A patient has come to the clinic and been diagnosed with Lyme disease. The physician has ordered oral tetracycline. What is important for the nurse to include in the teaching plan about tetracycline? (Select all that apply.) A) Do not take the drug with anything high in sodium content to keep from producing a state of hypernatremia in the body. B) Do not take the drug with foods or other drugs that contain calcium. C) Do not take the drug at the same time you take an iron supplement or with foods that are high in iron content. D) Avoid exposure to the sun when taking this drug as it can turn your skin purple. E) Avoid eating bananas at the same time you take this drug as the potassium content of the tetracycline can produce hyperkalemia in the body. Ans: B, C Feedback: The antibiotic tetracycline is not absorbed from the gastrointestinal (GI) tract if calcium or calcium products (e.g., milk) are present in the stomach. It cannot be taken with iron products because a chemical reaction occurs preventing absorption. Although tetracycline can increase sun sensitivity, it does not turn the skin purple. Patients who take tetracycline do not need to avoid eating bananas or foods that are high in potassium. 30. A nurse is caring for a patient taking multiple drugs and is concerned about a possible drugdrug interaction. What is the nurses first and best means of avoiding this problem? A) Consult a drug guide. B) Call the pharmacist. C) Contact the provider. D) Ask another nurse. Ans: A Feedback: Whenever two or more drugs are being given together, first consult a drug guide for a listing of clinically significant drugdrug interactions. Sometimes problems can be avoided by staggering the administration of the drugs or adjusting their dosages. Consulting the pharmacist is not wrong, but it would not be the first action to take. The nurse holds responsibility for his or her own practice so asking a health care provider or another nurse is based on the assumption that that professional is knowledgeable about all drugdrug interactions, which is likely not the case. 31. The nurse promotes optimal drug effectiveness by doing what? (Select all that apply.) A) Incorporate basic history and physical assessment factors into the plan of care. B) Evaluate the effectiveness of drugs after they have been administered. C) Modify the drug regimen to modify adverse or intolerable effects. D) Minimize the number of medications administered to patients. E) Examine factors known to influence specific drugs if they are to be effective. Ans: A, B, C, E Feedback: Incorporate basic history and physical assessment factors into any plan of care so that obvious problems can be identified and handled promptly. If a drug simply does not do what it is expected to do, further examine the factors that are known to influence drug effects. Frequently, the drug regimen can be modified to deal with that influence. Minimizing the number of medications administered is usually not an option because each drug is ordered for a reason of necessity for the patient. 32. The nurse administers a specific medication to an older adult patient every 4 hours. The patient has a history of chronic renal failure. Why would this patient be at risk for toxic drug levels? A) Cumulative effect B) First-pass effect C) Drug interactions D) Cross-tolerance effect Ans: A Feedback: If a drug is taken in successive doses at intervals that are shorter than recommended, or if the body is unable to eliminate a drug properly, the drug can accumulate in the body, leading to toxic levels and adverse effects. This is a cumulative effect. First-pass effect addresses the reduction of available drug when taken orally due to metabolism in the liver before the drug reaches the bloodstream. Drug interactions occur when taken with other drugs, food, or complementary alternative therapies. Cross-tolerance is resistance to drugs within the same class. 33. The patient, diagnosed with cancer, is receiving morphine sulfate (a potent narcotic pain reliever) to relieve cancer pain. Approximately every 7 days the medication is no longer effective in controlling the patients pain and a larger dose is needed to have the same effect. How might the nurse explain why this is happening? A) Tolerance B) Cumulation C) Interactions D) Addiction Ans: A Feedback: The body may develop a tolerance to some drugs over time. Tolerance may arise because of increased biotransformation of the drug, increased resistance to its effects, or other pharmacokinetic factors. When tolerance occurs, the amount of the drug no longer causes the same reaction. Therefore, increasingly larger doses are needed to achieve a therapeutic effect. Cumulative effect occurs when the drug is not properly eliminated and more of the drug is administered, resulting in toxic levels accumulating. Interactions occur when the drug reacts badly with another substance such as food, another drug, or an alternative or complementary therapy. Addiction is the psychological need for a substance. 34. While administering a medication that the nurse has researched and found to have limited effectiveness, the patient tells the nurse, I have read all about this drug and it is such a wonder drug. Im so lucky my doctor prescribed it because I just know it will treat my problem. The nurse suspects this drug will be more effective than usual for this patient because of what effect? A) Cumulative effect B) First-pass effect C) Placebo effect D) Cross-tolerance effect Ans: C Feedback: A drug is more likely to be effective if the patient thinks it will work than if the patient believes it will not work. This is called the placebo effect. If a drug is taken in successive doses at intervals that are shorter than recommended, or if the body is unable to eliminate a drug properly, the drug can accumulate in the body, leading to toxic levels and adverse effects. This is a cumulative effect. First-pass effect addresses the reduction of available drug when taken orally due to metabolism in the liver before the drug reaches the bloodstream. Cross-tolerance is resistance to drugs within the same class. 35. The nurse administers an intravenous medication with a half-life of 24 hours but recognizes what factors in this patient could extend the drugs half-life? (Select all that apply.) A) Gastrointestinal disease B) Kidney disease C) Liver disease D) Cardiovascular disease E) Route of administration Ans: B, C, D Feedback: Kidney disease could slow excretion and extend the drugs half-life. Liver disease could slow metabolism resulting in an extended half-life. Cardiovascular disease could slow distribution resulting in a longer half-life. Gastrointestinal disease would not impact half-life because the medication was injected directly into the bloodstream. Route of administration would not extend half-life because IV injection eliminates the absorption step in the process. Chapter 3. Patient Safety in Medication Administration MULTIPLE CHOICE 1. Where would the procedures and treatments directed by the health care provider be found? a Summary sheet . b Physicians order form . c Physicians progress notes . d History and physical examination form . ANS: B The physicians order form contains all procedures and treatments ordered by the health care provider. A summary sheet provides a brief overview of the hospital course at discharge. Physicians progress notes provide regular observations on the patients course of treatment and response. A history and physical examination form provides information about baseline information from the patient. DIF: Cognitive Level: Knowledge REF: p. 80 OBJ: 2 TOP: Nursing Process Step: Assessment MSC: NCLEX Client Needs Category: Safe, Effective Care Environment 2. Which action will the nurse take when it is determined that the narcotic count is incorrect while obtaining a medication from the narcotic area? a Determine the cause of the discrepancy at the end of the shift. . b Notify the health care provider stat. . c Call the nurse from the previous shift to determine if there was a discrepancy . earlier. d Report the discrepancy to the charge nurse immediately. . ANS: D Reporting the discrepancy to the charge nurse immediately enables the supervisory staff to narrow the time frame during which a medication was taken and not documented. The discrepancy needs to be addressed immediately, and therefore determining the cause of the discrepancy at the end of the shift is not the most appropriate action for the nurse to take. It is not appropriate to contact the health care provider for an incorrect narcotic count. The count would have been verified at shift change; calling the nurse from the previous shift is not an appropriate action for the nurse to take. DIF: Cognitive Level: Analysis REF: p. 95 OBJ: 3 TOP: Nursing Process Step: Implementation MSC: NCLEX Client Needs Category: Safe, Effective Care Environment 3. Which action will the nurse take if a dosage is unclear on a health care providers order? a Ask the patient what dosage was given in the past. . b . Ask another physician to determine the correct dosage. c . Tell the patient that the medication will not be given. d . Contact the health care provider to verify the correct dosage. ANS: D Any questionable orders should be verified by the health care provider who wrote the orders. The patient is not a reliable source of verification. The physician who wrote the order should verify it. It would be a medication error to withhold the dose instead of verifying it. DIF: Cognitive Level: Application REF: p. 99 OBJ: 5 TOP: Nursing Process Step: Planning MSC: NCLEX Client Needs Category: Physiological Integrity 4. What is the most reliable method to calculate a pediatric patients medication dosage? a Age . b Height . c Body surface area (BSA) . d Placement on a growth scale . ANS: C The most reliable method is by proportional amount of BSA or body weight. Because of the differences in weight among children, age is not a reliable method. Because of the differences in height among children, this is not a reliable method. Placement on a growth scale identifies how the child corresponds to other children on a percentile. Although it is determined by a specific measurement, the percentile identified would not be a specific measurement; therefore, this is not a reliable method. DIF: Cognitive Level: Comprehension REF: p. 101 OBJ: 10 TOP: Nursing Process Step: Assessment MSC: NCLEX Client Needs Category: Physiological Integrity 5. Which medication route provides the most rapid onset of a medication, but also poses the greatest risk of adverse effects? a Intradermal . b . Subcutaneous (subcut) c . Intramuscular (IM) d . Intravenous (IV) ANS: D IV medications are delivered directly into the bloodstream and avoid the first pass effect of the liver. Intradermal, subcut, and IM administration have a slower absorption rate. DIF: Cognitive Level: Knowledge REF: p. 102 OBJ: 10 TOP: Nursing Process Step: Assessment MSC: NCLEX Client Needs Category: Physiological Integrity 6. Which is known as the fifth vital sign? a Temperature . b Respirations . c Pain . d Pulse . ANS: C Pain is known as the fifth vital sign. DIF: Cognitive Level: Knowledge REF: p. 86 OBJ: 2 TOP: Nursing Process Step: Assessment MSC: NCLEX Client Needs Category: Physiological Integrity 7. Which is true regarding the unit dose drug distribution system? a The inventory is delivered to each nursing unit on a regular and recurring basis. . b The system delivers one dose of each medication to be administered until the . subsequent delivery of inventory. c The use of single dose packages of drugs dispensed to fill each dose . requirement as it is ordered. d The amount of inventory needed to dose all patients on the unit for a 24 hour . interval. ANS: C The unit dose drug distribution system uses single unit packages of drugs dispensed to fill each dose requirement as it is ordered. DIF: Cognitive Level: Comprehension REF: p. 93 OBJ: 7 TOP: Nursing Process Step: Implementation MSC: NCLEX Client Needs Category: Physiological Integrity 8. The nursing assessment identifies that the client is nauseated and cannot take acetaminophen (Tylenol) orally. Which is true regarding the substitution of this medication to suppository form? a It is standard practice when the patient is unable to take the ordered medication. . b It is acceptable if the patient agrees to the altered route form. . c It is preferable to having the patient miss a dose of the medication. . d It is contraindicated without an order from the health care provider. . ANS: D One dosage form of medication should never be substituted for another unless the prescriber is consulted; there can be a great variation in the absorption rate of the medication through different routes of administration. The substitution of one form for another is not standard practice, and is not acceptable or preferable without the prescribers order. DIF: Cognitive Level: Application REF: p. 99 OBJ: N/A TOP: Nursing Process Step: Implementation MSC: NCLEX Client Needs Category: Physiological Integrity 9. Which medication order requires nursing judgment and means administer if needed? a Morphine 4 mg IV stat . b Morphine 4 mg IV prior to procedure . c Morphine 4 mg IV four times a day . d Morphine 4 mg IV every 4 hours PRN . ANS: D PRN indicates for the nurse to administer morphine every 4 hours if needed and requires nursing judgment. Stat means the dose of morphine would be given immediately, not as needed. The orders for the dose of morphine to be given prior to the patients scheduled procedure and four times a day, do not indicate to give the dose as needed. DIF: Cognitive Level: Comprehension REF: p. 89 | p. 97 OBJ: 2 TOP: Nursing Process Step: Implementation MSC: NCLEX Client Needs Category: Physiological Integrity 10. What is medication reconciliation? a Comparing the patients current medication orders to all of the medications . actually being taken b The administration of high alert medications that have been ordered on . admission to an acute care facility c The completion of an incident report following a variance that resulted in a . serious complication d A printout of computerized patient data that identifies the times that all of the . ordered medications are to be administered ANS: A Medication reconciliation is the process of comparing a patients current medication orders to all of the medications that the patient is actually taking. Administering high alert medications and completing an incident report are not the same as medication reconciliation. A printout of computerized patient data that identifies the times that all of the ordered medications are to be administered is a description of the medication administration record (MAR), not a description of medication reconciliation. DIF: Cognitive Level: Knowledge REF: p. 98 OBJ: 4 | 9 | 10 TOP: Nursing Process Step: Assessment MSC: NCLEX Client Needs Category: Physiological Integrity 11. Which example best demonstrates safe drug administration by the nurse? a Administering an oral medication with the patient sitting upright . b Asking children to say their name before administering the medication . c Leaving the medications on the bedside stand after verifying patient . identification d Returning the unused portion of a medication to a stock supply bottle . ANS: A Sitting the patient upright for oral medications is safe medication practice. Children should never be asked their names as a means of positive identification. Remaining with a patient until the drug is swallowed is safe practice. Returning an unused portion of medication to the stock supply bottle is not safe medication practice. DIF: Cognitive Level: Application REF: p. 103 OBJ: 10 TOP: Nursing Process Step: Implementation MSC: NCLEX Client Needs Category: Physiological Integrity 12. The nurse determines that a prescribed medication has not been administered as ordered on the previous shift. What action will the nurse take? a Administer the medication immediately. . b Complete an incident report. . c Notify the nurse responsible for the error. . d Record the occurrence in the nurses notes. . ANS: B An incident report is completed when a medication error occurs. Depending on the medication and frequency of administration, the medication may not be given immediately. It is not the nurses responsibility to notify another nurse of the error. Medication errors are not recorded in the nurses notes. DIF: Cognitive Level: Application REF: p. 100 OBJ: 6 | 11 TOP: Nursing Process Step: Implementation MSC: NCLEX Client Needs Category: Physiological Integrity 13. A patients liquid cough medicine has been discontinued with one half of the bottle remaining. The home health nurse is aware that according to the U.S. Food and Drug Administration (FDA) guidelines on prescription medication disposal, the next step should be to: a save the remainder for another patient with the same prescription. . b flush the remainder down the toilet. . c read the drug label for specific disposal instructions. . d pour remaining medication into a hazardous waste container. . ANS: C The nurse must follow specific disposal instructions on the drug label or in the patient information leaflet that accompanies the medication. Prescription medications should not be shared among patients. Prescription drugs should not be flushed down the toilet unless specifically instructed to do so by the manufacturer. The first action to be taken is to follow disposal instructions on the label. If the drug label indicates it should be emptied into a hazardous waste container, measures should be taken to prevent leaking and/or accidental ingestion. DIF: Cognitive Level: Analysis REF: p. 96 OBJ: 7 TOP: Nursing Process Step: Implementation MSC: NCLEX Client Needs Category: Safe, Effective Care Environment; Physiological Integrity MULTIPLE RESPONSE 14. Who defines the standards of care for the practice of nursing? (Select all that apply.) a State boards of nursing . b Hospital policy and procedures . c Federal laws regulating health care facilities . d The Joint Commission . e Professional nursing associations . ANS: A, C, D, E Standards of care are defined by state boards of nursing, federal laws regulating health care facilities, The Joint Commission, and professional nursing associations such as the American Nurses Association. Individual hospital policies and procedures incorporate federal and state guidelines into their respective policies and procedures and are often more stringent than state and federal regulations. DIF: Cognitive Level: Knowledge REF: p. 79 | p. 91 | p. 92 OBJ: 1 TOP: Nursing Process Step: Assessment MSC: NCLEX Client Needs Category: Safe, Effective Care Environment 15. What must the nurse have before administering any medication? (Select all that apply.) a . A current license to practice b . A medication order signed by a practitioner licensed with prescription privileges c . Knowledge of the medication d Consultation with a pharmacist . e Knowledge of the clients diagnosis . ANS: A, B, C, E Physicians must be licensed to prescribe medications; nurses must be licensed to administer medications. Safe medication administration includes knowledge of the medication, pathophysiology of patient diagnoses, and pharmacodynamics of the ordered medication on the pathophysiology. It is not necessary for the nurse to consult with a pharmacist each time medication is to be administered to a patient. DIF: Cognitive Level: Comprehension REF: p. 80 OBJ: 1 TOP: Nursing Process Step: Implementation MSC: NCLEX Client Needs Category: Safe, Effective Care Environment 16. Which advantage(s) does the unit dose drug distribution system include? (Select all that apply.) a . There is decreased participation by the pharmacy. b . The pharmacist is able to analyze prescribed medications for each client for drug interactions and contraindications. c . There is less waste of medications. d . The time spent by nursing personnel preparing these medications is increased. e . Credit is given to the patient for unused medications. ANS: B, C, E Because the pharmacist has a profile of all medications for each patient, he or she is able to analyze prescribed medications for each patient for drug interactions and contraindications. This is an advantage of the unit dose drug distribution system. Less waste of medications is an advantage of the unit dose drug distribution system. Because each dose is individually packaged, credit can be given to the patient for unused medications. There is increased pharmacist involvement and better use of his or her extensive drug knowledge and nursing personnel time is decreased with this method. DIF: Cognitive Level: Knowledge REF: p. 93 OBJ: 7 TOP: Nursing Process Step: Implementation MSC: NCLEX Client Needs Category: Safe, Effective Care Environment 17. Which statement(s) is/are true regarding the types of medication orders? (Select all that apply.) a . Stat orders are the same as single dose orders. b . Standing orders indicate the number of specified doses of a medication to be given. c . Renewal orders facilitate physician review before continuance of high risk medications. d . PRN medications will designate a mandatory number of times the medication is to be administered. e . Verbal orders should be used as much as possible. ANS: B, C Standing orders state the frequency of medication dosages to be administered or indicate the time frame of administration. Renewal orders require the physician to review medications that have expired orders, as determined by facility policy. Renewal policies facilitate physician verification of the necessity to continue a medication beyond a usual time frame and help ensure patient safety. Single dose and stat orders are not the same. PRN medications are not ordered a mandatory number of times, although a maximum number might be specified. Verbal orders should be avoided whenever possible. DIF: Cognitive Level: Comprehension REF: p. 97 OBJ: 8 TOP: Nursing Process Step: Planning MSC: NCLEX Client Needs Category: Physiological Integrity 18. Which statement(s) is/are true regarding computerized prescriber order entry (CPOE)? (Select all that apply.) a . Integrates the ordering system with the pharmacy, laboratory, and nurses stations b . Provides instant access to online information to facilitate patient care needs c . Facilitates review of ordered medications for potential drug interactions d . Facilitates review of drugs for appropriateness of dosages e . Alleviates the need to perform mathematical computations ANS: A, B, C, D CPOE systems integrate patient information, provide instant access, facilitate review of ordered medications for potential drug interactions, and facilitate review of drugs for appropriateness of dosages. Alleviation of the need to perform mathematical computations is not a component of the CPOE system. DIF: Cognitive Level: Knowledge REF: p. 92 OBJ: 8 TOP: Nursing Process Step: Assessment MSC: NCLEX Client Needs Category: Physiological Integrity; Safe, Effective Care Environment 19. Which lab test(s) would be used to assess liver and/or renal function before administering medications? (Select all that apply.) a CBC . b LDH . c ALT . d Crs . e BUN . f aPTT . ANS: B, C, D, E Liver function tests include LDH (lactic dehydrogenase) and ALT (alanine aminotransferase). Renal function tests include Crs (serum creatinine) and BUN (blood urea nitrogen). Although a CBC (complete blood count) and an aPTT are useful in assessing the patient before administration of medication, they are not renal or hepatic function tests. DIF: Cognitive Level: Knowledge REF: p. 101 OBJ: N/A TOP: Nursing Process Step: Assessment MSC: NCLEX Client Needs Category: Physiological Integrity 20. The nurse is preparing to administer Lanoxin to a patient on the telemetry unit. In addition to understanding the patients diagnosis, the nurse must also know which characteristic(s) of the medication? (Select all that apply.) a . Chemical composition b . Adverse effects c . Expected actions d . Contraindications for use e . Usual dosing ANS: B, C, D, E The nurse must understand the individual patients diagnosis and symptoms that correlate with the rationale for drug use. The nurse should also know why a medication is ordered, expected actions, usual dosing, proper dilution, route and rate of administration, adverse effects, and contraindications for the use of a particular drug. It is not required that the nurse know the chemical composition of the medication prior to administration. DIF: Cognitive Level: Application REF: p. 80 OBJ: 5 TOP: Nursing Process Step: Implementation MSC: NCLEX Client Needs Category: Safe, Effective Care Environment; Physiological Integrity 21. The nurse transcribes an order to administer Valium 10 mg IV stat. This order is correctly interpreted by the nurse to mean it should be provided how? (Select all that apply.) a . As needed b . Immediately c . One time only d . In divided doses e . Intravenously ANS: B, C, E The stat order is generally used on an emergency basis. It means that the drug is to be administered as soon as possible, but only once. IV indicates the route is intravenous. A PRN order means administer if needed. The order would specify divided doses and amount per dose if indicated. Chapter 4. Regulations 1. A nurse is preparing a drug for administration to a patient. The drug does not have an indicated use for the patients medical diagnosis. What should the nurse do? A) Administer the drug as ordered. B) Question the prescriber concerning the ordered drug. C) Ask a coworker his or her thoughts about the ordered drug for the patient. D) Ask the patient why the drug has been prescribed for him or her. Ans: B Feedback: If the nurse is not sure about giving a drug, the order should be questioned. The nurse should never give a medication that is not clear. Mistakes do happen and the drug ordered, if not approved for the condition that the patient has, could be an error on someones part. The person who wrote the order should be questioned, not a co worker, who probably does not know why an off-label drug is being used. It would be unprofessional and inappropriate to ask the patient about the drug. 2. According to Center for Disease Control and Prevention (CDC) recommendations, what is the role of the nurse in preparing for the possibility of bioterrorism? A) Post updated information on signs and symptoms of infections caused by biological agents B) Provide guidelines for treating patients exposed to, or potentially exposed to, biological agents C) Remain current on recognition and treatment of infections caused by biological weapons D) Advocate for increased funding for research involving bioterrorism and patient treatment Ans: C Feedback: Nurses need to remain current about recognition of and treatment for those exposed to biological weapons because nurses are often called upon to answer questions, reassure the public, offer educational programs, and serve on emergency preparedness committees. The CDC posts updated information on signs and symptoms of infections caused by biological agents that nurses would read. The CDC also provides guidelines for how to treat patients exposed to biological agents and the nurse must remain current on this information. Although nurses could advocate for funding, this is not usually the role of the nurse. 3. How can the nurse find the most up-to-date information about emergency preparedness related to bioterrorism agents? A) Read textbooks devoted to the topic. B) Ask coworkers to explain current events. C) Read journal articles about bioterrorism agents. D) Visit the Centers for Disease Control and Prevention (CDC) Web site. Ans: D Feedback: The most current information will be found on the CDC Web site because new information can be posted immediately whereas textbooks and journal articles take time to print. Coworkers may or may not remain current on emergency preparedness and should not be the primary source of information. 4. The nurse is assessing a diabetic patient who has presented at the clinic reporting several hypoglycemic episodes during the past 3 weeks. The nurse questions the patient about the use of herbal or alternative therapies, suspecting what herbal remedy could cause the hypoglycemic episodes? A) St. Johns wort B) Kava C) Fish oil D) Ginseng Ans: D Feedback: Ginseng is known to decrease blood sugar levels. If the patient used this in combination with his or her oral antidiabetic agent, diet, and exercise, his or her blood sugar could drop below therapeutic levels. St. Johns wort interacts with many drugs, but not with antidiabetic agents. Kava is associated with liver toxicity. Fish oil has been associated with decreased coronary artery disease. 5. A 22-year-old patient calls the clinic and tells the nurse that she has been depressed and is thinking about taking St. Johns wort but wants to know if it is safe first. The nurse begins by questioning what other medications the patient takes and would be concerned about a drug-alternative drug interaction if the patient is also taking what type of medication? A) Antihistamines B) Analgesics C) Antibiotics D) Oral contraceptives Ans: D Feedback: St. Johns wort can interact with oral contraceptives that alter drug metabolism, which can decrease the effectiveness of the contraceptive. Analgesics, antibiotics, and antihistamines can be taken in combination with St. Johns wort without known adverse effects. 6. A patient tells the clinic nurse that he or she has been taking over-the- counter (OTC) Pepcid to relieve acid indigestion for several years. This is the first time the patient has ever reported this issue to a health care provider. As part of the teaching plan for this patient, the nurse explains what risk associated with not sharing OTC drug use with the provider? A) The OTC drug could be more expensive than seeking health care advice. B) The drug could mask symptoms of a serious problem that is undiagnosed. C) Use of the drug could cause a rebound effect of Pepcid. D) The drug could interact with several cold medicines. Ans: D Feedback: OTC drugs allow patients to self-diagnose and treat routine signs and symptoms without seeing a health care provider. This self-prescribed treatment, however, could mask a more serious underlying medical problem and result in a poor outcome for the patient. The issues of drug rebound and drug interaction need to be considered, but the safety issue related to self- diagnosis and self-prescription presents the greatest risk to the patient. Patients should always be encouraged to discuss the use of OTC products with their health care provider. 7. What patient populations would the nurse expect is most likely to be prescribed a drug for an off-label use? A) Adolescent and middle-aged adult patients B) Patients with diabetes or heart disease C) Obstetric and neonatal patients D) Pediatric and geriatric patients Ans: D Feedback: Drugs being used for an off-label purpose are commonly prescribed for pediatric and geriatric populations due to the lack of drug trial information and minimal premarket testing. Often a trial-and-error method is used in treating both the pediatric and geriatric populations when only adult information is known. The geriatric population responds to medication more like children because of their decreased ability to metabolize medications. Adolescents, especially later adolescents, use medications similarly to young adults as do middle-aged adults. Patients with different diagnoses are often involved in drug testing including those with diabetes and heart disease. Drugs are discouraged for use in obstetric patients. 8. A patient calls the clinic and asks to speak to a nurse. The patient questions the nurse about the use of a drug that was advertised on TV. The patient tells the nurse he or she is sure that the drug will make him or her feel the same way as described in the commercial. What response is most appropriate for the nurse to make? A) Im glad that you want to be involved in treatment decisions but you are not qualified to decide what medications are best for your condition. B) Its important to remember that drug advertisements emphasize the positive effects of drug therapy and not the adverse effects or contraindications. C) You need to remember that the drugs being advertised are much more expensive than other drugs that have the same effect. D) Ive seen those advertisements and I would want to take that medication too if I had the condition it was designed to treat. Ans: B Feedback: It would be important for the nurse to remind the patient that advertisements always emphasize the positive effects of drug therapy. The patient should not be discouraged from contributing to the plan of care by being told she is not adequately qualified to make decisions because no one is more qualified to make decisions about her own body. Although the drug may be more expensive, this is not a reason to choose or avoid a medication that could be more effective. Agreeing with the patient is not meeting the nurses obligation to teach and inform. 9. The clinic nurse is talking with a patient about information concerning a drug her or she bought online. What is the nurses responsibility to the patient concerning this information? A) Encourage the patient to seek information about drugs from a pharmacist. B) Explain that information obtained from the Internet is not always accurate. C) Offer the patient a drug reference guide to read and learn more about the drug. D) Interpret the information and explain it in terms that the patient will understand. Ans: D Feedback: The Internet can be a good reference for drug information. However, the amount and reliability of the information can be overwhelming. The nurse should always try to interpret the information and explain it in terms that the patient will understand. A pharmacist is a good resource person but may not be able to teach from a holistic perspective. Drug reference guides may be hard for the patient to understand and he or she would still need someone to interpret the information. 10. The triage nurse in the emergency department sees a patient suspected of abusing amphetamines brought in by friends. While assessing this patient, what would the nurse be likely to find if steroids are being abused? A) Hypertension B) Bradycardia C) Drowsiness D) Elated mood Ans: A Feedback: Increases in blood pressure, tachycardia, and insomnia are symptoms of amphetamine abuse. Elation can indicate abuse of cannabis. 11. The nursing instructor is discussing the off-label use of drugs. What group of drugs would the instructor tell the nursing students is often used for off- label indications? A) Drugs used to treat psychiatric problems B) Drugs used to treat gastrointestinal (GI) problems C) Drugs used to treat cardiovascular problems D) Drugs used to treat musculoskeletal problems Ans: A Feedback: Drugs often used for off-label indications include the drugs used to treat various psychiatric problems. Drugs used to treat GI, cardiovascular, or musculoskeletal problems do not fall in the category of frequent off-label uses. 12. The patient calls the clinic nurse and says, I looked this medication up on the Internet after it was prescribed yesterday and there is nothing in the literature about this drug being used to treat my disorder. Should I still take it? What is the nurses best response? A) No, stop taking it immediately until I can consult with the doctor because it is obvious a mistake was made. B) Oh, thats okay. Go ahead and take it because the doctor wouldnt order it if he or she didnt think it would be effective. C) It is quite common for drugs to be found to have positive effects for a condition not originally intended so it is safe to take. D) Let me talk with the physician about why this medication was ordered for you and I will call you back. Ans: D Feedback: Off-label use is relatively common because new information is gathered when the drug is used by large numbers of people that may indicate another condition for which the drug is effective. However, if the nurse does not know for a fact that the drug prescribed is the right drug for the patients condition, it is always best to consult with the prescriber to make sure the patient is taking the right drug and to avoid a medication error. The medication may be perfectly safe so the patient should not be told the doctor made a mistake. 13. When a drug is ordered off-label, what must the nurse be clear about before administering the drug? (Select all that apply.) A) Why the drug is being given B) Its potential for problems C) The research that has been done D) The age group it was pretested on E) The intended use Ans: A, B, E Feedback: Liability issues surrounding many of these uses are very unclear, and the nurse should be clear about the intended use, why the drug is being given, and its potential for problems. Knowing the age group it was pretested on and knowing the research that has been done are not factors the nurse needs to know before administering the drug. 14. It is important for the nurse to be aware of what related to the way drugs are marketed? A) The adverse effects the advertisements do not mention B) What magazines and Web sites contain the advertisement C) What patients are seeing in the advertisements about these drugs D) The name of the cheerful, happy models who are advertising these drugs Ans: C Feedback: As the marketing power for prescription drugs continues to grow, the nurse must be constantly aware of what patients are seeing, what the ads are claiming, and the real data behind the indications and contraindications for these hot drugs. The Food and Drug Administration regulates the information that needs to be contained within medication ads. Where the patient saw the ad and the actors in the ads are unimportant. 15. When evaluating information accessed over the Internet, an important question the nurse should teach the patient to ask is what? A) Is the information anecdotal? B) Where has this information been obtained? C) Is this information paid for by the drug company? D) How many patients have had input into the information? Ans: A Feedback: Many people do not know how to evaluate the drug-related information that they can access over the Internet. Is it accurate or anecdotal is an important concept for the nurse to teach the patient to assess to verify the accuracy of the information. Where the information came from is unimportant. It would be expected that all drug advertising is paid for by the drug company and this is not an important concern. Number of patients with input into the information is most likely none because information is gathered from health care professionals. 16. How has the patients access to drug information changed the way the patient interacts with the nurse and other health care providers? A) Patients share information from research reports with health care providers. B) Patients are contacting drug companies to see what their latest reports say. C) Patients are more likely to challenge the health care provider with their own research. D) Patients are more likely to self-prescribe and not obtain prescriptions from their health care provider. Ans: C Feedback: Access to consumer advertising, mass media health reports, and the Internet influence some patients to request specific treatments, to question therapy, and to challenge the health care provider. Consumers do not generally read research reports from medical facilities and contact drug companies to see what their reports say, and they cannot self-medicate because many of these drugs require a prescription to obtain them. 17. What can make a nurse or any health care provider lose credibility with the patient? A) Being unprepared to deal with the disease of the week B) Refusing to write prescriptions for the drug the patient requests C) Not being knowledgeable about diseases described on House D) Being prepared to discuss the role of concierge doctor Ans: A Feedback: Some health care providers have learned to deal with the disease of the week as seen on talk shows; others can be unprepared to deal with what was presented and may lose credibility with the patient. 18. Today, an abundance of information is available in the health care arena for consumers, resulting in the nurse encountering patients who have a much greater use of what? A) Over-the-counter (OTC) therapies B) Alternative therapies C) Prescription drugs D) Off-label drugs Ans: B Feedback: The patient now comes into the health care system burdened with the influence of advertising, the Internet, and a growing alternative therapy industry. Many patients no longer calmly accept whatever medication is selected for them. Indeed, an increasing number of patients are turning to alternative therapies with the belief that they will treat their disorder and reduce risk of adverse effects. Although more prescription drugs are used today, that is not related to abundant information. No indication exists of an increase in use of OTC or off-label drugs. 19. Because of the amount of care now being done in the home care setting, it is imperative that the nurse teach the patients what? (Select all that apply.) A) Care givers educational level. B) Generic names of medication C) Over-the-counter (OTC) drugs that need to be avoided D) Alleviation of adverse effects E) How to calculate safe dosages Ans: B, C, D Feedback: The responsibility of meeting the tremendous increase in teaching needs of patients frequently resides with the nurse. Patients need to know exactly what medications they are taking (generic and brand names), the dose of each medication, and what each is supposed to do. Patients also need to know what they can do to alleviate some of the adverse effects that are expected with each drug (e.g., small meals if gastrointestinal upset is common, use of a humidifier if secretions will be dried and make breathing difficult), which OTC drugs or alternative therapies they need to avoid while taking their prescribed drugs, and what to watch for that would indicate a need to call the health care provider. 20. What concerns might the nurse legitimately have related to the use of alternative therapies? (Select all that apply.) A) The Food and Drug Administration (FDA) does not test or regulate active ingredients. B) The incidental ingredients are clearly marked on the label. C) The dosage contained in each tablet may vary greatly. D) No alternative therapies have been found to be effective. E) Advertising of alternative products is not as restrictive or accurate. Ans: A, C, E Feedback: Alternative products are not controlled or tested by the FDA and advertising is not as restrictive or accurate as with classic drugs. Incidental ingredients are often unknown and strength of tablets may vary within the bottle depending on the conditions under which they were grown. While some alternative therapies have been found to be effective, there are others who have not been studied. 21. The nurse provides teaching to the patient using herbal therapies and includes what important information related to the effects of the herbal therapy? A) They can interact with prescription drugs. B) They always contain known ingredients. C) They are natural so they are effective and safe. D) The ingredients are natural, meaning toxicity is not a concern. Ans: A Feedback: Herbal therapies can produce unexpected adverse effects and toxic reactions, can interact with prescription drugs, and can contain various unknown ingredients that alter the therapies effectiveness and toxicity. 22. When patients do not understand the information provided with their medication, whose responsibility is it to help them sort through and comprehend the meaning? A) Care giver B) Nurse C) Patient D) Physician Ans: B Feedback: Many pharmacies provide written information with each drug that is dispensed, but trying to organize these sheets of information into a usable and understandable form is difficult for many patients. The nurse is often the one who needs to sort through the provided information to organize, simplify, and make sense of it for the patient. 23. The nurse is providing an inservice on alternative therapies for peers and explains that the term alternative therapies includes what? A) Holistic drug therapy B) Hospice care C) Nondrug measures D) Home care Ans: C Feedback: Herbal medicines and alternative therapies are found in ancient records and have often been the basis for discovery of an active ingredient that is later developed into a regulated medication. Today, alternative therapies can also include non-drug measures, such as imaging and relaxation. Options A, B, and D are not included in alternative therapies. 24. The patient calls the clinic and talks to the nurse saying, I found the same drug the provider prescribed on the Internet and it is much cheaper. Is it safe for me to order my drug from this site? What is the nurses best response? A) It is usually safe to order drugs from Internet Web sites if it is a reliable site. B) Most drugs ordered online come from another country and are safely used there. C) The drug you get will be the same chemical prescribed but the dosage may differ. D) The Food and Drug Administration (FDA) has issued warnings to consumers about the risk of taking unregulated drugs. Ans: D Feedback: The FDA has begun checking these drugs when they arrive in this country and have found many discrepancies between what was ordered and what is in the product, as well as problems in the storage of these products. Some foreign brand names are the same as brand names in this country but are associated with different generic drugs. The FDA has issued many warnings to consumers about the risk of taking some of these drugs without medical supervision, reminding consumers that they are not protected by U.S. laws or regulations when they purchase drugs from other countries. 25. With the need to protect our environment, what is it now important for the nurse to teach patients to do? A) Dispose of drugs no longer used on an annual basis. B) Flush drugs down the toilet. C) Bury unused in the yard. D) Throw unused pill bottles in the trash in original containers. Ans: A Feedback: Patients should go through their medicine cabinet annually and dispose of drugs no longer used. Unused drugs should not be flushed down the toilet or buried in the yard because they seep into the community water supply. Pills should be removed from their bottle and mixed with an undesirable substance to prevent someone from using the medication if found. 26. The nurse receives a call from a frantic mother saying, My child swallowed some of my birth control pills. Should I give Ipecac? What is the nurses best response? A) Yes, give Ipecac and follow the dosage directions on the bottle. B) Ipecac is not effective for this use so you should not give it to your child. C) Give the Ipecac only if you are absolutely sure your child swallowed the pills. D) No, dont give Ipecac because it will cause your child to vomit and make a mess. Ans: B Feedback: Ipecac is a drug that the Food and Drug Administration tested in 2003 and found, despite its use for many years, that it was not effective in inducing vomiting in children suspected of poisoning. As a result, it is no longer used. The mother should be instructed not to give it and to call poison control to get up-to-date instructions on how to deal with this emergency. Whether the pills were swallowed, this child requires appropriate intervention because it is better to err on the side of caution. Making a mess is not a concern. 27. The patient tells the nurse that he or she has begun ordering his or her medications over the Internet because it is cheaper. What statement made by the nurse in response to this information is accurate? A) All drugs are manufactured with the same quality controls. B) Any drug that is shipped into this country is safe to use. C) Foreign drugs may have the same name as domestic drugs, but they are not the same drug. D) If you order from Canada or Mexico, the drugs are safe because they undergo testing. Ans: C Feedback: The Food and Drug Administration has begun checking these drugs when they arrive in this country and have found many discrepancies between what was ordered and what is in the product, as well as problems in the storage of these products. Some foreign brand names are the same as brand names in this country but are associated with different generic drugs. Options A, B, and D are incorrect because not all drugs are manufactured the same and they are not always safe coming from another country. 28. The increasing number of patients who go to their health care provider and request a drug they have seen advertised on television or in a magazine has created what continuing challenge to health care providers? A) Treating infections appropriately B) Treating sicker patients C) Prescribing cost-effectively D) Staying knowledgeable about drug therapy Ans: D Feedback: As the marketing power for prescription drugs continues to grow, the health care provider must be constantly aware of what patients are seeing (or reading), what the commercials and ads are promising, and the real data behind the indications and contraindications for these hot drugs. It is a continuing challenge to stay up-to-date and knowledgeable about drug therapy. 29. Ipecac, formerly used as the drug of choice by parents for treatment of suspected poisoning in children, was tested by the Food and Drug Administration (FDA) in 2003. What was the finding of this testing? A) Ipecac is ineffective for its intended use. B) Ipecac is the safest treatment for poisoning in children. C) Ipecac was grandfathered in as an ineffective drug. D) Ipecac induces vomiting. Ans: A Feedback: Ipecac, a formerly standard over-the-counter drug, was used for many years by parents to induce vomiting in children in cases of suspected poisoning or suspected drug overdose. The drug was finally tested and in 2003, the FDA announced that it was not found to be effective for its intended use. Although it was grandfathered in as an effective drug, this was not what the study researched. Ipecac is not effective and does not consistently induce vomiting. 30. Federal guidelines state that when advertising a drug, if the company states what the drug is used for, what other information must also be included in the advertisement? (Select all that apply.) A) Symptoms B) Contraindications C) Adverse effects D) Precautions E) Cost Ans: B, C, D Feedback: If a drug advertisement states what the drug is used for, it must also state contraindications, adverse effects, and precautions. The advertisement does not have to state symptoms or cost. 31. The parent of a 2-year-old child is visiting his or her pediatric health care provider and shows the nurse the advertisement for allergy medication found in a magazine in the waiting room saying, This drug sounds like it would be far more effective to treat my sons asthma and Id only have to give it once a day. What is the nurses best response? A) Talk with your health care provider about this drug, but be aware that advertisements do not always provide all the important information you need to know. B) Oh, I need to throw that magazine away because so many people show me that ad and it is all complete nonsense with no truth to it at all. C) Ive been seeing amazingly positive results from that medication so you are absolutely right to want to give it to your child. D) That drug is dangerous and should not be given to children under the age of 5 unless there are no other good options. Ans: A Feedback: The health care provider should make the decision about what medications are to be prescribed, not the nurse. However, the nurse can make the mother aware of the fact that there is often more that goes into choosing the correct drug than the bit of information disclosed in the advertisement. Becoming upset with the mother, agreeing with the mother, or frightening the mother about the medication is the wrong approach for the nurse to use. 32. The local news has been discussing a specific rare disorder that killed a child in the community this week, describing the symptoms of the disease as including nasal congestion, ear pain, and a cough. The pediatricians office is receiving numerous calls asking to make appointments to rule out this rare disease. What is the nurses best action? A) Prepare a handout that describes the disorder discussed in the news in greater detail. B) Tell parents their child is experiencing the common cold and do not need to be seen. C) Direct all calls to the local news agency to answer questions and provide details. D) Become familiar with the disorder and screen each call for more specific symptoms. Ans: A Feedback: The nurse needs to not only become more familiar with the disorder in the news, but also needs to be prepared to teach parents about the disease of the week to allay fears so a handout with detailed information would allow the parents to have something to consult after leaving the providers office. Turning parents away without seeing their child will increase fears and the office will lose credibility for lack of interest in their childs well-being. Directing calls to the news agencies will not provide parents with essential information. Screening calls without seeing the child could be potentially very dangerous. 33. The nurse is teaching the patient how to safely use the Internet for health information and includes what information in the teaching plan? A) The Web site where information is obtained needs to be evaluated for credibility. B) Most information found on the Internet is accurate. C) Information on the Internet is most reliable when people give their reviews of the drug. D) Only a health care professional can tell whether a Web site is reliable. Ans: A Feedback: There are excellent sites for reliable drug information, but each site must be evaluated for credibility and the nurse can teach the patient things to look for to increase confidence in the site. However, a lot of information on the Internet is not accurate; the patient needs to learn how to recognize unreliable information when he or she comes across these sites. Just because a person reviews a drug and gives it multiple stars or a thumbs up does not mean the drug is any more effective or useful in the patients care. 34. The nursing instructor is teaching the class about how prescription drugs become over-the-counter (OTC) drugs and lists what factor as preventing a drug from becoming classified as OTC? A) If the patient cannot reliably self-diagnose the condition the drug is intended to treat B) If it would mask signs and symptoms of an underlying problem, the drug remains available by prescription only. C) If the drug would cause toxic effects if not taken as directed, it remains a prescription drug. D) OTC drugs must not have any adverse effects that could harm the patient. Ans: A Feedback: If a diagnosis requires medical intervention, such as hyperlipidemia, which can only be diagnosed through laboratory studies, there is no point in making the drug an OTC medication. Most, if not all, OTC drugs have the capacity to mask signs and symptoms of an underlying disease so this is not a factor in deciding if a drug can be sold OTC. All drugs have the potential for toxic effects if not taken as directed and virtually all drugs have the potential for adverse effects. 35. The nurse needs to ask what specific questions when collecting a drug history? (Select all that apply.) A) Do you take any over-the-counter medications? B) Do you take any herbal supplements? C) Do you use any alternative therapies? D) Do you take any natural supplements or vitamins? E) What unusual therapies do you take? Ans: A, B, D Feedback: The nurse needs to specifically question the patients use of over-the-counter drugs, herbal supplements, natural supplements, and vitamins. Use of terms like alternative therapies or unusual therapies is too vague and may not elicit the kind of information needed. Chapter 5.- Chapter 8 Prescriptions and LabelsChapter 6. Basic Review of MathematicsChapter 7. Measurement SystemsChapter 8. Dosage Calculations 1. The nurse is calculating a drug dosage and converting from milligrams to grams. What measurement system is the nurse using? A) Metric system B) Apothecary system C) Household system D) Avoirdupois system Ans: A Feedback: The metric system is the most widely used system of measurement in the world; it is based on the decimal system. The gram is the basic unit of solid measure, and the liter unit of liquid measure. The apothecary system uses the grain as the basic unit of solid measure. The household system uses the pound as the basic unit of measure. The avoirdupois system uses ounces and grains, but it is mostly used by drug manufacturers for bulk medications. 2. The nurse teaches a young mother the importance of administering appropriate dosages of acetaminophen (Tylenol) and determines further teaching is needed when the mother makes what statement? A) The childrens dosage will change with time as they grow. B) My babys dose of Tylenol is about 1 half an adult dose. C) It is important to give the right dose to prevent toxic effects of the medication. D) My childrens dose of Tylenol should be based on their weight or age. Ans: B Feedback: A childs dose is never based on an adults dose. A childs dosage is based on weight and age and will change with age as they grow. Larger than directed dosages can result in toxic effects of this medication. 3. A nurse calculates the pediatric patients medication dosage using Clarks rule and uses what formula? A) Infants age in months/150 months times the average adult dose B) Childs age in years/childs age in years plus 12 times the average adult dose C) Weight of child in pounds/150 pounds times the average adult dose D) Surface area in square meters/1.73 times the average adult dose Ans: C Feedback: Clarks rule uses the childs weight to calculate the dose and assumes the adult dose is based on a 150-pound person. Frieds rule applies to a child younger than 1 year of age and assumes that an adult dose would be appropriate for a child who is 12.5 years (150 months) old. Youngs rule applies to children 1 to 12 years of age. Surface area calculation of a childs dose is determined with the use of a nomogram including the childs height and weight. 4. The nurse receives a new medication order for a patient to administer 240 mg of medication per day in equally divided doses every 6 hours. How many mg of the drug should the nurse administer for each dose? Ans: 60 mg Feedback: Because there are 24 hours in a day, giving a drug every 6 hours would mean giving the drug 4 times a day. Because the total daily dose is 240 mg, dividing that dose by 4 would mean each dose should be 60 mg. 5. A physician orders 500 mL of IV solution be administered over 8 hours. If the IV infusion set delivers 15 drops per mL, how many drops per minute should the nurse administer to the patient? A) 15 drops/min B) 20 drops/min C) 32 drops/min D) 64 drops/min Ans: A Feedback: If a patient was to receive 500 mL in 8 hours, dividing 500 by 8 would mean that the patient would receive 62.5 mL in 1 hour, or 60 min. Setting up the equation, 15 drops/mL/X equals 62.5 mL/60 min; cross-multiplying, the answer will be 15 drops/min. 6. The nurse is teaching a diabetic patient to self-administer Humulin insulin, supplied in a vial labeled 100 units/mL. The provider has ordered 32 units of Humulin insulin to be taken each morning. How many mL of insulin would the patient prepare for one dose? A) 0.032 mL B) 0.32 mL C) 3.2 mL D) 0.64 mL Ans: B Feedback: There are 100 units in each mL. Divide that amount by 32 units for the answer (0.32 mL). 7. The provider orders a maintenance dose of oral aminophylline, 3 mg/kg every 6 hour. The patient weighs 50 kg. How many mg should the nurse administer to the patient in a 24-hour period? Ans: 600 milligram Feedback: The patients weight times the number of milligram/kilogram will provide daily dosage of medication: 50 kg 3 mg/kg = 150 mg per dose. The patient is to receive a dose every 6 hours. The number of hours in a day divided by the number of hours separating each dose supplies the number of dosages the patient receives per day: 24 hours 6 hours between doses = 4 doses per day. If each dose is 150 mg and the patient receives 4 of these doses a day, the total amount of medication received is 150 mg 4 daily doses = 600 mg. 8. The physician writes an order for oxazepam for a 6-year-old child. The nurse verifies that there is no established dosage for children 6 to 12 years of age for oxazepam. The nurse knows that the usual adult dose is 10 mg tid. What would the nurse calculate the appropriate dose to be? A) 0.03 mg tid B) 0.3 mg tid C) 1.8 mg tid D) 3.3 mg tid Ans: D Feedback: Because the nurse knows only the childs age, the nurse would need to use Youngs rule to determine the appropriate dosage. The formula for Youngs rule is: Childs dose = childs age in years (childs age + 12) average adult dose. Using the information provided in the question: Dose = 6 (6 + 12) 10 mg = 6 18 10 = 0.33 10 = 3.3. 9. A newly admitted patient has orders to receive 1,000 mL of normal saline IV over 8 hours. If the IV infusion set is a microdrip set that delivers 60 drops per mL, how many drops per minute should the nurse administer to the patient? A) 60 drops/min B) 125 drops/min C) 240 drops/min D) 480 drops/min Ans: B Feedback: If a patient was to receive 1,000 mL in 8 hours, dividing 1000 by 8 would mean that the patient would receive 125 mL in 1 hour, or 60 minutes. Setting up the equation, 60 drops/mL X = 125 mL/60 minutes; cross-multiplying, the answer is 125 drops/min. 10. The nurse is preparing to administer cefadroxil 1 g PO. The medication is supplied in 500-mg tablets. How many tablets will the nurse administer? A) 0.5 tablet B) 1 tablet C) 2 tablets D) 3 tablets Ans: C Feedback: Convert 1 g to mg by multiplying 1 g times 1,000 mg. There are 500-mg in each tablet. Dividing the 1000 mg prescribed dosage by 500-mg available dosage, the answer is two tablets. 11. The nurse begins administering 500 mL of 5% dextrose and water solution at 01:00 to run over 4 hours. At 02:00, the nurse administers 80 mg gentamicin in 50 cc normal saline to infuse over 30 minutes. How many mL of fluid will the nurse administer to the patient between 02:00 and 03:00? A) 175 mL B) 150 mL C) 125 mL D) 100 mL Ans: A Feedback: The patient is receiving 500 mL over 4 hours. To determine how much fluid is infusing per hour = 500 mL 4 = 125. In addition to the 125 mL of IV solution, the patient also receives 50 mL of gentamicin during the 02:00 to 03:00 hour. 125 mL + 50 mL = 175 total mL of fluid received during this hour. 12. An adult patient with renal cancer, weighing 95 kg, is to receive vincristine 25 mcg/kg/day IV. What is the dosage of vincristine that the nurse should administer to the patient daily in mg? Ans: 2.375 mg Feedback: This order requires 25 mcg of medication for every 1 kg of body weight. The patient weighs 95 kg. To determine total dosage multiply weight times mcg of medication: 25 95 = 2,375 mcg. Convert mcg to mg by moving the decimal three places to the left, or you can divide 2,375 by 1,000 because there are 1,000 mcg per mg. 13. The nurse is preparing medication for a 30-month-old child with otitis media in the right ear. The child weighs 33 pounds. The physician has ordered Keflex, 50 mg/kg/d in equally divided doses every 8 hours. The medication concentration is 250 mg/5 mL. How many milliliters should the nurse give the toddler for each dose? Ans: 5 mL Feedback: To calculate the correct dosage, the nurse first converts the childs weight from pounds to kilograms by dividing weight in pounds by 2.2 (2.2 lb = 1 kg). 33 pounds 2.2 pounds/kg = 15 kg. The child is to receive 50 mg for every kilogram. To determine this childs dosage multiply weight times daily dose (15 kg 50 mg/kg = 750 mg). Thus, 750 mg is to be administered in equally divided dosages every 8 hours, or 3 times a day. 750 mg 3 = 250 mg/dose. There is 250 mg in 5 mL of medication so the patient would be given 5 mL. 14. An adolescent is admitted to the intensive care unit with diabetic ketoacidosis. The nurse prepares a continuous insulin infusion of 100 units (U) regular insulin in 500 milligram normal saline. When documenting this medication, how many units of regular insulin will this patient receive per milligram of IV solution? A) 0.175 U/milligram B) 0.2 U/milligram C) 0.25 U/milligram D) 0.5 U/milligram Ans: B Feedback: The problem tells us there is 100 U/500 milligram. To determine how many units are in each milligram, divide both numbers by 500: 100 U 500/500 milligram 500 = 0.2 U/1 milligram. 15. The patient drinks 18 ounces of fluid at lunchtime. How many milliliters of intake will the nurse document? A) 1.7 mL B) 0.6 mL C) 540 mL D) 54 mL Ans: C Feedback: 1 ounce = 30 mL. Using the ratio-and-proportion method: 1 oz/30 mL = 18 oz/X. Cross-multiply to determine the patient drank 540 mL of fluid. 16. The pediatric nurse is caring for a child who weighs 44 pounds. The physician has ordered methylprednisolone sodium succinate (Solu-Medrol), 0.03 mg/kg/d IV in normal saline. How many milligrams of medication will the nurse prepare? A) 6.5 B) 6 C) 0.65 D) 0.6 Ans: D Feedback: First convert the childs weight to kilograms by dividing 44 pounds by 2.2 kg/1 pound = 20 kg. Multiply the dosage times the childs weight: 20 kg 0.03 mg/kg/d = 0.6 mg/d 17. The nurse is to infuse 100 mL of 5% dextrose and water solution containing an IV antibiotic over 30 minutes. The infusion set delivers 10 gtt/mL. How many drops per minute will the nurse administer? A) 33 gtt/min B) 30.3 gtt/min C) 30 gtt/min D) 3 gtt/min Ans: A Feedback: Use the following ratio to determine how many drops of fluid to administer per minute: Using the information from this problem: Because it is not possible to deliver 0.3 drops, round 33.3 to 33 gtt/min. 18. An 80-year-old patient with internal bleeding is admitted through the emergency room after a motor vehicle accident. The physician has ordered 2 units of packed red blood cells (1 unit is 250 mL) to infuse over 1 hour each. The drip rate on the blood administration set is10 gtt/mL. The nurse administers how many drops per minute to infuse the blood as ordered? A) 47 B) 42 C) 37 D) 32 Ans: B Feedback: Use the following ratio to determine how many drops of fluid to administer per minute: Using the information from this problem: Because it is not possible to deliver 0.7 of a drop, round 41.7 to 42 minutes 19. The physician prescribes 250 mg of a drug. The information on the drug vial says the concentration is 500 mg/mL. How much of the drug will the nurse prepare? A) 0.25 mL B) 0.33 mL C) 0.5 mL D) 0.75 mL Ans: C Feedback: To determine amount to prepare: 500 mg/1 mL = 250 mg/X. Cross-multiply to determine the nurse will prepare 0.5 mL. 20. An 81-year-old patient with congestive heart failure has been sent to a cardiologist who prescribes digoxin (Lanoxin) 0.125 mg PO every morning. The pharmacy dispenses pills that contain 0.25 mg of Lanoxin. How many pills should the nurse teach the patient to take every morning? A) 2 B) 1.5 C) 1 D) 0.5 Ans: D Feedback: 0.25 mg/1 tablet = 0.125 mg dose/X. Cross-multiply to determine 0.5 or 1/2 of a tablet is to be taken daily. The nurse may request the pharmacy dispense a different concentration to prevent the patient from having to cut the tablet in half. 21. A patient with diabetic ketoacidosis is to receive a continuous infusion of regular insulin. The physician orders 1 L of 5% dextrose and water solution to run at 150 mL/h once the patients blood glucose has reached 250 mg/dL. The drip factor of the tubing is 15 gtt/mL. How many drops per minute will the nurse deliver? A) 0.25 gtt/min B) 62 gtt/min C) 37 gtt/min D) 250 gtt/min Ans: C Feedback: Use the following ratio to determine how many drops of fluid to administer per minute: Using the information from this problem: 37.5 can be rounded to 37 or 38 gtt/min. 22. A patient has orders to receive 2 L of IV fluid over a 24-hour period with this amount to be infused in the first 10 hours of treatment. How many milliliters per hour will the nurse administer during the first 10 hours of the infusion? A) 50 mL/h B) 100 mL/h C) 83 mL/h D) 200 mL/h Ans: B Feedback: of the 2 liters is to infuse in the first 10 hours. of 2 = 1 L; 1 L = 1,000 mL to infuse over 10 hours = 100 mL/L. 23. A patient is going to have bowel surgery in the morning. The physician orders 500 mL of GoLytely PO to be administered at 5 PM this evening. How many liters will the nurse administer? A) 1 B) C) D) Ans: C Feedback: Cross-multiply to learn 500 mL = or 0.5 L. 24. The patient returns from the postanesthesia care unit (PACU) with the following order: morphine 3 mg IV every 2 hours as needed for relief of pain. The vial reads morphine, 4 mg/mL. How many milliliters of morphine will the nurse administer? A) 1 mL B) 0.75 mL C) 0.5 mL D) 0.25 mL Ans: B Feedback: Using the figures from this problem: 4 mg/1 mL = 3 mg/X. Cross-multiply yielding 0.75 mL. 25. A patient is admitted with a deep vein thrombosis in his or her left calf. The physician orders Heparin, 7,500 units subcutaneously every 12 hours. The medication vial reads Heparin, 10,000 units/mL. How many milliliters does the nurse administer? A) 0.5 mL B) 0.75 mL C) 1 mL D) 1.25 mL Ans: A Feedback: Using the information supplied by the problem: 10,000 units/1 mL = 7,500/ X. Cross-multiplying yields X = 0.75 mL for each dose. 26. A patient with an acute myocardial infarction is admitted to the coronary care unit. The physician has ordered heparin 25,000 units in 250 mL normal saline to infuse at a rate of 600 units/h. The nurse sets the infusion pump to deliver how many milliliters in an hour? A) 8 B) 7 C) 6 D) 5 Ans: C Feedback: First determine the number of units per mL = 25,000 units/250 mL = 100 units/1 mL. Next use the ratio and proportion method to determine the number of milliliters needed to supply 600 units/h. 100 units/1 mL = 600 units/X. Cross-multiplying yields 6 mL needed every hour to supply the required dose. 27. The physician has ordered 30 mg of Demerol IM for relief of a severe migraine headache. The package insert reads meperidine hydrochloride (Demerol) 50 mg/mL. How many milliliters would the nurse administer? A) 1.6 B) 1 C) 0.6 D) 0.5 Ans: C Feedback: Filling in the information from the problem: 50 mg/1 mL = 30 mg/X mL. Cross-multiply yielding 0.6 mL required to administer a 30-mg dosage. 28. A patient is experiencing pain, so the physician orders codeine grain every 4 hours. How many milligrams of codeine would the nurse administer? A) 15 mg B) 30 mg C) 60 mg D) 120 mg Ans: B Feedback: The simplest way to convert measurements from one system to another is to set up a ratio and proportion equation. The ratio containing two known equivalent amounts is placed on one side of an equation, and the ratio containing the amount you wish to convert and its unknown equivalent is placed on the other side. 60 mg/1 grain = flexion range of motion. Cross-multiplying yields 30 mg. 29. A 79-year-old female patient presents at the clinic complaining of constipation for 1 week. The nurse practitioner prescribes Milk of Magnesia 2 teaspoons by mouth as needed for relief of constipation. How many milliliter will the nurse administer? A) 30 mL B) 7.5 mL C) 10 mL D) 15 mL Ans: C Feedback: Use the ratio and proportion method to convert from household system to metric system. 5 mL/1 tsp = X mL/2 tsp. Cross-multiplying yields 10 mL = 2 tsp. 30. A patient has orders to receive 3,000 mL of IV fluid at a rate of 150 mL/h. If the infusion starts at 08:00, when would it be finished? A) 20:00 B) 23:00 C) 01:00 D) 04:00 Ans: D Feedback: Amount of fluid to infuse = 3,000 mL; rate of infusion = 150 mL. 3,000 150 mL = 20 hours to infuse. There are 24 hours in a day 20 hours = 4 hours. The infusion will complete in 4 hours before 08:00 (08:00 04:00 = 04:00 hour) so the infusion completes at 04:00. 31. The nurse recognizes that what system is being used when seeing a medication ordered in minims? A) Apothecary B) Metric C) Household D) Avoirdupois Ans: A Feedback: The apothecary system used minims as the basic measure of liquid and is rarely used today. The metric system uses liters as the basic unit of measurement for fluid, while the household system uses ounces and the avoirdupois system uses ounces with a different conversion amount. 32. The nurse is calculating the patients intake and output record and converts ounces to milliliters. What systems is this nurse converting from and to? A) From household to metric B) From metric to household C) From household to apothecary D) From apothecary to Avoirdupois Ans: A Feedback: Ounces can be either household or Avoirdupois, although they are not equal measurements and milliliters is a metric measurement. Since Avoirdupois to metric is not an available choice, the correct answer is household to metric. 33. What is the most accurate method for the nurse to use when determining a pediatric dosage? A) A nomogram using body surface area B) Youngs rule C) Frieds rule D) Clarks rule Ans: A Feedback: The most accurate means of determining a pediatric drug dosage is a nomogram using body surface area because the weight and body surface area of two children who are of the same age can be significantly different. Youngs rule, Frieds rule, and Clarks rule are based on the childs age and the usual adult dosage and are rarely used today unless no other method will suffice for a specific drug. 34. The nurse determines a childs body surface area is 0.4 m2 and the average adult dosage of the medication is 500 mg. The medication is supplied in liquid form with 500mg/5 mL. How many milliliter will the nurse administer? A) 3.46 mL B) 1.73 mL C) 0.5 mL D) 12 mL Ans: A Feedback: The formula for calculating the childs dose using body surface area is surface area in m2 divided by 1.73 and then multiplied by adult dosage. Using the information supplied in this problem (0.4 1.73) 500 mg = 346 mg. Using the ratio and proportion method, 500 mg/5 mL = 346 mg/X mL; cross-multiplying yields 3.46 mL for the nurse to administer. 35. The physician orders acetaminophen (Tylenol) 15 mg/kg q4 hours PRN (as needed) for pain. The drug is supplied with 160 mg/5 mL. The infant weighs 12 pounds. How many milliliter of medication will the nurse administer? A) 2.5 mL B) 5.6 mL C) 12.4 mL D) 10.7 mL Ans: A Feedback: Begin by converting the infants weight to kilograms (12 pounds 2.2 pounds/ kg = 5.45 kg). The order says to give 15 mg/kg. To determine this childs dosage multiply weight and 15 mg/kg (5.45 kg 15 mg/kg = 81.75 mg/dose). Finally, use the ratio and proportion method to calculate how to prepare the medication: 160 mg/5 mL = 81.75 mg/X mL; cross-multiplying yields 2.5 mL. Chapter 9. Enteral Medications and Administration MULTIPLE CHOICE 1. In which position would the nurse place a patient before the administration of an enteral feeding? a Supine . b Semi-Fowlers . c Left lateral . d Prone . ANS: B To facilitate gastric emptying, the patient should be placed in a semi-Fowlers position (30 degree head of bed [HOB] elevation) for 30 minutes before the start of the feeding. Aspiration is a risk during enteral feedings in the supine position and in the left lateral position. The prone position would prevent gastric emptying and increase the risk of aspiration. DIF: Cognitive Level: Application REF: p. 134 OBJ: 4 TOP: Nursing Process Step: Implementation MSC: NCLEX Client Needs Category: Physiological Integrity 2. Which type of lubricant would the nurse use to administer a rectal suppository? a Petroleum jelly . b Mineral oil . c Water soluble . d Anesthetic . ANS: C Water soluble lubricants should be used with rectal suppository administration. When not available, water can be used to moisten mucosal surfaces. Petroleum based and oil based lubricants can harbor bacteria and promote infection. Unless the patient has pain in the rectal area (in which case another route of administration should be considered), anesthetic should not be required. DIF: Cognitive Level: Knowledge REF: p. 136 OBJ: 5 TOP: Nursing Process Step: Implementation MSC: NCLEX Client Needs Category: Physiological Integrity 3. Which is a characteristic of medication administration via the rectal route? a Irritation of the mouth . b Nausea and vomiting . c Bypassing of the digestive enzymes . d Use of the first pass metabolism . ANS: C Rectal administration bypasses the digestive enzymes because the medication is absorbed directly into the bloodstream. Rectal administration bypasses the oral cavity and does not affect the gag reflex or upset the stomach. Rectal administration bypasses first pass metabolism. DIF: Cognitive Level: Knowledge REF: p. 124 OBJ: 5 TOP: Nursing Process Step: Implementation MSC: NCLEX Client Needs Category: Physiological Integrity 4. Which medications are provided in dried, powdered form compressed into small disks? a Pills . b Capsules . c Tablets . d Lozenges . ANS: C Tablets are dried, powdered drugs that have been compressed into small disks. Pills are an obsolete dose form that is no longer manufactured as a result of the development of capsules and compressed tablets. Capsules are small cylindrical gelatin containers that hold dry powder or liquid medicine. Lozenges are small aromatic medicated candies, such as cough drops. DIF: Cognitive Level: Comprehension REF: p. 125 OBJ: 1 TOP: Nursing Process Step: Implementation MSC: NCLEX Client Needs Category: Physiological Integrity 5. Which action by the nurse is appropriate when administering enteric coated tablets? a Administer with an antacid. . b Crush the tablet and mix with applesauce. . c Encourage the patient to drink a full glass of water. . d Instruct the patient to place the medication between the cheek and teeth. . ANS: C Drinking a full glass of water ensures the medication reaches the stomach and is diluted to decrease the potential for irritation. Administering with an antacid would alter the dissolution of the enteric coated tablet because it has a coating that resists dissolution in the acidic pH of the stomach but is dissolved in the intestines. Enteric coated tablets must not be crushed and must be swallowed. DIF: Cognitive Level: Application REF: p. 125 OBJ: 1 TOP: Nursing Process Step: Implementation MSC: NCLEX Client Needs Category: Physiological Integrity 6. Which route of administration would be ordered by the health care provider if a patient is vomiting? a Gastrostomy tube . b Intradermal . c Ophthalmic . d Rectal . ANS: D The rectal route is a good alternative when nausea or vomiting is present. Gastrostomy is used for patients who cannot swallow or have had oral surgery; drugs administered by this route would be placed in the gastrointestinal tract and are inappropriate for the patient with vomiting. Intradermal routes are used for allergy testing. Ophthalmic medications are for use in the eye. DIF: Cognitive Level: Application REF: p. 124 OBJ: 1 TOP: Nursing Process Step: Implementation MSC: NCLEX Client Needs Category: Physiological Integrity 7. After entering the patients room to administer oral medications, which action will the nurse take first? a Assist the patient to sit upright. . b Check the patients identification. . c Inform the patient about the medications. . d Offer the patient something to drink. . ANS: B Checking the patients identification is the first nursing action once at the bedside. Assisting the patient to sit upright is appropriate when administering oral medications, but this is not the first thing the nurse would do. Providing information about medications is appropriate, but it is not the first thing the nurse would do. Offering the patient something to drink to facilitate swallowing of medications is appropriate, but it is not the first thing the nurse would do. DIF: Cognitive Level: Application REF: p. 128 OBJ: 1 TOP: Nursing Process Step: Implementation MSC: NCLEX Client Needs Category: Physiological Integrity 8. The nurse is preparing to administer a medication in tablet form to a patient. In administering this medication, the nurse will encourage the patient to: a drink a large amount of water prior to administration so that swallowing is . easier. b place the medication on the front of the tongue. . c keep the head forward while swallowing. . d minimize the amount of fluid taken following medication administration. . ANS: C The patient should be encouraged to keep the head forward while swallowing. The patient should be allowed to drink a small amount of water to moisten the mouth, so that swallowing the medication is easier. The patient should be instructed to place the medication well back on the tongue. Drinking a full glass of fluid should be encouraged to ensure that the medication reaches the stomach and is diluted to decrease the potential for irritation. DIF: Cognitive Level: Application REF: p. 129 OBJ: 1 TOP: Nursing Process Step: Implementation MSC: NCLEX Client Needs Category: Physiological Integrity 9. When assessing aspirated stomach contents, the nurse notes the color to be green with sediment. The nurse is aware that this most likely represents fluid. a pleural . b gastric . c intestinal . d tracheobronchial . ANS: B Gastric fluid appears green with sediment or off white. Pleural fluid appears clear to straw colored. Intestinal fluid appears yellow (bile colored). Tracheobronchial fluid appears off white or tan. DIF: Cognitive Level: Analysis REF: p. 133 OBJ: 4 TOP: Nursing Process Step: Assessment MSC: NCLEX Client Needs Category: Physiological Integrity MULTIPLE RESPONSE 10. Oral drug administration includes which principle(s)? (Select all that apply.) a Dependable rate of absorption . b Most economical . c . Insulin able to be administered via this route d . Drugs delivered directly by the oral, rectal, or nasogastric (NG) methods e . Dosage forms are convenient and readily available ANS: B, E Oral administration is the most economical, convenient, and readily available. Absorption from oral medications can vary depending on many factors. Insulin cannot be administered via the oral route. In oral drug administration, drugs are only delivered via the oral route. DIF: Cognitive Level: Comprehension REF: p. 124 OBJ: 1 TOP: Nursing Process Step: Implementation MSC: NCLEX Client Needs Category: Physiological Integrity 11. The nurse is administering an oral medication to a 90 year old patient who has difficulty swallowing pills. One of the medications to be administered is a spansule type capsule. What nursing consideration(s) should be applied in this case? (Select all that apply.) a Wash hands before preparing medications and before administration. . b Crush medications and administer with a soft food, such as applesauce. . c Check the patients ID band with the MAR to ensure patient rights are followed. . d Have an 8 ounce glass of water available. . e Check with the pharmacist to see if the spansule medication comes in a liquid . form. ANS: A, C, D, E Hands should be washed before and after medication preparation. Always check the patients identification before administering medication. Have water available to the patient when administering this medication. Giving the medication in liquid form, if available, would be much more comfortable for the patient. Spansule medications are time released and should not be crushed. DIF: Cognitive Level: Application REF: pp. 128-129 OBJ: 1 TOP: Nursing Process Step: Implementation MSC: NCLEX Client Needs Category: Physiological Integrity 12. Which receptacle(s) is/are commonly used in the hospital with pediatric oral medications? (Select all that apply.) a Oral syringe . b Baby bottle full of formula . c Infant feeding nipple . d Teaspoon . e Medicine dropper . ANS: A, C, E An oral syringe or plastic medicine cup would be most accurate. An infant feeding nipple is commonly used for pediatric patients. A medicine dropper may be used to administer medications to pediatric patients. A full bottle of formula is too large a volume in which to administer medication. A teaspoon does not minimize the risk of spilling when administering medication to a pediatric patient. DIF: Cognitive Level: Comprehension REF: p. 127 OBJ: 3 TOP: Nursing Process Step: Implementation MSC: NCLEX Client Needs Category: Physiological Integrity 13. Which data will the nurse document when administering a PRN oral pain medication to a patient? (Select all that apply.) a Date, time, drug name, dosage, and route of administration . b Essential patient education about the drug completed . c Administration receptacle used . d Signs and symptoms of adverse drug effects . e Evaluation of therapeutic effectiveness . ANS: A, B, D, E Date, time, drug name, dosage, and route of administration are included in the seven rights of medication administration. The patient should be educated about the drug being administered. The nurse should observe for adverse drug effects. Evaluating the drugs therapeutic effectiveness is important. Unless the receptacle used is unusual, it does not need to be noted. DIF: Cognitive Level: Comprehension REF: pp. 129-130 OBJ: 1 TOP: Nursing Process Step: Implementation MSC: NCLEX Client Needs Category: Physiological Integrity 14. NG medication administration includes which principle(s)? (Select all that apply.) a The tube must be assessed for correct placement. . b All medications can be combined into one syringe. . c Tablets and capsules should be dissolved in water. . d The suction source should be immediately reconnected. . e Flush the tube with 30 mL of water after drug administration. . ANS: A, C, E It is essential to verify correct placement of an NG tube. Solid medications must be crushed and dissolved in water before administration (with the exception of enteric coated tablets). Capsules should be opened and granules or powder sprinkled into 30 mL of water to dissolve (with the exception of timed released capsules). Flushing the tube serves to clear the tube and ensure that the drug has been transported to the intestine. Incompatible medications should not be combined. Suction will evacuate the medication from the patient. DIF: Cognitive Level: Application REF: pp. 132-133 OBJ: 4 TOP: Nursing Process Step: Implementation MSC: NCLEX Client Needs Category: Physiological Integrity 15. Which nursing action(s) would be appropriate when administering a disposable enema? (Select all that apply.) a Position the patient on the left side. . b Allow the solution to flow in by gravity. . c Instruct the patient to hold the solution 30 minutes before defecating. . d Maintain the six rights of medication administration. . e Lubricate the rectal tube. . ANS: A, C, D, E To facilitate flow into the large intestine, patients should be positioned on the left side. The solution should be held for 30 minutes before defecating. Enemas are medications, so the seven rights of medication administration should be followed. Lubrication of the rectal tube will facilitate insertion into the rectum. Gravity will not facilitate the administration of a small volume of enema solution administered from a bottle. DIF: Cognitive Level: Knowledge REF: pp. 137-138 OBJ: 5 TOP: Nursing Process Step: Implementation MSC: NCLEX Client Needs Category: Physiological Integrity 16. In preparing to administer medications to a patient with an NG tube, which would be appropriate to give through that route? (Select all that apply.) a Liquid medication . b Tablets crushed and diluted in 30 mL of water . c Enteric coated tablets crushed and diluted in 30 mL of water . d Capsules emptied into 30 mL of water . e Timed release capsules emptied into 30 mL of water . f Suppositories . ANS: A, B, D Liquid forms of medications are preferable. Tablets may be crushed and diluted in water. Capsules may be opened and the contents added to approximately 1 ounce of water. Enteric coated medications and timed release capsules should never be broken for administration. Suppositories are not given via NG route. DIF: Cognitive Level: Application REF: pp. 132-133 OBJ: 4 TOP: Nursing Process Step: Implementation MSC: NCLEX Client Needs Category: Physiological Integrity 17. An adult patient is to receive 10 mL of cough syrup at 0800. The nurse can prepare to administer this medication in a(n): (Select all that apply.) a souffl cup. . b medicine cup. . c oral syringe. . d teaspoon. . e nipple. . ANS: B, C The medicine cup is a plastic container with three scales to measure liquid medications. An oral syringe comparable to the volume to be measured can be used for smaller volumes such as 10 mL. A souffl cup is a small paper or plastic cup used to transport solid medication forms such as a capsule or tablet to the patient to prevent contamination by handling. A teaspoon is equal to 5 mL. An infant feeding nipple with additional holes may be used for administering oral medications to infants. Chapter 10. Parenteral Medications and Administration MULTIPLE CHOICE 1. A patient is diagnosed with cancer and requires 6 months of chemotherapy infusions. Which type of intravenous (IV) access device will likely be used? a Peripheral venous access device . b Midline catheter . c Winged needle venous access device . d Implantable venous infusion port . ANS: D Implantable venous infusion ports are placed into central veins for long term therapy. Chemotherapy treatment is often irritating and best tolerated in the larger central veins. Peripheral lines are not used for administration of chemotherapy because of the risk of extravasation. A midline catheter is intended only for a 2 to 4 week interval, less than the projected length of time for chemotherapy infusion. Winged needles are for use in peripheral veins that are too small for ongoing infusion of chemotherapy. DIF: Cognitive Level: Application REF: p. 172 | p. 174 OBJ: 5 TOP: Nursing Process Step: Implementation MSC: NCLEX Client Needs Category: Physiological Integrity 2. The nurse notes that a patient with cardiac disease has IV heparin infusing and that it is behind by 2 hours. What is the best nursing action? a Increase the IV rate and recheck in 1 hour. . b Change the infusion rate to TKO. . c Discontinue the solution using aseptic technique. . d Contact the health care provider for consultation. . ANS: D The patient has a history of cardiac problems and is receiving a critical care medication, IV heparin. In this case, contacting the patients health care provider would be appropriate to avoid harm. Increasing the infusion rate might place the patient into fluid overload and might infuse too much heparin in a short time. Reducing the infusion rate to TKO or discontinuing the solution would put the schedule even further behind. DIF: Cognitive Level: Application REF: p. 180 OBJ: 8 TOP: Nursing Process Step: Implementation MSC: NCLEX Client Needs Category: Safe, Effective Care Environment 3. What is the composition of hypotonic intravenous solutions such as 0.45% NaCl? a Fewer dissolved particles than blood . b Approximately the same number of dissolved particles as blood . c Higher concentrations of dissolved particles than blood . d Electrolytes and dextrose . ANS: A Hypotonic solutions have fewer dissolved particles than blood. Half normal saline does not contain dextrose. DIF: Cognitive Level: Knowledge REF: p. 176 OBJ: 4 TOP: Nursing Process Step: Assessment MSC: NCLEX Client Needs Category: Physiological Integrity 4. Which condition would the nurse expect to be treated with an isotonic solution? a Fluid overload . b . Hemorrhagic shock c . Cellular dehydration d . Cerebral edema ANS: B Isotonic solutions have approximately the same osmolality as blood. Isotonic fluids are ideal replacement fluids for patients experiencing an intravascular fluid deficit that occurs in conditions such as acute blood loss from hemorrhage and gastrointestinal bleeding. Isotonic fluids increase vascular volume, thus counteracting hypovolemia and hypotension. Administering isotonic solutions for fluid overload would exacerbate the problem. Hypotonic solutions are administered for cellular dehydration. Hypertonic solutions are administered for cerebral edema. DIF: Cognitive Level: Application REF: p. 176 OBJ: 4 TOP: Nursing Process Step: Implementation MSC: NCLEX Client Needs Category: Physiological Integrity 5. The nurse determines that an elderly patients IV of D50.2NS with 20 mEq KCl at 75 mL/hr is running 3 hours behind. After determining the IV site is patent, what action will the nurse take? a Call the health care provider to obtain an order to decrease the IV rate. . b Administer a bolus to make up the deficit. . c Recalculate the flow rate and slowly make up the fluids. . d Maintain the ordered rate. . ANS: D The safest action is to maintain the ordered rate. The health care provider should be consulted if the patient has not received critical IV replacement therapy. Increasing an IV rate without a health care providers order can be detrimental for patients who have cardiac, renal, or circulatory impairment. Normal aging process results in decreased cardiac, renal, circulatory function. The rate ordered is the one the provider intended for the administration of fluids; changing it to fit the prevailing situation is not appropriate. The bolus technique should only be used for the administration of medications or fluid challenges in patients who need a volume of IV fluid quickly. The flow rate must be consistent with the providers order. DIF: Cognitive Level: Application REF: p. 169 OBJ: 9 TOP: Nursing Process Step: Implementation MSC: NCLEX Client Needs Category: Safe, Effective Care Environment 6. Which technique by the nurse accurately maintains asepsis of a peripheral IV access device? a Wear gloves when hanging all IV solutions. . b Apply a topical antibiotic ointment to the insertion site. . c Change fluid administration sets according to institutional policy. . d Flush with heparin before use. . ANS: C Generally all IV solution bag and bottles should be changed every 24 hours to minimize the development of new infections. IV administration sets used to deliver blood and blood products are changed after each unit is administered. Administration sets to deliver lipids and TPN are often changed every 4 hours, whereas administration sets for maintenance fluids may be changed every 72 hours. It is important to follow institutional policies. All IV bags, bottles, and administration sets should be labeled with the date, time, and nurses initials of the set change. Wearing gloves is not required for maintenance of routine infusion. Topical antibiotics may promote fungal infections and antimicrobial resistance. A peripheral line that is infusing should not need an anticoagulant to maintain patency. DIF: Cognitive Level: Application REF: p. 179 OBJ: 9 TOP: Nursing Process Step: Implementation MSC: NCLEX Client Needs Category: Safe, Effective Care Environment 7. Which needle is used to access implanted infusion devices? a Jamshidi . b Huber . c Gigli . d Crutchfield . ANS: B The Huber needle is a special noncoring 90-degree needle used to penetrate the skin and septum of the implanted device. The Jamshidi needle is used for biopsy purposes such as bone marrow. The Gigli saw is a wire with serrations used to cut through cranial bone. Crutchfield tongs are used to stabilize the cervical spine by traction in cases of fracture. DIF: Cognitive Level: Comprehension REF: p. 174 OBJ: 6 TOP: Nursing Process Step: Implementation MSC: NCLEX Client Needs Category: Physiological Integrity 8. The nurse assesses erythema, warmth, and burning pain along the patients IV site. Which complication is this patient most likely experiencing? a Air embolism . b Extravasation . c Phlebitis . d Pulmonary edema . ANS: C Erythema, warmth, and tenderness along the course of the vein and swelling are signs of phlebitis. Air embolism occurs as a result of an air bubble entering the vascular system, and shortness of breath, chest pain, and hypotension are indicative of this complication. Extravasation is the leakage of an irritant and is accompanied by redness, warmth or coolness, swelling, and a dull ache to severe pain at the venipuncture site. Pulmonary edema is caused by fluid infusing too rapidly; dyspnea, cough, anxiety, rales, and possible cardiac dysrhythmias are indicative of pulmonary edema. DIF: Cognitive Level: Comprehension REF: p. 196 OBJ: 9 TOP: Nursing Process Step: Evaluation MSC: NCLEX Client Needs Category: Physiological Integrity 9. An elderly patient receiving an infusion of an isotonic fluid at 100 mL/hr complains of dyspnea. The nurse notes shallow rapid respirations and a cough that produces frothy sputum. Which is the priority nursing action? a Assess the urine output. . b Elevate the head of the bed. . c Encourage the patient to cough. . d Maintain the IV rate. . ANS: B Elevating the head of the bed is an appropriate action for signs and symptoms of pulmonary edema. Urine output is important to assess, but it is not the priority nursing action. Encouraging the patient to cough and take deep breaths is not the priority nursing action. The IV rate should be slowed immediately based on the signs and symptoms the patient is displaying. DIF: Cognitive Level: Application REF: p. 198 OBJ: 9 TOP: Nursing Process Step: Implementation MSC: NCLEX Client Needs Category: Physiological Integrity 10. A diabetic patient requires the administration of insulin continuously at home. Which system would most likely be used in this instance? a Central line catheter . b Microdrip set . c Piggyback system . d Syringe pump . ANS: D Syringe pumps are used in patients with diabetes. A central line is not appropriate for the diabetic patient requiring insulin. A microdrip set is a type of IV tubing that is used when small volumes of fluid are given to patients with fluid volume concerns. A piggyback system is a type of administration set that connects to a primary setup and administers a small volume over 20 to 60 minutes. DIF: Cognitive Level: Application REF: pp. 171-172 OBJ: 3 TOP: Nursing Process Step: Implementation MSC: NCLEX Client Needs Category: Physiological Integrity 11. A patient is admitted with hypovolemia resulting from lack of fluid intake and requires an infusion of isotonic fluids. Which IV solution will the nurse administer? a D50.2 NS . b D5W . c 0.45 NS . d 0.9 NS . ANS: D 0.9 NS is an isotonic solution appropriate for hypovolemia. D50.2 NS, D5W, and 0.45 NS are hypotonic solutions. DIF: Cognitive Level: Comprehension REF: p. 176 OBJ: 4 TOP: Nursing Process Step: Implementation MSC: NCLEX Client Needs Category: Physiological Integrity 12. Which potential complication will the nurse expect in patients with a venous access device? a Circulatory overload . b Extravasation . c Infection . d Pain . ANS: C Because venipuncture alters skin integrity, the patient is vulnerable to infection at all times. Circulatory overload is a concern but does not occur with any type of venous access device because the device may just be used for administration of small volumes of drugs (e.g., chemotherapy in cancer patients). Extravasation is a potential complication when there is infusion of an irritating chemical. IV drug administration is usually more comfortable for patients than other routes, and pain would not be considered a complication. DIF: Cognitive Level: Application REF: p. 197 OBJ: 9 TOP: Nursing Process Step: Assessment MSC: NCLEX Client Needs Category: Physiological Integrity 13. A patient has a peripherally inserted central catheter (PICC) line inserted to continue IV antibiotic therapy at home. With proper care, how long can this type of venous access device remain in place? a 2 months . b 4 months . c 6 months . d 12 months . ANS: D PICC lines routinely remain in place for 1 to 3 months, but can last for a year or more if cared for properly. DIF: Cognitive Level: Knowledge REF: p. 173 OBJ: 3 TOP: Nursing Process Step: Implementation MSC: NCLEX Client Needs Category: Physiological Integrity 14. In assessing a patient with a central venous access device, which sign or symptom indicates that the patient is experiencing an air embolism? a Chest pain . b Erythema . c Frothy sputum . d Sweating . ANS: A Chest pain is a symptom associated with air embolism. Erythema occurs with infiltration or extravasation. Frothy sputum occurs with circulatory overload or pulmonary edema. Sweating is indicative of a pulmonary embolism. DIF: Cognitive Level: Application REF: p. 198 OBJ: 9 TOP: Nursing Process Step: Assessment MSC: NCLEX Client Needs Category: Physiological Integrity 15. Following the insertion of a central venous access device, the nurse notes a weak, thready pulse and decreased blood pressure. The patient complains of shortness of breath and palpitations. Which action will the nurse take first? a Place the patient on the left side. . b Reassess vital signs. . c Stop the infusion. . d Verify placement of the device. . ANS: A Signs and symptoms indicate an air embolism. The nurses immediate action will be to place the patient onto his or her left side. The nurse has determined change in pulse and blood pressure already, and although it is appropriate to reassess, it is not the first action the nurse will take. There is no indication that anything is infusing into this venous access device. Verifying the placement of the device is not the first action the nurse would take. DIF: Cognitive Level: Application REF: p. 198 OBJ: 9 TOP: Nursing Process Step: Implementation MSC: NCLEX Client Needs Category: Physiological Integrity 16. The nurse is about to administer a prescribed medication IV push into a patients Hickman catheter. When providing this medication, the nurse will first: a administer the prescribed drug. . b flush with saline. . c flush with heparin. . d prepare a pump. . ANS: B Drugs given by IV push or bolus through a Hickman catheter generally follow the SASH guideline: saline flush first; administer the prescribed drug; saline flush following the drug; heparin flush line. A pump is not used when a drug is administered by push technique. DIF: Cognitive Level: Application REF: p. 179 OBJ: 6 | 9 TOP: Nursing Process Step: Implementation MSC: NCLEX Client Needs Category: Safe, Effective Care Environment; Physiological Integrity 17. A 90 year old woman is admitted to an acute care facility with the diagnosis of pneumonia. She has a past medical history of diabetes mellitus, hypertension, and right sided mastectomy. When starting an IV for infusion of antibiotic therapy, the nurse will: a insert the IV catheter into the left hand. . b use a lower extremity vein for insertion. . c choose the left radial artery for insertion. . d attempt insertion into the left antecubital space vein. . ANS: D IV insertion should not be initiated in an arm with compromised lymphatic or venous flow such as a mastectomy. The left antecubital space vein would be a good choice for this patient given her age and medical history. In the older adult, using the veins in the hand area may be a poor choice because of the fragility of the skin and veins in this area. When possible, the veins of the lower extremities should be avoided for IV insertion because of the danger of developing thrombi and emboli. IV therapy should never be started in an artery. DIF: Cognitive Level: Application REF: p. 180 OBJ: 6 TOP: Nursing Process Step: Implementation MSC: NCLEX Client Needs Category: Safe, Effective Care Environment MULTIPLE RESPONSE 18. What will the nurse explain when teaching a patient about a PICC line? (Select all that apply.) a The catheter may have a single or double lumen. . b There is greater risk of clotting and infiltration with this type of catheter. . c The patient will be receiving infusions continuously to ensure patency. . d The tip of the catheter may be open or valved. . e The catheter may be used for drawing blood. . ANS: A, D PICC lines may have more than one lumen. The catheter may have an open tip or a valved (Groshong) tip. The risk of infiltration and clotting is less than with other types of central lines. The line should be flushed with a saline heparin solution after every use, or daily, in order to maintain patency if it is not in continuous use. PICC lines are not appropriate for blood drawing because of their small size. DIF: Cognitive Level: Comprehension REF: p. 173 OBJ: 3 TOP: Nursing Process Step: Implementation MSC: NCLEX Client Needs Category: Physiological Integrity 19. Which patient assessment finding(s) suggest(s) extravasation of an IV solution? (Select all that apply.) a Coolness . b Edema . c Fever . d Pain at venipuncture site . e Redness at the site . f Shortness of breath . ANS: A, B, D, E Coolness, edema, pain, and redness are indicative of extravasation. Fever does not indicate extravasation. Shortness of breath does not indicate extravasation. DIF: Cognitive Level: Comprehension REF: p. 197 OBJ: 9 TOP: Nursing Process Step: Assessment MSC: NCLEX Client Needs Category: Physiological Integrity 20. The nurse assesses a patients right hand IV site to be infiltrated. Appropriate nursing actions include: (Select all that apply.) a stopping the infusion. . b attempting to aspirate the medication. . c elevating the affected limb. . d checking capillary refill. . e removing the catheter as directed by policy. . ANS: A, C, D, E For an infiltration, stop the infusion. Elevate the affected limb. Assess for circulatory compromise; check capillary refill and pulses proximal and distal to the area of infiltration. If the infiltration is caused by an IV solution, remove the catheter as directed by policy. For extravasation, attempts may be made to aspirate the medication. Chapter 11. Integumentary System Medications MULTIPLE CHOICE 1. The nurse is educating a patient about diabetes. Based on recommendations from the American Diabetes Association, which statement by the nurse is best regarding site rotation? a Insulin injection sites should always be in the abdomen to ensure absorption . into the stomach. b . It is important to rotate injection sites systematically within one area before progressing to a new site for injection. c . Following exercise, site rotation is not indicated because the circulation in the muscles will absorb the medication efficiently. d . If you aspirate, site rotation can be done every other day to avoid developing problems with absorption. ANS: B The American Diabetes Association Clinical Practice recommendations include rotating injections systematically at one site before progressing to another. Insulin is not absorbed into the stomach. Failure to rotate sites can result in lipohypertrophy or lipoatrophy. When subcutaneous (subcut) insulin is administered, aspiration should never be performed. DIF: Cognitive Level: Application REF: p. 160 | p. 165 OBJ: 2 TOP: Nursing Process Step: Implementation MSC: NCLEX Client Needs Category: Safe, Effective Care Environment 2. Which technique by the nurse is accurate when administering heparin to a thin, older adult patient? a Aspirate before injecting the medication. . b Inject at a 45-degree angle. . c Inject at a 90-degree angle. . d Massage site following injection. . ANS: B For thin individuals, the skin may need to be pinched and a 45-degree angle used to avoid administration into the muscle. Heparin should never be aspirated. Subcut injections are properly administered at a 45-degree angle. The injection site of heparin should never be massaged. DIF: Cognitive Level: Application REF: p. 160 OBJ: 2 TOP: Nursing Process Step: Implementation MSC: NCLEX Client Needs Category: Physiological Integrity 3. The nurse is preparing to administer kindergarten immunizations at the local health clinic. Which anatomic site would be best for the injection of the immunizations containing 0.5 mL? a Rectus femoris . b Dorsogluteal . c Deltoid . d Ventrogluteal . ANS: C The deltoid muscle is often used because of its easy access and the fact that it can tolerate 0.5 mL of medication volume. Having the child disrobe is not efficient in this setting. DIF: Cognitive Level: Application REF: p. 164 OBJ: 5 TOP: Nursing Process Step: Implementation MSC: NCLEX Client Needs Category: Physiological Integrity 4. A 65-year-old man who weighs 180 lb (81.8 kg) is to receive 1.5 mL of a viscous antibiotic by intramuscular (IM) injection. Which needle and syringe will be used? a 5/8 inch, 25-gauge needle with 5 mL syringe . b 1 inch, 28-gauge needle with 4 mL syringe . c 1 1/2 inch, 21-gauge needle with 3 mL syringe . d 3 inch, 16-gauge needle with 1.5 mL syringe . ANS: C It is important to correlate the syringe size to the size of the patient and the tissue mass. The usual amount injected intramuscularly is 0.5 to 2 mL. Needle lengths commonly used for adults are 1 to 1 1/2 inches long. A longer needle may be used for a significantly obese adult. Commonly used needle gauges for IM injections are 20 to 22 gauge. A 5/8 inch, 25-gauge needle is too short and too small to administer a viscous medication. A 1 inch, 28-gauge needle is too small to administer a viscous medication. A 3 inch, 16-gauge needle is too large and too long to administer medication to a patient this size. DIF: Cognitive Level: Application REF: p. 161 OBJ: 3 TOP: Nursing Process Step: Implementation MSC: NCLEX Client Needs Category: Physiological Integrity 5. Which is the preferred IM site for injecting a 6-month-old child? a Dorsogluteal . b . Abdominal c . Vastus lateralis d . Deltoid muscle ANS: C The vastus lateralis is generally the preferred IM site in infants because it has the largest muscle mass for that age group. The muscles are not well developed in other areas for this age group. The dorsogluteal muscle is not developed well enough in a child this age to provide a safe site for injection. The abdominal muscles are not appropriate for IM injection. The deltoid muscle is not developed well enough in a child this age to provide a safe site for injection. DIF: Cognitive Level: Knowledge REF: p. 162 OBJ: 5 TOP: Nursing Process Step: Implementation MSC: NCLEX Client Needs Category: Physiological Integrity 6. Which angle is appropriate when administering an IM medication in the dorsogluteal site to a 46 year old obese man? a 45 degrees . b 60 degrees . c 75 degrees . d 90 degrees . ANS: D A 90 degree angle is used to reach the IM area of the dorsogluteal site. A 45-, 60-, or 75-degree angle does not ensure that the needle will penetrate to the muscle in an obese patient. DIF: Cognitive Level: Application REF: p. 163 OBJ: 3 TOP: Nursing Process Step: Implementation MSC: NCLEX Client Needs Category: Physiological Integrity 7. Which parenteral route has the longest absorption time? a Intradermal . b Subcut . c IM . d Intravenous (IV) . ANS: A Absorption rate is determined by the proximity of the medication to the vascular system. Medication injected into an intradermal site is farther away from the vascular system than the other sites. Therefore, absorption in this site is the slowest. Subcut tissue is more vascular than intradermal tissue. IM tissue is more vascular than intradermal tissue. IV administration places medication directly into the vascular system. DIF: Cognitive Level: Knowledge REF: p. 157 OBJ: 1 TOP: Nursing Process Step: Assessment MSC: NCLEX Client Needs Category: Safe, Effective Care Environment 8. Which site is identified by the posterior superior iliac spine and greater trochanter? a Ventrogluteal . b Dorsogluteal . c Vastus lateralis . d Rectus femoris . ANS: B The dorsogluteal site is identified by drawing an imaginary line from the posterior superior iliac spine to the greater trochanter of the femur. The injection is then administered at any point between the imaginary straight line and below the curve of the iliac crest (hipbone). DIF: Cognitive Level: Comprehension REF: p. 163 OBJ: 4 TOP: Nursing Process Step: Implementation MSC: NCLEX Client Needs Category: Physiological Integrity 9. Which nursing action is accurate when administering an IM injection using the Z track method? a Use a 1-inch needle. . b Add 0.5 mL of air to the syringe. . c Vigorously massage the injection site. . d Pinch up the skin. . ANS: B Adding 0.5 mL of air ensures that the drug will clear the needle. A 1-inch needle may not ensure deep muscle penetration. Massaging the injection site could cause the medication to leak into the muscle tissue. The skin should be stretched, not pinched up. DIF: Cognitive Level: Comprehension REF: p. 166 OBJ: 5 TOP: Nursing Process Step: Implementation MSC: NCLEX Client Needs Category: Physiological Integrity 10. Which gauge needles are used for subcut injections? a 14 to 16 gauge . b 18 to 21 gauge . c 22 to 24 gauge . d 25 to 29 gauge . ANS: D Commonly used gauges for subcut injection are 25 to 29 gauge. Needles that are 14 to 16 gauge are used for administration of blood or large volumes of fluid in a short period of time. Needles that are 18 to 21 gauge are used for routine parenteral fluid administration. Needles that are 22 to 24 gauge are used for administering fluids or medication via small veins. DIF: Cognitive Level: Knowledge REF: p. 160 OBJ: 2 TOP: Nursing Process Step: Implementation MSC: NCLEX Client Needs Category: Physiological Integrity 11. When is it acceptable to use the deltoid muscle as an injection site for infants? a . Medication is irritating. b . Dose is a long acting medication. c . Child is combative. d Volume is quite small. . ANS: D The deltoid site should be used in infants only when the volume is quite small, the medication is nonirritating, and the dose will be quickly absorbed. Irritating and long acting medications should be injected in deep muscle tissue such as the vastus lateralis. A combative child should be adequately restrained and injected in a fairly large muscle mass. DIF: Cognitive Level: Comprehension REF: p. 164 OBJ: 5 TOP: Nursing Process Step: Implementation MSC: NCLEX Client Needs Category: Physiological Integrity 12. Which action by the nurse is most accurate when administering an intradermal injection? a . Insert the needle at a 45-degree angle. b . Inject 0.1 mL. c . Use a 22-gauge needle. d . Wipe the site with alcohol after injection. ANS: B Small volumes, usually 0.1 mL, are injected. The needle is inserted at a 15-degree angle. A 26- gauge needle is used. After injection, the site should not be wiped with alcohol. DIF: Cognitive Level: Application REF: p. 157 OBJ: 1 TOP: Nursing Process Step: Implementation MSC: NCLEX Client Needs Category: Physiological Integrity 13. Which assessment by the nurse is most important to obtain prior to performing allergy sensitivity testing? a . Identify areas of loose connective tissue. b . Question the patient about frequency of exercise. c . Determine if the patient is using aspirin. d . Palpate and measure the size of induration. ANS: C It is important to determine if the patient has taken any antihistamines or anti inflammatory agents for 24 to 48 hours prior to allergy testing. Loose connective tissue is assessed prior to administering injections subcutaneously. Exercise routine is not significant prior to allergy testing. Palpation and measurement of the size of induration are done after the procedure. DIF: Cognitive Level: Application REF: p. 157 OBJ: 1 TOP: Nursing Process Step: Assessment MSC: NCLEX Client Needs Category: Physiological Integrity 14. The nurse administers a skin prick test (SPT) to a patient at 9 AM. The earliest time the nurse can expect to read the test is: a . the next day at 9 AM. b . by 9 PM the same day. c . by 9:10 AM the same day. d . one week from the date and time of administration. ANS: C The SPT can be read in 10 to 20 minutes after administration, depending on protocol. DIF: Cognitive Level: Application REF: p. 159 OBJ: 1 TOP: Nursing Process Step: Implementation MSC: NCLEX Client Needs Category: Physiological Integrity 15. The nurse cleansing the skin surface of a patient prior to injection will start at the: a . periphery and work inward in a back and forth motion. b . periphery and work inward in a cyclical motion. c . injection site and work outward in a straight line. d . injection site and work outward in a circular motion. ANS: D The skin surface is cleansed prior to injection starting at the injection site and working outward in a circular motion toward the periphery. DIF: Cognitive Level: Application REF: p. 165 OBJ: 3 TOP: Nursing Process Step: Implementation MSC: NCLEX Client Needs Category: Physiological Integrity MULTIPLE RESPONSE 16. The nurse is preparing to administer allergy sensitivity testing to a patient. Which statement(s) pertain(s) to this type of administration? (Select all that apply.) a . Allergy sensitivity testing requires the intradermal route. b . Injections are made into the loose connective tissue. c . Equipment needed includes gloves, antiseptic pledget, metric ruler, and physicians order sheet. d . Record of previous injection sites is needed. e . The needle should be inserted at a 15-degree angle with the needle bevel up. ANS: A, C, E Allergy sensitivity testing requires the intradermal route. Before administering allergy testing, gathering of equipment and physicians orders are necessary. This route is injected into the dermal layer of skin, using a 15-degree angle. Connective tissue, having poor blood supply, is not appropriate for injection of medication. Previous injection sites would not factor into the decision about where to conduct allergy testing. DIF: Cognitive Level: Comprehension REF: p. 157 OBJ: 1 TOP: Nursing Process Step: Implementation MSC: NCLEX Client Needs Category: Physiological Integrity 17. Which statement(s) about administering medications parenterally is/are true? (Select all that apply.) a . Subcut absorption is slower than intradermal absorption. b . Two mL or less should be administered in a subcut site. c . The gluteal area is recommended for children. d . Needles 1 to 1 1/2 inches in length are common for IM injections. e . The scapular areas of the back may be used for intradermal injections. ANS: B, D, E No more than 2 mL of medication should be injected into a subcut site. Needle length of 1 to 1 1/2 inches is common for IM injections. The upper chest, scapular areas of the back, and the inner aspect of the forearms are commonly used as sites for intradermal injections. Subcutaneous absorption is faster than intradermal absorption. Because of the undeveloped muscle mass in children, the gluteal area is not recommended for IM injection. DIF: Cognitive Level: Knowledge REF: pp. 157-167 OBJ: 1 | 2 | 3 TOP: Nursing Process Step: Implementation MSC: NCLEX Client Needs Category: Physiological Integrity 18. The vastus lateralis muscle is appropriate for injections for which patient(s)? (Select all that apply.) a . Children younger than 3 years of age b . Elderly c . Debilitated d . Nonambulatory e . Ambulatory f . Healthy ANS: A, E, F Children younger than 3 years of age, ambulatory patients, and otherwise healthy patients may receive injections in the vastus lateralis. Elderly, debilitated, and nonambulatory patients may have reduced muscle mass in the vastus lateralis, and therefore poor absorption of injected medications. DIF: Cognitive Level: Comprehension REF: p. 162 OBJ: 4 | 5 TOP: Nursing Process Step: Assessment MSC: NCLEX Client Needs Category: Physiological Integrity 19. The nurse administers B12 IM to a patient in a long term care facility. After administering this medication, the nurse will: (Select all that apply.) a carefully recap the needle. . b identify the patient. . c . massage site of injection. d . dispose of the used needle according to policy. e . apply a small bandage to the site. ANS: D, E After administering an IM injection, the nurse should dispose of used needles according to policy and apply a small bandage to the site. Needles should never be recapped following use. The patient requires identification before the injection is given. After the needle is removed, gentle pressure should be applied to the site. Massage can increase the pain if the muscle mass is stressed by the amount of medication given. Chapter 12. Musculoskeletal System Medications 1. The nurse is calculating a drug dosage and converting from milligrams to grams. What measurement system is the nurse using? A) Metric system B) Apothecary system C) Household system D) Avoirdupois system Ans: A Feedback: The metric system is the most widely used system of measurement in the world; it is based on the decimal system. The gram is the basic unit of solid measure, and the liter unit of liquid measure. The apothecary system uses the grain as the basic unit of solid measure. The household system uses the pound as the basic unit of measure. The avoirdupois system uses ounces and grains, but it is mostly used by drug manufacturers for bulk medications. 2. The nurse teaches a young mother the importance of administering appropriate dosages of acetaminophen (Tylenol) and determines further teaching is needed when the mother makes what statement? A) The childrens dosage will change with time as they grow. B) My babys dose of Tylenol is about 1 half an adult dose. C) It is important to give the right dose to prevent toxic effects of the medication. D) My childrens dose of Tylenol should be based on their weight or age. Ans: B Feedback: A childs dose is never based on an adults dose. A childs dosage is based on weight and age and will change with age as they grow. Larger than directed dosages can result in toxic effects of this medication. 3. A nurse calculates the pediatric patients medication dosage using Clarks rule and uses what formula? A) Infants age in months/150 months times the average adult dose B) Childs age in years/childs age in years plus 12 times the average adult dose C) Weight of child in pounds/150 pounds times the average adult dose D) Surface area in square meters/1.73 times the average adult dose Ans: C Feedback: Clarks rule uses the childs weight to calculate the dose and assumes the adult dose is based on a 150-pound person. Frieds rule applies to a child younger than 1 year of age and assumes that an adult dose would be appropriate for a child who is 12.5 years (150 months) old. Youngs rule applies to children 1 to 12 years of age. Surface area calculation of a childs dose is determined with the use of a nomogram including the childs height and weight. 4. The nurse receives a new medication order for a patient to administer 240 mg of medication per day in equally divided doses every 6 hours. How many mg of the drug should the nurse administer for each dose? Ans: 60 mg Feedback: Because there are 24 hours in a day, giving a drug every 6 hours would mean giving the drug 4 times a day. Because the total daily dose is 240 mg, dividing that dose by 4 would mean each dose should be 60 mg. 5. A physician orders 500 mL of IV solution be administered over 8 hours. If the IV infusion set delivers 15 drops per mL, how many drops per minute should the nurse administer to the patient? A) 15 drops/min B) 20 drops/min C) 32 drops/min D) 64 drops/min Ans: A Feedback: If a patient was to receive 500 mL in 8 hours, dividing 500 by 8 would mean that the patient would receive 62.5 mL in 1 hour, or 60 min. Setting up the equation, 15 drops/mL/X equals 62.5 mL/60 min; cross-multiplying, the answer will be 15 drops/min. 6. The nurse is teaching a diabetic patient to self-administer Humulin insulin, supplied in a vial labeled 100 units/mL. The provider has ordered 32 units of Humulin insulin to be taken each morning. How many mL of insulin would the patient prepare for one dose? A) 0.032 mL B) 0.32 mL C) 3.2 mL D) 0.64 mL Ans: B Feedback: There are 100 units in each mL. Divide that amount by 32 units for the answer (0.32 mL). 7. The provider orders a maintenance dose of oral aminophylline, 3 mg/kg every 6 hour. The patient weighs 50 kg. How many mg should the nurse administer to the patient in a 24-hour period? Ans: 600 milligram Feedback: The patients weight times the number of milligram/kilogram will provide daily dosage of medication: 50 kg 3 mg/kg = 150 mg per dose. The patient is to receive a dose every 6 hours. The number of hours in a day divided by the number of hours separating each dose supplies the number of dosages the patient receives per day: 24 hours 6 hours between doses = 4 doses per day. If each dose is 150 mg and the patient receives 4 of these doses a day, the total amount of medication received is 150 mg 4 daily doses = 600 mg. 8. The physician writes an order for oxazepam for a 6-year-old child. The nurse verifies that there is no established dosage for children 6 to 12 years of age for oxazepam. The nurse knows that the usual adult dose is 10 mg tid. What would the nurse calculate the appropriate dose to be? A) 0.03 mg tid B) 0.3 mg tid C) 1.8 mg tid D) 3.3 mg tid Ans: D Feedback: Because the nurse knows only the childs age, the nurse would need to use Youngs rule to determine the appropriate dosage. The formula for Youngs rule is: Childs dose = childs age in years (childs age + 12) average adult dose. Using the information provided in the question: Dose = 6 (6 + 12) 10 mg = 6 18 10 = 0.33 10 = 3.3. 9. A newly admitted patient has orders to receive 1,000 mL of normal saline IV over 8 hours. If the IV infusion set is a microdrip set that delivers 60 drops per mL, how many drops per minute should the nurse administer to the patient? A) 60 drops/min B) 125 drops/min C) 240 drops/min D) 480 drops/min Ans: B Feedback: If a patient was to receive 1,000 mL in 8 hours, dividing 1000 by 8 would mean that the patient would receive 125 mL in 1 hour, or 60 minutes. Setting up the equation, 60 drops/mL X = 125 mL/60 minutes; cross-multiplying, the answer is 125 drops/min. 10. The nurse is preparing to administer cefadroxil 1 g PO. The medication is supplied in 500-mg tablets. How many tablets will the nurse administer? A) 0.5 tablet B) 1 tablet C) 2 tablets D) 3 tablets Ans: C Feedback: Convert 1 g to mg by multiplying 1 g times 1,000 mg. There are 500-mg in each tablet. Dividing the 1000 mg prescribed dosage by 500-mg available dosage, the answer is two tablets. 11. The nurse begins administering 500 mL of 5% dextrose and water solution at 01:00 to run over 4 hours. At 02:00, the nurse administers 80 mg gentamicin in 50 cc normal saline to infuse over 30 minutes. How many mL of fluid will the nurse administer to the patient between 02:00 and 03:00? A) 175 mL B) 150 mL C) 125 mL D) 100 mL Ans: A Feedback: The patient is receiving 500 mL over 4 hours. To determine how much fluid is infusing per hour = 500 mL 4 = 125. In addition to the 125 mL of IV solution, the patient also receives 50 mL of gentamicin during the 02:00 to 03:00 hour. 125 mL + 50 mL = 175 total mL of fluid received during this hour. 12. An adult patient with renal cancer, weighing 95 kg, is to receive vincristine 25 mcg/kg/day IV. What is the dosage of vincristine that the nurse should administer to the patient daily in mg? Ans: 2.375 mg Feedback: This order requires 25 mcg of medication for every 1 kg of body weight. The patient weighs 95 kg. To determine total dosage multiply weight times mcg of medication: 25 95 = 2,375 mcg. Convert mcg to mg by moving the decimal three places to the left, or you can divide 2,375 by 1,000 because there are 1,000 mcg per mg. 13. The nurse is preparing medication for a 30-month-old child with otitis media in the right ear. The child weighs 33 pounds. The physician has ordered Keflex, 50 mg/kg/d in equally divided doses every 8 hours. The medication concentration is 250 mg/5 mL. How many milliliters should the nurse give the toddler for each dose? Ans: 5 mL Feedback: To calculate the correct dosage, the nurse first converts the childs weight from pounds to kilograms by dividing weight in pounds by 2.2 (2.2 lb = 1 kg). 33 pounds 2.2 pounds/kg = 15 kg. The child is to receive 50 mg for every kilogram. To determine this childs dosage multiply weight times daily dose (15 kg 50 mg/kg = 750 mg). Thus, 750 mg is to be administered in equally divided dosages every 8 hours, or 3 times a day. 750 mg 3 = 250 mg/dose. There is 250 mg in 5 mL of medication so the patient would be given 5 mL. 14. An adolescent is admitted to the intensive care unit with diabetic ketoacidosis. The nurse prepares a continuous insulin infusion of 100 units (U) regular insulin in 500 milligram normal saline. When documenting this medication, how many units of regular insulin will this patient receive per milligram of IV solution? A) 0.175 U/milligram B) 0.2 U/milligram C) 0.25 U/milligram D) 0.5 U/milligram Ans: B Feedback: The problem tells us there is 100 U/500 milligram. To determine how many units are in each milligram, divide both numbers by 500: 100 U 500/500 milligram 500 = 0.2 U/1 milligram. 15. The patient drinks 18 ounces of intake will the nurse document? fluid at lunchtime. How many milliliters of A) 1.7 mL B) 0.6 mL C) 540 mL D) 54 mL Ans: C Feedback: 1 ounce = 30 mL. Using the ratio-and-proportion method: 1 oz/30 mL = 18 oz/X. Cross-multiply to determine the patient drank 540 mL of fluid. 16. The pedia physician 0.03 mg/k the nurse tric nurse is caring for a child who weighs 44 pounds. The has ordered methylprednisolone sodium succinate (Solu-Medrol), g/d IV in normal saline. How many milligrams of medication will prepare? A) 6.5 B) 6 C) 0.65 D) 0.6 Ans: D Feedback: First convert the childs weight to kilograms by dividing 44 pounds by 2.2 kg/1 pound = 20 kg. Multiply the dosage times the childs weight: 20 kg 0.03 mg/kg/d = 0.6 mg/d 17. The nurse is to infuse 100 mL of 5% dextrose and water solution containing an IV antibiotic over 30 minutes. The infusion set delivers 10 gtt/mL. How many drops per minute will the nurse administer? A) 33 gtt/min B) 30.3 gtt/min C) 30 gtt/min D) 3 gtt/min Ans: A Feedback: Use the following ratio to determine how many drops of fluid to administer per minute: Using the information from this problem: Because it is not possible to deliver 0.3 drops, round 33.3 to 33 gtt/min. 18. An 80-year-old patient with internal bleeding is admitted through the emergency room after a motor vehicle accident. The physician has ordered 2 units of packed red blood cells (1 unit is 250 mL) to infuse over 1 hour each. The drip rate on the blood administration set is10 gtt/mL. The nurse administers how many drops per minute to infuse the blood as ordered? A) 47 B) 42 C) 37 D) 32 Ans: B Feedback: Use the following ratio to determine how many drops of fluid to administer per minute: Using the information from this problem: Because it is not possible to deliver 0.7 of a drop, round 41.7 to 42 minutes 19. The physician prescribes 250 mg of a drug. The information on the drug vial says the concentration is 500 mg/mL. How much of the drug will the nurse prepare? A) 0.25 mL B) 0.33 mL C) 0.5 mL D) 0.75 mL Ans: C Feedback: To determine amount to prepare: 500 mg/1 mL = 250 mg/X. Cross-multiply to determine the nurse will prepare 0.5 mL. 20. An 81-year-old patient with congestive heart failure has been sent to a cardiologist who prescribes digoxin (Lanoxin) 0.125 mg PO every morning. The pharmacy dispenses pills that contain 0.25 mg of Lanoxin. How many pills should the nurse teach the patient to take every morning? A) 2 B) 1.5 C) 1 D) 0.5 Ans: D Feedback: 0.25 mg/1 tablet = 0.125 mg dose/X. Cross-multiply to determine 0.5 or 1/2 of a tablet is to be taken daily. The nurse may request the pharmacy dispense a different concentration to prevent the patient from having to cut the tablet in half. 21. A patient with diabetic ketoacidosis is to receive a continuous infusion of regular insulin. The physician orders 1 L of 5% dextrose and water solution to run at 150 mL/h once the patients blood glucose has reached 250 mg/dL. The drip factor of the tubing is 15 gtt/mL. How many drops per minute will the nurse deliver? A) 0.25 gtt/min B) 62 gtt/min C) 37 gtt/min D) 250 gtt/min Ans: C Feedback: Use the following ratio to determine how many drops of fluid to administer per minute: Using the information from this problem: 37.5 can be rounded to 37 or 38 gtt/min. 22. A patient has orders to receive 2 L of IV fluid over a 24-hour period with this amount to be infused in the first 10 hours of treatment. How many milliliters per hour will the nurse administer during the first 10 hours of the infusion? A) 50 mL/h B) 100 mL/h C) 83 mL/h D) 200 mL/h Ans: B Feedback: of the 2 liters is to infuse in the first 10 hours. of 2 = 1 L; 1 L = 1,000 mL to infuse over 10 hours = 100 mL/L. 23. A patient is going to have bowel surgery in the morning. The physician orders 500 mL of GoLytely PO to be administered at 5 PM this evening. How many liters will the nurse administer? A) 1 B) C) D) Ans: C Feedback: Cross-multiply to learn 500 mL = or 0.5 L. 24. The patient returns from the postanesthesia care unit (PACU) with the following order: morphine 3 mg IV every 2 hours as needed for relief of pain. The vial reads morphine, 4 mg/mL. How many milliliters of morphine will the nurse administer? A) 1 mL B) 0.75 mL C) 0.5 mL D) 0.25 mL Ans: B Feedback: Using the figures from this problem: 4 mg/1 mL = 3 mg/X. Cross-multiply yielding 0.75 mL. 25. A patient is admitted with a deep vein thrombosis in his or her left calf. The physician orders Heparin, 7,500 units subcutaneously every 12 hours. The medication vial reads Heparin, 10,000 units/mL. How many milliliters does the nurse administer? A) 0.5 mL B) 0.75 mL C) 1 mL D) 1.25 mL Ans: A Feedback: Using the information supplied by the problem: 10,000 units/1 mL = 7,500/ X. Cross-multiplying yields X = 0.75 mL for each dose. 26. A patient with an acute myocardial infarction is admitted to the coronary care unit. The physician has ordered heparin 25,000 units in 250 mL normal saline to infuse at a rate of 600 units/h. The nurse sets the infusion pump to deliver how many milliliters in an hour? A) 8 B) 7 C) 6 D) 5 Ans: C Feedback: First determine the number of units per mL = 25,000 units/250 mL = 100 units/1 mL. Next use the ratio and proportion method to determine the number of milliliters needed to supply 600 units/h. 100 units/1 mL = 600 units/X. Cross-multiplying yields 6 mL needed every hour to supply the required dose. 27. The physician has ordered 30 mg of Demerol IM for relief of a severe migraine headache. The package insert reads meperidine hydrochloride (Demerol) 50 mg/mL. How many milliliters would the nurse administer? A) 1.6 B) 1 C) 0.6 D) 0.5 Ans: C Feedback: Filling in the information from the problem: 50 mg/1 mL = 30 mg/X mL. Cross-multiply yielding 0.6 mL required to administer a 30-mg dosage. 28. A patient is experiencing pain, so the physician orders codeine grain every 4 hours. How many milligrams of codeine would the nurse administer? A) 15 mg B) 30 mg C) 60 mg D) 120 mg Ans: B Feedback: The simplest way to convert measurements from one system to another is to set up a ratio and proportion equation. The ratio containing two known equivalent amounts is placed on one side of an equation, and the ratio containing the amount you wish to convert and its unknown equivalent is placed on the other side. 60 mg/1 grain = flexion range of motion. Cross-multiplying yields 30 mg. 29. A 79-year-old female patient presents at the clinic complaining of constipation for 1 week. The nurse practitioner prescribes Milk of Magnesia 2 teaspoons by mouth as needed for relief of constipation. How many milliliter will the nurse administer? A) 30 mL B) 7.5 mL C) 10 mL D) 15 mL Ans: C Feedback: Use the ratio and proportion method to convert from household system to metric system. 5 mL/1 tsp = X mL/2 tsp. Cross-multiplying yields 10 mL = 2 tsp. 30. A patient has orders to receive 3,000 mL of IV fluid at a rate of 150 mL/h. If the infusion starts at 08:00, when would it be finished? A) 20:00 B) 23:00 C) 01:00 D) 04:00 Ans: D Feedback: Amount of fluid to infuse = 3,000 mL; rate of infusion = 150 mL. 3,000 150 mL = 20 hours to infuse. There are 24 hours in a day 20 hours = 4 hours. The infusion will complete in 4 hours before 08:00 (08:00 04:00 = 04:00 hour) so the infusion completes at 04:00. 31. The nurse recognizes that what system is being used when seeing a medication ordered in minims? A) Apothecary B) Metric C) Household D) Avoirdupois Ans: A Feedback: The apothecary system used minims as the basic measure of liquid and is rarely used today. The metric system uses liters as the basic unit of measurement for fluid, while the household system uses ounces and the avoirdupois system uses ounces with a different conversion amount. 32. The nurse is calculating the patients intake and output record and converts ounces to milliliters. What systems is this nurse converting from and to? A) From household to metric B) From metric to household C) From household to apothecary D) From apothecary to Avoirdupois Ans: A Feedback: Ounces can be either household or Avoirdupois, although they are not equal measurements and milliliters is a metric measurement. Since Avoirdupois to metric is not an available choice, the correct answer is household to metric. 33. What is the most accurate method for the nurse to use when determining a pediatric dosage? A) A nomogram using body surface area B) Youngs rule C) Frieds rule D) Clarks rule Ans: A Feedback: The most accurate means of determining a pediatric drug dosage is a nomogram using body surface area because the weight and body surface area of two children who are of the same age can be significantly different. Youngs rule, Frieds rule, and Clarks rule are based on the childs age and the usual adult dosage and are rarely used today unless no other method will suffice for a specific drug. 34. The nurse determines a childs body surface area is 0.4 m2 and the average adult dosage of the medication is 500 mg. The medication is supplied in liquid form with 500mg/5 mL. How many milliliter will the nurse administer? A) 3.46 mL B) 1.73 mL C) 0.5 mL D) 12 mL Ans: A Feedback: The formula for calculating the childs dose using body surface area is surface area in m2 divided by 1.73 and then multiplied by adult dosage. Using the information supplied in this problem (0.4 1.73) 500 mg = 346 mg. Using the ratio and proportion method, 500 mg/5 mL = 346 mg/X mL; cross-multiplying yields 3.46 mL for the nurse to administer. 35. The physician orders acetaminophen (Tylenol) 15 mg/kg q4 hours PRN (as needed) for pain. The drug is supplied with 160 mg/5 mL. The infant weighs 12 pounds. How many milliliter of medication will the nurse administer? A) 2.5 mL B) 5.6 mL C) 12.4 mL D) 10.7 mL Ans: A Feedback: Begin by converting the infants weight to kilograms (12 pounds 2.2 pounds/ kg = 5.45 kg). The order says to give 15 mg/kg. To determine this childs dosage multiply weight and 15 mg/kg (5.45 kg 15 mg/kg = 81.75 mg/dose). Finally, use the ratio and proportion method to calculate how to prepare the medication: 160 mg/5 mL = 81.75 mg/X mL; cross-multiplying yields 2.5 mL. Chapter 13. Nervous System Medications 1. The central nervous system (CNS) cells, where the impulses for the sympathetic nervous system (SNS) originate, are located where? A) Cranium and sacral area of the spinal cord B) Hypothalamus and the medulla C) Nerve membranes D) Thoracic and lumbar sections of the spinal cord Ans: D Feedback: The SNS is also called the thoracolumbar system because the CNS cells, where the impulses for the SNS originate, are located in the thoracic and lumbar sections of the spinal cord. The parasympathetic nervous system (PNS) is called the craniosacral system because the CNS neurons, where the impulses for the PNS originate, are found in the cranium and the sacral area of the spinal cord. Alpha2-receptors are located on nerve membranes and the hypothalamus and medulla are located where the main nerve centers for the autonomic nervous system (ANS) are located. 2. The nurse assesses that the patient is having a sympathetic response when noting what manifestations? A) Decrease in sweating, decrease in respirations, and pupil constriction B) Decrease in heart rate and perfusion, and an increase in inflammatory reactions C) Increase in blood pressure, bronchodilation, and decreased bowel sounds D) Increased motility and secretions in the GI tract, and constriction of bronchi and pupils Ans: C Feedback: When stimulated, the sympathetic nervous system prepares the body to flee or to turn and fight (Figure 29.3). Cardiovascular activity increases, as do blood pressure, heart rate, and blood flow to the skeletal muscles. Respiratory efficiency also increases; bronchi dilate to allow more air to enter with each breath, and the respiratory rate increases. Pupils dilate to permit more light to enter the eye, to improve vision in darkened areas (which helps a person to see to fight or flee). Sweating increases to dissipate heat generated by the increased metabolic activity. 3. An anatomy and physiology instructor scratches chalk across the blackboard causing a screeching sound. Several students get a feeling like their hair is standing on end. This response is part of the sympathetic stress reaction and is called what? A) Diaphoresis B) Diuresis C) Piloerection D) Vasoconstriction Ans: C Feedback: The goose flesh or hair standing on end reaction that occurs as part of the stress response is called piloerection. Diaphoresis refers to sweating. Diuresis is the loss of water through the kidneys. Vasoconstriction is a muscle contraction in the blood vessel leading to blood vessel narrowing. 4. The nurse administers a drug to the patient whose heart rate is bradycardic aimed at increasing heart rate and myocardial activity. What adrenergic receptor is this drug stimulating? A) Alpha1 B) Alpha2 C) Beta1 D) Beta2 Ans: C Feedback: Beta1-receptors are found in cardiac tissue where they can stimulate increased myocardial activity and increased heart rate. Alpha1-receptors are found in blood vessels, in the iris, and in the urinary bladder. Alpha2- receptors are located on nerve membranes and act as modulators of norepinephrine release. Beta1-receptors are found in smooth muscle in blood vessels, in the bronchi, in the periphery, and in uterine muscle. 5. The nurse administers a drug that stimulates the parasympathetic nervous system. What physiological response would indicate the drug is working? A) Vasoconstriction B) Increased gastrointestinal (GI) motility C) Increased heart rate D) Pupil dilation Ans: B Feedback: When the parasympathetic nervous system is stimulated, the result is increased GI motility, decreased GI secretions, decreased heart rate, and pupillary constriction, which all result from stimulation of the sympathetic nervous system. 6. When the nurse administers a drug that stimulates the nicotinic receptors, what manifestation would indicate the drug is working? A) Increased gastrointestinal (GI) motility B) Decrease in heart rate C) Muscle contraction D) Pupil constriction Ans: C Feedback: Nicotinic receptors are located in the central nervous system (CNS), the adrenal medulla, the autonomic ganglia, and the neuromuscular junction. Stimulation of nicotinic receptors causes muscle contractions, autonomic responses, and release of norepinephrine from the adrenal medulla. Increased GI motility, decreased heart rate, and pupil constriction are the result of stimulation of the muscarinic receptors. 7. A young woman who lives alone comes home at night to find a man in her apartment. What body responses would be expected for the young woman? A) Increased blood pressure (BP), increased heart rate, and pupil dilation B) Decrease sweating, decreased BP, and increased heart rate C) Pupil constriction, increased respiratory rate, and decreased heart rate D) Increased sweating, decreased respiratory rate, and increased BP Ans: A Feedback: When stimulated by a stressful or fearful situation, the sympathetic nervous system (SNS) prepares the body to flee or to turn and fight. Cardiovascular activity increases as do blood pressure, heart rate, and blood flow to skeletal muscles. Respiratory rate increases, pupils dilate, and sweating increases. Decrease in sweating, BP, heart rate, respiratory rate, and pupil constriction indicate stimulation of the parasympathetic nervous system, which would not be stimulated by fear. 8. When there is stimulation of the sympathetic nervous system (SNS), blood is diverted away from the gastrointestinal (GI) tract. What might the nurse assess that would indicate this diversion of blood flow to the GI tract? A) Increased blood glucose levels B) Decreased bowel sounds C) Increased blood pressure D) Decreased immune reactions Ans: B Feedback: When blood is diverted away from the GI tract, bowel sounds decrease and digestion slows dramatically, sphincters are constricted, and bowel evacuation cannot occur. Increased blood glucose levels, elevated blood pressure, and decreased immune reaction are due to SNS stimulation but are not concerned with the GI tract. 9. A patient is being admitted to the floor following a motor vehicle accident. Because of the stressful nature of the event, the nurse anticipates the patient will continue to have a sympathetic stress reaction during the postoperative period. When monitoring the patients serum electrolytes, what will the nurse closely monitor? A) Increased calcium B) Decreased potassium C) Increased chloride D) Decreased sodium Ans: B Feedback: Aldosterone, also released with adrenal stimulation, retains sodium and water and causes the excretion of potassium in the urine. As water is retained, sodium is also retained, therefore increasing serum levels. Chloride levels are unlikely to change significantly. Calcium is not involved. 10. What does the body require in order to produce acetylcholine? A) Tyramine B) Tyrosine from the diet C) Choline from the diet D) Bilirubin from the liver Ans: C Feedback: Acetylcholine (ACh) is an ester of acetic acid and an organic alcohol called choline. Cholinergic nerves use choline, obtained in the diet, to produce ACh. Tyramine and tyrosine are associated with norepinephrine production. Bilirubin is not a neurotransmitter. 11. Which of these is a neurotransmitter? A) Calcium B) Cholinesterase C) Acetylcholine (ACh) D) Monoamine oxidase Ans: C Feedback: The last step in the production of the neurotransmitter involves choline acetyltransferase, an enzyme that is also produced within cholinergic nerves. Just like norepinephrine, the ACh is produced in the nerve and travels to the end of the axons, where it is packaged into vesicles. Calcium is an electrolyte and not a neurotransmitter. Cholinesterase breaks down acetylcholine and is an enzyme, not a neurotransmitter. Monoamine oxidase is an enzyme that breaks down norepinephrine. 12. Neurons that use acetylcholine as its neurotransmitter are what type of neurons? A) Cholinergic B) Dopaminergic C) GABA-ergic D) Serotonergic Ans: A Feedback: Neurons that use ACh as their neurotransmitter are called cholinergic neurons. Other options are incorrect. 13. The nursing student learns that the hypothalamus serves what purpose? A) Causes the secretion of adrenocorticotropic hormone (ACTH) B) Controls voluntary movement C) Secretes norepinephrine D) Helps maintain red blood cell production Ans: A Feedback: The hypothalamus causes the secretion of ACTH, leading to a release of the adrenal hormones including cortisol, which suppresses the immune and inflammatory reactions to preserve energy that otherwise, might be used by these activities. The hypothalamus does not have a role in controlling voluntary movement. The adrenal glands secrete norepinephrine. Red blood cells are produced in the bone marrow and are stimulated to produce the cells by epoetin secreted by the kidney. 14. Central nervous system drugs bind to receptors embedded in the cell membranes of neurons. Cholinergic receptors have been classified as what? A) Muscarinic B) Hormones C) Enzymes D) Proteins Ans: A Feedback: Cholinergic receptors or acetylcholine receptors are found on organs and muscles. They have been classified as muscarinic receptors and nicotinic receptors. Cholinergic receptors are not classified as hormones, enzymes, or proteins. 15. What helps to prevent overstimulation of effector sites on nerve membranes? A) Alpha1-receptors B) Alpha2-receptors C) Beta1-receptors D) Beta2-receptors Ans: C Feedback: Alpha2-receptors are located on nerve membranes and act as modulators of norepinephrine release. When norepinephrine is released from a nerve ending, it crosses the synaptic cleft to react with its specific receptor site. Some of it also flows back to react with the alpha-receptor on the nerve membrane. This causes a reflex decrease in norepinephrine release. In this way, the Alpha2-receptor helps to prevent overstimulation of effector sites. 16. The sympathetic nervous system (SNS) is associated with a fight-or-flight reaction. What reaction is the parasympathetic nervous system is associated? A) Recover and repair B) Respond and return C) Rest and digest D) Calm and peace Ans: C Feedback: Although the SNS is associated with the stress reaction and expenditure of energy, the parasympathetic nervous system is associated with activities that help the body to store or conserve energy, a rest-and-digest response. 17. The control systems of the body act in many ways to maintain homeostasis. These homeostatic control systems regulate the functions of the cell, integrate the functions of different organ systems, and do what else? A) Control vital functions B) Feed cells under stress C) Act on invading organisms D) Shut down the body at death Ans: A Feedback: In many areas, the parasympathetic nervous system works in opposition to the sympathetic nervous system. This allows the autonomic system to maintain a fine control over vital functions. This is a homeostatic control system. Homeostatic control systems do not feed cells when they are under stress, they do not act on invading organisms, and do they shut down the body at death. 18. The nurse administers a medication that stimulates the parasympathetic nervous system (PNS). What manifestations would indicate the medication is working? (Select all that apply.) A) Hyperactive bowel sounds B) Increased saliva production C) Elevated heart rate D) Urinary incontinence E) Constricted pupils Ans: A, B, E Feedback: PNS stimulation results in increased motility and secretions in the gastrointestinal (GI) tract to promote digestion and absorption of nutrients: decreased heart rate and contractility to conserve energy and provide rest for the heart; constriction of the bronchi, with increased secretions; relaxation of the GI and urinary bladder sphincters, allowing evacuation of waste products; pupillary constriction, which decreases the light entering the eye and decreases stimulation of the retina. While urinary sphincters relax, they do not lose control so incontinence would not be an expected manifestation. 19. The nurse administers a parasympathetic stimulator that only stimulates nicotinic receptors. What effects would the nurse expect to assess? (Select all that apply.) A) Muscle contraction B) Slowing heart rate C) Increased bladder contraction D) Signs and symptoms of a stress reaction E) Release of epinephrine from adrenal medulla Ans: A, D, E Feedback: Stimulation of nicotinic receptors causes muscle contractions, autonomic responses such as signs and symptoms of a stress reaction, and release of norepinephrine and epinephrine from the adrenal medulla. Stimulation of muscarinic receptors causes pupil constriction, increased gastrointestinal (GI) motility and secretions (including saliva), increased urinary bladder contraction, and a slowing of the heart rate. 20. Neurotransmitters are small molecules that exert their actions through specific proteins, called receptors, embedded in the postsynaptic membrane. Where are neurotransmitters synthesized? A) In the dendrite terminal B) In the presynaptic junction C) In the postsynaptic junction D) In the axon terminal Ans: D Feedback: Norepinephrine is made by the nerve cells using tyrosine, obtained in the diet. Dihydroxyphenylalanine (dopa) is produced by a nerve, using tyrosine from the diet and other chemicals. With the help of the enzyme dopa decarboxylase, the dopa is converted to dopamine, which in turn is converted to norepinephrine in the axon terminals of adrenergic cells. The norepinephrine then is stored in granules or storage vesicles within the cell. 21. The patient is undergoing chronic stress and has a prolonged sympathetic response. What type of drug could this patient receive to reduce the sympathetic response? (Select all that apply.) A) A drug that reduces sympathetic response B) A drug that increases sympathetic response C) A drug that reduces parasympathetic response D) A drug that increases parasympathetic response E) A drug that reduces central nervous system (CNS) response Ans: A, D, E Feedback: Decreasing sympathetic response would reduce the stress response, whereas increasing parasympathetic response would have the same effect. Although not optional, a medication that slowed down the entire nervous system would also work. In many areas, the parasympathetic nervous system works in opposition to the sympathetic nervous system (SNS). This allows the autonomic system to maintain a fine control over internal homeostasis. For example, the SNS increases heart rate, whereas the parasympathetic nervous system decreases it. Thus, the autonomic nervous system can influence heart rate by increasing or decreasing sympathetic activity or by increasing or decreasing parasympathetic activity. 22. Where are Alpha1-receptors found? (Select all that apply.) A) Blood vessels B) The iris C) Nerve membranes D) Urinary bladder E) Stomach sphincters Ans: A, B, D Feedback: Alpha1-receptors are found in blood vessels, in the iris, and in the urinary bladder. Alpha2-receptors are located on nerve membranes. Option E is a distracter. 23. The nurse administers a drug to treat hypertension that causes vasodilation of blood vessels. What is the drug stimulating? A) Alpha1-receptors B) Alpha2-receptors C) Beta1-receptors D) Beta2-receptors Ans: C Feedback: In blood vessels, beta2 stimulation leads to vasodilation. Stimulation of Alpha1-receptors causes vasoconstriction. Beta1 and Alpha2-receptors are not involved with blood vessels 24. The nurse administers a drug that stimulates beta2 receptors. What type of health condition would this drug treat? A) Heart disease B) High lipid levels C) Diabetes D) Respiratory disease Ans: D Feedback: Beta2-receptors are found in the smooth muscle in blood vessels, in the bronchi, in the periphery, and in uterine muscle. Beta2-receptors also cause dilation in the bronchi. Beta1-receptor stimulation would improve some heart disease and are responsible for increased lipolysis. Because beta2- receptors increase release of glucagon and the breakdown of glycogen, increasing serum glucose levels, stimulation of these receptors would exacerbate diabetes. 25. What is another name for the parasympathetic nervous system? A) Craniosacral system B) Cephalocaudal system C) Preganglionic system D) Thoracolumbar system Ans: A Feedback: The parasympathetic system is sometimes called the craniosacral system because the central nervous system neurons that originate parasympathetic impulses are found in the cranium (one of the most important being the vagus or tenth cranial nerve) and in the sacral area of the spinal cord. The thoracolumbar system is the sympathetic nervous system. The other options are distractors. 26. The nurse administers a medication that stimulates the muscarinic receptors. What types of manifestations will the nurse assess in this patient that indicate the drug is working? A) Pupil dilation B) Increased activity of bowel sounds C) Increased heart rate D) Muscle contractions Ans: B Feedback: Stimulation of muscarinic receptors increases gastrointestinal (GI) motility which would cause increased activity of bowel sounds. Other effects include pupil constriction, increased urinary bladder contraction, and a slowing of the heart rate. Stimulation of nicotinic receptors cause muscle contractions. 27. When muscarinic receptors are stimulated, what happens physiologically in the body? (Select all that apply.) A) Pupil constriction B) Pupil dilation C) Increased secretions D) Increased bladder contraction E) Increased heart rate Ans: A, C, D Feedback: Stimulation of muscarinic receptors causes pupil constriction, increased gastrointestinal (GI) motility and secretions (including saliva), increased urinary bladder contraction, and a slowing of the heart rate. Pupils are constricted, not dilated and heart rate slows, it does not increase. 28. The nurse administers a drug that stimulates the nicotinic receptors. What assessment findings would indicate effectiveness of the drug? (Select all that apply.) A) Muscle contractions B) Release of norepinephrine from the adrenal medulla C) Signs and symptoms of a stress reaction D) Urinary incontinence E) Hyperactive bowel sounds Ans: A, B, C Feedback: Stimulation of nicotinic receptors causes muscle contractions, autonomic responses such as signs and symptoms of a stress reaction, and release of norepinephrine and epinephrine from the adrenal medulla. Urinary incontinence would not be associated with stimulation of the nicotinic receptors and increased bowel activity would result from muscarinic receptors. 29. The body makes norepinephrine by using what from the diet? A) Tyrosine B) Thiamine C) Tryptophan D) Trichonosis Ans: A Feedback: Norepinephrine is made by adrenergic nerves using tyrosine from the diet. Therefore the other options are incorrect. 30. The nurse administers a drug that causes vasoconstriction, contracted piloerection muscles, pupil dilation, closure of salivary sphincter, and male sexual emission. What receptor is this drug stimulating? A) Alpha1-receptors B) Alpha2-receptors C) Beta1-receptors D) Beta2-receptors Ans: A Feedback: Stimulation of Alpha1-receptors results in vasoconstriction of blood vessels, increased peripheral resistance with increased blood pressure, contracted piloerection muscles, pupil dilation, thickened salivary secretions, closure of the urinary bladder sphincter, and male sexual emission. None of the other receptors, when stimulated, would have this effect. 31. The nurse accompanies the physician into the patients room and remains after the patient is told he has cancer and it is likely to be terminal. The patients respirations become rapid and deep, pupils dilate, and measurement of vital signs indicates the patients heart rate and blood pressure are elevated. What type of response is the nurse assessing? A) Sympathetic nervous system (SNS) response B) Parasympathetic nervous system (PNS) response C) Muscarinic receptor stimulation response D) Nicotinic receptor stimulation response Ans: A Feedback: When stimulated, the SNS prepares the body to flee or to turn and fight. Cardiovascular activity increases, as do blood pressure, heart rate, and blood flow to the skeletal muscles. Respiratory efficiency also increases; bronchi dilate to allow more air to enter with each breath, and the respiratory rate increases. Pupils dilate to permit more light to enter the eye to improve vision in darkened areas. PNS would lower heart rate and blood pressure and would constrict pupils. Stimulation of muscarinic receptors cause pupil constriction, increased gastrointestinal (GI) motility and secretions (including saliva), increased urinary bladder contraction, and a slowing of the heart rate. Stimulation of nicotinic receptors causes muscle contractions, autonomic responses such as signs and symptoms of a stress reaction, and release of norepinephrine and epinephrine from the adrenal medulla. 32. Muscarinic and nicotinic receptors are part of what system? A) The limbic system B) The reticular activating system C) The sympathetic nervous system D) The parasympathetic nervous system Ans: D Feedback: Cholinergic receptors or acetylcholine receptors of the parasympathetic nervous system are found on organs and muscles. They have been classified as muscarinic receptors and nicotinic receptors. Because these receptors are part of the parasympathetic nervous system, all other options are incorrect. 33. After the effector cell has been stimulated by acetylcholine (ACh), what enzyme stops this stimulation and allows the effector membrane to repolarize? A) Decarboxylase B) Norepinephrine C) Acetylcholinesterase D) Catecholamine Ans: C Feedback: After the effector cell has been stimulated by ACh, stimulation of the receptor site must be terminated and destruction of any ACh must occur. The destruction of ACh is carried out by the enzyme acetylcholinesterase. This enzyme reacts with the ACh to form a chemically inactive compound. The breakdown of the released ACh is accomplished in 1/1,000 second, and the receptor is vacated, allowing the effector membrane to repolarize and be ready for the next stimulation. Dopa decarboxylase is an enzyme that converts dopa to dopamine. Norepinephrine is a catecholamine as are dopamine, serotonin, and epinephrine. 34. The nurse is teaching a class about the autonomic nervous system for critical care nurses. What statements, if made by the nurse during the class, are accurate? (Select all that apply.) A) Adrenergic receptors respond to norepinephrine. B) Adrenergic receptors are part of the sympathetic nervous system. C) Cholinergic receptors are part of the parasympathetic nervous system. D) Cholinergic receptors include alpha- and beta- receptors. E) Cholinergic and adrenergic receptors are part of the autonomic nervous system. Ans: A, B, C, E Feedback: The sympathetic nervous system contains the adrenergic receptors that respond to norepinephrine and include alpha- and beta-receptors. The parasympathetic nervous system contains the cholinergic receptors including the muscarinic and nicotinic receptors that respond to acetylcholine. Together the sympathetic and parasympathetic systems, including cholinergic and adrenergic receptors, make up the autonomic nervous system. Option D is incorrect. 35. What statement correctly explains the nerve impulse transmission? A) The impulse travels from the central nervous system (CNS) to the preganglionic neuron to the ganglia to the postganglionic neuron to the neuroeffector cells. B) The impulse travels from the preganglionic neuron to the CNS to the ganglia to the postganglionic neuron to the neuroeffector cells. C) The impulse travels from the preganglionic neuron to the ganglia to the postganglionic neuron to the CNS to the neuroeffector cell. D) The impulse travels from the CNS, to the neuroeffector cells, to the preganglionic neuron to the ganglia to the postganglionic neuron. Ans: A Feedback: The autonomic nervous system does not send impulses directly to the periphery. Instead, axons from CNS neurons end in ganglia, or groups of nerve bodies that are packed together, located outside of the CNS. These ganglia receive information from the preganglionic neuron that started in the CNS and relay that information along postganglionic neurons. The postganglionic neurons transmit impulses to the neuroeffector cellsmuscles, glands, and organs. Chapter 14. Eye and Ear Medications MULTIPLE CHOICE 1. The nurse is preparing a patient for an ophthalmic examination. Which action occurs when the nurse instills eye drops to produce mydriasis? a Drying of tears in the eyes . b Extreme dilation of the pupil . c Opening of the canal of Schlemm . d Paralysis of the ciliary muscle . ANS: B Dilating the eye before eye examinations allows for better visualization of the interior of the globe. Anticholinergic drugs may produce drying of tears in the eye as an adverse effect of use. Obstruction of the canal of Schlemm results in glaucoma. Paralysis of the ciliary muscle is cycloplegia. DIF: Cognitive Level: Application REF: p. 678 OBJ: 6 TOP: Nursing Process Step: Implementation MSC: NCLEX Client Needs Category: Physiological Integrity 2. Which type of medication would be used to dilate the pupils before an eye examination? a Osmotics . b Adrenergic agent . c Beta adrenergic agent . d Corticosteroid . ANS: B Adrenergic agents are sympathomimetic. They cause pupil dilation, increased outflow of aqueous humor, vasoconstriction, relaxation of ciliary muscle, and a decrease in the formation of aqueous humor. Adrenergic agents are used to lower intraocular pressure (IOP) in open angle glaucoma, relieve congestion and hyperemia, and produce mydriasis for ocular examinations. Osmotics are given to reduce IOP. Beta adrenergics are given for the treatment of asthma. Corticosteroids are given for acute allergic reactions in the eye. DIF: Cognitive Level: Comprehension REF: p. 688 OBJ: 4 TOP: Nursing Process Step: Evaluation MSC: NCLEX Client Needs Category: Physiological Integrity 3. Which medication is used to produce miosis following a diagnostic procedure? a Pilocarpine (Pilocar) . b Mannitol (Osmitrol) . c Atropine (Isopto Atropine) . d Epinephrine (EpiPen) . ANS: A Pilocarpine is a direct acting cholinergic agent that is used to counter the effects of mydriatic and cycloplegic agents after surgery or eye ophthalmoscopic examinations. Cholinergic agents also reduce IOP in glaucoma patients by widening the filtration angle that permits outflow of aqueous humor. Mannitol is an osmotic diuretic given to decrease ocular pressure by drawing aqueous humor from the eye. Atropine is a mydriatic agent given to dilate the pupil. Epinephrine may be used to treat certain types of glaucoma. DIF: Cognitive Level: Comprehension REF: p. 687 OBJ: 6 TOP: Nursing Process Step: Evaluation MSC: NCLEX Client Needs Category: Physiological Integrity 4. Which discharge instruction will the nurse include for a patient sent home from the clinic who is taking an adrenergic ophthalmic solution for an acute inflammation? a Headaches and eye pain are adverse effects to be reported to the health care . provider immediately. b Mouth dryness should be reported immediately. . c Avoid driving or operating machinery until blurring subsides. . d Halos or yellow rings around objects will be seen while taking this medication. . ANS: C Adrenergic agents cause the smooth muscle of the ciliary body and iris to relax, producing mydriasis. Blurred vision will temporarily occur until the patient can adjust to the increased light coming into the eyes. Activities such as driving or operating machinery should be avoided until vision stabilizes. Sunglasses help reduce the brightness. These are common, mild adverse effects and usually resolve with continued therapy. Mouth dryness is a common adverse effect. Halos are a sign of acute closed angle glaucoma. DIF: Cognitive Level: Application REF: pp. 688-689 OBJ: 4 | 6 TOP: Nursing Process Step: Implementation MSC: NCLEX Client Needs Category: Health Promotion and Maintenance 5. What is the action of timolol maleate (Timoptic), a beta adrenergic blocking agent? a Draws aqueous humor from the eye into the circulatory network . b . Increases the production of aqueous humor c . Increases the outflow of aqueous humor d . Decreases the production of aqueous humor ANS: D Timolol maleate is a beta adrenergic blocking agent used to reduce increased intraocular pressure. The exact mechanism of action of these medications is unknown, but they are believed to reduce the production of aqueous humor. Timolol is believed to decrease production of aqueous humor. DIF: Cognitive Level: Comprehension REF: p. 689 OBJ: 6 TOP: Nursing Process Step: Evaluation MSC: NCLEX Client Needs Category: Health Promotion and Maintenance 6. What is the mechanism of action of osmotic agents when used to decrease IOP? a Promoting outflow of the aqueous humor into the tear ducts . b Increasing plasma osmolarity and drawing extracellular fluid into the blood . c Blocking production of aqueous humor . d Decreasing viscosity of the tears and allowing fluid to drain away from the eye . ANS: B Osmotic agents elevate the osmotic pressure of the plasma, causing fluid from the extravascular spaces to be drawn into the blood. The effect on the eye is reduction of volume of intraocular fluid, which produces a decrease in IOP. Osmotic agents do not promote flow of aqueous humor into tear ducts, block production of aqueous humor, or decrease viscosity of tears. DIF: Cognitive Level: Comprehension REF: pp. 683-684 OBJ: 6 TOP: Nursing Process Step: Implementation MSC: NCLEX Client Needs Category: Physiological Integrity 7. Which is a potential serious adverse effect associated with mannitol (Osmitrol)? a Bradycardia . b Fluid overload . c Anaphylaxis . d Fever . ANS: B Mannitol, an osmotic agent, acts on blood volume by pulling fluid from the tissue spaces into the general circulation (blood). Patients should be assessed at regularly scheduled intervals for signs and symptoms of fluid overload, pulmonary edema, or heart failure. Osmotic agents are not likely to cause bradycardia, anaphylaxis, or fever. DIF: Cognitive Level: Knowledge REF: p. 684 OBJ: 6 TOP: Nursing Process Step: Evaluation MSC: NCLEX Client Needs Category: Physiological Integrity 8. The nurse is caring for a patient immediately following a right sided trabeculectomy. When positioning this patient, the nurse will encourage a position. a prone . b right side lying . c left side lying . d Trendelenburg . ANS: C Following right sided trabeculectomy, the patient is usually positioned on the back or on the unoperated side. Prone position, right side lying, and Trendelenburg positions are not recommended after a trabeculectomy. DIF: Cognitive Level: Application REF: p. 682 OBJ: 4 TOP: Nursing Process Step: Implementation MSC: NCLEX Client Needs Category: Safe, Effective Care Environment 9. The nurse is assisting with applanation tonometry on a patient at the ophthalmologists office. The results indicate the patients reading to be 15 mm Hg. The nurse interprets this result as IOP. a decreased . b normal . c slightly increased . d severely increased . ANS: B Normal IOP using an applanation tonometer is 10 to 21 mm Hg. DIF: Cognitive Level: Analysis REF: p. 682 OBJ: 2 | 4 TOP: Nursing Process Step: Evaluation MSC: NCLEX Client Needs Category: Physiological Integrity MULTIPLE RESPONSE 10. Which statement(s) about aqueous humor is/are true? (Select all that apply.) a Bathes and feeds the lens, posterior surface of the cornea, and iris . b Maintains the iris color . c Drains out of the eye through drainage channels located near the junction of the . cornea and sclera d Manufactures fluid for tear production . e Flows out of the canal of Schlemm into the venous system of the eye . ANS: A, C, E Aqueous humor bathes and feeds the lens, posterior surface of the cornea, and iris. After it is formed, the fluid flows forward between the lens and the iris into the anterior chamber. Aqueous humor drains out of the eye through drainage channels located near the junction of the cornea and sclera. Aqueous humor drains into a meshwork leading into the canal of Schlemm and into the venous system of the eye. Eye color is not dependent on the aqueous humor. Tears are produced by the lacrimal glands in the upper and lower eyelids. DIF: Cognitive Level: Comprehension REF: pp. 678-679 OBJ: 1 | 2 TOP: Nursing Process Step: Assessment MSC: NCLEX Client Needs Category: Physiological Integrity 11. A factory worker had a chemical inadvertently splashed into his right eye. An eyewash was used at the work site. Which nursing assessment(s) would be important to include? (Select all that apply.) a Visual acuity . b Presence of pain, blurred or halo vision, or lack of vision . c Type of chemical . d Presence of nystagmus . e Presence of contacts or use of eyeglasses . ANS: A, B, C, E Important nursing assessments would include visual acuity and baseline vital signs. These assessments would also include subjective data such as pain, clarity, and acuity of vision; observation of any physical abnormalities of the eye or lid; pupil characteristics; drainage or excessive tearing; edema or redness; and interventions completed before arrival. Important nursing assessments would include the type and cause of injury and whether contacts or eyeglasses are worn. Nystagmus is involuntary, rhythmic, repeated oscillations of one or both eyes. The cause is unknown. It would be unrelated to chemical eye injury. DIF: Cognitive Level: Application REF: p. 681 OBJ: 4 TOP: Nursing Process Step: Assessment MSC: NCLEX Client Needs Category: Safe, Effective Care Environment 12. What information will the nurse include when instructing a patient on the correct method of instilling eye drops? (Select all that apply.) a With an infection, prevent cross contamination and use a separate source of . medication and droppers for each eye. b Wash hands before and after administration. . c Place the lid on the surface area as instructed to avoid contamination. . d Never touch the tip of the dropper or opening of the ointment container. . e Wipe eye from the outer to inner canthus. . ANS: A, B, C, D Separate medication sources should be used when an eye infection is present to avoid cross contamination. Washing of hands before and after instillation will prevent infection. The medication lid should not lie with the opening down on any surface. Any medications instilled into the eye should be sterile. The medication tip should not touch anything, including the eye, face, or fingers. Wipe the eye from the inner canthus outward. DIF: Cognitive Level: Application REF: pp. 679-680 OBJ: 5 TOP: Nursing Process Step: Implementation MSC: NCLEX Client Needs Category: Physiological Integrity 13. A patient recently diagnosed with glaucoma is to begin drug therapy with carbonic anhydrase inhibitors. For which assessment(s) would the nurse need to contact the health care provider? (Select all that apply.) a Electrolyte levels . b Any signs of gastric symptoms before initiating drug therapy . c Allergy to sulfonamides . d Patient history of menopause . e Elevated IOP levels . ANS: A, C Baseline electrolyte studies, weight, hydration data, vital signs, and mental status should be obtained before beginning drug therapy. Carbonic anhydrase inhibitors should be held and the health care provider notified if the patient is pregnant or allergic to sulfonamides. If the patient is experiencing gastrointestinal symptoms, the medication should be administered with milk or food. Menopause is unrelated to the condition of glaucoma or its treatment. Elevated IOP levels are associated with glaucoma and are an expected condition. They do not need to be reported to the health care provider. DIF: Cognitive Level: Application REF: pp. 685-686 OBJ: 4 | 5 TOP: Nursing Process Step: Implementation MSC: NCLEX Client Needs Category: Physiological Integrity 14. What is the purpose of administering a cycloplegic agent? (Select all that apply.) a Facilitate examination of the eye . b Facilitate surgery on the eye . c Cause pupillary dilation . d Paralyze the ciliary muscle . e Decrease the production of aqueous humor . ANS: A, B, D The eye is easier to examine in some cases if the ciliary muscle is paralyzed. Surgery on the eye is easier when the ciliary muscle is paralyzed. Cycloplegic agents are used to paralyze the ciliary muscle in preparation for examination or surgery. Drugs that cause pupillary dilation are called mydriatic agents. Cycloplegic agents do not decrease the production of aqueous humor. DIF: Cognitive Level: Comprehension REF: p. 691 OBJ: 6 TOP: Nursing Process Step: Implementation MSC: NCLEX Client Needs Category: Safe, Effective Care Environment 15. Which are important teaching points for the nurse to review with a patient recently diagnosed with open angle glaucoma? (Select all that apply.) a The disease will cause damage to the optic disc if left untreated. . b Symptoms are sudden and painful when the disease begins. . c Loss of peripheral vision is a common trigger for diagnosis. . d Total blindness may result if the glaucoma is not treated. . e Glaucoma is not a serious disease and will cause only mild inconvenience to the . patient. f Treatment is only necessary when symptoms are bothersome. . ANS: A, C, D IOP builds up and, if not treated, will damage the optic disc. Initially, the patient has no symptoms, but over the years, peripheral vision is gradually lost. If glaucoma is left untreated, total blindness may result. Patients with glaucoma are initially asymptomatic. Glaucoma is a serious disease that, if left untreated, may result in total blindness. If glaucoma is not treated, IOP builds up and will damage the optic disc. DIF: Cognitive Level: Comprehension REF: pp. 680-681 OBJ: 3 | 4 TOP: Nursing Process Step: Implementation MSC: NCLEX Client Needs Category: Physiological Integrity 16. What information will the nurse include when teaching the patient and family about postoperative care for a trabeculectomy? (Select all that apply.) a Use aseptic technique for all dressing changes and medication administration. . b Place the patient on the operated side. . c . Avoid heavy lifting. d . Redness in the eye, pain, and swelling are common occurrences after surgery. e . Avoid straining on defecation. ANS: A, C, D, E Teach the patient and family proper hygiene and eye care techniques to ensure that medications, dressings, and/or surgical wounds are not contaminated during necessary eye care. Explain and enforce activity and exercise restrictions. To prevent an increase in IOP, instruct the patient to avoid heavy lifting, straining on defecation, coughing, or bending and placing the head in a dependent position. Teach the patient and family about signs and symptoms of infections and when and how to report them to allow early recognition and treatment of possible infection. Straining increases IOP and should be avoided. The patient is positioned on the unoperated side or the back. DIF: Cognitive Level: Application REF: pp. 682-683 OBJ: 4 TOP: Nursing Process Step: Implementation MSC: NCLEX Client Needs Category: Physiological Integrity 17. The nurse is educating a patient about a newly prescribed cholinergic agent. When relaying common adverse effects of this type of medication, the nurse will include information about: (Select all that apply.) a conjunctival irritation. . b headache. . c salivation. . d hypotension. . e bradycardia. . ANS: A, B Conjunctival irritation and headache are common adverse effects of cholinergic agents. Salivation, hypotension, and bradycardia are systemic adverse effects of cholinergic agents and may indicate toxicity. Chapter 15. Endocrine System Medications 1. A patient suspected of having Cushings disease comes to the clinic. What drug might the nurse administer to test for adrenal function and responsiveness? A) Corticotropin B) Menotropins C) Thyrotropin alfa D) Chorionic gonadotropin Ans: A Feedback: Corticotropin (ACTH) and cosyntropin are used for diagnostic purposes to test adrenal function and responsiveness. Menotropin is a purified preparation of gonadotropins and is used as a fertility drug. Thyrotropin alfa is used as adjunctive treatment for radioiodine ablation of thyroid tissue remnants in patients who have undergone a near-total to total thyroidectomy for well-differentiated thyroid cancer and who do not have evidence of metastatic thyroid cancer. Chorionic gonadotropin acts like luteinizing hormone and stimulates the production of testosterone and progesterone. 2. A nurse is working at a fertility clinic. Today she is administering ganirelix acetate (Antagon) to a woman participating in a fertility research program. By what route is this drug administered? A) Orally B) Subcutaneously C) Intramuscularly D) IV Ans: B Feedback: Ganirelix acetate (Antagon) is administered subcutaneously and cannot be administered orally, intramuscularly, or intravenously. 3. A patient is taking leuprolide (Lupron) to treat prostatic cancer. The nurse caring for this patient is careful to monitor for what? A) Diarrhea B) Urinary retention C) Peripheral edema D) Increased appetite Ans: C Feedback: Peripheral edema is an identified adverse effect of leuprolide therapy. Constipation not diarrhea; urinary frequency not urinary retention; and anorexia not increased appetite are also identified adverse effects. 4. After administering somatropin (Saizen) to an 11-year-old patient with growth failure, what outcome would indicate that the drug should be stopped? A) Early sexual development B) Thyroid overactivity C) Closure of the epiphyses in long bones D) Gynecomastia Ans: C Feedback: Closure of the epiphyses is a sign that the drug should be stopped. Early sexual development, thyroid overactivity, and gynecomastia would not be associated with this drug. 5. The nurse administers desmopressin (DDAVP) to the patient to treat diabetes insipidus. What assessment finding would indicate to the nurse that the desmopressin is producing a therapeutic effect? A) Decreased urine output B) Decreased water reabsorption C) Increased plasma osmolarity D) Decreased blood volume Ans: A Feedback: Desmopressin produces its antidiuretic activity in the kidneys, causing the cortical and medullary parts of the collecting duct to become permeable to water, thereby increasing water reabsorption and decreasing urine formation. These activities reduce plasma osmolarity and increase blood volume. 6. A patient with diabetes insipidus is taking desmopressin (DDAVP). He or she is complaining of drowsiness, lightheadedness, and headache. What does the nurse suspect that he is experiencing? A) An allergic reaction B) Dehydration C) Depression D) Water intoxication Ans: D Feedback: The adverse effects associated with the use of desmopressin include water intoxication (drowsiness, light-headedness, headache, coma, convulsions) related to the shift to water retention and resulting electrolyte imbalance. An allergic reaction, dehydration, or depression would not be associated with these symptoms and desmopressin. 7. What drug would the nurse expect to administer to an AIDS patient with cachexia? A) Bromocriptine (Parlodel) B) Somatropin (Saizen) C) Desmopressin (DDAVP) D) Leuprolide (Lupron) Ans: B Feedback: Indications for somatropin therapy include cachexia, long-term treatment of children with growth failure associated with various deficiencies, girls with Turners syndrome, AIDS-related wasting, growth hormone deficiency in adults, and treatment of growth failure in children of small gestational age who do not achieve catch-up growth by 2 years of age. Bromocriptine mesylate is indicated for the treatment of Parkinsons disease, hyperprolactinemia associated with pituitary adenomas, female infertility associated with hyperprolactinemia, and acromegaly; and short-term treatment of amenorrhea or galactorrhea. Desmopressin is indicated for the treatment of neurogenic diabetes insipidus and hemophilia A. Leuprolide is used as an antineoplastic agent for treatment of specific cancers. 8. What symptoms will the nurse instruct the patient taking octreotide (Sandostatin) to report to the health care provider? A) Abdominal pain B) Alteration in consciousness C) Changes in vision D) Muscle cramps Ans: A Feedback: Octreotide and lanreotide have commonly been associated with the development of acute cholecystitis, cholestatic jaundice, biliary tract obstruction, and pancreatitis, which would present with abdominal pain, so patients should be taught to report this symptom. The drug is not associated with alteration in consciousness, changes in vision, or muscle cramps. 9. The nurse has been caring for a child who has been receiving growth hormone therapy for several years. When the child returns for evaluation following a sudden growth spurt, what nursing diagnosis will the nurse add to the plan of care? A) Disturbed body image B) Deficient knowledge regarding drug therapy C) Imbalanced nutrition: Less than body requirements related to metabolic changes D) Risk for disproportionate growth Ans: C Feedback: A child who is taking growth hormone may experience sudden growth, which will require increased nutritional intake, so it is important to include nutritional needs in the plan of care. More than likely an increase in caloric intake and nutrients will be necessary. Most children who are small for their age see growth as a positive thing and not a disturbed body image. After taking the drug for several years, the patient should have received adequate teaching from the nurse ot make deficient knowledge unlikely. The child should not be at risk for, but have a diagnosis of, disproportionate growth as the reason for taking the medication. 10. The nurse receives an order to administer leuprolide 5 mcg/kg subcutaneously to a child with precocious puberty. The child weighs 30 kg. What is the correct dosage for this child? A) 150 mcg B) 6 mcg C) 68 mcg D) 330 mcg Ans: A Feedback: Multiply the childs weight times the mg/kg: 30 5 = 150 mcg/dose. 11. The nurse is caring for a patient with infertility related to hyperprolactinemia. What drug would the nurse recognize was ordered to treat this problem? A) Bromocriptine mesylate B) Somatropin C) Leuprolide D) Desmopressin Ans: A Feedback: Bromocriptine mesylate is indicated for the treatment of female infertility associated with hyperprolactinemia. Somatropin is indicated for the treatment of growth failure, Turners syndrome, AIDS wasting and cachexia, and growth hormone deficiency in adults. Leuprolide is used as antineoplastic agent for treatment of specific cancers and for treatment of endometriosis and precocious puberty that results from hypothalamic activity. Desmopressin is used for the treatment of neurogenic diabetes insipidus, von Willebrands disease, hemophilia; and is currently being studied for the treatment of chronic autonomic failure. 12. A child weighing 14.5 kg has von Willebrands disease. How much desmopressin (DDAVP) should be administered? A) 4.4 mcg B) 9.6 mcg C) 10.3 mcg D) 21.1 mcg Ans: A Feedback: The normal dosage of desmopressin used to treat von Willebrands disease is 0.3 mcg/kg. Multiply this dosage times the childs weight: 14.5 kg 0.3 g = 4.4 g. 13. What is the purpose of releasing hormones secreted by the hypothalamus? A) Stimulating or inhibiting release of hormones from the pituitary B) Stimulating organs within the body to secrete hormones C) Allowing the secretion of hormones from the hypothalamus D) Stimulating other glands to release hormones Ans: A Feedback: The hypothalamus uses various hormones or factors to either stimulate or inhibit the release of hormones from the anterior pituitary. These do not stimulate other organs, the hypothalamus, or other glands to release hormones. 14. A 48-year-old patient with acromegaly is not a candidate for other therapy. What medication, administered subcutaneously, would the nurse caring for the patient expect the physician to order? A) Gonadorelin hydrochloride (Factrel) B) Octreotide (Sandostatin) C) Nafarelin (Synarel) D) Gonadorelin acetate (Lutrepulse) Ans: B Feedback: Octreotide (Sandostatin) is a treatment for acromegaly in adults who are not candidates for, or cannot tolerate, other therapy. Gonadorelin hydrochloride, nafarelin, and gonadorelin acetate are not indicated for treating acromegaly. 15. A patient is brought to the emergency department after a motor vehicle accident. The patient is hemorrhaging, indicating that which hormone is being secreted to restore blood volume? A) Growth hormone (GH) B) Follicle-stimulating hormone (FSH) C) Antidiuretic hormone (ADH) D) Adrenocorticotropic hormone (ACTH) Ans: C Feedback: ADH possesses antidiuretic, hemostatic, and vasopressor properties. During hemorrhage, GH, FSH and ACTH are not involved in blood volume restoration. 16. The nurse administers somatropin to a child with impaired growth due to a deficiency of endogenous growth hormone during what period of growth and development? A) Before the start of elementary school B) Any time before age 18 C) Before the child reaches 5 feet in height D) Before epiphyses close Ans: D Feedback: Somatropin is contraindicated in the presence of closed epiphyses so the drug can be given at any time before closure of the epiphyses. Age and height are not an indication impacting when the drug is given so long as the epiphyses remain open. 17. The nurse would teach a patient receiving octreotide for acromegaly of the importance of baseline and periodic what? A) Ultrasound evaluation of the gallbladder B) Magnetic resonance imaging (MRI) of the brain C) Serum glucose levels D) Complete blood counts Ans: A Feedback: Arrange for baseline and periodic ultrasound evaluation of the gallbladder for patients receiving octreotide because common adverse effects of the drug are acute cholecystitis, cholestatic jaundice, biliary tract obstruction, and pancreatitis. A patient taking octreotide does not need baseline and periodic MRIs of the brain, serum glucose levels, or complete blood counts. 18. The nurse is caring for a patient with neurogenic diabetes insipidus and administers what drug to treat the condition? A) Dexamethasone (Decadron) B) Desmopressin (DDAVP, Stimate) C) Methylprednisolone (Solu-Medrol) D) Physostigmine (Antilirium) Ans: B Feedback: Synthetic preparations of antidiuretic hormone (ADH), which are purer and have fewer adverse effects, are used to treat diabetes insipidus. Only one ADH preparation is currently available, desmopressin. Dexamethasone, methylprednisolone, and physostigmine would not be indicated for treatment of this disorder. 19. The nurse transcribes an order for menotropin (Pergonal), which will be administered to have what effect? A) Promote development of secondary sex characteristics B) Induce ovulation C) Promote bone growth D) Treat diabetes insipidus Ans: B Feedback: Menotropin (Pergonal) is used as fertility drug to stimulate ovulation and spermatogenesis. Pergonal is not used to promote the development of secondary sex hormones, to promote bone growth, or to treat diabetes insipidus. 20. The nurse administers desmopressin to treat a patient with diabetes insipidus. Assessment of what laboratory studies would indicate the drug is working? A) Increased serum sodium levels B) Increased red blood cell count C) Decreased urine specific gravity D) Reduced urine glucose levels Ans: A Feedback: Patients with diabetes insipidus (DI) produce large amounts of dilute urine with a decrease in serum sodium levels. Administering desmopressin would reduce urine output and allow sodium levels to rise. Urine specific gravity would increase as the urine production slows and urine becomes more concentrated. There is no impact on red blood cell production with diabetes insipidus or its treatment. The urine of a patient with DI does not contain glucose. 21. The nurse would administer desmopressin (DDAVP) cautiously, with careful monitoring, to the patient with what co-morbidity? (Select all that apply.) A) Hyponatremia B) Asthma C) Severe renal dysfunction D) Gastrointestinal (GI) disease E) Epilepsy Ans: A, B, E Feedback: Caution should be used with any known vascular disease because of its effects on vascular smooth muscle, epilepsy, asthma, and with hyponatremia, which could be exacerbated by the effects of the drug. The drug is contraindicated and should not be used in patients with severe renal dysfunction. GI disease is not a caution or contraindication. 22. The nurse is caring for a 25-year-old female patient who is receiving chorionic gonadotropin alfa. What would be an appropriate nursing diagnosis for this patient? A) Anxiety related to injection of medication B) Acute pain related to need for injections C) Imbalanced nutrition: More than body requirements D) Evaluate effectiveness of the teaching plan Ans: B Feedback: Nursing diagnoses related to drug therapy might include acute pain related to need for injections. Not all patients are anxious at the thought of an injection so more information would be needed. The patient is more likely to have reduced nutritional intake because of gastrointestinal (GI) adverse effects. Evaluating the effectiveness of the teaching plan is not a nursing diagnosis. 23. When providing patient teaching to the family of a 12-year-old child receiving Somatropin, the nurse stresses the need to notify prescriber if what manifestation occurs? A) Severe hip or knee pain B) Development of a bruise C) Severe hypertension D) Tachycardia Ans: A Feedback: The adverse effects that most often occur when using a growth hormone (GH) include the development of antibodies to GH and subsequent signs of inflammation and autoimmune-type reactions, such as swelling and joint pain, and the endocrine reactions of hypothyroidism and insulin resistance. It would not be necessary to notify the physician for development of a bruise. The health care provider should always be notified if a patient develops severe hypertension or tachycardia but this would not be related to administration of somatropin so it would not be included in drug teaching. 24. What medication would the nurse expect the physician to prescribe as palliative treatment for advanced prostate cancer? A) Histrelin (Vantas) B) Ganirelix (Antagon) C) Nafarelin (Synarel) D) Somatropin (Nutropin) Ans: A Feedback: Histrelin (Vantas) is used to provide palliative treatment for advanced prostate cancer. Nafarelin (Synarel) is used for treatment of endometriosis and precocious puberty; Ganirelix (Antagon) is used for inhibition of premature luteinizing hormone surge in women undergoing controlled ovarian stimulation as part of a fertility program; Somatropin (Nutropin) is used for treatment of children with growth failure due to lack of growth hormone or to chronic renal failure. 25. A 4-year-old is tested and found to have deficient growth hormone (GH). What does this condition cause? A) Gigantism B) Acromegaly C) Syndrome of inappropriate antidiuretic hormone (SIADH) D) Dwarfism Ans: D Feedback: GH deficiency in children results in short stature (dwarfism). Excess production of GH results in gigantism and acromegaly. SIADH is caused by excessive production of antidiuretic hormone. 26. What hormone does the posterior pituitary gland store and release? A) Antidiuretic hormone (ADH) B) Follicle-stimulating hormone (FSH) C) Growth hormone (GH) D) Thyroid stimulating hormone (TSH) Ans: A Feedback: The posterior pituitary stores two hormones produced in the hypothalamus: ADH, also known as vasopressin and oxytocin. The posterior pituitary does not store FSH, GH, or TSH. 27. For what purposes might the nurse administer a hypothalamic hormone? (Select all that apply.) A) Diagnostic testing B) Synthesis of growth factors C) Prevent aging D) Treating some forms of cancer E) Adjuncts in fertility programs Ans: A, D, E Feedback: The hypothalamic hormones are not all available for pharmacological purposes. Those available are used mostly for diagnostic testing, for treating some forms of cancer, or as adjunctive therapies in fertility programs. They would not be used to prevent aging or for the synthesis of growth factor. 28. The nurse is caring for a patient diagnosed with syndrome of inappropriate antidiuretic hormone (SIADH) which the nurse recognizes is caused by what? A) Excessive secretion of antidiuretic hormone (ADH) B) Insufficient secretion of ADH C) Excessive secretion of androgens D) Insufficient secretion of antidepressants Ans: A Feedback: SIADH occurs with excessive secretion of ADH. Insufficient secretion of ADH causes diabetes insipidus. ADH is not related to androgenic or antidepressant actions. 29. The nurse explains the role of the posterior pituitary gland is to do what? A) It synthesizes different hormones. B) It makes two hormones. C) It stores hormones. D) It controls most of the metabolic functions of the body. Ans: C Feedback: The posterior pituitary stores two hormones produced in the hypothalamus: antidiuretic hormone (ADH, also known as vasopressin) and oxytocin. The posterior pituitary is anatomically an extension of the hypothalamus and is composed mainly of nerve fibers. Although it does not manufacture any hormones itself, it stores and releases hormones synthesized in the hypothalamus. The hypothalamus of the brain and the pituitary gland interact together to control most metabolic functions of the body and to maintain homeostasis. 30. The nurse is caring for a patient receiving tolvaptan and digoxin. What drugdrug interaction will the nurse assess for when reviewing this patients laboratory results? A) Elevated serum sodium levels B) Reduced digoxin levels C) Elevated serum potassium levels D) Tolvaptan toxicity Ans: C Feedback: Tolvaptan should be used with care with digoxin, which could cause hyperkalemia, so the nurse must carefully monitor serum potassium levels. The combination of drugs would not cause reduced digoxin levels or tolvaptan toxicity. The indication for administering tolvaptan would be to treat hyponatremia, so an elevation in serum sodium levels to normal range would be an indication the drug was working but would not be a result of a drug-drug interaction. 31. The nurse is caring for a patient who has been receiving long-term growth hormone treatment to stimulate growth. What diagnostic testing would the nurse expect to see ordered as a standard part of the treatment plan? (Select all that apply.) A) Blood sugar level B) Serum electrolytes C) X-ray of the long-bones D) Nasal examination E) Bone density studies Ans: A, B, C Feedback: Periodic radiography of the long bones, as well as monitoring of blood sugar levels and electrolytes, should be a standard part of the treatment plan for children who receive any of the hypothalamic or pituitary agents. There would be no indication for nasal assessment because growth hormone is not administered nasally. Bone density would not be impacted by these drugs. 32. What is the nurses priority assessment when administering hypothalamic or pituitary agents to older adults? A) Hydration and nutrition B) Balance and fall risk C) Cognitive function D) Urinary incontinence Ans: A Feedback: Older adults may be more susceptible to the imbalances associated with alterations in the endocrine system. They should be evaluated periodically during treatment for hydration and nutrition, as well as for electrolyte balance. These drugs would not be expected to impact balance, cognitive function, or to cause incontinence so these would not be the priority assessment. 33. The mother of a child awaiting a renal transplant asks the nurse whether growth hormone could be effective for her child. What is the nurses best response? A) Growth hormone is only used to treat short stature in children who do not produce enough growth hormone naturally. B) Growth hormone has been used successfully in children with renal failure but there is a lot to consider and you should talk to the doctor about it. C) Administration of growth hormone requires frequent injections so it would be cruel for a child with existing medical problems. D) You do not want to use growth hormone if you do not have to. What would make you even think of something like that? Ans: B Feedback: Growth hormone therapy is used with children with renal failure but the decision carries risks as well as benefits so it is important for the mother to talk to the childs primary physician and nephrologist to get recommendations for its safety. It is not just used in children with inadequate growth hormone secretion. It is never correct for the nurse to castigate the mother for a question so telling her it would be cruel or that she should not use it if it is not essential would destroy the patient/nurse relationship. 34. The nurse is asked to explain how to administer somatropin (Saizen) to the mother of a 6-year-old. How would the nurse explain how this drug is administered? A) It requires only a very small needle and doesnt hurt much at all. B) There will no longer be any need to rotate sites because it uses a needleless system. C) This system will be used until your son gets older and is more tolerant of needles. D) This delivers a fine mist through the skin without needles and far less discomfort. Ans: D Feedback: Saizen uses the cool.click delivery system, which is a neon-colored, needle- free system that delivers the drug through the skin using a fine mist. Tests have shown a bioequivalency of this method with standard injection techniques, and the young patients who must use this drug are much less resistant to the dosing. There are no needles but site of injection still require rotation to avoid skin damage. The child does not have to go back to a needle system when he gets older. Chapter 16. Cardiovascular System Medications Cardiotonic: 1. A patient presents to the emergency department with rales, wheezing, and blood-tinged sputum. What does the nurse recognize that these symptoms indicate? A) Cardiomyopathy B) Cardiomegaly C) Valvular heart disease D) Pulmonary edema Ans: D Feedback: In left-sided heart failure, the left ventricle pumps inefficiently resulting in a backup of blood into the lungs causing pulmonary vessel congestion and fluid leaks into the alveoli and lung tissue. As more fluid continues to collect in the alveoli, pulmonary edema develops. The patient will present with rales, wheezes, blood-tinged sputum, low oxygenation, and development of a third heart sound. Cardiomyopathy can occur as a result of a viral infection, alcoholism, anabolic steroid abuse, or a collagen disorder. It causes muscle alterations and ineffective contraction and pumping. Cardiomegaly is an enlargement of the heart due to compensatory mechanisms in congestive heart failure (CHF) and leads to ineffective pumping and eventually exacerbated CHF. Valvular heart disease leads to an overload of the ventricles because the valves do not close adequately causing blood to leak backward. This causes muscle stretching and increased demand for oxygen and energy. 2. What electrolyte inactivates troponin and allows actin and myosin to form a bridge enabling the muscle fibers to contract? A) Magnesium B) Calcium C) Potassium D) Sodium Ans: B Feedback: Calcium must be present to deactivate troponin so that actin and myosin can react to form actinomycin bridges. Potassium, sodium, and magnesium do not affect troponin. 3. A 62-year-old African American man diagnosed with congestive heart failure and hypertension has BiDil included in his drug therapy. What nursing assessment finding would indicate the patient is developing a complication from this drug? A) Alopecia B) Photosensitivity C) Anorexia D) Orthostatic hypotension Ans: D Feedback: Orthostatic hypotension is an adverse effect of a combination of isosorbide dinitrate and hydralazine called BiDil. This could lead to safety concerns and should be addressed in drug teaching for this patient. Alopecia, photosensitivity, and anorexia are not adverse effects related to this drug. 4. The nurse is preparing digoxin for an infant. What is the nurses priority intervention? A) To perform hand hygiene B) To have another nurse check dosage calculations C) To check the childs apical pulse D) To identify the patient by checking the ID bracelet Ans: B Feedback: The margin of safety for the dosage of this drug is very narrow in children. The dosage needs to be very carefully calculated and should be double- checked by another nurse before administration. The other options are important and the nurse should implement all three. However, these actions are of lower priority. 5. A patient asks the nurse what cardiac glycosides do to improve his condition. What is the nurses best response? A) They decrease the force of myocardial contractions. B) They help renal blood flow and increase urine output. C) They increase heart rate. D) They increase conduction velocity. Ans: B Feedback: Cardiac glycosides increase intracellular calcium and allow more calcium to enter myocardial cells. This action causes an increased force of myocardial contraction, an increased cardiac output, and renal perfusion that increases urine output. Cardiac glycosides also serve to slow the heart rate and decrease conduction velocity. 6. What is the priority nursing assessment for a patient who is about to begin digoxin therapy? A) Blood glucose levels B) Neurological function C) Kidney function D) Liver function Ans: C Feedback: Digoxin is primarily excreted unchanged in the urine, so caution should be exercised if renal impairment is present. Blood glucose levels and neurological and liver function would not be a priority assessment related to digoxin therapy. 7. A triage nurse in the emergency department is assessing a 78-year-old man. It is determined that the patient is experiencing severe digoxin toxicity. What drug will the nurse administer immediately? A) Inamrinone (Inocor) B) Digoxin immune Fab (Digibind) C) Verapamil hydrochloride (Calan) D) Quinidine sulfate Ans: B Feedback: Digoxin immune Fab is an antigen-binding fragment derived from specific antidigoxin antibodies. The drug is used for the treatment of life-threatening digoxin intoxication when serum levels are greater than 10 ng/mL. Inamrinone is a phosphodiesterase inhibitor that acts as a cardiotonic agent. Verapamil hydrochloride is a calcium channel blocker. Quinidine is an antiarrhythmic agent that when taken with digoxin increases both the therapeutic and toxic effects of digoxin. 8. The nurse is providing patient teaching to a patient who has been prescribed digoxin. The patient tells the nurse that she occasionally use herbals and other alternative therapies. What herb would the nurse warn the patient to avoid taking with digoxin? A) Black cohosh B) Ginseng C) Saw palmetto D) Valerian Ans: B Feedback: Digoxin toxicity can occur if the drug is taken concurrently with licorice, ginseng, or hawthorn. St. Johns wort and psyllium have been shown to decrease the effectiveness of digoxin, so that combination should be avoided. There is no drug-to-drug interaction with black cohosh, saw palmetto, or valerian. 9. The nurse administers an IV phosphodiesterase inhibitor. What drug will result in forming a precipitate if given via the same IV line without adequate flushing? A) Albuterol (Proventil) B) Nifedipine (Procardia) C) Furosemide (Lasix) D) Lovastatin (Mevacor) Ans: C Feedback: Furosemide, when given with a phosphodiesterase inhibitor, forms precipitates; therefore, this combination should be avoided. Use alternate lines if both of these drugs are being given IV. There are no known drug-to- drug interactions with albuterol, nifedipine, or lovastatin. 10. A 6-year-old child weighing 60 pounds has been prescribed oral digoxin (Lanoxin) 30 mcg/kg as a loading dose. How many milligram will she be given? A) 0.218 mg B) 0.418 mg C) 0.618 mg D) 0.818 mg Ans: D Feedback: First, using the formula: 2.2 pounds and 60 pounds: multiplied by kg, determine the childs weight in kg (60/2.2 = 27.27 kg). Next, using the formula: amount of drug prescribed times weight in kg, determine the dose in mcg the child should receive (30 multiplied by 27.27 = 0. 818 mcg). Then to determine the amount of mg the child should receive, use the formula: 1 mg: 1,000 mcg = X mg: 818 mcg (818/1,000 = 0.818 mg). 11. The nurse assesses the patient before administering digoxin (Lanoxin) and withholds the drug and notifies the physician with what finding? A) Respiratory rate falls below 14 B) History reveals liver failure C) Pulse is 44 beats/min D) Blood pressure is 72/40 mm Hg Ans: C Feedback: Monitor apical pulse for 1 full minute before administering the drug to assess for adverse effects. Hold the dose if the pulse is less than 60 beats/ min in an adult or less than 90 beats/min in an infant; retake pulse in 1 hour. If pulse remains low, document pulse, withhold the drug, and notify the prescriber. 12. When a drug is said to increase the force of contraction of the heart muscle, the nurse appropriately uses what term? A) Positive chronotropic B) Positive inotropic C) Negative inotropic D) Negative dromotropic Ans: B Feedback: Sympathetic stimulation causes an increase in heart rate, blood pressure, and rate and depth of respirations, as well as a positive inotropic effect (increased force of contraction) on the heart and an increase in blood volume (through the release of aldosterone). A negative inotropic effect would be one that decreased the force of a contraction. A negative dromotropic effect is one that slows the conduction of the impulse through the atrioventricular node. A positive chronotropic effect is one that increases the heart rate. 13. The nurse, caring for a patient who is receiving cardiac glycosides to treat heart failure, will teach the patient to follow what diet? A) High sodium, low potassium, high fat B) Low sodium, low potassium, low fat C) High iron, high calcium, high potassium D) Low sodium, high potassium, low fat Ans: D Feedback: Restrict dietary sodium to reduce edema in patients receiving cardiac glycosides. If the patient is hyponatremic or using a potassium-losing diuretic, increase potassium in diet, as well as limit fat intake to reduce weight and atherogenic activity. 14. What order for a digitalizing dose of digoxin (Lanoxin) for a 62-year-old man would the nurse consider appropriate and safe to administer? A) 1.25 mg IV now B) 0.75 mg orally now C) 0.25 mg orally every day D) 1 mg intramuscularly every 4 hours 24 hours Ans: B Feedback: Digoxin: Adult: loading dose 0.75 to 1.25 mg orally or 0.125 to 0.25 mg IV, then oral maintenance dose of 0.125 to 0.25 mg/d; decrease dose with renal impairment. 15. After administering an IV dose of digoxin, the nurse would expect to see effects within what period of time? A) 30 to 120 minutes B) 5 to 30 minutes C) 1 hour D) 2 hours Ans: B Feedback: The drug has a rapid onset of action and rapid absorption (30 to 120 minutes when taken orally, 5 to 30 minutes when given IV). 16. The patient taking digoxin (Lanoxin) has developed an infection. What antibiotic can the nurse safely administer to this patient? A) Zithromax B) Erythromycin C) Tetracycline D) Cyclosporine Ans: A Feedback: Zithromax may be given without impacting the effects of digoxin. There is a risk of increased therapeutic effects and toxic effects of digoxin if it is taken with verapamil, amiodarone, quinidine, quinine, erythromycin, tetracycline, or cyclosporine. If digoxin is combined with any of these drugs, it may be necessary to decrease the digoxin dose to prevent toxicity. If one of these drugs has been part of a medical regimen with digoxin and is discontinued, the digoxin dose may need to be increased. 17. The 96-year-old patient is receiving digoxin (Lanoxin) and furosemide (Lasix). In the morning, the patient complains of a headache and nausea. What will the nurse do first? A) Contact the patients physician immediately. B) Check her laboratory values and vital signs. C) Administer acetaminophen and Maalox. D) Give her clear liquids and have her lie down. Ans: B Feedback: The nurse will check the patients digoxin level and electrolytes. Assessing vital signs is important because the risk of cardiac arrhythmias could increase due to the patients receiving furosemide, which is a potassium- losing diuretic. The adverse effects most frequently seen with the cardiac glycosides include headache, weakness, drowsiness, and vision changes (a yellow halo around objects is often reported). Gastrointestinal (GI) upset and anorexia also commonly occur. Only after checking lab values and assessing vital signs might the nurse call the physician. Acetaminophen and Maalox would not be indicated. Having her lie down and restricting her diet to clear liquids would be appropriate but not the first actions. 18. The nurse provides teaching about digoxin to the 62-year-old patient. The nurse evaluates patient understanding and determines further teaching is needed when the patient says she will do what? A) Take the medication daily in the morning. B) Take her pulse before taking her dose. C) Weigh herself daily at the same time. D) Take the medication with a meal. Ans: D Feedback: Avoid administering the oral drug with food or antacids to avoid delays in absorption. The other answers are appropriate actions for the patient to take when self-administering digoxin. 19. When administering milrinone (Primacor), the nurse will assess the patient for what common adverse effect? A) Hypoglycemia B) Confusion C) Hypotension D) Seizures Ans: C Feedback: The adverse effects most frequently seen with these drugs are ventricular arrhythmias (which can progress to fatal ventricular fibrillation), hypotension, and chest pain. Hypoglycemia, confusion, and seizures are not generally adverse effects of milrinone. 20. The nurse administers a cardiac glycoside for what therapeutic effect? A) To decrease cardiac output B) To decrease afterload C) To increase ventricular rate D) To increase the force of the contraction of the heart Ans: D Feedback: Cardiac glycosides exert a negative cardiotonic and positive inotropic effect. They do not decrease cardiac output, decrease afterload, or increase the ventricular rate of the heart. 21. After administering digoxin, what assessment finding would indicate to the nurse that the drug was having the desired effect? A) Increased heart size B) Increased urinary output C) Decreased respiratory rate D) Increased heart rate Ans: B Feedback: As cardiac output improves, so does urinary output due to increased renal perfusion. Respiratory rate and heart size would not be impacted by the drug, although ventricular hypertrophy is a common finding in patients with heart failure. Heart rate would decrease as the force of contraction increases, ejecting more blood with each contraction. 22. What assessment finding would indicate the patients left-sided heart failure is worsening? A) Increased jugular venous pressure B) Liver enlargement C) Increased crackles in lung fields D) Increased pulse rate Ans: C Feedback: Fluid may accumulate in the lungs due to left sided heart failure. Patients may evidence dyspnea, tachypnea, and orthopnea. Right-sided failure would include increased jugular venous pressure and liver enlargement. Pulse rate could increase or decrease depending on medications administered. 23. Which drug is in the class of drugs called human B-type natriuretic peptides? A) Bosentan (Tracleer) B) Milrinone (Primacor) C) Digoxin (Lanoxin) D) Nesiritide (Natrecor) Ans: D Feedback: Nesiritide is the only drug currently available in a class of drugs called human B-type natriuretic peptides. Digoxin is a cardiac glycoside. Milrinone is a phosphodiesterase inhibitor. Bosentan is an endothelin receptor antagonist. 24. The nurse reviews the patients lab results and recognizes the patient is at risk for digoxin toxicity due to what electrolyte imbalance? A) Hyperkalemia B) Hypokalemia C) Hypernatremia D) Hyponatremia Ans: B Feedback: Electrolyte abnormalities (e.g., increased calcium, decreased potassium, decreased magnesium) could alter the action potential and change the effects of the drug. Hypokalemia and hypomagnesemia increase cardiac excitability and ectopic pacemaker activity leading to dysrhythmias. 25. What common action do both cardiac glycosides and phosphodiesterase inhibitors have in common related to therapeutic action? A) Blocking the enzyme phosphodiesterase B) Increasing cellular calcium C) Developing ventricular arrhythmias D) Metabolizing in the liver and excreted in the urine Ans: B Feedback: The phosphodiesterase inhibitors block the enzyme phosphodiesterase. This blocking effect leads to an increase in myocardial cell cyclic adenosine monophosphate (cAMP), which increases calcium levels in the cell (Figure 44.4). Increased cellular calcium causes a stronger contraction and prolongs the effects of sympathetic stimulation, which can lead to vasodilation, increased oxygen consumption, and arrhythmias. Digoxin also increases intracellular calcium and allows more calcium to enter myocardial cells during depolarization. 26. The nurse evaluates an improvement in the patients heart failure (HF) status based on what assessment finding? A) Using fewer pillows to sleep B) Increased skin turgor C) Heart rate regular D) Improved mental status Ans: A Feedback: The degree of HF is often calculated by the number of pillows required to get relief (e.g., one-pillow, two-pillow, or three-pillow orthopnea). Fluid overload is associated with HF so skin turgor is not an indicator of improvement. Regular heart rate and normal cognition can be found with acute flare-ups of HF so these findings would not indicate improvement. 27. The nurse suspects the patient may have toxic levels of digoxin in the bloodstream when what is assessed? (Select all that apply.) A) Irregular heart rhythms B) Nausea C) Anorexia D) Headache E) Peripheral edema Ans: A, B, C Feedback: Digoxin toxicity is a serious syndrome that can occur when digoxin levels are too high. The patient may present with anorexia, nausea, vomiting, malaise, depression, irregular heart rhythms including heart block, atrial arrhythmias, and ventricular tachycardia. Peripheral edema is indicative of heart failure, not digoxin toxicity. Headache is not usually associated with digoxin toxicity. 28. The patient has been prescribed inamrinone (Inocor). Before administering the drug the nurse needs to know the drug has what pharmacokinetic effect? A) Decrease in cyclic adenosine monophosphate (cAMP) B) Decrease in cardiac output C) Increase in cardiac preload D) Increase in cAMP Ans: D Feedback: Inamrinone is a phosphodiesterase inhibitor that blocks the enzyme phosphodiesterase. This blocking effect leads to an increase in myocardial cell cAMP, which increases calcium levels in the cell. These drugs do not decrease cardiac output or increase cardiac preload. 29. The patient has just been prescribed milrinone (Primacor). The nurse recognizes the drug is contraindicated due to the patients allergy to what? A) Penicillins B) Salicylates C) Opioids D) Bisulfites Ans: D Feedback: Phosphodiesterase inhibitors are contraindicated in the presence of allergy to the drug or to bisulfites. Penicillins, salicylates, and opioids have no contraindications when used with milrinone. 30. The nurse administers a human B-type natriuretic peptide with the expectation it will have what action? A) Decrease blood volume B) Increase force of cardiac contraction C) Reduce venous return D) Lighten the hearts workload Ans: C Feedback: Human B-type natriuretic peptides are normally produced by myocardial cells as a compensatory response to increased cardiac workload and increased stimulation by the stress hormones. They bind to endothelial cells, leading to dilation and resulting in decreased venous return, peripheral resistance, and cardiac workload. They also suppress the bodys response to the stress hormones, leading to increased fluid loss and further decrease in cardiac workload. Diuretics decrease blood volume, cardiac glycosides increase force of contraction, and vasodilators lighten the hearts workload. 31. The nurse expects the patients heart failure (HF) is caused by what diagnosis that is responsible for 95% of the cases diagnosed? A) Cardiomyopathy B) Hypertension C) Congenital anomaly D) Coronary artery disease (CAD) Ans: D Feedback: CAD is the leading cause of HF, accounting for approximately 95% of the cases diagnosed. CAD results in an insufficient supply of blood to meet the oxygen demands of the myocardium. Consequently, the muscles become hypoxic and can no longer function efficiently. When CAD evolves into a myocardial infarction, muscle cells die or are damaged, leading to an inefficient pumping effort. Cardiomyopathy, hypertension, and congenital anomaly are rarely associated with heart failure. 32. The nurse prepares to administer a phosphodiesterase inhibitor by what route? A) Oral B) IV C) Subcutaneous D) Intramuscular Ans: B Feedback: Phosphodiesterase inhibitors are only given IV. They cannot be given orally, intramuscularly, or subcutaneously. 33. The nurse is caring for a patient who has digoxin toxicity. As the nurse assesses the changes in the patients daily activities, what finding could indicate the cause of the toxic level? A) The patient has been sleeping more lately. B) The patient took nitroglycerin for chest pain twice yesterday. C) The patients daughter brought her a bag of licorice that she has been enjoying. D) The patients intake of sodium increased lately because shes been eating seafood. Ans: C Feedback: Increased digoxin toxicity has been reported with ginseng, hawthorn, and licorice. Patients should be advised to avoid these combinations. Increased sodium intake will exacerbate the patients heart failure, which might explain why she is sleeping more and requiring nitroglycerin but these actions did not contribute to the digoxin toxicity. 34. The nurse admits a patient to the constant care unit with a digoxin level of 11 ng/mL and a serum potassium level of 5.2 mEq/L. Digoxin immune Fab is administered. The next day, the patients digoxin level remains elevated. What action does the nurse anticipate? A) Administer digoxin immune Fab again. B) Administer a reduced dosage of digoxin. C) Continue to monitor the patients digoxin level daily. D) Notify the health care provider of the elevated level. Ans: C Feedback: Serum digoxin levels will be very high and unreliable for about 3 days after the digoxin immune Fab infusion because of the high levels of digoxin in the blood. The patient should not be redigitalized for several days to 1 week after digoxin immune Fab has been used, because of the potential of fragments remaining in the blood. There is no need to notify the health care provider or to administer digoxin immune Fab again. 35. The nurse is preparing to administer a digitalizing dosage of digoxin to a geriatric patient. What factors will the nurse assess for first to avoid digoxin toxicity? (Select all that apply.) A) Renal function B) Low body mass C) Hydration D) Assessment of pulse E) Cognitive function Ans: A, B, C Feedback: Factors that may contribute to elevated digoxin levels include impaired renal function, low body mass, and dehydration. Assessment of pulse and cognitive function are always important when caring for a geriatric patient, but they will not contribute to elevated digoxin levels. Arrthytimic:1. A patient with a history of atrial fibrillation has had a worsening of his or her condition. The nurse knows that the drug of choice for long-term stabilization of atrial fibrillation following electrocardioversion is what? A) Disopyramide (Norpace) B) Moricizine (Ethmozine) C) Procainamide (Pronestyl) D) Quinidine (generic) Ans: D Feedback: Quinidine is often the drug of choice for long-term stabilization of atrial fibrillation after the rhythm is converted by electrocardioversion. Quinidine is a class I antiarrhythmic and stabilizes the cell membrane by binding to sodium channels, depressing phase 0 of the action potential, and changing the duration of the action potential. Disopyramide, moricizine, and procainamide are all used in the treatment of life-threatening ventricular arrhythmias. 2. What class of antiarrhythmics drug blocks potassium channels, prolonging phase 3 of the action potential and slowing the rate and conduction of the heart? A) I B) II C) III D) IV Ans: C Feedback: The class III antiarrhythmics block potassium, prolonging phase 3 of the action potential, which prolongs repolarization and slows the rate and conduction of the heart. Class I drugs block the sodium channels in the cell membrane during an action potential. Class II drugs are beta-adrenergic blockers that block beta-receptors, causing a depression of phase 4 of the action potential. Class IV drugs block calcium channels in the cell membrane leading to a depression of depolarization and a prolongation of phases I and II of repolarization, which slows automaticity. 3. The nurse is caring for a pediatric patient who has been diagnosed with paroxysmal atrial tachycardia. The order reads digoxin 10 mcg/kg orally. The child weighs 44 pounds. How many mcg will the nurse administer? A) 50 mcg B) 100 mcg C) 150 mcg D) 200 mcg Ans: D Feedback: The nurse will administer 200 mcg. (2.2 pounds: 1 kg = X: 20 kg, 20(10) = 200 mcg). 4. The nurse is providing drug teaching about quinidine in preparation for the patients discharge. The nurse discusses drugfood interactions and advises the patient to drink what? A) Apple juice B) Grapefruit juice C) Milk D) Orange juice Ans: A Feedback: Apple juice would be the best choice. Quinidine requires a slightly acidic urine (normal state) for excretion. Patients receiving quinidine should avoid foods that alkalinize the urine (e.g., citrus juices, vegetables, antacids, milk products), which could lead to increased quinidine levels and toxicity. Grapefruit juice has been shown to interfere with the metabolism of quinidine, leading to increased serum levels and toxic effects; this combination should be avoided. 5. A patient has had sotalol (Betapace) ordered for treatment of a ventricular arrhythmia. What will the nurse consider when administering the drug? A) Sotalol has a very short duration of action B) Food increases the bioavailability of the drug C) Absorption of sotalol is decreased by the presence of food D) The drug is best administered intramuscularly Ans: C Feedback: This drug should not be taken with food because absorption is decreased. The drug should be given 1 hour before or 2 hours after a meal. Adenosine, not sotalol, has a very short duration of action and food increases the bioavailability of propranolol. Sotalol is administered by oral route only. 6. The nurse is caring for a patient who reports insomnia since starting the antiarrhythmic agent prescribed for him or her. What antiarrhythmic agent would the nurse expect this patient is taking? A) Disopyramide (Norpace) B) Amiodarone (Cordarone) C) Procainamide (Pronestyl) D) Propranolol (Inderal) Ans: D Feedback: Class II antiarrhythmics can cause insomnia. The adverse effects associated with class II antiarrhythmics are related to the effects of blocking beta- receptors in the sympathetic nervous system. CNS effects include dizziness, insomnia, unusual dreams, and fatigue. Disopyramide and procainamide are class I agents and do not cause insomnia. Amiodarone is a class III drug and is not associated with insomnia. 7. A patient with impaired renal function is to receive dofetilide (Tikosyn) for conversion of atrial fibrillation. What is the nurses priority assessment before administering the drug? A) Check the patients creatinine level. B) Measure the urine output. C) Listen to breath sounds. D) Measure the PR interval on the electrocardiogram. Ans: A Feedback: When giving dofetilide to a patient with renal dysfunction, the dosage must be calculated according to the patients creatinine level to ensure the therapeutic effect while limiting toxicity. This drug can only be administered by oral route. Intake and output as well as breath sounds may need to be assessed but are not related to administering the drug. A patient in atrial fibrillation will not have a measurable PR interval. 8. A nurse is caring for a patient who has had disopyramide (Norpace), ordered. Before administering disopyramide (Norpace) what is the nurses priority action to maintain safety? A) Offer the patient something to drink. B) Ask the patient if he or she needs to void. C) Raise all side rails. D) Place the call button within reach. Ans: C Feedback: When administering disopyramide, the nurse should make sure that all side- rails are up. The central nervous system effects of the drug can include dizziness, drowsiness, fatigue, twitching, mouth numbness, slurred speech, vision changes, and tremors that can progress to convulsions. The other three options are appropriate and placing the call button within reach can also be considered a safety measure; however nothing has a higher priority than raising the beds side-rails. 9. The nurse sees a patient in an outpatient setting who is given a new prescription for an antiarrhythmic medication to treat premature atrial contractions (PAC). The nurse has limited time with the patient and addresses what priority nursing diagnosis? A) Decreased cardiac output B) Alteration in comfort C) Deficient knowledge D) Potential for injury Ans: C Feedback: The patient received a new prescription and needs information about how to take the medication, when to call the provider, and potential adverse effects so the priority nursing diagnosis is deficient knowledge. It is unknown what arrhythmia the patient is treating and without this information it is impossible to know what the impact on cardiac output, comfort and potential for injury may be. 10. You are caring for a patient who takes an antiarrhythmic agent. What would be a priority nursing assessment before administering this drug? A) Assess mental status. B) Assess breath sounds. C) Assess pulses and blood pressure. D) Assess urine output. Ans: C Feedback: The nurse should continually monitor cardiac rate and rhythm when administering an antiarrhythmic agent to detect potentially serious adverse effects and to evaluate drug effectiveness. All of the other options are appropriate assessments but are not the priority assessment. 11. The nurse is caring for a patient receiving propranolol. What problems, reported by the patient, does the nurse suspect is caused by the drug? (Select all that apply.) A) Seizures B) Rash C) Atrioventricular (AV) block D) Bronchospasm E) Dreams Ans: C, D, E Feedback: The adverse effects associated with class II antiarrhythmics are related to the effects of blocking beta-receptors in the sympathetic nervous system. Central nervous system effects include dizziness, insomnia, unusual dreams, and fatigue. Cardiovascular symptoms can include hypotension, bradycardia, AV block, arrhythmias, and alterations in peripheral perfusion. Respiratory effects can include bronchospasm and dyspnea. GI problems frequently include nausea, vomiting, anorexia, constipation, and diarrhea. Other effects to anticipate include a loss of libido, decreased exercise tolerance, and alterations in blood glucose levels. Seizures and rash are not usually associated with the adverse effects of propranolol. 12. The nurse teaches the patient receiving propranolol (Inderal) at home for management of a ventricular dysrhythmia to monitor what parameter? A) Daily fluid intake B) Daily blood pressure C) Weekly weight D) Weekly pulse Ans: B Feedback: Hypotension can occur with propranolol, which is a beta-blocker, so patients should check their blood pressure and pulse every day. Propranolol does not require that fluid intake be measured. propranolol also will not require daily weight taking, but it may be necessary with a diagnosis of propranolol. 13. The emergency department nurse is administering IV lidocaine to a patient. What adverse effect of lidocaine therapy should the nurse assess for? A) Dysphagia B) Dizziness C) Excessive bruising D) Tinnitus Ans: B Feedback: Adverse effects include dizziness, light-headedness, fatigue, arrhythmias, cardiac arrest, nausea, vomiting, anaphylactoid reactions, hypotension, and vasodilation. Dysphagia, bruising, and tinnitus are not usually associated with lidocaine. 14. The nurse administers a bolus of lidocaine and follows it with a continuous infusion at what rate? A) 0.25 to 0.75 mg/min B) 1 to 4 mg/min C) 6 to 8 mg/min D) 10 to 20 mg/min Ans: B Feedback: Lidocaine may be delivered at 1 to 4 mg/min after a bolus. Therefore, the other options are incorrect. 15. The nurse administers what drug to terminate supraventricular tachycardia? A) Lidocaine (Lidocaine Parenteral) B) Flecainide (Tambocor) C) Adenosine (Adenocard) D) Dronedarone (Multaq) Ans: C Feedback: Adenosine depresses conduction at the atrioventricular node and is used to restore NSR (normal sinus rhythm) in patients with paroxysmal supraventricular tachycardia. Adenosine is used to treat supraventricular tachycardias, including those caused by the use of alternate conduction pathways in adults. Lidocaine is used to treat life-threatening ventricular arrhythmias during myocardial infarction or cardiac surgery; it is also used as a bolus injection in emergencies when monitoring is not available to document exact arrhythmia. Flecainide is used to treat life-threatening ventricular arrhythmias in adults; prevention of paroxysmal atrial tachycardia (PAT) in symptomatic patients with no structural heart defect. Dronedarone is used to treat paroxysmal or persistent atrial fibrillation or atrial flutter in patients with multiple risk factors for coronary artery disease who are currently in sinus rhythm or scheduled for conversion. 16. For what condition would the nurse expect to administer lidocaine via IV drip? A) Decrease in arterial oxygen saturation (SaO2) B) Increase in blood pressure C) Multiple premature ventricular contractions (PVCs) D) Increase in intracranial pressure (ICP) Ans: C Feedback: Lidocaine drips are indicated for the treatment of life-threatening ventricular arrhythmias such as long or frequent runs of PVCs. Lidocaine would not be indicated for the treatment of hypoxia, hypertension, or increased ICP. 17. The nurse is caring for a patient who takes an antiarrhythmic agent and is reporting a complete lack of appetite. Which cardiac antiarrhythmic agent would the nurse suspect the patient is taking? A) Diltiazem (Cardizem) B) Propranolol (Inderal) C) Lidocaine (Lidocaine Parenteral) D) Amiodarone (Cordarone) Ans: B Feedback: Propranolol frequently causes gastrointestinal (GI) problems such as nausea, vomiting, anorexia, constipation, and diarrhea. Diltiazem could cause nausea and vomiting but would not cause anorexia. Lidocaine can lead to changes in taste, nausea, and vomiting but does not cause anorexia. Amiodarone has adverse effects including nausea, vomiting, GI distress, weakness, dizziness, hypotension, heart failure, arrhythmia, a potentially fatal liver toxicity, and ocular abnormalities but does not cause anorexia. 18. A 92-year-old patient is being sent home on disopyramide (Norpace) for a ventricular arrhythmia. He asks the nurse why he must continue to take this drug. The nurses best response would be that failure to treat a ventricular arrhythmia may what? A) Lead to renal failure B) Result in hypertension C) Result in death D) Cause heart failure Ans: C Feedback: Ventricular arrhythmias cause a dramatic reduction in cardiac output and will result in death if not treated. The patient needs to be taught the importance of taking his medication every day as ordered. Generally, ventricular arrhythmias are not associated with renal failure, hypertension, or heart failure. 19. The nurse is providing drug teaching for a patient who is to be discharged taking dofetilide (Tikosyn). What drug will the nurse teach the patient to avoid due to a drug-drug interaction? A) Cimetidine B) Furosemide C) Acetaminophen D) Antacids Ans: A Feedback: There is an increased risk of serious adverse effects if dofetilide is combined with ketoconazole, cimetidine, or verapamil, so these combinations should be avoided. There is no known increased risk associated with furosemide, acetaminophen, or antacids but other specific drugdrug interaction are reported, so it is important to check a current drug handbook before administering these medications. 20. The patient asks the nurse, Will I have to take this antiarrhythmic agent for the rest of my life? The nurse, having reviewed the CAST study, responds by saying what? A) Yes, you will have to take this drug for life. B) The drug is indicated for short-term treatment of life-threatening ventricular arrhythmias. C) This drug may need to be changed but youll take an antiarrhythmic for life. D) After the arrhythmia is corrected, the drug will be stopped. Ans: B Feedback: The CAST study, a large research study run by the National Heart and Lung Institute, found that long-term treatment of arrhythmias may have an uncertain effect on mortality, and in some cases may actually lead to increased cardiac death, which is the basis for the current indication for antiarrhythmics to be used only short-term to treat life-threatening ventricular arrhythmias. 21. What patient factor would result in the nurse administering a reduced dosage of disopyramide (Norpace)? A) Dehydration B) Hypertension C) Renal impairment D) Chronic diarrhea Ans: C Feedback: Caution should be used with renal or hepatic dysfunction, which could interfere with the biotransformation and excretion of these drugs. Caution is not indicated with the findings of dehydration, hypertension, or chronic diarrhea. 22. The nurse is administering an intravenous infusion of amiodarone (Cordarone). What should the nurse be aware of? A) The possible drugdrug interaction with nonsteroidal anti-inflammatory drugs (NSAIDs) B) The possible development of very serious cardiac arrhythmias C) The possible development of peripheral edema D) The possible development of a fatal renal toxicity Ans: B Feedback: Amiodarone has been associated with a potentially fatal liver toxicity rather than a renal toxicity, ocular abnormalities, and the development of very serious cardiac arrhythmias. Sotalol may have a loss of effectiveness if combined with NSAIDs. Diltiazem is associated with peripheral edema. 23. The nurse reviews the patients medical history and determines class II antiarrhythmics are contraindicated due to the patients history of what condition? A) Asthma B) Colitis C) Migraine headache D) Antidiarrheals Ans: A Feedback: Class II antiarrhythmics are contraindicated in patients with asthma because they could worsen the condition due to blockage of beta-receptors. They are not contraindicated in patients with colitis, migraine headache, or diarrhea. 24. What would the nurse teach the diabetic patient to monitor for when beginning a class II antidysrhythmic drug regimen in addition to insulin? A) Weight loss B) Reduced peripheral perfusion C) Hypoglycemia D) Exercise intolerance Ans: C Feedback: Increased hypoglycemia is possible if these drugs are combined with insulin; so patients should be taught to monitor their blood sugar levels often. Recurrent hypoglycemic episodes may indicate the need to reduce insulin dosage, but this advice would need to come from the health care provider. Class II antiarrhythmic drugs are normally not associated with weight loss, reduced peripheral perfusion, or exercise intolerance. 25. The nurse assesses the patients rhythm strip and notes several premature ventricular contractions, which the nurse recognizes is caused by what? A) Stimulation of the ventricles from an ectopic focus in the ventricles B) Stimulation of the ventricles from the atrioventricular node C) Stimulation of the ventricles from the Purkinje fibers D) Stimulation of the ventricles from the bundle of His Ans: A Feedback: Premature ventricular contractions are stimulations of the cells caused by an ectopic focus in the ventricles causing an early contraction. The source of these ectopic foci could be anywhere within the ventricles, but the end result is reduced cardiac output due to reduced force of contraction. 26. When the nurse is caring for a patient with a cardiac arrhythmia, the priority goal for the patient is what? A) To maintain nutritional intake B) To maintain fluid intake C) To maintain cardiac output D) To maintain urine output Ans: C Feedback: Disruptions in the normal rhythm of the heart can interfere with myocardial contractions and affect the cardiac output, the amount of blood pumped with each beat. Arrhythmias that seriously disrupt cardiac output can be fatal. Therefore, the primary goal of treating a cardiac arrhythmia is to maintain adequate cardiac output to support life. The other goals may be important to individual patient care, but sustaining life takes priority. 27. The nurse is caring for a 3-year-old awaiting a heart transplant who requires an antiarrhythmic agent to control a supraventricular arrhythmia. What drug, if ordered, would the nurse question? A) Digoxin B) Propranolol C) Procainamide D) Verapamil Ans: D Feedback: Verapamil should be avoided in children and, if ordered, would require the nurse to question the drug. Adenosine, propranolol, procainamide, and digoxin have been successfully used to treat supraventricular arrhythmias, with propranolol and digoxin being the drugs of choice for long-term management. 28. The nurse suspects drug toxicity in the patient who has been receiving lidocaine by infusion over the past 2 days to control a ventricular arrhythmia. What assessment would the nurse perform to determine the accuracy of the suspicion of toxicity? A) Neurological assessment B) Serum lidocaine level C) Renal function studies D) Hepatic function studies Ans: B Feedback: If lidocaine is used for ventricular arrhythmias related to cardiac surgery or digoxin toxicity, serum levels should be monitored regularly to determine the appropriate dose and to avoid the potential for serious proarrhythmias and other adverse effects. Neurological assessment may indicate adverse effects but would not confirm lidocaine toxicity. Renal and hepatic function would not confirm lidocaine toxicity. 29. The nurse performs an electrocardiogram and finds the older adult patient is in atrial fibrillation (AF). Time of onset is unknown but could be as long as 3 months earlier when the patient was last assessed. What drug will the nurse expect to be ordered? A) Anticoagulant B) Digoxin C) Quinidine D) Ibutilide Ans: A Feedback: If the onset of AF is not known and it is suspected that the atria may have been fibrillating for longer than 1 week, the patient is better off staying in AF without drug therapy or electrocardioversion. Prophylactic oral anticoagulants are given to decrease the risk of clot formation and emboli being pumped into the system. In 2011, the American Heart Association and American College of Cardiology endorsed dabigatran (Pradaxa) as the anticoagulant of choice for prophylaxis in AF. Conversion in this case could result in potentially life-threatening embolization of the lungs, brain, or other tissues. Administration of other antiarrhythmics would not be indicated. 30. What nursing assessment will the nurse perform to determine the hemodynamic effect of the patients arrhythmia? A) Obtain an electrocardiographic rhythm strip. B) Obtain a serum drug level. C) Assess the patients level of consciousness. D) Assess the patients blood pressure (BP) and pulse rate. Ans: D Feedback: BP and pulse rate are indicators of the hemodynamic effect of arrhythmias and are nursing measures that do not require a physicians order. Obtaining an electrocardiogram or checking drug levels requires a physicians order. The patient will be conscious and alert with non-life-threatening arrhythmias, even when cardiac output is reduced. Serum drug levels would indicate the therapeutic or toxic level of drugs in the body but would not indicate hemodynamic effects of the drug. 31. What class of antiarrhythmics will the nurse administer to the patient in symptomatic paroxysmal tachycardia because it markedly depresses phase 0 with extreme slowing of conduction? A) Class Ib B) Class Ic C) Class II D) Class III Ans: B Feedback: Class Ic drugs markedly depress phase 0, with a resultant extreme slowing of conduction, but have little effect on the duration of the action potential. As a result, they are indicated for the treatment of paroxysmal tachycardia. Class Ib drugs depress phase 0 somewhat and actually shorten the duration of the action potential. The class II antiarrhythmics are beta-adrenergic blockers that block beta-receptors, causing a depression of phase 4 of the action potential. The class III antiarrhythmics block potassium channels and slow the outward movement of potassium during phase 3 of the action potential, thus prolonging it. 32. What class of antiarrhythmic agents does the nurse administer to slow the outward movement of potassium during phase 3 of the action potential? A) Class Ib B) Class Ic C) Class II D) Class III Ans: D Feedback: The class III antiarrhythmics block potassium channels and slow the outward movement of potassium during phase 3 of the action potential, prolonging it. Class Ib drugs depress phase 0 somewhat and actually shorten the duration of the action potential. Class Ic drugs markedly depress phase 0, with a resultant extreme slowing of conduction, but have little effect on the duration of the action potential. The class II antiarrhythmics are beta- adrenergic blockers that block beta-receptors, causing a depression of phase 4 of the action potential. 33. What class of antiarrhythmic agents does the nurse administer to depress generation of the action potentials and delaying phases 1 and 2 of repolarization? A) Class Ib B) Class II C) Class III D) Class IV Ans: D Feedback: The class IV antiarrhythmics block the movement of calcium ions across the cell membrane, depressing the generation of action potentials and delaying phases 1 and 2 of repolarization, which slows automaticity and conduction. Class Ib drugs depress phase 0 somewhat and actually shorten the duration of the action potential. The class II antiarrhythmics are beta-adrenergic blockers that block beta-receptors, causing a depression of phase 4 of the action potential. The class III antiarrhythmics block potassium channels and slow the outward movement of potassium during phase 3 of the action potential, thus prolonging it. 34. The nurse is teaching a class for newly hired cardiac care nurses and is discussing dronedarone (Multaq). What statement, if made by the nurse, is accurate? (Select all that apply.) A) Dronedarone has properties of all four classes of antiarrhythmics. B) Dronedarone reduces the risk of hospitalization in patients in atrial fibrillation. C) Dronedarone has many drugdrug interactions that need to be reviewed. D) It is an oral drug taken once a day. E) Common adverse effects of dronedarone include ventricular arrhythmias. Ans: A, B, C Feedback: Dronedarone has properties of all four classes of antiarrhythmics and the mechanism by which it helps suppress atrial arrhythmias is not fully understood. It is used to reduce the risk of hospitalization in patients with paroxysmal or persistent atrial fibrillation or flutter who have risk factors for cardiovascular disease and who are in sinus rhythm or are scheduled to be converted to sinus rhythm. The drug is taken orally twice a day. Many drugdrug interactions have been associated with the drug and this situation should always be reviewed before starting or stopping any drugs while on this drug. Grapefruit juice should not be consumed while taking this drug. The most common adverse effects seen with dronedarone are heart failure, prolonged QT interval, nausea, diarrhea, and rash. It should never be used during pregnancy because it has been associated with fetal abnormalities. 35. What class of antiarrhythmic agent would the nurse be most likely to administer to a lactating new mother? A) Class Ib B) Class II C) Class III D) Class IV Ans: B Feedback: Class I, III, and IV agents should not be used during lactation; if they are needed, another method of feeding the baby should be used. This leaves only class II drugs for consideration if the mother is lactating. Anginas: 1. A nurse is providing patient teaching to a patient who has been experiencing unstable angina. What will the nurses explanation of this condition include? A) A coronary vessel has become completely occluded and is unable to deliver blood to your heart. B) The pain is caused by a spasm of a blood vessel, not just from the vessel narrowing. C) There is serious narrowing of a coronary artery that is causing a reduction in oxygen to the heart. D) Your bodys response to a lack of oxygen in the heart muscle is pain. Ans: C Feedback: Unstable angina is described as increased narrowing of coronary arteries with the heart experiencing episodes of ischemia even at rest. If a coronary vessel is completely occluded and unable to deliver blood to the cardiac muscle, a myocardial infarction has occurred. Prinzmetals angina is an unusual form of angina caused by spasm of the blood vessel and not just by vessel narrowing. Although pain is the bodys response to ischemia in the heart muscle, this description could encompass angina or a myocardial infarction and is not specific enough to explain the condition. 2. The nurse cautions the patient taking nadolol (Corgard) for angina that they may experience what adverse effect? A) Dry mouth B) Decreased exercise tolerance C) Constipation D) Problems with urination Ans: B Feedback: Nadolol is a beta-blocker that can cause a decreased tolerance to exercise because of the inability to experience the effects of the stress reaction. Dry mouth, constipation, and problems with urination are effects often seen with anticholinergic drugs but not with beta-blockers. 3. A patient who has been taking cyclosporine to prevent rejection of a kidney transplant has had diltiazem ordered. Why would the nurse question this order? A) Serious diltiazem toxicity could occur. B) The combination may result in elevated or even toxic cyclosporine levels. C) The combination could lead to kidney rejection. D) A kidney recipient would not effectively excrete the diltiazem. Ans: B Feedback: Potentially serious adverse effects to keep in mind include increased serum levels and toxicity of cyclosporine if they are taken with diltiazem. This combination is not associated with diltiazem toxicity. A functioning implanted kidney should still excrete diltiazem. This drug would not cause rejection of a transplanted kidney. 4. A nurse is teaching the patient newly prescribed sublingual nitroglycerin how to take the medication. What will the nurse instruct the patient to do first? A) To check his radial pulse B) To place the tablet in the buccal cavity C) To take a sip of water D) To lie down for 15 minutes before administration Ans: C Feedback: The nurse should instruct the patient to take a sip of water to moisten the mucous membranes so the tablet will dissolve quickly. The patient does not need to take his pulse or lie down before drug administration. For sublingual administration, the patient will place the tablet under his tongue and not in the buccal cavity (cheek area). 5. The nurse, caring for a patient taking a beta-blocker and a nitrate to treat angina, recognizes the need for careful monitoring as the result of what secondary diagnosis? A) Chronic obstructive pulmonary disease (COPD) B) Rheumatoid arthritis (RA) C) Irritable bowel syndrome (IBS) D) Chronic urinary tract infection (UTI) Ans: A Feedback: The nurse should assess for COPD, because the effect of beta-blockers in reducing effects of the sympathetic nervous system could exacerbate the respiratory condition. RA, IBS, and chronic UTI are not affected by the use of beta-blockers or nitrates to treat angina. 6. What drug would the nurse administer to the patient to control angina caused by atherosclerosis that would also slow the development of further plaque buildup on the arterial wall? A) Diltiazem (Cardizem) B) Propranolol (Inderal) C) Amyl Nitrates (generic) D) Isosorbide dinitrate (Isordil) Ans: A Feedback: Diltiazem is a calcium channel blocker that is indicated to treat Prinzmetals angina, chronic angina, effort-associated angina, and hypertension. Research has indicated these agents slow the development of atherosclerosis. Beta- blockers are indicated for long-term management of angina caused by atherosclerosis, but they do not slow the development of plaque deposits on the artery wall. Propranolol is a beta-blocker. Isosorbide dinitrate and amyl nitrate are nitrates and are indicated for relief acute anginal pain, but they are not used to prevent angina and have no effect on the progression of atherosclerosis. 7. An older adult patient who is taking metformin (Glucophage) has just been seen in the clinic. The doctor has ordered metoprolol (Toprol) for angina. What assessment data should the nurse monitor due to this drug combination? A) Blood pressure B) Blood glucose C) Heart rate D) Intake and output Ans: B Feedback: Metformin is an antidiabetic drug and the nurse should monitor the patients blood glucose frequently throughout the day. The patient will not have the usual signs and symptoms of hypoglycemia or hyperglycemia. Blood pressure, heart rate, and intake and output would not be affected by this drugdrug combination. 8. A patient is to receive diltiazem (Cardizem) 360 mg/d orally in four divided doses. How many mg will the nurse administer per dose? A) 30 mg B) 60 mg C) 90 mg D) 120 mg Ans: C Feedback: The patient will take 90 mg/dose (360 divided by 4 doses equals 90 mg/ dose). Therefore, the other options are incorrect. 9. The patient, diagnosed with angina, tells the nurse he is having chest pain. There is an order for oral sublingual nitroglycerin as needed. What action should the nurse take? A) Place two nitroglycerin tablets under the patients tongue and call the physician. B) Place one tablet under the patients tongue and repeat every 5 minutes for total of three tablets until pain has been relieved. C) Have the patient swallow a tablet with a full glass of water and repeat in 10 minutes. D) Apply a nitroglycerin transdermal patch to the patients back. Ans: B Feedback: The correct administration for sublingual administration is to place one tablet under the patients tongue and repeat every 5 minutes for a total of three tablets until pain is relieved. If pain is not relieved after three sublingual tablets, the health care provider should be notified. Transdermal application would be inappropriate and nitroglycerin is not swallowed. Administering two tablets at one time would be an inappropriate dosage and could cause serious adverse effects. 10. The nurse is caring for a patient who is complaining of chest pain. The nurse is to administer 40 mg of isosorbide dinitrate (Isordil) to the patient. What is the nurses priority assessment before administering the drug? A) Jaundice B) Headache C) Anemia D) Sinusitis Ans: C Feedback: The nurse should assess for anemia because the decrease in cardiac output could be detrimental in a patient who already has a decreased ability to deliver oxygen because of a low red blood cell count. Jaundice and sinusitis would not be a contraindication to the drug. Headache is an adverse effect of isosorbide and would be expected after administration of the drug. 11. A 49-year-old patient is admitted with uncontrolled chest pain. He is currently taking nitroglycerin (Nitrostat). His physician orders nifedipine (Adalat) added to his regimen. The nurse should observe the patient for what adverse effects? A) Hypokalemia B) Renal insufficiency C) Hypotension D) Hypoglycemia Ans: C Feedback: Both nitroglycerin and nifedipine have hypotension as a potential adverse effect so frequent assessment of blood pressure is important. Other cardiovascular effects include bradycardia, peripheral edema, and heart block. Skin effects include flushing and rash. Adverse effects do not include renal insufficiency, hypokalemia, or hypoglycemia. 12. What statements by the 54-year-old patient indicates an understanding of the nurses teaching about how to take sublingual nitroglycerin? A) A headache means a toxic level has been reached. B) I can take up to 3 tablets at 5-minute intervals. C) I can take as much nitroglycerin as I need because it is not habit forming. D) If I become dizzy after taking the medication, I should stop taking it. Ans: B Feedback: Sublingual nitroglycerin may be taken at 5-minute intervals up to a maximum of three doses to relieve anginal chest pain. Headaches are very common due to vasodilation and do not indicate a toxic level. Nitroglycerin causes significant peripheral vasodilation in addition to its therapeutic effects of coronary artery dilation so no more than three tablets should be taken, even though it is not habit forming. Dizziness could be an adverse effect of the drug or a manifestation of inadequate cardiac output, but it would not indicate the patient should stop taking it. 13. When the nurse administers a beta-adrenergic blocker to the patient with angina, the nurse expects the drug will help to control angina, but it also has what other effect? A) Increased heart rate B) Increased oxygen consumption C) Decreased strength of heart muscle contraction D) Decreased urinary output Ans: C Feedback: Beta-blockers competitively block beta-adrenergic receptors in the heart and kidneys, decreasing the influence of the sympathetic nervous system on these tissues and the excitability of the heart. As a result, it decreases the strength of cardiac contraction, reducing cardiac output, which results in lowered blood pressure and decreased cardiac workload. It does not impact urinary output. 14. The nurse is caring for a patient who takes nitroglycerin sublingually. When providing patient education, the nurse would tell the patient that she can expect relief of chest pain within what period of time? A) 1 to 3 minutes B) 5 to 10 minutes C) 15 to 20 minutes D) 30 to 60 minutes Ans: A Feedback: Nitroglycerin acts within 1 to 3 minutes. Other options are incorrect. 15. When providing patient education about nitroglycerin to the patient, what would the nurse include in the teaching plan about a nitroglycerin patch? A) It only has to be administered once a week. B) It is more effective than tablets in treating angina. C) It has a longer duration of action. D) It is faster acting than the tablets. Ans: C Feedback: Transdermal nitroglycerin has a long 24-hour duration of action compared with the sublingual form that lasts 30 to 60 minutes or oral tablets that last 8 to 12 hours. Transdermal patches are neither more nor less effective, but rather it is the speed of onset and duration of action that differ. 16. Which statement by the patient would lead the nurse to believe that he has understood the teaching provided regarding angina? A) I will not exercise because it precipitates angina. B) As long as I take the medicine, I need make no lifestyle change. C) There is no correlation between my hypertension and angina. D) Heavy meals and cigarette smoking can precipitate an angina attack. Ans: D Feedback: Avoid stressful activities, especially in combination. For example, if you eat a big meal, do not drink coffee or alcoholic beverages with that meal. If you have just eaten a big meal, do not climb stairs; rest for a while. However, exercise is important and should not be eliminated, but managed in coordination with other activities. Smoking causes vasoconstriction that can result in angina attacks so lifestyle changes like reducing fat and calories in the diet, moderate exercise, reducing alcohol intake and avoiding smoking are all healthful choices. Hypertension does increase the risk of angina and coronary artery disease. 17. The nurse is caring for a patient who is taking a calcium-channel blocker. What adverse effects would the nurse caution this patient about? A) Hypertension and tachycardia B) Headache and dizziness C) Itching and rash D) Nausea and diarrhea Ans: B Feedback: The adverse effects associated with these drugs are related to their effects on cardiac output and on smooth muscle. Central nervous system (CNS) effects include dizziness, light-headedness, headache, and fatigue. Gastrointestinal (GI) effects can include nausea and hepatic injury related to direct toxic effects on hepatic cells. Cardiovascular effects include hypotension, bradycardia, peripheral edema, and heart block. Skin effects include flushing and rash. The adverse effects do not, however, include diarrhea, hypertension, tachycardia, or itching. 18. The nurse assesses patients receiving nifedipine (Adalat) for what adverse effects? A) Ascites B) Asthma C) Peripheral edema D) Tetany Ans: C Feedback: Peripheral edema, heart block, bradycardia, and hypotension can occur with calcium-channel blockers. Asthma, ascites, and tetany are not associated with these drugs. 19. What adverse reaction does the nurse anticipate if the 56-year-old patient takes his beta-blocker with clonidine? A) Hypertension B) Bradycardia C) Angina D) Syncope Ans: A Feedback: A paradoxical hypertension occurs when clonidine is given with beta- blockers, and an increased rebound hypertension with clonidine withdrawal may also occur. It is best to avoid this combination. Bradycardia, angina, and syncope are not associated with this drug combination. 20. An asthmatic patient taking beta-blockers should be assessed by the nurse for what potential adverse reaction? A) Bronchospasm B) Hypoglycemia C) Pleural effusion D) Pneumonia Ans: A Feedback: Bronchospasm can occur with beta blockade. The patient would not have to be observed for hypoglycemia, pleural effusion, or pneumonia. 21. The nurse should instruct the patient to take what action if three nitroglycerin tablets taken sublingually are not effective in eliminating chest pain? A) To call 911 B) To call her health care provider C) To lie down after taking an aspirin D) To take more tablets until pain subsides Ans: A Feedback: Instruct patient that a sublingual dose may be repeated in 5 minutes if relief is not felt, for a total of three doses, if pain persists, the patient or a family member should call 911 to ensure proper medical support if a myocardial infarction should occur. She should not waste time by calling the health care provider; she can lie down while waiting for the ambulance to arrive, she should not take more tablets. 22. The nurse is caring for a patient who takes metoprolol for angina. The patient asks how long it takes for the medicine to work. What is the nurses best response? A) 15 minutes B) 30 minutes C) 1 hour D) 90 minutes Ans: A Feedback: Oral metoprolol has an onset of action of 15 minutes. Other options are incorrect. 23. The nurse teaches the patient wearing a nitroglycerin patch to avoid what? A) Exercise B) Alcoholic beverages C) Milk products D) Synthetic fabrics Ans: B Feedback: Patients should be taught to avoid or at least decrease use of coffee, cigarettes, and alcoholic beverages. There is no need to avoid exercise, milk, or synthetic fabrics. 24. The nurse is caring for a patient who is taking a sustained-release (SR) oral nitrate. How should the nurse instruct this patient to take the medication? A) With water B) Sublingually until absorbed C) With milk or milk products D) 1 hour after eating Ans: A Feedback: Give SR forms with water, and caution the patient not to chew or crush them, because these preparations need to reach the gastrointestinal (GI) tract intact to avoid overdosage. They are not dissolved sublingually but swallowed whole. They are best taken on an empty stomach 1 hour before meals. 25. An adult patient who experiences angina pectoris with exertion is informed by the nurse that the leading cause of angina is what? A) Smoking B) Inadequate cardiac output C) Infarction of the myocardium D) Coronary atherosclerosis Ans: D Feedback: The person with atherosclerosis has a classic supply-and-demand problem. The heart may function without a problem until increases in activity or other stresses place a demand on it to beat faster or harder. Normally, the heart would stimulate the vessels to deliver more blood when this occurs, but the narrowed vessels are not able to respond and cannot supply the blood needed by the working heart. The heart muscle then becomes hypoxic. This imbalance between oxygen supply and demand is manifested as pain, or angina pectoris, which literally means suffocation of the chest. Atherosclerosis of the coronary artery can block the coronary artery completely leading to infarction. Smoking causes further vasoconstriction, increasing risk of myocardial infarction or angina. Damage to the heart muscle causes a decrease in cardiac output. 26. The nurse is caring for a patient prescribed ranolazine. The patient asks why this drug is different from the beta-blocker that he was previously taking. What is the nurses best response? A) This drug does not slow your heart rate. B) This drug increases myocardial oxygen demand. C) This drug slows the QT intervals. D) This maintains blood pressure with no hypotensive effects. Ans: A Feedback: The newest drug approved for the treatment of angina is a piperazine acetamide agent called ranolazine. The mechanism of action of this drug is not understood. It prolongs QT intervals, does not slow heart rate or blood pressure, but decreases myocardial oxygen demand. 27. After the patients anginal pain is relieved he says to the nurse, That nitroglycerin works great. How does it do that? What is the nurses best response? A) Nitroglycerin decreases the amount of oxygen needed by the heart to function. B) Nitroglycerin makes the coronary arteries open much wider. C) Nitroglycerin promotes growth of new, smaller arteries to supply oxygen to the heart. D) Nitroglycerin decreases preload and afterload. Ans: A Feedback: The main effect of nitrates seems to be related to the drop in blood pressure that occurs. The vasodilation causes blood to pool in veins and capillaries, decreasing preload, while the relaxation of the vessels decreases afterload. The combination of these effects greatly reduces the cardiac workload and the demand for oxygen, thus bringing the supply-and-demand ratio back into balance. Because coronary artery disease causes a stiffening and lack of responsiveness in the coronary arteries, the nitrates probably have very little effect on increasing blood flow through the coronary arteries, so it would be incorrect to say that the coronary arteries become much wider. Although both preload and afterload are reduced, this is an explanation that the patient would not understand, so it is inappropriate. Nitroglycerin does not promote growth of compensatory circulation. 28. An adult patient has had symptoms of unstable angina during admission to the hospital. What is the most appropriate nursing diagnosis? A) Deficient knowledge about underlying disease and methods for avoiding complications B) Anxiety related to fear of death C) Ineffective tissue perfusion (total body) related to reduced oxygen supply to the heart D) Noncompliance related to failure to accept necessary lifestyle changes Ans: C Feedback: The most appropriate nursing diagnosis is ineffective tissue perfusion related to reduced oxygen supply to the heart because this is the cause of the patients pain. Further assessment would be needed to determine whether the patient lacks knowledge, fears death, or has made the necessary lifestyle changes. 29. The nurse is preparing to administer sublingual nitroglycerin to a patient for the first time and warns that the patient may experience what right after administration? A) Nervousness or paresthesia B) Throbbing headache or dizziness C) Drowsiness or blurred vision D) Tinnitus or diplopia Ans: B Feedback: Headache and dizziness commonly occur at the start of nitroglycerin therapy. When administering nitroglycerin, the nurse must use caution to avoid self-contamination, especially with the topical paste formulation because the nurse can experience the same symptoms. However, the patient usually develops a tolerance. Nervousness, paresthesia, drowsiness, blurred vision, tinnitus, and diplopia do not occur as a result of nitroglycerin therapy. 30. A patient is diagnosed with Prinzmetals angina. What drug would the nurse expect to administer to this patient? A) Nadolol B) Diltiazem C) Propranolol D) Metoprolol Ans: B Feedback: Calcium channel blockers are indicated for the treatment of Prinzmetals angina because these drugs relieve coronary artery vasospasm, increasing blood flow to the muscle cells. Diltiazem is a calcium channel blocker. Beta- blockers are not indicated for the treatment of Prinzmetals angina because they could cause vasospasm due to blocking of beta-receptor sites. Propranolol, nadolol, and metoprolol are beta-blockers. 31. The nurse is caring for a patient diagnosed with human immunodeficiency virus (HIV) and newly diagnosed angina. What drug would the nurse question if ordered? A) Ranolazine (Ranexa) B) Nitroglycerin (generic) C) Propranolol D) Diltiazem Ans: A Feedback: Drugdrug interactions can occur with ketoconazole, diltiazem, verapamil, macrolide antibiotics, and HIV protease inhibitors; these combinations should be avoided because ranolazine levels may become extremely high. The nurse should get a complete list of current medications and consult with the health care provider regarding drug interactions before administering the medication as ordered. Nitroglycerin, propranolol, and diltiazem have no contraindication with medications used to treat HIV. 32. Other than angina, what other medical condition might the nurse treat by administering nitroglycerin? A) Muscular dystrophy B) Pulmonary embolisms C) Polycythemia D) Anal fissures Ans: D Feedback: In 2011, a nitroglycerin in ointment form, Rectiv 0.4%, was approved for the treatment of moderate to severe anal fissures. There is no documented benefit to the use of nitroglycerin to treat muscular dystrophy, pulmonary embolisms, or polycythemia. 33. What would the nurse teach the patient about potency of nitroglycerin? (Select all that apply.) A) The tablet should fizzle or burn when placed under the tongue. B) Protect the drug from heat and light. C) Always replace when past the expiration date. D) Older tablets may require you to use two tablets at one time. E) Nitroglycerin does not lose its potency easily. Ans: A, B, C Feedback: Ask the patient if the tablet fizzles or burns, which indicates potency. Always check the expiration date on the bottle and protect the medication from heat and light because these drugs are volatile and lose potency easily. 34. The nurse teaches the patient how to use topical nitroglycerin and includes what teaching points in the teaching plan? (Select all that apply.) A) Rotate application sites. B) Assess the skin for signs of breakdown. C) Make sure no one touches the side with the medication. D) Do not shower with the patch in place. E) Increase fluid intake to avoid hypotension. Ans: A, B, C Feedback: Rotate the sites of topical forms of nitroglycerin to lower the risk of skin abrasion and breakdown; monitor for signs of skin breakdown to arrange for appropriate skin care as needed. Care should be taken not to touch the side of the patch with the medication by the patient or anyone assisting in applying the patch. The patient may shower with the patch in place. There is no need to increase fluid intake but patients should be encouraged to maintain adequate intake. 35. The provider orders isosorbide dinitrate as oral maintenance drug to prevent angina. What dosage would the nurse administer without need to question the dose? A) 2.5 mg B) 80 mg C) 40 mg D) 5 mg Ans: B Feedback: A maintenance oral dose of isosorbide dinitrate is 10 to 40 mg every 6 hours. Sublingual dose is 2.5 to 5 mg, sustained release is 40 to 80 mg, and the chewable tablet is 5 mg. Blood Pressure: 1. A nurse is providing patient teaching to a patient who has been experiencing unstable angina. What will the nurses explanation of this condition include? A) A coronary vessel has become completely occluded and is unable to deliver blood to your heart. B) The pain is caused by a spasm of a blood vessel, not just from the vessel narrowing. C) There is serious narrowing of a coronary artery that is causing a reduction in oxygen to the heart. D) Your bodys response to a lack of oxygen in the heart muscle is pain. Ans: C Feedback: Unstable angina is described as increased narrowing of coronary arteries with the heart experiencing episodes of ischemia even at rest. If a coronary vessel is completely occluded and unable to deliver blood to the cardiac muscle, a myocardial infarction has occurred. Prinzmetals angina is an unusual form of angina caused by spasm of the blood vessel and not just by vessel narrowing. Although pain is the bodys response to ischemia in the heart muscle, this description could encompass angina or a myocardial infarction and is not specific enough to explain the condition. 2. The nurse cautions the patient taking nadolol (Corgard) for angina that they may experience what adverse effect? A) Dry mouth B) Decreased exercise tolerance C) Constipation D) Problems with urination Ans: B Feedback: Nadolol is a beta-blocker that can cause a decreased tolerance to exercise because of the inability to experience the effects of the stress reaction. Dry mouth, constipation, and problems with urination are effects often seen with anticholinergic drugs but not with beta-blockers. 3. A patient who has been taking cyclosporine to prevent rejection of a kidney transplant has had diltiazem ordered. Why would the nurse question this order? A) Serious diltiazem toxicity could occur. B) The combination may result in elevated or even toxic cyclosporine levels. C) The combination could lead to kidney rejection. D) A kidney recipient would not effectively excrete the diltiazem. Ans: B Feedback: Potentially serious adverse effects to keep in mind include increased serum levels and toxicity of cyclosporine if they are taken with diltiazem. This combination is not associated with diltiazem toxicity. A functioning implanted kidney should still excrete diltiazem. This drug would not cause rejection of a transplanted kidney. 4. A nurse is teaching the patient newly prescribed sublingual nitroglycerin how to take the medication. What will the nurse instruct the patient to do first? A) To check his radial pulse B) To place the tablet in the buccal cavity C) To take a sip of water D) To lie down for 15 minutes before administration Ans: C Feedback: The nurse should instruct the patient to take a sip of water to moisten the mucous membranes so the tablet will dissolve quickly. The patient does not need to take his pulse or lie down before drug administration. For sublingual administration, the patient will place the tablet under his tongue and not in the buccal cavity (cheek area). 5. The nurse, caring for a patient taking a beta-blocker and a nitrate to treat angina, recognizes the need for careful monitoring as the result of what secondary diagnosis? A) Chronic obstructive pulmonary disease (COPD) B) Rheumatoid arthritis (RA) C) Irritable bowel syndrome (IBS) D) Chronic urinary tract infection (UTI) Ans: A Feedback: The nurse should assess for COPD, because the effect of beta-blockers in reducing effects of the sympathetic nervous system could exacerbate the respiratory condition. RA, IBS, and chronic UTI are not affected by the use of beta-blockers or nitrates to treat angina. 6. What drug would the nurse administer to the patient to control angina caused by atherosclerosis that would also slow the development of further plaque buildup on the arterial wall? A) Diltiazem (Cardizem) B) Propranolol (Inderal) C) Amyl Nitrates (generic) D) Isosorbide dinitrate (Isordil) Ans: A Feedback: Diltiazem is a calcium channel blocker that is indicated to treat Prinzmetals angina, chronic angina, effort-associated angina, and hypertension. Research has indicated these agents slow the development of atherosclerosis. Beta- blockers are indicated for long-term management of angina caused by atherosclerosis, but they do not slow the development of plaque deposits on the artery wall. Propranolol is a beta-blocker. Isosorbide dinitrate and amyl nitrate are nitrates and are indicated for relief acute anginal pain, but they are not used to prevent angina and have no effect on the progression of atherosclerosis. 7. An older adult patient who is taking metformin (Glucophage) has just been seen in the clinic. The doctor has ordered metoprolol (Toprol) for angina. What assessment data should the nurse monitor due to this drug combination? A) Blood pressure B) Blood glucose C) Heart rate D) Intake and output Ans: B Feedback: Metformin is an antidiabetic drug and the nurse should monitor the patients blood glucose frequently throughout the day. The patient will not have the usual signs and symptoms of hypoglycemia or hyperglycemia. Blood pressure, heart rate, and intake and output would not be affected by this drugdrug combination. 8. A patient is to receive diltiazem (Cardizem) 360 mg/d orally in four divided doses. How many mg will the nurse administer per dose? A) 30 mg B) 60 mg C) 90 mg D) 120 mg Ans: C Feedback: The patient will take 90 mg/dose (360 divided by 4 doses equals 90 mg/ dose). Therefore, the other options are incorrect. 9. The patient, diagnosed with angina, tells the nurse he is having chest pain. There is an order for oral sublingual nitroglycerin as needed. What action should the nurse take? A) Place two nitroglycerin tablets under the patients tongue and call the physician. B) Place one tablet under the patients tongue and repeat every 5 minutes for total of three tablets until pain has been relieved. C) Have the patient swallow a tablet with a full glass of water and repeat in 10 minutes. D) Apply a nitroglycerin transdermal patch to the patients back. Ans: B Feedback: The correct administration for sublingual administration is to place one tablet under the patients tongue and repeat every 5 minutes for a total of three tablets until pain is relieved. If pain is not relieved after three sublingual tablets, the health care provider should be notified. Transdermal application would be inappropriate and nitroglycerin is not swallowed. Administering two tablets at one time would be an inappropriate dosage and could cause serious adverse effects. 10. The nurse is caring for a patient who is complaining of chest pain. The nurse is to administer 40 mg of isosorbide dinitrate (Isordil) to the patient. What is the nurses priority assessment before administering the drug? A) Jaundice B) Headache C) Anemia D) Sinusitis Ans: C Feedback: The nurse should assess for anemia because the decrease in cardiac output could be detrimental in a patient who already has a decreased ability to deliver oxygen because of a low red blood cell count. Jaundice and sinusitis would not be a contraindication to the drug. Headache is an adverse effect of isosorbide and would be expected after administration of the drug. 11. A 49-year-old patient is admitted with uncontrolled chest pain. He is currently taking nitroglycerin (Nitrostat). His physician orders nifedipine (Adalat) added to his regimen. The nurse should observe the patient for what adverse effects? A) Hypokalemia B) Renal insufficiency C) Hypotension D) Hypoglycemia Ans: C Feedback: Both nitroglycerin and nifedipine have hypotension as a potential adverse effect so frequent assessment of blood pressure is important. Other cardiovascular effects include bradycardia, peripheral edema, and heart block. Skin effects include flushing and rash. Adverse effects do not include renal insufficiency, hypokalemia, or hypoglycemia. 12. What statements by the 54-year-old patient indicates an understanding of the nurses teaching about how to take sublingual nitroglycerin? A) A headache means a toxic level has been reached. B) I can take up to 3 tablets at 5-minute intervals. C) I can take as much nitroglycerin as I need because it is not habit forming. D) If I become dizzy after taking the medication, I should stop taking it. Ans: B Feedback: Sublingual nitroglycerin may be taken at 5-minute intervals up to a maximum of three doses to relieve anginal chest pain. Headaches are very common due to vasodilation and do not indicate a toxic level. Nitroglycerin causes significant peripheral vasodilation in addition to its therapeutic effects of coronary artery dilation so no more than three tablets should be taken, even though it is not habit forming. Dizziness could be an adverse effect of the drug or a manifestation of inadequate cardiac output, but it would not indicate the patient should stop taking it. 13. When the nurse administers a beta-adrenergic blocker to the patient with angina, the nurse expects the drug will help to control angina, but it also has what other effect? A) Increased heart rate B) Increased oxygen consumption C) Decreased strength of heart muscle contraction D) Decreased urinary output Ans: C Feedback: Beta-blockers competitively block beta-adrenergic receptors in the heart and kidneys, decreasing the influence of the sympathetic nervous system on these tissues and the excitability of the heart. As a result, it decreases the strength of cardiac contraction, reducing cardiac output, which results in lowered blood pressure and decreased cardiac workload. It does not impact urinary output. 14. The nurse is caring for a patient who takes nitroglycerin sublingually. When providing patient education, the nurse would tell the patient that she can expect relief of chest pain within what period of time? A) 1 to 3 minutes B) 5 to 10 minutes C) 15 to 20 minutes D) 30 to 60 minutes Ans: A Feedback: Nitroglycerin acts within 1 to 3 minutes. Other options are incorrect. 15. When providing patient education about nitroglycerin to the patient, what would the nurse include in the teaching plan about a nitroglycerin patch? A) It only has to be administered once a week. B) It is more effective than tablets in treating angina. C) It has a longer duration of action. D) It is faster acting than the tablets. Ans: C Feedback: Transdermal nitroglycerin has a long 24-hour duration of action compared with the sublingual form that lasts 30 to 60 minutes or oral tablets that last 8 to 12 hours. Transdermal patches are neither more nor less effective, but rather it is the speed of onset and duration of action that differ. 16. Which statement by the patient would lead the nurse to believe that he has understood the teaching provided regarding angina? A) I will not exercise because it precipitates angina. B) As long as I take the medicine, I need make no lifestyle change. C) There is no correlation between my hypertension and angina. D) Heavy meals and cigarette smoking can precipitate an angina attack. Ans: D Feedback: Avoid stressful activities, especially in combination. For example, if you eat a big meal, do not drink coffee or alcoholic beverages with that meal. If you have just eaten a big meal, do not climb stairs; rest for a while. However, exercise is important and should not be eliminated, but managed in coordination with other activities. Smoking causes vasoconstriction that can result in angina attacks so lifestyle changes like reducing fat and calories in the diet, moderate exercise, reducing alcohol intake and avoiding smoking are all healthful choices. Hypertension does increase the risk of angina and coronary artery disease. 17. The nurse is caring for a patient who is taking a calcium-channel blocker. What adverse effects would the nurse caution this patient about? A) Hypertension and tachycardia B) Headache and dizziness C) Itching and rash D) Nausea and diarrhea Ans: B Feedback: The adverse effects associated with these drugs are related to their effects on cardiac output and on smooth muscle. Central nervous system (CNS) effects include dizziness, light-headedness, headache, and fatigue. Gastrointestinal (GI) effects can include nausea and hepatic injury related to direct toxic effects on hepatic cells. Cardiovascular effects include hypotension, bradycardia, peripheral edema, and heart block. Skin effects include flushing and rash. The adverse effects do not, however, include diarrhea, hypertension, tachycardia, or itching. 18. The nurse assesses patients receiving nifedipine (Adalat) for what adverse effects? A) Ascites B) Asthma C) Peripheral edema D) Tetany Ans: C Feedback: Peripheral edema, heart block, bradycardia, and hypotension can occur with calcium-channel blockers. Asthma, ascites, and tetany are not associated with these drugs. 19. What adverse reaction does the nurse anticipate if the 56-year-old patient takes his beta-blocker with clonidine? A) Hypertension B) Bradycardia C) Angina D) Syncope Ans: A Feedback: A paradoxical hypertension occurs when clonidine is given with beta- blockers, and an increased rebound hypertension with clonidine withdrawal may also occur. It is best to avoid this combination. Bradycardia, angina, and syncope are not associated with this drug combination. 20. An asthmatic patient taking beta-blockers should be assessed by the nurse for what potential adverse reaction? A) Bronchospasm B) Hypoglycemia C) Pleural effusion D) Pneumonia Ans: A Feedback: Bronchospasm can occur with beta blockade. The patient would not have to be observed for hypoglycemia, pleural effusion, or pneumonia. 21. The nurse should instruct the patient to take what action if three nitroglycerin tablets taken sublingually are not effective in eliminating chest pain? A) To call 911 B) To call her health care provider C) To lie down after taking an aspirin D) To take more tablets until pain subsides Ans: A Feedback: Instruct patient that a sublingual dose may be repeated in 5 minutes if relief is not felt, for a total of three doses, if pain persists, the patient or a family member should call 911 to ensure proper medical support if a myocardial infarction should occur. She should not waste time by calling the health care provider; she can lie down while waiting for the ambulance to arrive, she should not take more tablets. 22. The nurse is caring for a patient who takes metoprolol for angina. The patient asks how long it takes for the medicine to work. What is the nurses best response? A) 15 minutes B) 30 minutes C) 1 hour D) 90 minutes Ans: A Feedback: Oral metoprolol has an onset of action of 15 minutes. Other options are incorrect. 23. The nurse teaches the patient wearing a nitroglycerin patch to avoid what? A) Exercise B) Alcoholic beverages C) Milk products D) Synthetic fabrics Ans: B Feedback: Patients should be taught to avoid or at least decrease use of coffee, cigarettes, and alcoholic beverages. There is no need to avoid exercise, milk, or synthetic fabrics. 24. The nurse is caring for a patient who is taking a sustained-release (SR) oral nitrate. How should the nurse instruct this patient to take the medication? A) With water B) Sublingually until absorbed C) With milk or milk products D) 1 hour after eating Ans: A Feedback: Give SR forms with water, and caution the patient not to chew or crush them, because these preparations need to reach the gastrointestinal (GI) tract intact to avoid overdosage. They are not dissolved sublingually but swallowed whole. They are best taken on an empty stomach 1 hour before meals. 25. An adult patient who experiences angina pectoris with exertion is informed by the nurse that the leading cause of angina is what? A) Smoking B) Inadequate cardiac output C) Infarction of the myocardium D) Coronary atherosclerosis Ans: D Feedback: The person with atherosclerosis has a classic supply-and-demand problem. The heart may function without a problem until increases in activity or other stresses place a demand on it to beat faster or harder. Normally, the heart would stimulate the vessels to deliver more blood when this occurs, but the narrowed vessels are not able to respond and cannot supply the blood needed by the working heart. The heart muscle then becomes hypoxic. This imbalance between oxygen supply and demand is manifested as pain, or angina pectoris, which literally means suffocation of the chest. Atherosclerosis of the coronary artery can block the coronary artery completely leading to infarction. Smoking causes further vasoconstriction, increasing risk of myocardial infarction or angina. Damage to the heart muscle causes a decrease in cardiac output. 26. The nurse is caring for a patient prescribed ranolazine. The patient asks why this drug is different from the beta-blocker that he was previously taking. What is the nurses best response? A) This drug does not slow your heart rate. B) This drug increases myocardial oxygen demand. C) This drug slows the QT intervals. D) This maintains blood pressure with no hypotensive effects. Ans: A Feedback: The newest drug approved for the treatment of angina is a piperazine acetamide agent called ranolazine. The mechanism of action of this drug is not understood. It prolongs QT intervals, does not slow heart rate or blood pressure, but decreases myocardial oxygen demand. 27. After the patients anginal pain is relieved he says to the nurse, That nitroglycerin works great. How does it do that? What is the nurses best response? A) Nitroglycerin decreases the amount of oxygen needed by the heart to function. B) Nitroglycerin makes the coronary arteries open much wider. C) Nitroglycerin promotes growth of new, smaller arteries to supply oxygen to the heart. D) Nitroglycerin decreases preload and afterload. Ans: A Feedback: The main effect of nitrates seems to be related to the drop in blood pressure that occurs. The vasodilation causes blood to pool in veins and capillaries, decreasing preload, while the relaxation of the vessels decreases afterload. The combination of these effects greatly reduces the cardiac workload and the demand for oxygen, thus bringing the supply-and-demand ratio back into balance. Because coronary artery disease causes a stiffening and lack of responsiveness in the coronary arteries, the nitrates probably have very little effect on increasing blood flow through the coronary arteries, so it would be incorrect to say that the coronary arteries become much wider. Although both preload and afterload are reduced, this is an explanation that the patient would not understand, so it is inappropriate. Nitroglycerin does not promote growth of compensatory circulation. 28. An adult patient has had symptoms of unstable angina during admission to the hospital. What is the most appropriate nursing diagnosis? A) Deficient knowledge about underlying disease and methods for avoiding complications B) Anxiety related to fear of death C) Ineffective tissue perfusion (total body) related to reduced oxygen supply to the heart D) Noncompliance related to failure to accept necessary lifestyle changes Ans: C Feedback: The most appropriate nursing diagnosis is ineffective tissue perfusion related to reduced oxygen supply to the heart because this is the cause of the patients pain. Further assessment would be needed to determine whether the patient lacks knowledge, fears death, or has made the necessary lifestyle changes. 29. The nurse is preparing to administer sublingual nitroglycerin to a patient for the first time and warns that the patient may experience what right after administration? A) Nervousness or paresthesia B) Throbbing headache or dizziness C) Drowsiness or blurred vision D) Tinnitus or diplopia Ans: B Feedback: Headache and dizziness commonly occur at the start of nitroglycerin therapy. When administering nitroglycerin, the nurse must use caution to avoid self-contamination, especially with the topical paste formulation because the nurse can experience the same symptoms. However, the patient usually develops a tolerance. Nervousness, paresthesia, drowsiness, blurred vision, tinnitus, and diplopia do not occur as a result of nitroglycerin therapy. 30. A patient is diagnosed with Prinzmetals angina. What drug would the nurse expect to administer to this patient? A) Nadolol B) Diltiazem C) Propranolol D) Metoprolol Ans: B Feedback: Calcium channel blockers are indicated for the treatment of Prinzmetals angina because these drugs relieve coronary artery vasospasm, increasing blood flow to the muscle cells. Diltiazem is a calcium channel blocker. Beta- blockers are not indicated for the treatment of Prinzmetals angina because they could cause vasospasm due to blocking of beta-receptor sites. Propranolol, nadolol, and metoprolol are beta-blockers. 31. The nurse is caring for a patient diagnosed with human immunodeficiency virus (HIV) and newly diagnosed angina. What drug would the nurse question if ordered? A) Ranolazine (Ranexa) B) Nitroglycerin (generic) C) Propranolol D) Diltiazem Ans: A Feedback: Drugdrug interactions can occur with ketoconazole, diltiazem, verapamil, macrolide antibiotics, and HIV protease inhibitors; these combinations should be avoided because ranolazine levels may become extremely high. The nurse should get a complete list of current medications and consult with the health care provider regarding drug interactions before administering the medication as ordered. Nitroglycerin, propranolol, and diltiazem have no contraindication with medications used to treat HIV. 32. Other than angina, what other medical condition might the nurse treat by administering nitroglycerin? A) Muscular dystrophy B) Pulmonary embolisms C) Polycythemia D) Anal fissures Ans: D Feedback: In 2011, a nitroglycerin in ointment form, Rectiv 0.4%, was approved for the treatment of moderate to severe anal fissures. There is no documented benefit to the use of nitroglycerin to treat muscular dystrophy, pulmonary embolisms, or polycythemia. 33. What would the nurse teach the patient about potency of nitroglycerin? (Select all that apply.) A) The tablet should fizzle or burn when placed under the tongue. B) Protect the drug from heat and light. C) Always replace when past the expiration date. D) Older tablets may require you to use two tablets at one time. E) Nitroglycerin does not lose its potency easily. Ans: A, B, C Feedback: Ask the patient if the tablet fizzles or burns, which indicates potency. Always check the expiration date on the bottle and protect the medication from heat and light because these drugs are volatile and lose potency easily. 34. The nurse teaches the patient how to use topical nitroglycerin and includes what teaching points in the teaching plan? (Select all that apply.) A) Rotate application sites. B) Assess the skin for signs of breakdown. C) Make sure no one touches the side with the medication. D) Do not shower with the patch in place. E) Increase fluid intake to avoid hypotension. Ans: A, B, C Feedback: Rotate the sites of topical forms of nitroglycerin to lower the risk of skin abrasion and breakdown; monitor for signs of skin breakdown to arrange for appropriate skin care as needed. Care should be taken not to touch the side of the patch with the medication by the patient or anyone assisting in applying the patch. The patient may shower with the patch in place. There is no need to increase fluid intake but patients should be encouraged to maintain adequate intake. 35. The provider orders isosorbide dinitrate as oral maintenance drug to prevent angina. What dosage would the nurse administer without need to question the dose? A) 2.5 mg B) 80 mg C) 40 mg D) 5 mg Ans: B Feedback: A maintenance oral dose of isosorbide dinitrate is 10 to 40 mg every 6 hours. Sublingual dose is 2.5 to 5 mg, sustained release is 40 to 80 mg, and the chewable tablet is 5 mg. Blood Pressure : 1. A nurse is providing patient teaching to a patient who has been experiencing unstable angina. What will the nurses explanation of this condition include? A) A coronary vessel has become completely occluded and is unable to deliver blood to your heart. B) The pain is caused by a spasm of a blood vessel, not just from the vessel narrowing. C) There is serious narrowing of a coronary artery that is causing a reduction in oxygen to the heart. D) Your bodys response to a lack of oxygen in the heart muscle is pain. Ans: C Feedback: Unstable angina is described as increased narrowing of coronary arteries with the heart experiencing episodes of ischemia even at rest. If a coronary vessel is completely occluded and unable to deliver blood to the cardiac muscle, a myocardial infarction has occurred. Prinzmetals angina is an unusual form of angina caused by spasm of the blood vessel and not just by vessel narrowing. Although pain is the bodys response to ischemia in the heart muscle, this description could encompass angina or a myocardial infarction and is not specific enough to explain the condition. 2. The nurse cautions the patient taking nadolol (Corgard) for angina that they may experience what adverse effect? A) Dry mouth B) Decreased exercise tolerance C) Constipation D) Problems with urination Ans: B Feedback: Nadolol is a beta-blocker that can cause a decreased tolerance to exercise because of the inability to experience the effects of the stress reaction. Dry mouth, constipation, and problems with urination are effects often seen with anticholinergic drugs but not with beta-blockers. 3. A patient who has been taking cyclosporine to prevent rejection of a kidney transplant has had diltiazem ordered. Why would the nurse question this order? A) Serious diltiazem toxicity could occur. B) The combination may result in elevated or even toxic cyclosporine levels. C) The combination could lead to kidney rejection. D) A kidney recipient would not effectively excrete the diltiazem. Ans: B Feedback: Potentially serious adverse effects to keep in mind include increased serum levels and toxicity of cyclosporine if they are taken with diltiazem. This combination is not associated with diltiazem toxicity. A functioning implanted kidney should still excrete diltiazem. This drug would not cause rejection of a transplanted kidney. 4. A nurse is teaching the patient newly prescribed sublingual nitroglycerin how to take the medication. What will the nurse instruct the patient to do first? A) To check his radial pulse B) To place the tablet in the buccal cavity C) To take a sip of water D) To lie down for 15 minutes before administration Ans: C Feedback: The nurse should instruct the patient to take a sip of water to moisten the mucous membranes so the tablet will dissolve quickly. The patient does not need to take his pulse or lie down before drug administration. For sublingual administration, the patient will place the tablet under his tongue and not in the buccal cavity (cheek area). 5. The nurse, caring for a patient taking a beta-blocker and a nitrate to treat angina, recognizes the need for careful monitoring as the result of what secondary diagnosis? A) Chronic obstructive pulmonary disease (COPD) B) Rheumatoid arthritis (RA) C) Irritable bowel syndrome (IBS) D) Chronic urinary tract infection (UTI) Ans: A Feedback: The nurse should assess for COPD, because the effect of beta-blockers in reducing effects of the sympathetic nervous system could exacerbate the respiratory condition. RA, IBS, and chronic UTI are not affected by the use of beta-blockers or nitrates to treat angina. 6. What drug would the nurse administer to the patient to control angina caused by atherosclerosis that would also slow the development of further plaque buildup on the arterial wall? A) Diltiazem (Cardizem) B) Propranolol (Inderal) C) Amyl Nitrates (generic) D) Isosorbide dinitrate (Isordil) Ans: A Feedback: Diltiazem is a calcium channel blocker that is indicated to treat Prinzmetals angina, chronic angina, effort-associated angina, and hypertension. Research has indicated these agents slow the development of atherosclerosis. Beta- blockers are indicated for long-term management of angina caused by atherosclerosis, but they do not slow the development of plaque deposits on the artery wall. Propranolol is a beta-blocker. Isosorbide dinitrate and amyl nitrate are nitrates and are indicated for relief acute anginal pain, but they are not used to prevent angina and have no effect on the progression of atherosclerosis. 7. An older adult patient who is taking metformin (Glucophage) has just been seen in the clinic. The doctor has ordered metoprolol (Toprol) for angina. What assessment data should the nurse monitor due to this drug combination? A) Blood pressure B) Blood glucose C) Heart rate D) Intake and output Ans: B Feedback: Metformin is an antidiabetic drug and the nurse should monitor the patients blood glucose frequently throughout the day. The patient will not have the usual signs and symptoms of hypoglycemia or hyperglycemia. Blood pressure, heart rate, and intake and output would not be affected by this drugdrug combination. 8. A patient is to receive diltiazem (Cardizem) 360 mg/d orally in four divided doses. How many mg will the nurse administer per dose? A) 30 mg B) 60 mg C) 90 mg D) 120 mg Ans: C Feedback: The patient will take 90 mg/dose (360 divided by 4 doses equals 90 mg/ dose). Therefore, the other options are incorrect. 9. The patient, diagnosed with angina, tells the nurse he is having chest pain. There is an order for oral sublingual nitroglycerin as needed. What action should the nurse take? A) Place two nitroglycerin tablets under the patients tongue and call the physician. B) Place one tablet under the patients tongue and repeat every 5 minutes for total of three tablets until pain has been relieved. C) Have the patient swallow a tablet with a full glass of water and repeat in 10 minutes. D) Apply a nitroglycerin transdermal patch to the patients back. Ans: B Feedback: The correct administration for sublingual administration is to place one tablet under the patients tongue and repeat every 5 minutes for a total of three tablets until pain is relieved. If pain is not relieved after three sublingual tablets, the health care provider should be notified. Transdermal application would be inappropriate and nitroglycerin is not swallowed. Administering two tablets at one time would be an inappropriate dosage and could cause serious adverse effects. 10. The nurse is caring for a patient who is complaining of chest pain. The nurse is to administer 40 mg of isosorbide dinitrate (Isordil) to the patient. What is the nurses priority assessment before administering the drug? A) Jaundice B) Headache C) Anemia D) Sinusitis Ans: C Feedback: The nurse should assess for anemia because the decrease in cardiac output could be detrimental in a patient who already has a decreased ability to deliver oxygen because of a low red blood cell count. Jaundice and sinusitis would not be a contraindication to the drug. Headache is an adverse effect of isosorbide and would be expected after administration of the drug. 11. A 49-year-old patient is admitted with uncontrolled chest pain. He is currently taking nitroglycerin (Nitrostat). His physician orders nifedipine (Adalat) added to his regimen. The nurse should observe the patient for what adverse effects? A) Hypokalemia B) Renal insufficiency C) Hypotension D) Hypoglycemia Ans: C Feedback: Both nitroglycerin and nifedipine have hypotension as a potential adverse effect so frequent assessment of blood pressure is important. Other cardiovascular effects include bradycardia, peripheral edema, and heart block. Skin effects include flushing and rash. Adverse effects do not include renal insufficiency, hypokalemia, or hypoglycemia. 12. What statements by the 54-year-old patient indicates an understanding of the nurses teaching about how to take sublingual nitroglycerin? A) A headache means a toxic level has been reached. B) I can take up to 3 tablets at 5-minute intervals. C) I can take as much nitroglycerin as I need because it is not habit forming. D) If I become dizzy after taking the medication, I should stop taking it. Ans: B Feedback: Sublingual nitroglycerin may be taken at 5-minute intervals up to a maximum of three doses to relieve anginal chest pain. Headaches are very common due to vasodilation and do not indicate a toxic level. Nitroglycerin causes significant peripheral vasodilation in addition to its therapeutic effects of coronary artery dilation so no more than three tablets should be taken, even though it is not habit forming. Dizziness could be an adverse effect of the drug or a manifestation of inadequate cardiac output, but it would not indicate the patient should stop taking it. 13. When the nurse administers a beta-adrenergic blocker to the patient with angina, the nurse expects the drug will help to control angina, but it also has what other effect? A) Increased heart rate B) Increased oxygen consumption C) Decreased strength of heart muscle contraction D) Decreased urinary output Ans: C Feedback: Beta-blockers competitively block beta-adrenergic receptors in the heart and kidneys, decreasing the influence of the sympathetic nervous system on these tissues and the excitability of the heart. As a result, it decreases the strength of cardiac contraction, reducing cardiac output, which results in lowered blood pressure and decreased cardiac workload. It does not impact urinary output. 14. The nurse is caring for a patient who takes nitroglycerin sublingually. When providing patient education, the nurse would tell the patient that she can expect relief of chest pain within what period of time? A) 1 to 3 minutes B) 5 to 10 minutes C) 15 to 20 minutes D) 30 to 60 minutes Ans: A Feedback: Nitroglycerin acts within 1 to 3 minutes. Other options are incorrect. 15. When providing patient education about nitroglycerin to the patient, what would the nurse include in the teaching plan about a nitroglycerin patch? A) It only has to be administered once a week. B) It is more effective than tablets in treating angina. C) It has a longer duration of action. D) It is faster acting than the tablets. Ans: C Feedback: Transdermal nitroglycerin has a long 24-hour duration of action compared with the sublingual form that lasts 30 to 60 minutes or oral tablets that last 8 to 12 hours. Transdermal patches are neither more nor less effective, but rather it is the speed of onset and duration of action that differ. 16. Which statement by the patient would lead the nurse to believe that he has understood the teaching provided regarding angina? A) I will not exercise because it precipitates angina. B) As long as I take the medicine, I need make no lifestyle change. C) There is no correlation between my hypertension and angina. D) Heavy meals and cigarette smoking can precipitate an angina attack. Ans: D Feedback: Avoid stressful activities, especially in combination. For example, if you eat a big meal, do not drink coffee or alcoholic beverages with that meal. If you have just eaten a big meal, do not climb stairs; rest for a while. However, exercise is important and should not be eliminated, but managed in coordination with other activities. Smoking causes vasoconstriction that can result in angina attacks so lifestyle changes like reducing fat and calories in the diet, moderate exercise, reducing alcohol intake and avoiding smoking are all healthful choices. Hypertension does increase the risk of angina and coronary artery disease. 17. The nurse is caring for a patient who is taking a calcium-channel blocker. What adverse effects would the nurse caution this patient about? A) Hypertension and tachycardia B) Headache and dizziness C) Itching and rash D) Nausea and diarrhea Ans: B Feedback: The adverse effects associated with these drugs are related to their effects on cardiac output and on smooth muscle. Central nervous system (CNS) effects include dizziness, light-headedness, headache, and fatigue. Gastrointestinal (GI) effects can include nausea and hepatic injury related to direct toxic effects on hepatic cells. Cardiovascular effects include hypotension, bradycardia, peripheral edema, and heart block. Skin effects include flushing and rash. The adverse effects do not, however, include diarrhea, hypertension, tachycardia, or itching. 18. The nurse assesses patients receiving nifedipine (Adalat) for what adverse effects? A) Ascites B) Asthma C) Peripheral edema D) Tetany Ans: C Feedback: Peripheral edema, heart block, bradycardia, and hypotension can occur with calcium-channel blockers. Asthma, ascites, and tetany are not associated with these drugs. 19. What adverse reaction does the nurse anticipate if the 56-year-old patient takes his beta-blocker with clonidine? A) Hypertension B) Bradycardia C) Angina D) Syncope Ans: A Feedback: A paradoxical hypertension occurs when clonidine is given with beta- blockers, and an increased rebound hypertension with clonidine withdrawal may also occur. It is best to avoid this combination. Bradycardia, angina, and syncope are not associated with this drug combination. 20. An asthmatic patient taking beta-blockers should be assessed by the nurse for what potential adverse reaction? A) Bronchospasm B) Hypoglycemia C) Pleural effusion D) Pneumonia Ans: A Feedback: Bronchospasm can occur with beta blockade. The patient would not have to be observed for hypoglycemia, pleural effusion, or pneumonia. 21. The nurse should instruct the patient to take what action if three nitroglycerin tablets taken sublingually are not effective in eliminating chest pain? A) To call 911 B) To call her health care provider C) To lie down after taking an aspirin D) To take more tablets until pain subsides Ans: A Feedback: Instruct patient that a sublingual dose may be repeated in 5 minutes if relief is not felt, for a total of three doses, if pain persists, the patient or a family member should call 911 to ensure proper medical support if a myocardial infarction should occur. She should not waste time by calling the health care provider; she can lie down while waiting for the ambulance to arrive, she should not take more tablets. 22. The nurse is caring for a patient who takes metoprolol for angina. The patient asks how long it takes for the medicine to work. What is the nurses best response? A) 15 minutes B) 30 minutes C) 1 hour D) 90 minutes Ans: A Feedback: Oral metoprolol has an onset of action of 15 minutes. Other options are incorrect. 23. The nurse teaches the patient wearing a nitroglycerin patch to avoid what? A) Exercise B) Alcoholic beverages C) Milk products D) Synthetic fabrics Ans: B Feedback: Patients should be taught to avoid or at least decrease use of coffee, cigarettes, and alcoholic beverages. There is no need to avoid exercise, milk, or synthetic fabrics. 24. The nurse is caring for a patient who is taking a sustained-release (SR) oral nitrate. How should the nurse instruct this patient to take the medication? A) With water B) Sublingually until absorbed C) With milk or milk products D) 1 hour after eating Ans: A Feedback: Give SR forms with water, and caution the patient not to chew or crush them, because these preparations need to reach the gastrointestinal (GI) tract intact to avoid overdosage. They are not dissolved sublingually but swallowed whole. They are best taken on an empty stomach 1 hour before meals. 25. An adult patient who experiences angina pectoris with exertion is informed by the nurse that the leading cause of angina is what? A) Smoking B) Inadequate cardiac output C) Infarction of the myocardium D) Coronary atherosclerosis Ans: D Feedback: The person with atherosclerosis has a classic supply-and-demand problem. The heart may function without a problem until increases in activity or other stresses place a demand on it to beat faster or harder. Normally, the heart would stimulate the vessels to deliver more blood when this occurs, but the narrowed vessels are not able to respond and cannot supply the blood needed by the working heart. The heart muscle then becomes hypoxic. This imbalance between oxygen supply and demand is manifested as pain, or angina pectoris, which literally means suffocation of the chest. Atherosclerosis of the coronary artery can block the coronary artery completely leading to infarction. Smoking causes further vasoconstriction, increasing risk of myocardial infarction or angina. Damage to the heart muscle causes a decrease in cardiac output. 26. The nurse is caring for a patient prescribed ranolazine. The patient asks why this drug is different from the beta-blocker that he was previously taking. What is the nurses best response? A) This drug does not slow your heart rate. B) This drug increases myocardial oxygen demand. C) This drug slows the QT intervals. D) This maintains blood pressure with no hypotensive effects. Ans: A Feedback: The newest drug approved for the treatment of angina is a piperazine acetamide agent called ranolazine. The mechanism of action of this drug is not understood. It prolongs QT intervals, does not slow heart rate or blood pressure, but decreases myocardial oxygen demand. 27. After the patients anginal pain is relieved he says to the nurse, That nitroglycerin works great. How does it do that? What is the nurses best response? A) Nitroglycerin decreases the amount of oxygen needed by the heart to function. B) Nitroglycerin makes the coronary arteries open much wider. C) Nitroglycerin promotes growth of new, smaller arteries to supply oxygen to the heart. D) Nitroglycerin decreases preload and afterload. Ans: A Feedback: The main effect of nitrates seems to be related to the drop in blood pressure that occurs. The vasodilation causes blood to pool in veins and capillaries, decreasing preload, while the relaxation of the vessels decreases afterload. The combination of these effects greatly reduces the cardiac workload and the demand for oxygen, thus bringing the supply-and-demand ratio back into balance. Because coronary artery disease causes a stiffening and lack of responsiveness in the coronary arteries, the nitrates probably have very little effect on increasing blood flow through the coronary arteries, so it would be incorrect to say that the coronary arteries become much wider. Although both preload and afterload are reduced, this is an explanation that the patient would not understand, so it is inappropriate. Nitroglycerin does not promote growth of compensatory circulation. 28. An adult patient has had symptoms of unstable angina during admission to the hospital. What is the most appropriate nursing diagnosis? A) Deficient knowledge about underlying disease and methods for avoiding complications B) Anxiety related to fear of death C) Ineffective tissue perfusion (total body) related to reduced oxygen supply to the heart D) Noncompliance related to failure to accept necessary lifestyle changes Ans: C Feedback: The most appropriate nursing diagnosis is ineffective tissue perfusion related to reduced oxygen supply to the heart because this is the cause of the patients pain. Further assessment would be needed to determine whether the patient lacks knowledge, fears death, or has made the necessary lifestyle changes. 29. The nurse is preparing to administer sublingual nitroglycerin to a patient for the first time and warns that the patient may experience what right after administration? A) Nervousness or paresthesia B) Throbbing headache or dizziness C) Drowsiness or blurred vision D) Tinnitus or diplopia Ans: B Feedback: Headache and dizziness commonly occur at the start of nitroglycerin therapy. When administering nitroglycerin, the nurse must use caution to avoid self-contamination, especially with the topical paste formulation because the nurse can experience the same symptoms. However, the patient usually develops a tolerance. Nervousness, paresthesia, drowsiness, blurred vision, tinnitus, and diplopia do not occur as a result of nitroglycerin therapy. 30. A patient is diagnosed with Prinzmetals angina. What drug would the nurse expect to administer to this patient? A) Nadolol B) Diltiazem C) Propranolol D) Metoprolol Ans: B Feedback: Calcium channel blockers are indicated for the treatment of Prinzmetals angina because these drugs relieve coronary artery vasospasm, increasing blood flow to the muscle cells. Diltiazem is a calcium channel blocker. Beta- blockers are not indicated for the treatment of Prinzmetals angina because they could cause vasospasm due to blocking of beta-receptor sites. Propranolol, nadolol, and metoprolol are beta-blockers. 31. The nurse is caring for a patient diagnosed with human immunodeficiency virus (HIV) and newly diagnosed angina. What drug would the nurse question if ordered? A) Ranolazine (Ranexa) B) Nitroglycerin (generic) C) Propranolol D) Diltiazem Ans: A Feedback: Drugdrug interactions can occur with ketoconazole, diltiazem, verapamil, macrolide antibiotics, and HIV protease inhibitors; these combinations should be avoided because ranolazine levels may become extremely high. The nurse should get a complete list of current medications and consult with the health care provider regarding drug interactions before administering the medication as ordered. Nitroglycerin, propranolol, and diltiazem have no contraindication with medications used to treat HIV. 32. Other than angina, what other medical condition might the nurse treat by administering nitroglycerin? A) Muscular dystrophy B) Pulmonary embolisms C) Polycythemia D) Anal fissures Ans: D Feedback: In 2011, a nitroglycerin in ointment form, Rectiv 0.4%, was approved for the treatment of moderate to severe anal fissures. There is no documented benefit to the use of nitroglycerin to treat muscular dystrophy, pulmonary embolisms, or polycythemia. 33. What would the nurse teach the patient about potency of nitroglycerin? (Select all that apply.) A) The tablet should fizzle or burn when placed under the tongue. B) Protect the drug from heat and light. C) Always replace when past the expiration date. D) Older tablets may require you to use two tablets at one time. E) Nitroglycerin does not lose its potency easily. Ans: A, B, C Feedback: Ask the patient if the tablet fizzles or burns, which indicates potency. Always check the expiration date on the bottle and protect the medication from heat and light because these drugs are volatile and lose potency easily. 34. The nurse teaches the patient how to use topical nitroglycerin and includes what teaching points in the teaching plan? (Select all that apply.) A) Rotate application sites. B) Assess the skin for signs of breakdown. C) Make sure no one touches the side with the medication. D) Do not shower with the patch in place. E) Increase fluid intake to avoid hypotension. Ans: A, B, C Feedback: Rotate the sites of topical forms of nitroglycerin to lower the risk of skin abrasion and breakdown; monitor for signs of skin breakdown to arrange for appropriate skin care as needed. Care should be taken not to touch the side of the patch with the medication by the patient or anyone assisting in applying the patch. The patient may shower with the patch in place. There is no need to increase fluid intake but patients should be encouraged to maintain adequate intake. 35. The provider orders isosorbide dinitrate as oral maintenance drug to prevent angina. What dosage would the nurse administer without need to question the dose? A) 2.5 mg B) 80 mg C) 40 mg D) 5 mg Ans: B Feedback: A maintenance oral dose of isosorbide dinitrate is 10 to 40 mg every 6 hours. Sublingual dose is 2.5 to 5 mg, sustained release is 40 to 80 mg, and the chewable tablet is 5 mg. Blood Pressure: 1. What does the clinic nurse anticipate as initial drug therapy for a 39-year-old African American man who is 25 pounds overweight and newly diagnosed with hypertension? A) An angiotensin-converting enzyme (ACE) inhibitor B) A beta-blocker C) A calcium channel blocker D) A diuretic Ans: D Feedback: African Americans are at highest risk for developing hypertension with men more likely than women to develop the disease. African Americans have documented differences in response to antihypertensive therapy. They are most responsive to single-drug therapy and diuretics. The first line use of a diuretic is in combination with diet and other lifestyle changes. The use of a calcium channel blocker and/or alpha-adrenergic blocker should follow. African Americans are less responsive to ACE inhibitors and beta-blocker. 2. The clinic nurse assesses a patient taking benazepril (Lotensin) to control hypertension. What is the priority nursing assessment related to this drug? A) Mental illness B) Hepatic disease C) Renal disease D) Peptic ulcer disease Ans: C Feedback: Benazepril is an angiotensin-converting enzyme inhibitor; drugs in this class are contraindicated in the presence of impaired renal function. Mental illness, hepatic disease, or peptic ulcer disease is not a contraindication with this drug. 3. A patient has been prescribed losartan (Cozaar) for hypertension. What patient teaching points will the nurse include about this drug include? A) Report onset of a cough or fever to health care provider. B) Limit fluid intake to decrease urinary output. C) Monitor blood pressure once a week. D) Take the drug late in the day to prevent sleepiness. Ans: A Feedback: Losartan is an angiotensin IIreceptor blocker that is associated with a cough, back pain, fever, muscle weakness, and upper respiratory tract infections, so the patient should be taught to report a fever or cough to his health care provider. Fluid intake should be normal and the drug is best taken in the morning. Blood pressure should be monitored daily, especially when first starting the drug when adverse effects are not yet known. 4. A patient newly diagnosed with hypertension has just been given a prescription for medication. Along with promoting safety, what is the other goal of the nurses teaching plan? A) A blood pressure of 120/80 B) Medication compliance C) A discussion with his insurance company about the cost of the drug D) Verbalization of why drugs should be kept out of the reach of children Ans: B Feedback: The nurse provides thorough patient teaching, including the name of the drug, dosage prescribed, measures to avoid adverse effects, warning signs of problems, and the need for periodic monitoring and evaluation, to enhance patient knowledge about drug therapy and to promote compliance. All of the options are realistic outcomes for this patient. However, compliance is a great concern for people who are in need of hypertensive agents. It would be most important for his or her health care provider to know that he or she has filled the prescription and is taking his or her medication as prescribed. Keeping the drug out of the reach of children would be a safety measure. 5. A stepped care management approach to treating hypertension includes weight loss, smoking cessation, decreased use of alcohol, reducing salt in the diet, and increased physical exercise. In which step of a stepped-care management approach will the nurse teach about these changes in lifestyle? A) Step 1 only B) Steps 1 and 2 C) Steps 1, 2, and 3 D) Steps 1, 2, 3, and 4 Ans: D Feedback: Lifestyle changes are encouraged in all four steps and should be advocated for the maintenance of good health. These changes are stressed in steps 1 and 2 in the hope that drug therapy will not be necessary. However, these changes should occur in steps 3 and 4 as well. 6. A patient taking diltiazem (Cardizem) for hypertension has come to the clinic for a follow-up appointment. What adverse effects would the nurse assess the patient for? A) Chest pain and pale skin B) Shortness of breath and wheezing C) Peripheral edema and bradycardia D) Tachycardia and increased energy level Ans: C Feedback: Cardiovascular adverse effects of diltiazem include bradycardia, peripheral edema, and hypotension. Skin flushing and rash may occur. There should be no effect on the lungs and usually this drug causes fatigue rather than increased energy levels. 7. A patient who works on road construction has been diagnosed with hypertension. After attempting to decrease his blood pressure with lifestyle changes and a mild diuretic, it is determined that he will need to be placed on an angiotensin-converting enzyme (ACE) inhibitor. Based on his occupation, what is the nurses priority assessment? A) Chronic constipation B) Excessive sweating on the job C) Three large meals a day D) One beer every night Ans: B Feedback: A patient taking an ACE inhibitor should be sure to maintain fluid intake, so excessive sweating on the job places him at risk for a drop in fluid volume. Excessive sweating, vomiting, diarrhea, or dehydration need to be monitored and treated if they occur while taking an ACE inhibitor. Six smaller meals rather than three larger ones would be better and should be encouraged. However, this could be a problem with his job. The best treatment for constipation would be to increase fluid and fiber and one beer a night would be within reason when considering alcohol intake. 8. What is the nurses priority to assess before giving a female patient her prescription for an angiotensin IIreceptor blocker (ARB)? A) Do you eat something when you take your medications? B) How much physical exercise do you get? C) When was your last menstrual period (LMP)? D) Have you always weighed 130 pounds since you grew up? Ans: C Feedback: It would be important to know when the patients LMP occurred and that the patient was not pregnant. These drugs can cause fetal abnormalities and fetal death. The other questions are appropriate and would help the nurse plan care for the patient; however, it would not be as important as assessing for the possibility of pregnancy before beginning of therapy. The nurse should teach the patient the need to avoid pregnancy using a barrier contraceptive. 9. A patient taking a calcium channel blocker is seen in the clinic and diagnosed with drug toxicity. When collecting the nursing history, what finding would indicate the likely cause of this drug toxicity? A) Intake of alcohol B) The use of eggs in the diet C) The ingestion of grapefruit juice D) Intake of aged cheese Ans: C Feedback: The calcium channel blockers are a class of drugs that interact with grapefruit juice. When grapefruit juice is present in the body, the concentrations of calcium channel blockers increase, sometimes to toxic levels. Advise patients to avoid drinking grapefruit juice taking a calcium channel blocker. If a patient on a calcium channel blocker reports toxic effects, ask whether he or she is drinking grapefruit juice. Use of alcohol could be important if the patient was ingesting large amounts, but that would not be the most likely cause of drug toxicity. Eggs and cheese should not exert any fooddrug interaction. 10. An older adult patient is taking a sustained-release antihypertensive drug. What is the nurses priority teaching point about this medication? A) Take your blood pressure only at night. B) Swallow the drug whole and do not to cut, crush, or chew it. C) Take the drug before bedtime. D) Use over-the-counter (OTC) drugs to control headache or cold symptoms. Ans: B Feedback: Sustained-release drugs are suspended in a matrix system that allows a steady release of the drug over time. Sustained-release drugs cannot be cut, crushed, or chewed; it destroys the matrix system and allows absorption of the complete dose all at once. Older patients should be especially cautioned about sustained-release antihypertensives that cannot be cut, crushed, or chewed to avoid the potential for excessive dosing if these drugs are inappropriately cut. Many OTC drugs contain ingredients that increase blood pressure and so are not recommended for patients with hypertension. The patient can take his or her blood pressure any time during the day but should take the drug in the morning. 11. A patient has been started on losartan (Cozaar), an angiotensin IIreceptor blocker (ARB), for hypertension. After 6 weeks of therapy, it is decided that the losartan alone is not controlling the patients hypertension. What does the nurse anticipate will be added to the losartan regimen for better control of this patients hypertension? A) Candesartan (Atacand) B) Hydrochlorothiazide (Hyzaar) C) Captopril (Capoten) D) Antidiuretic hormone (ADH) Ans: B Feedback: When losartan therapy is started, maximal effects on blood pressure usually occur within 3 to 6 weeks. If losartan alone does not control blood pressure, a low dose of a diuretic may be added. A combination product of losartan and hydrochlorothiazide (Hyzaar) is available. Adding a second ARB such as candesartan or captopril would risk causing toxic adverse effects. ADH causes retention of water in the nephrons, which would further increase blood pressure. 12. The nurse is providing drug teaching for a patient who is prescribed enalapril (Vasotec). What drug specific adverse effect will the nurse include in the drug teaching? A) Sedation B) Persistent cough C) Tachycardia D) Rash Ans: B Feedback: Benazepril, enalapril, and fosinopril are generally well tolerated but cause an unrelenting cough, possibly related to adverse effects in the lungs, where the angiotensin-converting enzyme is inhibited, which may lead patients to discontinue the drug. This persistent cough develops in approximately 10% to 20% of patients. 13. The nurse provides drug teaching to the patient prescribed captopril (Capoten). What statement made by the patient does the nurse interpret to mean teaching has been effective? A) I will limit my fluid intake to 1,200 mL daily. B) I will call my doctor if I bruise easily or become extremely tired. C) I will move from a reclining to a standing position slowly. D) I will increase my intake of foods high in potassium. Ans: B Feedback: Captopril has been associated with a sometimes-fatal pancytopenia, cough, and unpleasant gastrointestinal (GI) distress so the patient should be alert to symptoms related to anemia, reduction in platelets, or infection. There is no need to increase potassium intake because a slight rise in potassium level is associated with this drug. Orthostatic hypotension is not a listed adverse effect. There is no need to limit fluid intake for most patients unless they have a comorbid disorder. 14. The nurse provides dietary teaching to the patient with hypertension and determines the patient understood the information when what meal is selected? A) Hot dogs, baked beans, and cole slaw B) French fries, grilled hamburger, and cola drink C) Grilled chicken, green salad with dressing, and baked apple D) Bologna sandwich with mayonnaise, potato chips, and a chocolate-chip cookie Ans: C Feedback: Chicken, salad, and fruit are all low in sodium. The other meal options all contain foods high in sodium (i.e., hot dogs, French fries, processed meats like bologna, and potato chips). 15. The nurse works with the patient, diagnosed with hypertension, and the patients family to determine the goal of drug therapy for the patient taking an antihypertensive medication is what? A) Maintaining compliance B) Maintaining the blood pressure within normal limits C) Maintaining a fluid volume balance D) Maintaining homeostasis Ans: B Feedback: Helping the patient to maintain the blood pressure within normal limits is the goal of drug therapy. How blood pressure is maintained within normal limits may involve balancing fluid volume and patient compliance with the plan of care, but these are interventions and not the goal of therapy. Returning the patient to homeostasis comes before maintaining homeostasis. 16. While studying the antihypertensive drugs, the nursing students learn that the pressure in the cardiovascular system is determined by three elements. What are they? (Select all that apply.) A) Heart rate B) Stroke volume C) Preload D) Total peripheral resistance E) Pulse pressure Ans: A, B, D Feedback: The pressure in the cardiovascular system is determined by three elements: heart rate, stroke volume, or the amount of blood that is pumped out of the ventricle with each heartbeat (primarily determined by the volume of blood in the system), and total peripheral resistance, or the resistance of the muscular arteries to the blood being pumped through. The preload and the pulse pressure are not factors that regulate pressure in the cardiovascular system. 17. The nurse recognizes blood pressure is determined by three elements including what? A) Peripheral resistance B) Pulse pressure C) Renal blood flow D) Preload Ans: A Feedback: The pressure in the cardiovascular system is determined by three elements: heart rate, stroke volume, and peripheral resistance. The small arterioles are thought to be the most important factors in determining peripheral resistance. Because they have the smallest diameter, they are able to almost stop blood flow into capillary beds when they constrict, building up tremendous pressure in the arteries behind them as they prevent the blood from flowing through. 18. When a combination of drugs needs to be incorporated into the drug regimen for a hypertensive patient, what type of diuretic would the nurse expect to administer? A) Thiazide B) Loop C) Potassium-sparing D) Osmotic Ans: A Feedback: A somewhat controversial study, the Antihypertensive and Lipid-Lowering Treatment to Prevent Heart Attack Trial (ALLHAT), reported in 2002 that patients taking the less expensive, less toxic diuretics did better and had better blood pressure control than patients using other antihypertensive agents. Replications of this study have supported its findings, and the use of a thiazide diuretic is currently considered the first drug used in the stepped- care management of hypertension. 19. The nurse is caring for a patient who has been noncompliant with treatment for hypertension. The nurse explains that untreated hypertension increases the risk of what? (Select all that apply.) A) Renal disease B) Cerebral infarction C) Heart failure D) Cholecystitis E) Migraine headache Ans: A, B, C Feedback: Hypertension is a common chronic disorder. It is estimated that at least 20% of the people in the United States have hypertension. Hypertension increases risks of myocardial infarction, heart failure, cerebral infarction and hemorrhage, and renal disease. It does not increase the risk of cholecystitis or migraine headache. 20. The nurse administers ambrisentan (Letairis) to the patient diagnosed with pulmonary arterial hypertension. What single indicator would the nurse use to evaluate the effectiveness of this medication? A) Oxygen saturation B) Resting respiratory rate C) Exercise tolerance D) Breath sounds Ans: C Feedback: Although it is certainly important to assess all aspects of oxygenation and ventilation, the single best indicator of drug effectiveness is improved exercise tolerance. Many patients can oxygenate at rest and maintain a normal resting respiration, but activity increases oxygen demand, which is when signs of pulmonary hypertension are best seen. 21. The pediatric nurse examines a child with mildly elevated blood pressure who is 10% above the upper weight limits for his or her age. What is the nurses priority teaching point? A) Encourage activity and begin weight loss diet. B) Explain how to administer diuretics to reduce adverse effects. C) Involve social services in monitoring this childs diet. D) Explain the most common adverse effects of calcium channel blockers. Ans: A Feedback: Treatment of childhood hypertension should be done very cautiously because the long-term effects of the antihypertensive agents are not known. Lifestyle changes should be instituted before drug therapy is started if at all possible. Weight loss and increased activity may bring an elevated blood pressure back to normal in many children. As a result, the priority teaching point is to help parents understand how to adapt the childs diet to reduce weight and introduce family activities to increase exercise. Drug teaching would only be required if lifestyle changes is inadequate to lower blood pressure. There is no need to involve social services. 22. The nurse is caring for an obese child with hypertension and slightly elevated serum glucose levels. What would the nurse anticipate will be ordered if lifestyle changes do not return blood pressure to an acceptable limit? A) Lifestyle changes B) Mild diuretic C) Calcium channel blocker. D) Beta-blocker Ans: B Feedback: Lifestyle changes should be instituted before drug therapy if at all possible. If drug therapy is used, a mild diuretic may be tried first, with monitoring of blood glucose and electrolyte levels on a regular basis. Calcium channel blockers have been used to treat hypertension in children and may be among the first considerations if drug therapy other than mild diuretics is needed. Beta-blockers have been used with success in some children; adverse effects may limit their usefulness in others. The safety and efficacy of the angiotensin-converting-enzyme (ACE) inhibitors and the angiotensin- receptor blockers (ARBs) have not been established in children. 23. The emergency department nurse is asked to prepare a nitroprusside (Nitropress) infusion for a patient being brought to the hospital in an ambulance. The nurse knows this drug is only used in what circumstances? A) Hypertensive emergencies B) Hypertension in a patient having a myocardial infarction C) Hypertension complicated by symptoms of a stroke D) Hypertension associated with diabetic ketoacidosis Ans: A Feedback: Most of the vasodilators are reserved for use in severe hypertension or hypertensive emergencies. These include hydralazine, minoxidil, and nitroprusside. The presence of absence of a comorbidity does not increase the likelihood of use. These drugs are used when blood pressure is extremely high and needs to be reduced quickly. 24. The nurse cares for a diabetic patient with uncontrolled hypertension who has been prescribed losartan (Cozaar). The health care provider changes this patients prescription to losartan with hydrochlorothiazide (Hyzaar). What benefits does the nurse anticipate this patient will receive from this drug therapy? (Select all that apply.) A) Slows progression of diabetic nephropathy in type 2 diabetes B) Increases excretion of fluid and sodium resulting in lower blood volume C) Alters electrolyte and acidbase balance D) Improves control of blood pressure due to combination therapy E) Produces far fewer adverse effects resulting from the combination Ans: A, B Feedback: Many patients require more than one type of antihypertensive to achieve good control of their blood pressure. There are now many fixed-combination drugs available for treating hypertension. This allows for fewer tablets or capsules each day, making it easier for the patient to comply with drug therapy. Losartan slows the progression of diabetic nephropathy in patients with hypertension and type 2 diabetes. Hydrochlorothiazide is a diuretic that will increase excretion of fluid and sodium resulting in a lower circulating blood volume that will help to reduce blood pressure, preload, and afterload. The two drugs will work together to better control the patients blood pressure. Alteration in electrolytes and acidbase is a reasonable expectation but it is an adverse effect and not a benefit. There are likely to be more adverse effects when taking drugs that are not fixed combinations. 25. The patient is diagnosed with secondary hypertension. What will be the focus of nursing care to treat this patient? A) Administering epinephrine B) Administering antihypertensives C) Promoting healthy lifestyle D) Treating the cause Ans: A Feedback: Most people have essential, or primary, hypertension with no known cause that is treated with medications and lifestyle changes. Patients with secondary hypertension means there is a known cause of the blood pressure elevation. There are many different things that can result in secondary hypertension; the focus of care is treating the cause whether that requires surgery, medication, or discontinuing a medication that is causing hypertension as an adverse effect. 26. The patient does not want to take medication to treat his or her hypertension if he or she can avoid it and asks the nurse if there is anything else he or she can do? What lifestyle changes will the nurse recommend? (Select all that apply.) A) Increase time spent exercising. B) Eliminate all salt from your diet. C) Reduce intake of fluids. D) Lose some weight. E) Try meditation. Ans: A, D, E Feedback: Exercising, losing weight, and meditation to reduce stress are all effective changes the patient can make in her lifestyle. However, if these do not bring blood pressure to within acceptable limits, the next step in the stepped-care management approach to treat hypertension would be to introduce a diuretic, beta-blocker, angiotensin-converting enzyme inhibitor, calcium channel blocker, or angiotensin IIreceptor blocker in addition to the lifestyle changes. It is not necessary, or even possible, to eliminate all sodium from the diet but cutting back will reduce water retention. Reducing intake of fluids is not a healthy option and would not be suggested unless comorbidities, such as possibility of heart failure, were present. 27. The nurse evaluates the patients lifestyle for factors that are contributing to his or her diagnosis of hypertension and then addresses the need to change what factor? A) Working outdoors in the sun B) Weight lifting at the gym C) Married with two children D) Exposure to high-frequency noise Ans: D Feedback: Factors that are known to increase blood pressure in some people include high levels of psychological stress, exposure to high-frequency noise, a high-salt diet, lack of rest, and genetic predisposition. Working outdoors in the sun may increase risk for skin cancer but does not contribute to hypertension. Weight lifting is good exercise, especially if he uses low weight and frequent repetition. Being married with two children is not a contributing factor for hypertension. 28. A patient is admitted to the intensive care unit in shock with hypotension. What is an appropriate nursing diagnosis for this patient? A) Impaired gas exchange B) Deficient fluid volume C) Risk for shock D) Ineffective peripheral tissue perfusion Ans: D Feedback: An appropriate nursing diagnosis would be ineffective peripheral tissue perfusion. If blood pressure becomes too low, the vital centers in the brain, as well as the rest of the tissues of the body, may not receive enough oxygenated blood to continue functioning. Because the patient is already in shock it would not be a risk diagnosis. There is no indication of altered gas exchange or deficient fluid volume. 29. The nurse is caring for a patient who takes ramipril (Altace) to treat hypertension. What would be an appropriate nursing diagnosis to include in this patients care plan? A) Ineffective tissue perfusion related to changes in cardiac output B) Acute pain related to skin effects and headache C) Altered gas exchange related to unrelenting cough D) Impaired body image Ans: A Feedback: Nursing diagnoses related to drug therapy might include ineffective tissue perfusion (total body) related to changes in cardiac output because ramipril is associated with adverse effects such as reflex tachycardia, chest pain, angina, heart failure, and cardiac arrhythmias. Although dermatitis and rash may occur, headaches are not an associated adverse effect of this drug. Benazepril, enalapril, and fosinopril can cause an unrelenting cough but ramipril is not associated with this adverse effect. Impaired body image would not be associated with this drug. 30. What drug is a safe and effective calcium channel blocker only if the nurse administers them as sustained-release or extended-release preparations to treat hypertension? A) Aliskiren (Tekturna) B) Diltiazem (Cardizem) C) Atenolol (Tenormin) D) Metoprolol (Lopressor) Ans: B Feedback: The calcium channel blockers available in immediate-release and sustained- release forms that are used in treating hypertension include amlodipine (Norvasc), felodipine (Plendil), isradipine (DynaCirc, DynaCirc CR), and nicardipine (Cardene, Cardene SR). Other calcium channel blockers are safe and effective for this use only if they are given as sustained-release or extended-release preparations. These include diltiazem (Cardizem, Dilacor CR), nifedipine (Procardia XL), nisoldipine (Sular), and verapamil (Calan SR). Aliskiren (Tekturna) is a renin inhibitor. Atenolol (Tenormin) and Metoprolol (Lopressor) are beta-blockers, not calcium channel blockers. 31. The nurse works in a clinic that has many African American patients. What would the nurse need to consider when caring for patients with hypertension? (Select all that apply.) A) African American men are at highest risk for hypertension. B) African Americans respond best to single-drug therapy. C) African Americans are most responsive to angiotensin-converting enzyme inhibitors. D) Increased adverse effects occur when using thiazide and thiazide-like diuretics. E) First-line use of a calcium channel blocker with changes to lifestyle is best in African Americans. Ans: A, B, D Feedback: In the United States, African Americans are at highest risk for developing hypertension, with men more likely than women to develop the disease. African Americans are most responsive to single-drug therapy (as opposed to combination drug regimens). African Americans are less responsive to angiotensin-converting enzyme inhibitors and beta-blockers. Increased adverse effects (e.g., depression, fatigue, drowsiness) often occur when using thiazide and thiazide-like diuretics. Because African Americans are more responsive to diuretics, the treatment approach should include the first- line use of a diuretic in combination with diet and other lifestyle changes. 32. The nurse is caring for a patient newly prescribed iloprost (Ventavis). What is the nurses priority teaching point about this medication? A) Inhale this drug six to nine times a day during waking hours. B) Men should not touch this pill. C) It is safe for use in pregnancy. D) The drug causes peripheral dilation of blood vessels. Ans: A Feedback: Iloprost is an inhaled synthetic prostacyclin that directly dilates the pulmonary vascular bed, reducing pressure in the pulmonary vascular system, increasing gas exchange, and easing the signs and symptoms of pulmonary arterial hypertension. It is inhaled using a special delivery device six to nine times a day while awake. Patients report dizziness and syncope after using the drug and are therefore encouraged to change position slowly. Men and women should not ingest the drug or get it on their skin. It is a pregnancy category C drug and so is not considered safe in pregnancy. 33. No antihypertensive medication is safe during pregnancy but if the benefit for the mother outweighs the risk to the fetus, what classification of medication can the nurse administer to the hypertensive pregnant woman? A) Angiotensin-converting enzyme (ACE) inhibitor B) Angiotensin-receptor blocker (ARB) C) Renin inhibitor D) Calcium channel blocker Ans: D Feedback: ACE inhibitors, ARBs, and renin inhibitors should not be used during pregnancy, and women of childbearing age should be advised to use barrier contraceptives to prevent pregnancy while taking these drugs. Calcium channel blockers and vasodilators should not be used in pregnancy unless the benefit to the mother clearly outweighs the potential risk to the fetus. 34. The patient was unable to achieve an acceptable blood pressure with just lifestyle changes so in stage 2 of the stepped-care management of hypertension plan, an angiotensin-converting enzyme inhibitor was ordered. In step 3, when inadequate response was obtained from step 2, the nurse anticipates the provider will order what? A) A diuretic B) A beta-blocker C) A calcium channel blocker D) A vasodilator Ans: A Feedback: In step 3, another drug will be added for combined effect. However, fixed- combination drugs should only be used when the patient has been stabilized on each drug separately. A diuretic would be added before adding another class of medication unless the first drug was a diuretic. Vasodilators are generally used only in hypertensive emergencies. 35. The nurse is caring for a patient with hypertension who is preparing to be discharged from the hospital after suffering a myocardial infarction. What drug might the nurse administer that will both treat his hypertension and reduce myocardial oxygen consumption? A) Captopril B) Losartan C) Diltiazem D) Nitroprusside Ans: C Feedback: Diltiazem inhibits the movement of calcium ions across the membranes of cardiac and arterial muscle cells, depressing the impulse and leading to slowed conduction, decreased myocardial contractility, and dilation of arterioles, which lowers blood pressure and decreases myocardial oxygen consumption. Captopril, losartan, and nitroprusside do not have actions to reduce myocardial oxygen consumption Chapter 17. Immunological System Medications 1. A patient has been diagnosed with hairy cell leukemia. The patient is to begin taking interferon alfa 2b. What will the nurse include in her instructions to the patient concerning this drug? A) Avoid drinking alcohol while taking the drug. B) Continue to maintain maximal physical activity. C) Increase fluid intake while taking the drug. D) Treat constipation with over-the-counter laxatives. Ans: C Feedback: Interferon alfa 2b is metabolized in the kidney so adequate fluid intake is needed to promote metabolism and excretion of the drug as well as to minimize common adverse effects including dry skin and dizziness. Maintaining maximal physical activity is a good idea but has no relationship to the use of the drug. Fluids should be increased not decreased while taking the drug. Constipation is not an associated adverse effect of this medication. 2. The health care provider plans to inject an interferon directly into the patients wart. What interferon will the nurse prepare? A) Interferon alfa 2a (Roferon-A) B) Interferon alfacon 1 (Infergen) C) Interferon alfa n3 (Alferon N) D) Interferon beta 1a (Avonex) Ans: C Feedback: Interferon alf n3 is used for intralesional treatment of warts. Interferon alfa 2a is used in the treatment of leukemia. Interferon alfacon 1 is used in the treatment of chronic hepatitis C infection in adults. Interferon beta 1a is used to treat multiple sclerosis in adults. 3. A 30-year-old woman has been diagnosed with leukemia and will be using an immune modulator for treatment. What will be important to discuss with the patient when the nurse provides patient teaching about her treatment? A) The need to continue oral contraceptives B) The need to use barrier contraceptives while taking the drug C) The need to avoid sexual intercourse while taking the drug D) The importance of taking an aspirin daily to decrease the adverse effects of the drug Ans: B Feedback: A patient taking an immune modulator would be advised to use barrier contraceptives to prevent pregnancy. The interaction of the immune modulator and the oral contraceptive may interfere with the oral contraceptives ability to work properly. Asking patients to avoid sexual intercourse is not necessary if barrier methods are properly used. Daily aspirin would not decrease adverse effects of this drug. 4. The nurse has an order to administer oprelvekin (Neumega) to a patient for the first time. Before administering the drug, what allergy would the nurse want to specifically question the patient about? A) Egg products B) Escherichia coliproduced products C) Lactose intolerance D) Penicillin Ans: B Feedback: The interleukins are produced using deoxyribonucleic acid technology and E. coli bacteria. Patients with known allergy to E. coli products should not receive oprelvekin. The allergies to penicillin, egg products, or lactose intolerance would not be of concern with this drug. 5. The physician has decided to prescribe T- and B-cell suppressors for a patient diagnosed with psoriasis. What drug will be ordered for this patient? A) Alefacept (Amevive) B) Azathioprine (Imuran) C) Cyclosporine (Neoral) D) Glatiramer acetate (Copaxone) Ans: A Feedback: Alefacept is prescribed for patients with severe chronic plaque psoriasis. Cyclosporine is used to suppress rejection in a variety of transplant situations. Azathioprine is used to treat patients with rheumatoid arthritis and in prevention of rejection in renal homotransplants. Tacrolimus is used for prevention of rejection after renal or liver transplantation. 6. A patient who is receiving an immune suppressant has been admitted to the unit. What would be a priority action by the nurse? A) Monitor nutritional status. B) Provide patient teaching regarding the drug. C) Protect the patient from exposure to infection. D) Provide support and comfort measures in relation to adverse effects of the drug. Ans: C Feedback: Patients taking immune suppressant drugs are more susceptible to infection because the patients normal body defenses will be diminished. As a result, the priority action by the nurse would to protect the patient from exposure to infection through room selection, good hand hygiene, and taking care to avoid exposure to sick staff members. Teaching will need to include avoiding crowded places and people with known infection and those working in soil. Nutritional status is important as are comfort and support measures and other instructions concerning the drug. However, protecting the patient from infection should be the priority action. 7. A nurse is discussing interferon alfa 2b with a patient. What will the nurse encourage the patient to do while taking this drug? A) To avoid crowds B) To increase salt intake C) To decrease milk intake D) To eat three meals a day Ans: A Feedback: Potential adverse effects in addition to the types of conditions interferon alfa 2b is prescribed to contribute to the need for the patient to take care to avoid people with infections so the patient should be taught to avoid crowds whenever possible. Adverse effects include dizziness, confusion, rash, dry skin, anorexia, nausea, bone marrow suppression, and flu-like syndrome. Salt, diet, and milk do not interfere with this drug. 8. A 70-year-old patient with acute myelocytic leukemia is receiving sargramostim (Leukine). What is a priority nursing action for this patient? A) Providing a quiet environment B) Increasing fluids C) Providing comfort measures related to nausea D) Encouraging appropriate dietary intake Ans: B Feedback: A common adverse effect of this drug is vomiting and diarrhea. Due to the patients age it would be important to keep him hydrated. Vomiting and diarrhea can cause dehydration quickly in the elderly. Providing a quiet environment and comfort measures for the nausea would be important but not as critical as increasing fluids. Diet is very important to this patient; however, usually this drug causes a loss of appetite. Therefore, increasing fluids would be extremely important to the patients nutritional status. 9. The nurse is caring for a patient in the immediate postoperative period following cardiac transplantation who is receiving mycophenolate (CellCept) twice a day IV. What will the nurse teach the patient regarding drug therapy? (Select all that apply.) A) The drug will be given orally as soon as possible. B) Take the medication three times a day. C) Avoid people with contagious diseases. D) Ask a pharmacist about drugdrug interactions before taking any over-the- counter (OTC) drug. E) Never miss a dose of medication. Ans: A, C, D, E Feedback: The nurse will explain that the IV medication will be changed to oral therapy when the patient is able to tolerate oral medications. The patient will take the medication twice a day, not three times a day. Care should be taken to never miss a dose. The patient should avoid other medications that are hepatotoxic or nephrotoxic due to a risk of increased toxicity so the patient should be taught to always consult a doctor or pharmacist before beginning an OTC drug. Patients who have immune suppression must be taught how to reduce risk of infection, including avoiding people with contagious diseases, such as colds or viruses. 10. The nurse administers aldesleukin to a patient diagnosed with renal cell carcinoma. When assessing the patient a few days later, what abnormal findings would the nurse attribute to the medication? (Select all that apply.) A) Increased lymphocyte count B) Increased red blood cell count C) Increased platelet count D) Irregular pulse rate E) Increased blood pressure Ans: A, C, D Feedback: Aldesleukin activates human cellular immunity and inhibits tumor growth through increases in lymphocytes, platelets, and cytokines. Common adverse effects include hypotension, sinus tachycardia, arrhythmias, as well as pruritus, nausea, vomiting, diarrhea, anorexia, GI bleeding, bone marrow suppression, respiratory difficulties, fever, chills, pain, mental status changes, and dizziness. There is no impact on red blood cell count. It does not raise blood pressure. 11. While studying for a pharmacology test, a student asks his peers about interferons. What statement about interferons is accurate? A) They stimulate B-lymphocyte activity. B) They interfere with multiplication of stem cells. C) They stimulate growth and differentiation of lymphoid cells into lymphocytes. D) They interfere with the ability of viruses in infected cells to replicate. Ans: D Feedback: Interferons are substances naturally produced and released by human cells that have been invaded by viruses. They may also be released from cells in response to other stimuli, such as cytotoxic T-cell activity. Interferons do not stimulate B-lymphocyte activity, interfere with multiplication of stem cells, nor do they stimulate growth and differentiation of lymphoid cells into lymphocytes. 12. How do immune suppressants work when ordered for a patient who has had an organ transplant? A) Blocking normal effects of the immune system B) Stimulating immune system to fight off infection C) Working with corticosteroids to enhance healing D) Working with corticosteroids to promote suppressor cells Ans: A Feedback: Immune suppressants are used to block the normal effects of the immune system in cases of organ transplantation (in which nonself-cells are transplanted into the body and destroyed by the immune reaction) and in autoimmune disorders (in which the bodys defenses recognize self-cells as foreign and work to destroy them) in some cancers. Options B, C, and D are distracters for this question. 13. A patient has just been told that her cancer has metastasized to her right kidney. An interferon (Aldesleukin) has been prescribed to treat this metastasis. The patient asks why this interferon is ordered. What is the nurses best response? A) Aldesleukin has been shown to protect autologous tumor cells. B) Aldesleukin has been shown to inhibit tumor growth. C) Aldesleukin has been shown to enhance allogeneic stem-cell transplantation. D) Aldesleukin has been shown to have a direct proliferative effect on renal tumors. Ans: B Feedback: Aldesleukin is prescribed for metastatic renal cell carcinoma in adults and treatment of metastatic melanomas (orphan drug use) working by activating human cellular immunity and inhibiting tumor growth through increases in lymphocytes, platelets, and cytokines. Aldesleukin does not protect autologous tumor cells, enhance allogeneic stem-cell transplantation, or have a direct proliferative effect on renal tumors. 14. The nurse admits a patient who was newly diagnosed with Kaposis sarcoma to the unit. The physician has ordered an IV infusion of an interferon. What drug would be appropriate? A) Interferon beta1a B) Interferon gamma 1b C) Interferon alfa 2b D) Peginterferon alfa 2b Ans: C Feedback: Interferon alfa 2b indications include hairy cell leukemia, melanoma, AIDS- related Kaposis sarcoma, chronic hepatitis B and C infection, intralesional treatment of condyloma acuminatum in patients 18 years of age or older. No other interferons are indicated for treatment of Kaposis sarcoma. 15. The pharmacology instructor is talking about interferon. The instructor explains that agents, such as interferons, have more than one biologic function. What are the functions of interferons? (Select all that apply.) A) Antibacterial B) Antiviral C) Immunomodulatory D) Antiproliferative E) Anticancer Ans: B, C, D Feedback: Interferons act to prevent virus particles from replicating inside the cells. They also stimulate interferon receptor sites on noninvaded cells to produce antiviral proteins, which prevent viruses from entering the cell. In addition, interferons have been found to inhibit tumor growth and replication, to stimulate cytotoxic T-cell activity, and to enhance the inflammatory response. Options A and E are incorrect. 16. The nursing class is studying monoclonal antibodies. What monoclonal antibody reacts to human T cells, disabling them and acting as an immune suppressor? A) Adalimumab B) Cetuximab C) Rituximab D) Muromonab-CD3 Ans: D Feedback: Muromonab-CD3, the first monoclonal antibody approved for use, is a T- cellspecific antibody, that is available as an IV agent. It reacts as an antibody to human T cells, disabling the T cells, acting as an immune suppressor. Adalimumab is an antibody specific for human tumor necrosis factor. Cetuximab is an antibody specific to epidermal growth factor receptor sites. Rituximab is an antibody specific to sites on activated B lymphocytes. 17. What monoclonal antibody is used to prevent respiratory syncytial virus (RSV) in high risk children? A) Palivizumab B) Natalizumab C) Belimumab D) Eculizumab Ans: A Feedback: Palivizumab is specific to the antigenic site on respiratory syncytial virus (RSV); it inactivates that virus. It is used to prevent RSV disease in high-risk children. Natalizumab is an antibody specific to surface receptors on all leukocytes except neutrophils. Belimumab is a specific inhibitor of B- lymphocyte stimulator that inhibits the survival of B-lymphocytes and their differentiation into immune-globulin producing cells. Eculizumab binds to complement proteins and prevents the formation of the complement complex. 18. The nurse is caring for a patient with an allograft transplant. The physician orders a monoclonal antibody to prevent rejection of the transplant. What monoclonal antibody would the nurse expect to be ordered? A) Alemtuzumab B) Daclizumab C) Erlotinib D) Omalizumab Ans: B Feedback: Daclizumab is specific to interleukin-2 receptor sites on activated T lymphocytes; it reacts with those sites and blocks cellular response to allograft transplants. Alemtuzumab is an antibody specific for lymphocyte receptor sites used to treat chronic lymphocytic leukemia patients who have been treated with alkylating agents and have been failed by fludarabine therapy. Erlotinib is effective against specific malignant receptor sites. Omalizumab is an antibody to immunoglobulin E, an important factor in allergic reactions. 19. The pharmacology instructor is explaining interleukins to the class. What would be the best definition of interleukins? A) They are substances naturally produced and released by human cells that have been invaded by viruses. B) They block the inflammatory reaction and decrease initial damage to cells. C) They are chemicals used to communicate between leukocytes and stimulate immunity. D) They attach to specific receptor sites and respond to very specific situations. Ans: C Feedback: Interleukins are chemicals produced by T cells to communicate between leukocytes and stimulate cellular immunity and inhibit tumor growth. Immune suppressants block the inflammatory reaction and decrease initial damage to cells. Interferons are naturally produced and released by human cells that have been invaded by viruses. Monoclonal antibodies attach to specific receptor sites and respond to very specific situations. 20. The nurse is caring for a female patient, aged 62, who has been admitted for treatment of metastatic melanoma. What agent would the nurse anticipate the physician is likely to order? A) Aldesleukin B) Interferon alfa 2b C) Cyclosporine D) Ipilimumab Ans: D Feedback: Ipilimumab is a human cytotoxic T-cell antigen-4 blocking antibody. By blocking this site, T cells are activated and proliferate at a faster rate. It is used to treat patients with unresectable or metastatic melanoma. Aldesleukin is an interleukin, used for metastatic renal cell carcinoma in adults; a treatment of metastatic melanomas. Interferon alfa 2b is indicated for hairy cell leukemia, melanoma, AIDS-related Kaposis sarcoma, chronic hepatitis B and C infections, intralesional treatment of condyloma acuminatum in patients 18 years of age or older. Cyclosporine is a T and B cell suppressor and is indicated for prophylaxis for organ rejection in kidney, liver, and heart transplants (used with corticosteroids); treatment of chronic rejection in patients previously treated with other immunosuppressants; treatment of rheumatoid arthritis; and recalcitrant psoriasis. 21. While studying the T- and B-cell immune suppressors, the nursing students learn that the most commonly used immune suppressant is what? A) Cyclosporine (Sandimmune) B) Azathioprine (Imuran) C) Pimecrolimus (Elidel) D) Glatiramer (Copaxone) Ans: A Feedback: Several T- and B-cell immune suppressors are available for use. Of the numerous agents available, cyclosporine is the most commonly used immune suppressant. Options B, C, and D are all T- and B-cell immune suppressors, they are simply not the most commonly prescribed. 22. What interleukin receptor antagonist would the nurse anticipate is most likely to be ordered for a patient, 25 years old, who has not responded to traditional antirheumatic drugs? A) Natalizumab (Tysabri) B) Anakinra (Kineret) C) Eculizumab (Soliris) D) Adalimumab (Humira) Ans: B Feedback: Anakinra is used to reduce the signs and symptoms of moderately to severely active rheumatoid arthritis in patients 18 years of age and older who have not responded to the traditional antirheumatic drugs. Options A, C, and D are monoclonal antibodies, therefore they are incorrect answers. 23. A patient with chronic hepatitis C has been prescribed peginterferon alfa 2b (PEG-INTRON). By what route would the nurse administer this drug? A) Subcutaneously (SQ) B) Intramuscularly (IM) C) Intralesionally (IL) D) Orally Ans: A Feedback: PEG-INTRON, like many of the interferons, is administered subcutaneously. Avonex is given intramuscularly. Interferon alfa n3 is given intralesionally. There are no interferons given orally. 24. The patient has arrived in the short stay unit for an infusion of tositumomab with 131 tositumomab (Bexxar). Before beginning the infusion, the nurse assesses the patients vital signs and finds the patient has a temperature of 101.5F, What is the nurses priority action? A) Holding the infusion until patient is afebrile B) Notifying the physician C) Starting the infusion and inform the physician D) Treating the fever before beginning the therapy Ans: D Feedback: Monoclonal antibodies should be used cautiously with fever (treat the fever before beginning therapy). This makes Options A, B, and C incorrect. 25. A 72-year-old male patient has arrived at the outpatient unit to receive an infusion of alemtuzumab (Compath). The patient tells the nurse this is the second time his chronic lymphocytic leukemia has relapsed and the second time he will receive this drug because he failed alemtuzumab therapy after being treated with an alkylating agent. What is the priority nursing action? A) Calling the physician and questioning the order B) Washing your hands C) Beginning an intravenous infusion D) Canceling the infusion Ans: A Feedback: Monoclonal antibodies should be used cautiously in patients who have had previous administration of the monoclonal antibody (serious hypersensitivity reactions can occur with repeat administration). The nursing priority would be to question the order because the patient has already received alemtuzumab (Compath) previously and if the order is verified, this patient should be monitored very carefully, perhaps starting to infuse more slowly until the patients reaction can be determined. Only after questioning the order and having it verified would the nurse perform hand hygiene and begin the infusion. 26. The nurse is writing a plan of care for a patient receiving immune suppressants for leukemia. What would be an appropriate nursing diagnosis for this patient? A) Anxiety related to diagnosis and drug therapy B) Acute pain related to central nervous system (CNS), gastrointestinal (GI), and flu-like effects C) Risk for infection related to immune stimulation D) Imbalanced nutrition: More than body requirements Ans: B Feedback: Nursing diagnoses related to drug therapy might include: Acute pain related to CNS, GI, and flu-like effects. Anxiety related to diagnosis and drug therapy is a nursing diagnosis for a patient on an immune stimulant. There is no risk for infection related to immune stimulation unless an adverse effect occurs. Imbalanced nutrition would be less than body requirements due to flu-like symptoms resulting in diminished appetite. 27. The nurse is preparing a patient to receive immunosuppressant drugs on an outpatient basis. What is the priority for the nurse to arrange for this patient in the home environment? A) A caregiver who is skilled in cardiopulmonary resuscitation (CPR) B) A caregiver who will provide adequate nutrition C) Supportive care and comfort measures D) Arrange for a home care nurse to administer injections Ans: C Feedback: Arrange for supportive care and comfort measures for flu-like symptoms (rest, environmental control, acetaminophen) to decrease patient discomfort and increase therapeutic compliance. Patients may also need support and comfort measures related to diagnosis and drug therapy. Although knowledge of CPR and providing appropriate nutrition are always positive actions, they are not related to administration of immunosuppressants. The patient or caregiver can be taught to administer injections unless the medication is to be given IV, in which case the patient would go to an infusion center. 28. The nurse is caring for a patient who has a diagnosis of chronic hepatitis B infection and has been prescribed an immune stimulant. After teaching the patient about the treatment plan, how might the nurse evaluate the effectiveness of teaching? A) The patient can state where to go to get the medication. B) The patient can state who will administer the medication. C) The patient can state what positive effects to watch for. D) The patient can state specific measures to avoid adverse effects. Ans: D Feedback: The nurse would evaluate that the teaching plan was successful if the patient can name drug, dosage, adverse effects to watch for, and specific measures to avoid adverse effects. Knowing where to get the medication, who will administer it, and the positive effects to watch for would not be an adequate assessment of the teaching plan. 29. The patient underwent an allograft renal transplant 48 hours earlier and is showing signs of rejection. What drug would the nurse expect the physician to order? A) Muromonab B) Anakinra C) Mycophenolate D) Sirolimus Ans: A Feedback: Muromonab is indicated for the treatment of acute allograft rejection in patients undergoing renal transplantation. It also is indicated for the treatment of steroid-resistant acute allograft rejection in those receiving heart or liver transplants. Anakinra, mycophenolate, and sirolimus are useful for preventing renal or liver transplant rejection. 30. The nurse, working with a nursing student, is caring for a patient who is to receive interleukins. The student nurse asks you what happens physiologically when a patient receives interleukins. What is the nurses best response? A) It really helps the patient! B) The patient has increases in the number of natural killer cells. C) The patient has decreased cytokine activity. D) The patient gets really sick from flu-like symptoms and then they get better. Ans: B Feedback: When interleukins are administered, there are increases in the numbers of natural killer cells and lymphocytes, in cytokine activity, and in the number of circulating platelets. Options A, C, and D are incorrect. 31. The nurse is caring for a child requiring cyclosporine to prevent rejection. Cyclosporine is given to adults using a dosage of 15 mg/kg. The nurse calculates the childs dosage is 20 mg/kg. What is the nurses priority action? A) Administer the drug. B) Hold the dose and question the ordering provider. C) Complete an incident report if this dosage has already been given before. D) Notify the nursing supervisor of the medication error. Ans: A Feedback: The nurse would administer the medication as ordered because doses larger than those given to adults are often needed when cyclosporine is administered to children. This is not an error so the nurse would not hold the drug, question the provider, complete an incident report, or notify the nursing supervisor. 32. When caring for older adults receiving immune modulators, what are the nurses priorities of care? (Select all that apply.) A) Assess carefully for infection. B) Obtain baseline liver function studies and monitor follow-up studies. C) Determine dosage based on renal and liver function. D) Minimize teaching to avoid causing confusion. E) Encourage the family to visit often, especially young children. Ans: A, B, C Feedback: Older patients may be more susceptible to the effects of the immune modulators, partly because the aging immune system is less efficient and less responsive. These patients need to be monitored closely for infection, GI, renal, hepatic, and central nervous system effects. Baseline renal and liver function tests can help to determine whether a decreased dosage will be needed before beginning therapy. Because these patients are more susceptible to infection, they need to receive extensive teaching, not less teaching, about ways to avoid infection and injury. Contact with young children and large groups of people increase the risk of infection. 33. The nurse teaches the female patient receiving immune modulating drugs about the need to use barrier contraceptives. The patient says, I hate using barrier contraceptives. Why cant I just take oral contraceptives? What is the nurses best response? A) Effects of oral contraceptives may be altered by liver changes or changes in immune response. B) Oral contraceptives increase the action of immune modulating drugs so dosage needs to be reduced. C) Immune modulators make oral contraceptives ineffective because of hormonal impact of drugs. D) Oral contraceptives are acceptable if barrier contraceptives are distasteful, but only high-estrogen pills can be used. Ans: A Feedback: The use of barrier contraceptives is advised because the effects of oral contraceptives may be altered by liver changes or by changes in the bodys immune response, potentially resulting in unexpected pregnancy. The other options conflict with this information and are incorrect. 34. The nurse is caring for a young adult woman taking immune modulating medications who has been advised to use barrier contraceptives but she wants to start her family. What information can the nurse provide about these drugs to help this patient with her decision-making? A) Discuss the desire to start a family with the provider so risk can be minimized. B) Immune modulating drugs will need to be discontinued if pregnancy occurs. C) Immune modulating drugs have been proven to be highly teratogenic. D) Pregnancy is not an option when taking immune modulating drugs but adoption is an option. Ans: A Feedback: If a patient taking immune modulators becomes pregnant or decides that she wants to become pregnant, she should discuss this with her health care provider and review the risks associated with use of the drug or drugs being taken. Monoclonal antibodies should be used with caution during pregnancy and lactation. Because results of long-term studies of most of these drugs are not yet available, it may be prudent to advise patients taking these drugs to avoid pregnancy if possible. Immune modulating drugs do not need to be discontinued, but the safest drug should be prescribed. Most immune modulating drugs have not been studied and there is not enough information to know whether they are teratogenic. The nurse cannot tell a patient that pregnancy is not an option. 35. The nurse is caring for a patient diagnosed with rheumatoid arthritis (RA) who recently underwent a liver transplant. What immunosuppressant could this patient be prescribed that would treat both diagnoses? A) Anakinra (Kineret) B) Adalimumab (Humira) C) Sirolimus (Rapamune) D) Cyclosporine (Sandimmune) Ans: A Feedback: Anakinra is used to prevent rejection after kidney or liver transplantation and also reduces signs and symptoms of RA in patients who have had inadequate response to other drugs. Adalimumab would be effective for the patients RA but would not prevent rejection of the transplanted liver. Sirolimus is used to prevent rejection of kidney transplants but would not be effective for either of the patients diagnoses. Cyclosporine would be appropriate to prevent liver rejection but would not treat RA. Chapter 18. Pulmonary System Medications 1. A patient comes to the clinic with symptoms as seen in a cold. What group of upper respiratory drugs causes local vasoconstriction, which decreases blood flow and shrinks swollen membranes to improve air flow? A) Antitussives B) Decongestants C) Expectorants D) Mucolytics Ans: B Feedback: Decongestants cause local vasoconstriction that decreases blood flow to irritated and dilated capillaries of the mucous membranes lining the nasal passages and sinus cavities. This vasoconstriction leads to a shrinking of swollen membranes and opens clogged nasal passages promoting drainage of secretions and improved air flow. Antitussives either work directly on the medullary cough center of the brain or act as a local anesthetic on the respiratory passages blocking the effectiveness of the stretch receptors that stimulate a cough reflex. Expectorants liquefy lower respiratory tract secretions, reducing the viscosity of the secretions and so making it easier to cough them up. Mucolytics break down mucus to aid a person in coughing up thick tenacious secretions by separating cells that hold mucous material together. 2. What would be the nurses best response if a patient calls the clinic and reports that he has had a persistent cough for 2 weeks and asks the nurse for a recommendation for a cough medicine? A) Look for dextromethorphan as an ingredient in any OTC cough preparation. B) The doctor could order an antihistamine, which might dry up your secretions and stop the cough. C) You should come to the clinic to be evaluated. A cough that lasts that long might be an indication of an underlying medical problem. D) Drink a lot of fluids and take aspirin, which should reduce the irritation in your throat. Ans: C Feedback: A cough that has persisted for 2 weeks could be indicative of an underlying medical condition that should be addressed before treating the cough. The patient should be asked to come in for an evaluation. Dextromethorphan could help stop the cough, but suppressing the cough might not be in the patients best interest. Drying the mucosa with an antihistamine could aggravate the cough. Aspirin would not be indicated for relieving throat irritation. 3. The nurse is caring for a patient who is receiving acetylcysteine (Mucomyst) by face mask. What would be an appropriate nursing diagnosis? A) Impaired swallowing B) Risk for impaired skin integrity C) Risk for falls D) Sleep deprivation Ans: B Feedback: A patient receiving acetylcysteine by face mask should have the residue wiped off the face mask and her face with plain water to prevent skin breakdown. The appropriate nursing diagnosis would be risk for impaired skin integrity. Acetylcysteine does not cause impaired swallowing, produce any CNS effects that could increase the risk for falls, or impair the patients ability to sleep. 4. A nurse is caring for a 15-year-old patient with cystic fibrosis who has been prescribed dornase alfa (Pulmozyme) to relieve the buildup of secretions and keep airways open and functioning longer. What will the nurse instruct the patient to do concerning the use of this drug? A) Use in home nebulizer up to four times a day if needed. B) Stop all other medications while using the drug. C) Store the drug in the refrigerator, protected from light. D) Expect a severe headache after each use. Ans: C Feedback: Patients using dornase alfa should be cautioned to store the drug in the refrigerator, protected from light. Heat and light can cause the drug to break down and can decrease its therapeutic value. The drug should only be used up to two times a day and the patient should continue all other medications while using it. Dornase alfa is only a palliative therapy that improves respiratory symptoms. A severe headache is not associated with this drug. 5. A nurse in a three drops with respiratory problems has received orders for four patients. What medication order will the nurse question? A) Tetrahydrozoline (Tyzine) three drops in each nostril for an 8-year-old child B) Hydrocodone (Hycodan) 10 mg PO for a 5-year-old child C) Pseudoephedrine (Dorcol) 15 mg PO for a 2-year-old child D) Diphenhydramine (Benadryl) 5 mg/kg IM for a 10-year-old child Ans: B Feedback: The nurse should question the order for hydrocodone. The ordered dose, 10 mg, is an adult dose and should not be given to a 5-year-old. Patients from 2 to 12 years of age should be given between 1.25 and 5 mg/dose. The other medications are all correct dosages for the age of the child for whom they were prescribed. 6. Parents who treat their childrens cold and flu symptoms at home should be educated concerning the reading and understanding of over-the-counter (OTC) labels. Why is this statement true? A) Many of these preparations contain the same active ingredients so that inadvertent overdose is a common problem. B) Each product is best used for alleviating a particular symptom. C) Some of these products do not contain any drugs. D) Some of these products could interfere with breast-feeding. Ans: A Feedback: Parents need to be educated to read the labels of any OTC preparation they give their children. Many of these preparations contain the same ingredients and inadvertent overdose is a common problem. Giving a child a drug should not interfere with the mothers milk production. All these products contain chemicals, which are drugs; many can be used to treat more than one symptom. 7. A patient presents at the clinic with a dry nonproductive cough. The patient is diagnosed with bronchitis and it has been determined that assistance is needed in thinning the sputum so the cough can become productive. What does the nurse expect the physician will prescribe? A) Benzonatate (Tessalon) B) Guaifenesin (Mucinex) C) Dextromethorphan (Benylin) D) Hydrocodone (Hycodan) Ans: B Feedback: Because this patient needs to cough up respiratory secretions, he would likely be prescribed guaifenesin. This drug is an expectorant that liquefies lower respiratory secretions by reducing their viscosity and so making it easier for a patient to cough them up. Benzonatate, dextromethorphan, and hydrocodone are antitussives and are given to suppress the cough reflex. 8. A patient presents at the clinic with signs and symptoms of seasonal allergic rhinitis. The patient is prescribed a nasal steroid to relieve symptoms. Two days later, the patient calls the clinic and tells the nurse that he is frustrated and wants a new drug. What is the most appropriate response by the nurse? A) It may take up to 2 weeks to get the full clinical effect. Try to keep using the drug as ordered. B) The drug must not work for you. Lets change to an oral steroid. C) You probably are administering the drug incorrectly. Come in and we can review the process. D) You probably need to try a different nasal steroid. This one should be effective by now. Ans: A Feedback: Nasal steroids require about 2 weeks to reach their full clinical effect so the patient should be encouraged to use the drug for that length of time before changing drugs or giving up. The other responses could be appropriate if after 2 weeks the patient is still not getting relief. 9. A patient has an important presentation to make in 4 hours and he needs relief from the congestion of seasonal rhinitis. The patient calls the nurse, explains the situation, and tells the nurse that he cannot afford to be drowsy. Which antihistamine would be a good choice for this patient? A) Diphenhydramine (Benadryl) B) Dexchlorpheniramine (Polaramine) C) Loratadine (Claritin) D) Hydroxyzine (Atarax) Ans: C Feedback: The first-generation antihistamines, including diphenhydramine, dexchlorpheniramine, and hydroxyzine, are associated with drowsiness. Loratadine is one of the second-generation antihistamines, which have fewer anticholinergic effects and are less likely to cause drowsiness. 10. A 71-year-old man with a history of heart disease and diabetes has had an antihistamine prescribed. The nurse is concerned with this prescription because of the risk for what? A) Cardiac arrhythmias B) Increased salivation and choking C) Severe constipation D) Insomnia Ans: A Feedback: The patient has history of heart disease. Antihistamines have been associated with prolongation of the QT interval, which can lead to potentially fatal cardiac arrhythmias. Antihistamines dry the mucosa and are not associated with increased salivation or choking, can cause drowsiness, and are not associated with insomnia or severe constipation. 11. A 29-year-old female patient has sinusitis, so the physician orders a topical nasal decongestant. What instructions should be given? A) Avoid becoming pregnant during decongestant therapy. B) Increase fluids to 2 L/d. C) Restri ct fluids to 500 mL/d. D) Take the medication with meals. Ans: B Feedback: Institute other measures to help relieve the discomfort of congestion (e.g., humidity, increased fluid intake, cool environment, avoidance of smoke- filled areas) as appropriate. The medication does not need to be taken with meals or to restrict fluids. It would be inappropriate to tell the patient to avoid becoming pregnant. 12. What statement by a 61-year-old patient who is to take an antitussive with codeine indicates that the nurses teaching has been effective? A) I will take this medication anytime I start to cough. B) This medication may make me anxious and nervous. C) I should call the physician if I develop nausea, diarrhea, or stomach cramps while taking this medication. D) This medication can cause drowsiness, so I will avoid driving or using power equipment while I take it. Ans: D Feedback: Codeine is a CNS depressant and should not be combined with driving or heavy machinery activities. Antitussives are not intended to be taken with every coughing episode because the patient may overdose on the medication. The medication usually makes the patient drowsy rather than nervous and anxious. Codeine may cause GI upset, although it is usually constipating; some patients may complain of nausea and stomach distress while taking this medication. 13. What statement by the patient leads the nurse to believe that he needs additional instruction regarding his nasal decongestant? A) I will blow my nose before instilling the nasal spray. B) I will report any dizziness, drowsiness, or rapid pulse. C) I will drink 2,000 to 3,000 mL of fluid daily. D) I will use it only when I have nasal discharge. Ans: D Feedback: Decongestants decrease overproduction of secretions by causing local vasoconstriction to the upper respiratory tract (See Table 54.2). This vasoconstriction leads to a shrinking of swollen mucous membranes and tends to open clogged nasal passages, providing relief from the discomfort of a blocked nose and promoting drainage of secretions and improved airflow. The patient must understand proper administration, which includes clearing the nasal passages before inhaling the medication and increasing fluid intake and reporting adverse effects. The medication must be used on a regular basis to be effective. Option B is a distracter. 14. The nurse is giving discharge instructions to a patient with an upper respiratory infection who has been advised to take an over-the-counter (OTC) topical nasal decongestant. The nurse advises the patient about what common adverse reaction to these medications? A) Diarrhea B) Rhinitis medicamentosa C) Rash D) Headache Ans: B Feedback: An adverse effect that accompanies frequent or prolonged use of topical nasal decongestants is rebound congestion, technically called rhinitis medicamentosa. Other adverse reactions include disorientation, confusion, nausea, vomiting, fever, and dyspnea. Diarrhea, rash, and headache are not commonly associated with these drugs, however. 15. The nurse is caring for a patient who is taking dextromethorphan for cough suppression. The nurse will assess this patient for hypotension if he also takes which other medication? A) Calcium-channel blockers B) Monoamine oxidase (MAO) inhibitors C) Beta-blockers D) Thiazide diuretics Ans: B Feedback: Dextromethorphan should not be used in conjunction with MAO inhibitors because hypotension, fever, nausea, myoclonic jerks, and coma could occur. No known drugdrug interaction exists between dextromethorphan and calcium-channel blockers, beta-blockers, and thiazide diuretics. 16. The nurse is giving discharge instructions to the mother of a 3-month-old infant who has an upper respiratory tract infection and has been prescribed a pseudoephedrine nasal solution. What instructions are most important for the nurse to give to this mother? A) Instill the medication 20 to 30 minutes before feeding. B) Keep the baby on clear liquids until the nasal discharge has resolved. C) Start the baby on cereal, because she is having difficulty sucking right now. D) Give the medication immediately after feeding. Ans: A Feedback: Oral decongestants are drugs that are taken by mouth to decrease nasal congestion related to the common cold, sinusitis, and allergic rhinitis. They are also used to relieve the pain and congestion of otitis media. Opening of the nasal passage allows better drainage of the Eustachian tube, relieving pressure in the middle ear. It should be given prior to a feeding so that infant is able to suck more effectively. 17. The nurse is caring for a patient who does not have a respiratory disorder but has been prescribed acetylcysteine. What is an additional indication for acetylcysteine (Mucomyst)? A) Conversion of cardiac dysrhythmias B) Treatment of peptic ulcer disease C) Antidote for acetaminophen poisoning D) Decreased bronchospasm Ans: C Feedback: Acetylcysteine is used orally to protect liver cells from being damaged during episodes of acetaminophen toxicity because it normalizes hepatic glutathione levels and binds with a reactive hepatotoxic metabolite of acetaminophen. Acetylcysteine is not used for the conversion of cardiac dysrhythmias, for treatment of peptic ulcer disease, or for decreasing bronchospasm. 18. The nursing instructor is teaching the lab students the best position for the administration of nasal sprays. What position would the instructor teach the students? A) Supine B) Semi-Fowlers C) High Fowlers D) Side-lying Ans: C Feedback: Teach the patient to sit upright and press a finger over one nostril to close it. This body position is important to prevent excessive amounts of the medication running down the back of the throat. It needs to be in direct contact with the greatest amount of nasal mucosa and the high Fowlers position provides that. 19. The nurse advises that patient to avoid long-term use of nasal decongestants because it may lead to what condition? A) Mucosal ulcerations B) Decreased drainage C) Increased risk of infection D) Asthma Ans: A Feedback: Adverse effects associated with topical decongestants include local stinging and burning, which may occur the first few times the drug is used. If the sensation does not resolve, the drug should be discontinued, because it may indicate lesions or erosion of the mucous membranes. Nasal decongestants do not cause asthma or increased risk of infection. These medications do not decrease drainage from the nose as they shrink the nasal mucosa. 20. What disorders would the pharmacology instructor tell the nursing students may be exacerbated by the use of nasal decongestants? A) Pneumonia B) Rheumatoid arthritis C) Acid reflux D) Hypothyroidism Ans: D Feedback: Assess for possible contraindications or cautions; any history of allergy to the drug or a component of the drug vehicle; glaucoma, hypertension, diabetes, thyroid disease, coronary disease, and prostate problems, all of which could be exacerbated by the sympathomimetic effects. Nasal decongestants do not appear to exacerbate pneumonia, rheumatoid arthritis, or acid reflux. 21. The pharmacology instructor questions the students as to which classification of drugs is commonly found in over-the-counter (OTC) combination cold medications? A) Stimulants B) Opioids C) Oral decongestants D) Antitussives Ans: C Feedback: Oral decongestants are found in many OTC cold and flu preparations so that care must be taken to avoid inadvertent overdose when more than one such drug is used. Opioids are only available by prescription; stimulants and antitussives are not generally found in OTC combination cold medications. 22. A patient visits the clinic and is diagnosed with acute sinusitis. To promote sinus drainage, what medication might be ordered? A) Topical nasal steroid decongestants B) First-generation antihistamines C) Second-generation antihistamines D) Topical decongestants Ans: D Feedback: Topical decongestants are sympathomimetics, meaning that they imitate the effects of the sympathetic nervous system to cause vasoconstriction, leading to decreased edema and reduced inflammation of the nasal membranes. They are available as nasal sprays that are used to relieve the discomfort of nasal congestion that accompanies the common cold, sinusitis, and allergic rhinitis. Topical nasal steroid decongestants are used for the treatment of allergic rhinitis and to relieve inflammation after the removal of nasal polyps. First- and second-generation antihistamines are not ordered for sinusitis. 23. The nurse is caring for a patient who needs education on his medication therapy for allergic rhinitis. The patient is to take clemastine (Tavist) daily. In providing educational interventions regarding this medication, what is the most important instruction on the action of the medication? A) It blocks the effects of histamine. B) It is used to treat atrial and ventricular dysrhythmias. C) It competitively inhibits the rate-limiting enzyme in the liver. D) It leads to bronchodilation and relaxes smooth muscle in the bronchi. Ans: A Feedback: Clemastine blocks the effects of histamine at the histamine-1 receptor sites, decreasing the allergic response. It is prescribed to treat allergic rhinitis. Antiarrhythmic medications are used to treat atrial and ventricular dysrhythmias. Beta-hydroxy-beta-methylglutaryl coenzyme A reductase inhibitors are used to lower blood cholesterol by competitively inhibiting the rate-limiting enzyme in the liver. Beta-adrenergic agents lead to bronchodilation and stimulate beta2-adrenergic receptors in the smooth muscle of the bronchi and bronchioles. 24. The nursing instructor is discussing the administration of nasal spray with the nursing students. What information is most important to include in this discussion? A) Finish the bottle of nasal spray to clear the infection effectively. B) Nasal spray can be shared between family members only. C) Administer the nasal spray in a prone position. D) Overuse of nasal spray may cause rebound congestion. Ans: D Feedback: An adverse effect that accompanies frequent or prolonged use of decongestants is rebound vasodilation, clinically called rhinitis medicamentosa. The reflex reaction to vasoconstriction is a rebound vasodilation, which often leads to prolonged overuse of decongestants. The patient should hold his or her head back for maximum distribution of the spray. Only an individual patient should use the bottle of medication. 25. The clinic nurse is caring for a patient who has been prescribed fexofenadine for hay fever. When the nurse is assessing this patients medication history, what drug would make the nurse question the order for fexofenadine? A) Tetracycline B) Penicillin C) Gentamicin D) Ketoconazole Ans: D Feedback: Drugdrug interactions vary among antihistamines. For example, anticholinergic effects may be prolonged if diphenhydramine is taken with a monoamine inhibitor and the interaction of fexofenadine with ketoconazole or erythromycin may raise fexofenadine concentrations to toxic levels. This is not a concern with tetracycline, penicillin, or gentamicin. 26. A student nurse is doing research consisting of chart audits of 25 patients with diagnosed chronic pharyngitis; she is attempting to determine what medication has been prescribed most frequently. What type of medication would this nurse expect to find in most of the chart audits? A) Antitussives B) Nasal sprays C) Oral decongestants D) Mucolytics Ans: A Feedback: Antitussives are drugs that suppress the cough reflex. Many disorders involving the respiratory tract, including the common cold, sinusitis, pharyngitis, and pneumonia are accompanied by an uncomfortable, nonproductive cough. Persistent coughing can be exhausting and can cause muscle strain and further irritation of the respiratory tract. Nasal sprays, oral decongestants, and mucolytics are not generally prescribed for chronic pharyngitis, however. 27. The nurse is teaching a group of patients with allergic rhinitis about the use of their medications. What would be the most essential information to give these patients about preventing possible drug interactions? A) Over-the-counter (OTC) medications are safe to use. B) Use only one pharmacy so the pharmacist can check drug interactions. C) Read drug labels before taking OTC medications. D) Ask the pharmacy tech for assistance in selecting an OTC medication. Ans: C Feedback: Teach patients to read the OTC labels to avoid inadvertent overdose. It would be inappropriate to teach the patient to use only one pharmacy for OTC medications. OTC medications are generally safe to use if used correctly. Asking the pharmacy tech for help in selecting an OTC medication is appropriate but not the most essential information to give the patients. 28. A patient has been prescribed a nasal steroid and asks the nurse what the most common reason that this medication is used to treat. What is the nurses best response? A) Nasal steroids are only used to treat a sinus infection. B) Nasal steroids are used to treat allergic rhinitis. C) Nasal steroids are used for an infection in the adenoids. D) Nasal steroids are used for all acute upper respiratory infections. Ans: B Feedback: Because nasal steroids block the inflammatory response, their use is contraindicated in the presence of acute infections. The most common reason they are prescribed is for the treatment of allergic rhinitis or to relieve inflammation after the removal of nasal polyps. Nasal steroids are not used for a sinus infection, an infection in the adenoids, or any other acute upper respiratory infection. 29. The nurse is writing a care plan for a patient who has been prescribed a nasal steroid. What would be an appropriate nursing diagnosis for this patient? A) Disturbed sensory perception (kinesthetic) related to CNS effects B) Risk for injury related to suppression of inflammatory reaction C) Ineffective airway clearance related to bronchospasm D) Ineffective airway related to nasal obstruction Ans: B Feedback: Nursing diagnoses related to drug therapy might include acute pain related to local effects of the drug, risk for injury related to suppression of inflammatory reaction, and deficient knowledge regarding drug therapy. Nursing diagnosis for this patient does not include disturbed sensory perception, ineffective airway clearance, or ineffective airway. 30. The family nurse practitioner is caring for a Hispanic woman who is 83 years old. The patient has been noncompliant with the care regimen the nurse practitioner has previously outlined for treatment of sinusitis. What should the nurse practitioner do that can assist the patient in being more compliant with the prescribed treatment regimen? A) Provide instructions in writing. B) Provide the instructions in large type. C) Give the treatment instructions to a member of her family. D) Give the treatment instructions to the womans husband. Ans: A Feedback: The nurse instructs the patient about signs and symptoms that require follow-up and provides these instructions verbally and in writing. Instructions in alternate formats (e.g., large type, patients language) may be needed to increase the patients understanding and adherence to the treatment plan. Option B is incorrect because the situation does not indicate that the woman cannot read regular-sized type print. Options C and D are incorrect because the situation does not indicate that any family members accompanied the patient to see the family nurse practitioner. 31. The nurse is caring for a patient with cystic fibrosis who is receiving dornase alfa by nebulizer to help thin secretions. What statement by the patient indicates a need for further instruction? A) This medication will loosen up the sticky mucus that is in my lungs. B) I will try to cough after I receive this medication. C) Im glad that this medication will solve all of my problems. D) I will continue to use postural drainage and take my enzymes as ordered. Ans: C Feedback: Cystic fibrosis patients who receive dornase alfa should be cautioned about the need to continue all therapies for their cystic fibrosis because dornase alfa is only a palliative therapy that improves respiratory symptoms, and other therapies, such as coughing, postural drainage and enzymes, are still needed. 32. The nurse is caring for a patient who is scheduled to receive acetylcysteine because of an acetaminophen overdose. The nurse would notify the physician before administering the medication if the patient had which condition? (Select all that apply.) A) Bronchospasm B) Hypertension C) Nephrotic syndrome D) Peptic ulcer E) Esophageal varices Ans: A, D, E Feedback: Before administration, assess for possible contraindications or cautions: any history or allergy to the prescribed drugs and the presence of bronchospasm, which are contraindications to the use of these drugs, as well as findings of peptic ulcer and esophageal varices, which would require careful monitoring and cautious use. Options B and C are not correct. 33. A patient has been using guaifenesin for a cough that accompanied a common cold. The patient calls the nurse help line and states that she thinks she is having an adverse reaction to the medication. The nurse knows that which symptoms are adverse effects of this medication? (Select all that apply.) A) Nausea B) Rash C) Constipation D) Bleeding E) Headache Ans: A, B, E Feedback: Adverse effects of guaifenesin are nausea, vomiting, headache, dizziness, and rash. Constipation and bleeding are not adverse effects of guaifenesin. 34. A patient has been prescribed an antihistamine for treatment of allergic rhinitis. What statements by the patient indicate an understanding of this medication? (Select all that apply.) A) This medication will work best if I take it before I eat anything. B) I need to drink less fluid while I take this medication to help reduce the amount of mucus I have. C) I will use sugarless candies to help with the feelings of a dry mouth. D) I will use a humidifier in the bedroom while I sleep. E) This medication will probably cause my appetite to increase. Ans: A, C, D Feedback: Antihistamines should be taken on an empty stomach and the patient should force fluids, not drink less fluids. The patient may use sugarless candy to help with dry mouth and should increase room humidity. The patient may experience nausea or anorexia but not increased hunger. Options B and E are not correct. 35. An individual calls the nurse help line and asks what the drug diphenhydramine is used for. The nurse knows that the medication is prescribed for which conditions? (Select all that apply.) A) Urticaria B) Vasomotor rhinitis C) Productive cough D) Motion sickness E) Angioedema Ans: A, B, D, E Feedback: Diphenhydramine is used for the symptomatic relief of perennial and seasonal rhinitis, vasomotor rhinitis, allergic conjunctivitis, urticaria, and angioedema; it is also used for treating motion sickness and parkinsonism, as a nighttime sleep aid, and to suppress cough. It would not be used to treat a productive cough, because it is not an expectorant. Lower Respiratory Tract: 1. A 70-year-old man is being treated for chronic obstructive pulmonary disease (COPD) with theophylline (Theo-Dur). What will be a priority assessment by the nurse? A) Ingestion of fatty foods B) Weight C) Activity level D) Use of nicotine Ans: D Feedback: Nutritional status, weight, and activity level would be important for a nurse to know about a COPD patient. However, it would be most important for the nurse to know whether the patient smokes or uses tobacco in other ways or smoking cessation methods that involve nicotine. Nicotine increases the metabolism of theophyllines; the dosage may need to be increased to produce a therapeutic effect. 2. A 6-year-old child weighing 52 pounds has had a loading dose of aminophylline (Truphylline). The nurse is ready to administer the regular prescribed dose of 4 mg/kg every 4 hours for three doses. How many milligram will the patient receive in 12 hours? A) 44 mg B) 94 mg C) 284 mg D) 344 mg Ans: C Feedback: First, using the formula: 2.2 pounds and 52 pounds: times kg, determine the childs weight in kg (52/2.2 = 23.64). Next, using the formula of amount of drug prescribed times weight in kg, determine the amount the child will receive in one dose (4 times 23.64 = 94.56). To determine the mg in a 12- hour period multiply 94.56 times 3 = 283.68. Round to 284 mg. Options A, B, and D are not correct. 3. A patient is in the clinic to have blood drawn to assess his or her theophylline level. The patient is taking theophylline (Theo-Dur) and appears to being doing well on the drug. He or she reports no problems. What serum level will the nurse expect the patient to have? A) Between 0.5 to 5 mcg/mL B) Between 10 to 20 mcg/mL C) Between 25 to 35 mcg/mL D) Between 40 to 50 mcg/mL Ans: B Feedback: Therapeutic theophylline levels should be between 10 and 20 mcg/mL. A level between 0.5 and 5 mcg/mL would be low and would not produce a therapeutic effect. Levels between 25 and 50 mcg/mL would be too high and could cause serious adverse effects. 4. A patient presents to the emergency department (ED) having an acute asthma attack. An ED physician has ordered a sympathomimetic (epinephrine). The nurse expects what as the therapeutic effect of this drug? A) Decrease the inflammatory response in the airways B) Reduce the surface tension within the alveoli allowing for gas exchange C) Inhibit the release of histamine and slow-reacting substance of anaphylaxis (SRSA) to prevent the allergic asthmatic response D) Cause dilation of the bronchi with increased rate and depth of respiration Ans: D Feedback: Epinephrine will cause the bronchi to dilate and also cause the rate and depth of respiration to increase. Inhaled steroids decrease the inflammatory response and lung surfactants reduce the surface tension within the alveoli. Mast cell stabilizers inhibit the release of histamine and SRSA to prevent the allergic response. Options A, B, and C are not correct. 5. An inhaled sympathomimetic drug has been ordered for a teenage athlete who has exercise-induced asthma. What should the patient be instructed to do? A) Use the inhaler every day at the same time each day. B) Use the inhaler as soon as the symptoms start. C) Use the inhaler 30 to 60 minutes before exercising to ensure peak therapeutic levels when needed. D) Use the inhaler 2 to 3 hours before exercising to ensure peak effectiveness. Ans: C Feedback: Teaching a patient about using an inhaled sympathomimetic for management of exercise-induced asthma should include instructions to use the inhaler 30 to 60 minutes before exercising to ensure therapeutic levels when needed. The inhaler would not be used daily and waiting until symptoms occur will be too late for prevention. Options B and D are not correct. 6. A premature newborn is being treated for respiratory distress syndrome. The nurse teaches the parents about what adverse effect that can occur with the use of lung surfactants? A) Kidney dysfunction B) Cardiac arrhythmias C) High fever D) Collapsed lung Ans: D Feedback: Lung surfactants used therapeutically can cause many adverse effects including pneumothorax (collapsed lung), hypotension, pulmonary leak, hyperbilirubinemia, and sepsis. Other adverse effects may occur in the infant related to the degree of immaturity of the childs system and may not be related to the drug therapy. Options A, B, and C are not correct. 7. A patient who has chronic bronchial asthma has had a mast cell stabilizer prescribed. What drug would the physician prescribe? A) Ipratropium or budesonide B) Isoetharine or montelukast C) Nedocromil or cromolyn D) Aminophylline or caffeine Ans: C Feedback: Nedocromil and cromolyn are mast cell stabilizers used in the treatment of asthma. Aminophylline and caffeine are xanthines. Ipratropium is an anti- cholinergic drug and budesonide is a corticosteroid. Isoetharine is a sympathomimetic drug and montelukast is a leukotriene receptor antagonist. 8. A 76-year-old man with asthma is being treated with an anticholinergic. What will the nurse be careful to assess for? A) Cardiac arrhythmias B) Prostatic hypertrophy C) Thyroid conditions D) Parkinsonism Ans: B Feedback: Anticholinergics can produce urinary hesitancy and urinary retention, conditions that would aggravate the signs and symptoms of prostatic hypertrophy. Older patients given anti-cholinergics should be encouraged to empty the bladder before taking the drug. These drugs are used to treat parkinsonism. Thyroid conditions and cardiac arrhythmias are not cautions or contraindications to the use of these drugs. 9. A patient with chronic bronchial asthma is prescribed montelukast (Singulair). What will the nurse instruct the patient to avoid taking? A) Aspirin B) Penicillin C) Sertraline (Zoloft) D) Nifedipine (Procardia) Ans: A Feedback: The nurse would instruct the patient to avoid aspirin, which might cause an increased montelukast level and toxicity. The other options do not cause drugdrug interactions with montelukast. 10. A patient presents at the emergency department in acute respiratory distress. A quick assessment by the triage nurse indicates that the patient experienced difficulty breathing immediately after taking Combivent for the first time. The nurse suspects that the patient may be allergic to what? A) Aspirin B) Penicillin C) Peanuts D) Ragweed pollen Ans: C Feedback: Combivent is a combination drug of ipratropium and albuterol. The propellant used to make ipratropium has a cross-sensitivity to the antigen that causes peanut allergies. Aspirin, penicillin, and ragweed pollen are not associated with this drug. 11. The nurse has admitted a patient (who takes ipratropium) to the respiratory unit with an acute exacerbation of chronic obstructive pulmonary disease (COPD). While writing a plan of care for this patient, what would be the most appropriate nursing diagnosis to use? A) Deficient knowledge regarding alternative therapy B) Imbalanced nutrition: Less than body requirements C) Acute pain related to renal effects of the drug D) Disturbed thought processes related to central nervous system (CNS) effects Ans: B Feedback: Nursing diagnoses related to drug therapy might include acute pain related to CNS, gastrointestinal (GI), or respiratory effects of the drug; imbalanced nutrition: Less than body requirements, related to dry mouth and GI upset; and deficient knowledge regarding drug therapy. Options A, C, and D are not correct. 12. A patient, diagnosed with asthma, has been prescribed tiotropium (Spiriva). What should the nurse teach the patient about this drug? (Select all that apply.) A) It makes you fatigued. B) You need to stay out of direct sunlight. C) It is an anticholinergic. D) You only need to take it once a day. E) It has a rapid onset of action and a long duration. Ans: C, D, E Feedback: Tiotropium is the first drug approved for once-daily maintenance treatment of bronchospasm associated with chronic obstructive pulmonary disease (COPD). Patients who cannot tolerate the sympathetic effects of the sympathomimetics might respond to the anticholinergic drugs ipratropium (Atrovent) or tiotropium. Tiotropium has a rapid onset of action and a long duration, with a half-life of 5 to 6 days. Tiotropium is not associated with fatigue or photosensitivity. 13. The nurse is providing health teaching to a newly diagnosed asthmatic patient. The patient has been prescribed theophylline. What is contraindicated with the use of this drug? A) Using insulin B) Taking anti-inflammatory drugs C) Exercising D) Smoking cigarettes Ans: D Feedback: Nicotine increases the metabolism of xanthines in the liver so that xanthine dosage must be increased in patients who continue to smoke while using xanthines. In addition, extreme caution must be used if the patient decides to decrease or discontinue smoking because severe xanthine toxicity can occur. Bronchoconstriction is not caused by using insulin and anti-inflammatory drugs do not cause bronchoconstriction. Exercise with a physicians supervision is encouraged in individuals with asthma. 14. What action by the patient would indicate that the patient understands how to use an inhaler? A) The patient inhales as soon as the inhaler enters his or her mouth. B) The patient holds his or her breath for several seconds after releasing the medication. C) The patient administers three doses of medication within a 1-minute time frame. D) The patient exhales as soon as he or she compresses the inhaler. Ans: B Feedback: Holding the breath prevents exhalation of medication still remaining in the mouth. The patient should inhale when the canister is compressed, not as soon as the inhaler enters his or her mouth. The patient should only administer one dose of medication at a time and the patient should wait to exhale until after the breath has been held as long as possible. 15. A patient, newly diagnosed with chronic obstructive pulmonary disease (COPD), calls the clinic and asks the nurse to explain what the newly prescribed medications are for. What would be the most appropriate response by the nurse? A) The medications that have been ordered for you are what the physician thinks will help you the most. B) The medications that have been ordered for you are to help you breathe easier. C) The medications that have been ordered for you are designed to work together to help you feel better. D) The medications that have been ordered for you are to help relieve the inflammation and promote dilation of the bronchi. Ans: D Feedback: Drug treatment of asthma and COPD aims to relieve inflammation and promote bronchial dilation. The other options do not give the patient information about the use of these new medications. 16. The nurse is caring for a patient who is taking an adrenergic bronchodilator. In what disease process should adrenergic bronchodilators be used cautiously? A) Liver failure B) Renal failure C) Respiratory failure D) Heart failure Ans: D Feedback: Adrenergic drugs cause cardiac stimulation. Patients with liver failure, renal failure, or respiratory failure do not need to use adrenergic bronchodilators cautiously. 17. The patient is a 34-year-old man who recently started taking theophylline. The nurse knows that medication teaching has been successful when he agrees to what activity? A) Avoiding caffeine B) Eating foods high in potassium C) Limiting fluid intake to 1,000 mL a day D) Taking the medicine on an empty stomach Ans: A Feedback: Both theophylline and caffeine are xanthenes. Theophylline increases cardiac output and heart rate. Caffeine also stimulates heart rate. This can have an additive effect. Eating foods high in potassium, limiting fluid intake, or taking the medicine on an empty stomach are not indications that the patient has understood the nurses teaching. 18. The nurse caring for a 38-year-old patient started on albuterol (Proventil) should advise the patient that he or she may experience what adverse effect? A) Polydipsia B) Tachycardia C) Hypotension D) Diarrhea Ans: B Feedback: Adrenergic agents stimulate beta1-adrenergic receptors in the heart as well as beta2-adrenergic receptors in the lungs. Adrenergic agents do not cause polydipsia, hypotension, or diarrhea. 19. The nursing instructor is discussing bronchodilators with a group of nursing students. The students understand the instruction when they identify what drug is most effective in treating acute bronchospasm? A) Ipratropium bromide (Atrovent) B) Epinephrine (Adrenalin) C) Cromolyn (Intal) D) Ephedrine Ans: B Feedback: Epinephrine may be injected subcutaneously in an acute attack of bronchoconstriction, with therapeutic effects in 5 minutes that last 4 hours. It is considered the drug of choice for the treatment of acute bronchospasm. Ipratropium bromide has an onset of action of 15 minutes when inhaled with a duration of 3 to 4 hours. Cromolyn is not for use during acute times of bronchospasm but is used to help prevent bronchospasm. Ephedrine can be used in acute bronchospasm but epinephrine remains the drug of choice. 20. A patient with chronic obstructive pulmonary disease (COPD) presents at the emergency department in acute respiratory distress. The patients family tells the nurse that the patients problems began right after the patient took his or her first dose of ipratropium (Atrovent). What would the nurse suspect is the problem? A) An allergy to milk B) Overexertion by the patient C) Patient not taking the medications correctly D) An allergy to soy products Ans: D Feedback: The use of ipratropium or tiotropium is contraindicated in the presence of known allergy to the drug or to soy products or peanuts (the vehicle used to make ipratropium an aerosol contains a protein associated with peanut allergies) to prevent hypersensitivity reactions. An allergy to milk is not associated with sensitivity to ipratropium. Overexertion would not cause the patient to develop respiratory distress after using ipratropium for the first time. Misuse of the inhaler would not cause respiratory distress. 21. The nurse has provided health teaching for a 15-year-old boy newly diagnosed with asthma. What statement, made by the patient, indicates that he has a good understanding of the teaching the nurse has done regarding inhalers? A) I should hold my breath when administering a puff. B) The aerosol canister should be shaken well before using. C) I need to take three short quick breaths when I administer the inhaler. D) A second aerosol medication cannot be administered until 30 minutes after the first aerosol medication. Ans: B Feedback: Inhalers should be shaken well, immediately before each use. It would not be appropriate to teach the patient to hold his breath when administering a puff, to take three short quick puffs when administering the inhaler, or that a second aerosol medication should not be administered until 30 minutes after the first dose of aerosol medication. 22. The nurse is writing a plan of care for a patient newly admitted to the floor with an asthma attack that occurred while exercising. What would be the most appropriate intervention for this patient? A) Assist patients with moderate to severe asthma in obtaining a home nebulizer unit B) Try to prevent or reduce panic, which may initiate bronchospasm C) Teach patient to use an inhaler before exercising D) Monitor peak flow rates, especially in children Ans: C Feedback: Teach patient who use one of these drugs for exercise-induced asthma to use it 30 to 60 minutes before exercising to ensure peak therapeutic effects when they are needed. The most important intervention would be to use a bronchodilator as prophylaxis for a patient with exercise-induced asthma. It would not be monitoring peak flow rates, trying to prevent or reduce panic, or assisting patient in obtaining a home nebulizer unit. 23. A patient tells the nurse that a friend has recommended the use of caffeine to treat the patients asthma. The nurse counsels the patient to begin treatment immediately with the prescribed medication for what reason? A) Caffeine can aggravate the drugs used to treat asthma. B) Most natural products are less toxic or more potent than traditional asthma medications. C) Natural products decrease the adverse effects associated with adrenergic bronchodilators. D) Delays in appropriate treatment can have serious, even fatal, consequences. Ans: D Feedback: The xanthines, including caffeine and theophylline, come from a variety of naturally occurring sources. These drugs were formerly the main treatment choices for asthma and bronchospasm. However, because they have a relatively narrow margin of safety, and they interact with many other drugs, they are no longer considered the first-choice bronchodilators. Delays in appropriate treatment can have serious, even fatal, consequences. Natural products do not decrease the adverse effects associated with adrenergic bronchodilators. Natural products have not been proven to be less toxic or more potent than prescribed asthma medications. Caffeine does not aggravate drugs used to treat asthma, but it can have an additive effect. 24. Epinephrine, formerly the drug of choice for acute attacks of bronchoconstriction, has been replaced by what? A) Short-acting bronchodilators are the drug of choice in this situation. B) Nothing has replaced epinephrine as drug of choice in this situation. C) Self-administered metered-dose inhalers (MDIs) D) Long-acting beta2-adrenergic agonists (LABAs) Ans: B Feedback: Epinephrine, the prototype drug, is the drug of choice in adults and children for the treatment of acute bronchospasm, including that caused by anaphylaxis; it is also available for inhalation therapy. Because epinephrine is associated with systemic sympathomimetic effects, it is not the drug of choice for patients with cardiac conditions. Options A, C, and D are not correct. 25. A patient has an acute asthma attack. A bronchodilator is used to bring the exacerbation under control. What drug would be used to prevent acute bronchoconstriction? A) Salbutamol (Ventolin) B) Salmeterol (Serevent) C) Fenoterol (Berotec) D) Terbutaline (Bricanyl) Ans: B Feedback: Salmeterol (Serevent) and formoterol (Foradil) and are long-acting beta2- adrenergic agonists used only for prophylaxis of acute bronchoconstriction. Options A, C, and D are all short-acting beta2-adrenergic agonists and not used for prophylaxis. 26. A patient is prescribed salmeterol with dosage on a 4 to 6 hour schedule for treatment of exercise-induced asthma. What is the recommended dosing schedule of asthma experts regarding this drug? A) 30 minutes before exercise to prevent dyspnea during exercise B) Every 15 minutes during exercise to prevent dyspnea C) As needed to treat or prevent dyspnea during exercise D) Every 1 to 2 hours to treat or prevent dyspnea during exercise Ans: A Feedback: Salmeterol (Serevent) adult and pediatric (12-year-old and older): two puffs every 12 hours; or two puffs 30 to 60 minutes before exercise. Therefore, options B, C, and D are incorrect. 27. The pharmacology instructor is explaining the difference between bronchodilators and anti-inflammatory drugs. How does an anti- inflammatory drug reduce bronchoconstriction? A) Increases ability to metabolize medication B) Decreases formation of mucus secretions C) Increases reactivity to medication D) By decreasing airway hyperreactivity to various stimuli Ans: D Feedback: Bronchodilators, or antiasthmatics, are medications used to facilitate respirations by dilating the airways. They are helpful in symptomatic relief or prevention of bronchial asthma and for bronchospasm associated with chronic obstructive pulmonary disease (COPD). Reducing inflammation prevents and reduces bronchoconstriction by decreasing airway hyperreactivity to various stimuli that decreases mucosal edema and formation of mucus secretions that narrow airways. Anti-inflammatory drugs do not increase the ability to metabolize medication or increases reactivity to medication. 28. Why are inhaled steroids used to treat asthma and chronic obstructive pulmonary disease (COPD)? A) They act locally to decrease release of inflammatory mediators. B) They act locally to improve mobilization of edema. C) They act locally to increase histamine release. D) They act locally to decrease histamine release. Ans: A Feedback: When administered into the lungs by inhalation, steroids decrease the effectiveness of the inflammatory cells. This has two effects, which are decreased swelling associated with inflammation and promotion of beta- adrenergic receptor activity, which may promote smooth muscle relaxation and inhibit bronchoconstriction. Options B, C, and D are incorrect. 29. The nurse is caring for a patient with chronic obstructive pulmonary disease. The plan of care will focus on what patient problem? A) Pain B) Obstructed airway C) Activity intolerance D) Adverse effects of medication therapy Ans: B Feedback: Asthma, emphysema, chronic obstructive pulmonary disease (COPD), and respiratory distress syndrome (RDS) are pulmonary obstructive diseases. All but RDS involve obstruction of the major airways. RDS obstructs the alveoli. Pain, activity intolerance, and adverse effects of medication therapy are conditions identified to detect, manage, and minimize the unexpected outcomes the nurse should be especially aware of the potential for an obstructed airway in these patients. 30. The clinic nurse is caring for a patient who has just been diagnosed with chronic obstructive pulmonary disease (COPD). The patient asks the nurse what COPD means. What would be the nurses best response? A) It is an umbrella term for diseases like acute bronchitis. B) It means that the lungs have been damaged in such a way that there airflow is limited in and out of the lungs. C) It means your lungs cant expand and contract like they are supposed to, which makes it hard for you to breathe. D) It is a term that covers so many lung diseases I cant list them all. Ans: B Feedback: The obstruction of asthma, emphysema, and COPD can be related to inflammation that results in narrowing of the interior of the airway and to muscular constriction that results in narrowing of the conducting tube. With chronic inflammation, muscular and cilial action is lost, and complications related to the loss of these protective processes can occur, such as infections, pneumonia, and movement of inhaled substances deep into the respiratory system. In severe COPD, air is trapped in the lower respiratory tract, the alveoli degenerate and fuse together, and the exchange of gases is greatly impaired. Options A, C, and D are not incorrect, but option B is the best response. 31. The nurse is caring for a child who has been newly diagnosed with asthma. What environmental modifications should the nurse encourage the parents to make to help their child avoid future attacks? (Select all that apply.) A) Make sure the child begins herbal therapy as soon as possible. B) Avoid crowded areas as much as possible. C) Keep the child away from any known allergens. D) Encourage the child to use a broom to sweep the bedroom every day. E) Keep the child away from areas that are filled with cigarette smoke. Ans: B, C, E Feedback: Parents need to be encouraged to take measures to prevent acute attacks, including avoidance of known allergens, smoke-filled rooms, and crowded or dusty areas. OTC drugs and herbal remedies should be avoided if possible. The child should not be sweeping the bedroom because this will produce a lot of dust. 32. The nursing instructor is talking to a group of nursing students about the treatment regimen for children with asthma. The students indicate they understand the information when they identify which class of drugs that comprise this regimen? (Select all that apply.) A) Long-acting inhaled steroids B) Xanthines C) Leukotriene-receptor antagonists D) Topical steroid nasal decongestants E) Beta-agonists Ans: A, C, E Feedback: Antiasthmatics are frequently used in children. The leukotriene-receptor antagonists have been found to be especially effective for long-term prophylaxis in children. Acute episodes are best treated with a beta-agonist and then a long-acting inhaled steroid or a mast cell stabilizer. Xanthines (e.g., theophylline) have been used in children, but because of their many adverse effects and the better control afforded by newer agents, its use is reserved for patients who do not respond to other therapies. Topical steroid nasal decongestants may be used for symptom relief for nasal congestion but are not a regular part of asthma therapy in children. 33. The nurse has just admitted a patient with asthma and the emergency department doctor has ordered the patient to begin taking zafirlukast (Accolate). The nurse would hold the medication and contact the physician if the patient reported taking which medication at home? (Select all that apply.) A) Propranolol B) Warfarin C) Acetaminophen D) Ampicillin E) Terfenadine Ans: A, B, E Feedback: Use zafirlukast with caution if propranolol, theophylline, terfenadine, or warfarin is taken at the same time because increased toxicity can occur. Toxicity may also occur if these drugs are combined with calcium channel blockers, cyclosporine, or aspirin. No reported change of toxicity occurs when the patient is currently taking acetaminophen or ampicillin. 34. A patient with asthma is going to begin taking an inhaled steroid. The nurse teaching the patient that what adverse effects may occur when using this drug? (Select all that apply.) A) Headache B) Rebound congestion C) Sepsis D) Epistaxis E) Depression Ans: A, B, D Feedback: Adverse effects associated with the use of inhaled steroids include irritability, not depression, headache, rebound congestion, local infection, not sepsis and epistaxis. 35. The nursing instructor is discussing the use of sympathomimetics in patients who have acute bronchospasm. The instructor shares with the students that this classification of drugs is contraindicated or only used with great caution in patients with what disorders? (Select all that apply.) A) Hypothyroidism B) Cardiac disease C) Kidney disease D) Diabetes mellitus E) Peripheral vascular disease Ans: B, D, E Feedback: Before administering a sympathomimetic the nurse should assess for possible contraindications or cautions that include any known allergies to any drug in this class, cigarette use, cardiac disease, vascular disease, arrhythmias, diabetes, and hyperthyroidism. Sympathomimetics have no known adverse effects in hypothyroidism or kidney disease. Chapter 19. Gastrointestinal System Medications 1. What action does the histamine-2 antagonist administered by the nurse have on the human body that will help to prevent peptic ulcer disease? A) Destroys Helicobacter pylori B) Coats and protects the stomach lining C) Increases the pH of the secreted hydrochloric acid D) Reduces the amount of hydrochloric acid secreted Ans: D Feedback: Histamine-2 antagonists are administered to reduce the amount of hydrochloric acid secreted in the stomach, which helps to prevent peptic ulcer disease. H2 antagonists do not act as an antibiotic to kill bacteria (i.e., H. pylori) coat and protect the stomach lining like sucralfate (Carafate), or increase the pH of the secreted hydrochloric acid. 2. What classification of drugs does the nurse administer to treat peptic ulcers by suppressing the secretion of hydrochloric acid into the lumen of the stomach? A) Antipeptic agents B) Histamine-2 antagonists C) Proton pump inhibitors D) Prostaglandins Ans: C Feedback: Proton pump inhibitors suppress the secretion of hydrochloric acid into the lumen of the stomach. Antipeptic agents coat any injured area in the stomach to prevent further injury. H2 antagonists block the release of hydrochloric acid in response to gastrin. Prostaglandins inhibit secretion of gastrin and increase secretion of the mucous lining of the stomach. 3. The nurse is caring for a patient requiring digestive enzyme replacement therapy and establishes what appropriate nursing diagnosis for this patient? A) Acute pain B) Risk for constipation C) Risk for imbalanced nutrition D) Bowel incontinence Ans: C Feedback: The nurse would be concerned about the patients nutritional status because lack of digestive enzymes results in malnutrition. Replacement digestive enzymes help the patient improve digestion and absorption of essential nutrients. Effectiveness of the therapy is determined by the patients ability to maintain balanced nutrition. The other three options are not applicable to the use of replacement digestive enzymes. 4. What nursing interventions are included in the plan of care for a patient receiving antacids to relieve GI discomfort? A) Administer this drug with other drugs or food. B) Administer the antacid 1 hour before or 2 hours after other oral medications. C) Limit fluid intake to decrease dilution of the medication in the stomach. D) Have the patient swallow the antacid whole and do not crush or chew the tablet. Ans: B Feedback: A patient taking antacids should be advised to take the antacid 1 hour before or 2 hours after other oral medications. These tablets are often chewed to increase effectiveness. Limiting fluid intake can result in rebound fluid retention so that patients should be encouraged to maintain hydration. It is not necessary to take an antacid with other drugs, nor with food. 5. An adult patient is prescribed cimetidine (Tagamet). A nurse will instruct the patient that an appropriate dosage and frequency of cimetidine is what? A) 20 mg PO b.i.d B) 150 mg PO b.i.d C) 300 mg PO at bedtime D) 800 mg PO at bedtime Ans: D Feedback: An appropriate dosage and frequency for cimetidine is 800 mg PO at bedtime. Also, 300 mg can be taken q.i.d at meals and at bedtime. Ranitidine is taken 150 mg daily or b.i.d. Famotidine is taken 20 mg PO b.i.d. Nizatidine can be taken 150 to 300 mg PO at bedtime. 6. The nurse develops a discharge teaching plan for a patient who was prescribed pancreatic enzyme replacement and includes what important teaching point? A) Take the enzymes on an empty stomach. B) Crush the capsules and take with food. C) Avoid spilling the powder on the skin because it may be irritating. D) Pancreatin and pancrelipase are interchangeable. Ans: C Feedback: Patients receiving pancreatic enzymes should be taught to avoid spilling the powder on the skin because it can be very irritating. The enzymes should be taken with food and are often in a powder form. Pancreatin and pancrelipase are not interchangeable. 7. A patient comes to the clinic complaining of acid indigestion and tells the nurse he is tired of buying over-the-counter (OTC) antacids and wants a prescription drug to cure the problem. What would the nurse specifically assess for? A) Alkalosis B) Hypocalcemia C) Hypercholesterolemia D) Rebound tenderness at McBurneys point Ans: A Feedback: Prolonged or excessive use of OTC antacids can lead to the development of metabolic alkalosis. Many antacids contain calcium so that low calcium levels would be unlikely. Because metabolic alkalosis is a concern, metabolic acidosis is unlikely. High cholesterol levels are not associated with OTC antacid use. Rebound tenderness at McBurneys point is related to appendicitis and not antacid use. 8. A patient with a duodenal ulcer is receiving sucralfate for short-term treatment. What will the nurse advise the patient to avoid? A) Milk of Magnesia B) Tums C) Aluminum salts D) Proton pump inhibitors Ans: C Feedback: If aluminum salts (AlternaGEL) are taken concurrently with sucralfate, a risk of high aluminum levels and related aluminum toxicity exists. The combination of sucralfate and aluminum salts should be avoided or used with extreme caution. Adverse reactions with sucralfate are not associated with Milk of magnesia, Tums, or proton pump inhibitors. 9. A patient has been prescribed esomeprazole (Nexium). What statement by the patient does the nurse evaluate as indicating that he or she has a good understanding of his newly prescribed drug? A) I should open the capsules and crush the drug into applesauce. B) It is important that I take the drug after each meal. C) I need to swallow the drug whole and not chew the capsules. D) I should always take the drug with an antacid. Ans: C Feedback: Esomeprazole must be swallowed whole, not cut, crushed, or chewed, which would interfere with its effectiveness. The drug should not be taken with an antacid, which could interfere with absorption. The drug is taken once a day, not with each meal. 10. Which of these patients would the nurse expect to be the best candidate for misoprostol (Cytotec)? A) A 12-year-old with obsessive-compulsive disorder B) A 22-year-old pregnant patient C) A 46-year-old trial lawyer with hypertension D) An 83-year-old man with rheumatoid arthritis Ans: D Feedback: The 83-year-old man with rheumatoid arthritis is most likely to be taking nonsteroidal anti-inflammatory drugs (NSAIDs). Misoprostol is indicated for prevention of NSAID induced ulcers in adults at high risk for development of gastric ulcers. The other three patients would not be candidates for this drug. 11. When comparing the histamine-2 antagonists to each other the nurse recognizes that cimetidine (Tagamet) is more likely to cause which adverse effect? A) Dizziness B) Headache C) Gynecomastia D) Somnolence Ans: C Feedback: Cimetidine was the first drug in this class to be developed. It has been associated with antiandrogenic effects, including gynecomastia and galactorrhea. Dizziness, headache, and somnolence are associated with all histamine-2 antagonists. 12. What H2 antagonist would the nurse consider the drug of choice for a patient with advanced liver failure? A) Cimetidine B) Nizatidine C) Ranitidine D) Famotidine Ans: B Feedback: Nizatidine differs from the other three drugs in that it is eliminated by the kidneys, with no first-pass metabolism in the liver. It is the drug of choice for patients with liver disease or dysfunction. 13. When caring for a patient diagnosed with a peptic ulcer, the nurse administers omeprazole (Prilosec) along with what antibiotic to eradicate Helicobacter pylori? A) Gentamicin B) Ketoconazole C) Tetracycline D) Amoxicillin Ans: D Feedback: Gastric acid pump or proton pump inhibitors are recommended for the short- term treatment of active duodenal ulcers, gastroesophageal reflux disease, erosive esophagitis, and benign active gastric ulcer; for the long-term treatment of pathologic hypersecretory conditions; as maintenance therapy for healing of erosive esophagitis and ulcers; and in combination with amoxicillin and clarithromycin for the treatment of H. pylori infection. The other options are not antibiotics used to eradicate H. pylori. 14. What priority teaching point does the nurse include when instructing patients about the use of antacids? A) Reduce calorie intake to reduce acid production. B) Take before each meal and before bed. C) Be aware of risk of acid rebound with long-term use. D) Consider liquid diet if diarrhea occurs. Ans: C Feedback: Repeated use of antacids can result in rebound acid production because more gastrin is produced when pH of acid level decreases. Patients should be taught that long-term use of antacids requires follow-up care. Calorie and fluid intake does not need to be reduced because it is important to maintain nutrition, especially if diarrhea occurs. Antacids are taken at least 1 hour before or 2 hours after any other drug or meal. 15. For treatment of a gastric ulcer, what would the recommended dosing schedule of famotidine (Pepcid) be? A) 10 mg b.i.d B) 20 mg b.i.d C) 60 mg at bedtime D) 40 mg q am Ans: B Feedback: Famotidine should be administered 40 mg every day at bedtime or 20 mg b.i.d for treatment of a gastric ulcer. Options A, C, and D are not correct. 16. The 59-year-old patient has peptic ulcer disease and is started on sucralfate (Carafate). What is an appropriate nursing diagnosis related to this medication? A) Risk for constipation related to GI effects B) Risk for injury: bleeding C) Imbalanced nutrition related to nausea D) Deficient fluid volume Ans: A Feedback: The adverse effects associated with sucralfate are primarily related to its GI effects. Constipation is the most frequently seen adverse effect. Imbalanced nutrition, if seen, would be related to diarrhea or constipation and not nausea. Fluid volume deficit and bleeding are not common adverse effects of this drug. 17. The nurse is preparing a patient for discharge with a prescription for sucralfate (Carafate) and teaches the patient to take the medication when? A) With meals B) With an antacid before breakfast C) 1 hour before or 2 hours after meals and at bedtime D) After each meal Ans: C Feedback: Administer drug on an empty stomach, 1 hour before or 2 hours after meals and at bedtime, to ensure therapeutic effectiveness of the drug. Administer antacids, if ordered, between doses of sucralfate and not within 30 minutes of taking the drug. Options A, B, and D are not correct. 18. The patient will receive ranitidine (Zantac) 150 mg PO at bedtime. Prior to administration, the nurse will inform the patient that common adverse effects related to this medication include what? A) Tremors B) Headache C) Visual disturbances D) Anxiety Ans: B Feedback: Headache, dizziness, somnolence, and mental confusion may occur with H2 antagonists. Visual disturbances, tremors, and anxiety are not normally associated with ranitidine. 19. The nurse administers ranitidine (Zantac) cautiously to patients with evidence of what conditions? A) Renal disease B) Diabetes mellitus C) Pulmonary disease D) Migraine headaches Ans: A Feedback: All histamine-2 antagonists are eliminated through the kidneys; dosages need to be reduced in patients with renal impairment. No caution is necessary with Zantac therapy in people with diabetes, pulmonary disease, or migraine headaches. 20. The nurse would question an order for misoprostol if the patient was diagnosed with what condition? A) Diabetes B) Hypertension C) Arthritis D) Pregnancy Ans: D Feedback: This drug is contraindicated during pregnancy because it is an abortifacient. The other options are not correct. 21. The nurse would assess patients with what condition to determine whether there is a need for a saliva substitute? (Select all that apply.) A) Cystic fibrosis B) Stroke C) Cancer receiving chemotherapy or radiation therapy D) Myocardial infarction E) Chronic dry mouth Ans: B, C, E Feedback: A saliva substitute helps in conditions that result in dry mouthstroke, radiation therapy, chemotherapy, and other illnesses. Patients with myocardial infarction do not require this drug. Patients with cystic fibrosis receive pancreatic enzymes rather than saliva substitute. 22. After providing teaching to a patient prescribed lansoprazole (Prevacid), the nurse evaluates the patient understands the action of this medication when the patient makes what statement? A) The medication inhibits acid secretions. B) The medication is an antibiotic. C) The medication is an analgesic. D) The medication will promote healing of my ulcer. Ans: A Feedback: The gastric acid pump or proton pump inhibitors suppress gastric acid secretion by specifically inhibiting the hydrogenpotassium adenosine triphosphatase (H+,K+-ATPase) enzyme system on the secretory surface of the gastric parietal cells. The statement, The medication inhibits acid secretions, indicates that the patient understands that the medication inhibits acid secretion. This medication does not act as an antibiotic or analgesic, nor will it repair the ulcer. 23. What would the nurse consider an indication for the use of antacids? (Select all that apply.) A) Gastric hyperacidity B) Gastritis C) Peptic esophagitis D) Hiatal hernia E) Duodenal ulcer Ans: A, B, C, D Feedback: Antacids neutralize stomach acid by direct chemical reaction. They are recommended for the symptomatic relief of upset stomach associated with hyperacidity, as well as the hyperacidity associated with peptic ulcer, gastritis, peptic esophagitis, gastric hyperacidity, and hiatal hernia. Duodenal ulcer is not an indication for the use of an antacid. 24. A patient presents to the walk-in clinic complaining of vomiting and burning in his or her mid-epigastria. The nurse suspects peptic ulcer disease and knows that to confirm peptic ulcer disease, the physician is likely to order a diagnostic test to detect the possible presence of what? A) Infection with Helicobacter pylori B) Decreased stomach acid secretion C) Gastric irritation caused by nonsteroidal anti-inflammatory drugs (NSAIDs) D) Overconsumption of spicy foods Ans: A Feedback: Peptic ulcers may result from increased acid production, decrease in the protective mucous lining of the stomach, infection with Helicobacter pylori bacteria, or a combination of these. Peptic ulcers do not result from decreased acid secretion or overconsumption of spicy foods. While gastric irritation can be caused by NSAIDs gastric irritation can result from many different causes so this would not be specifically assessed for unless the patient was found to use NSAIDs frequently, which is not indicated by the question. 25. The nurse is providing education for a patient with peptic ulcer disease resulting from chronic nonsteroidal anti-inflammatory drug (NSAID) use who will begin a prescription of misoprostol (Cytotec). The nurse evaluates that the patient understands the actions of this drug when he or she says it does what? A) Reduces the stomachs volume of hydrochloric acid B) Increases the speed of gastric emptying C) Protects the stomachs lining D) Increases lower esophageal sphincter pressure Ans: C Feedback: Misoprostol is a synthetic prostaglandin that, like physiologic prostaglandin, protects the gastric mucosa. NSAIDs decrease prostaglandin production and predispose the patient to peptic ulceration. Misoprostol does not reduce gastric acidity (option A), improve emptying of the stomach (option B), or increase lower esophageal sphincter pressure (option D). 26. What drug combination will the nurse normally administer most often to treat a gastric ulcer? A) Antibiotics and histamine-2 antagonists B) H2 antagonists, antibiotics, and bicarbonate salts C) Bicarbonate salts, antibiotics, and ZES D) Antibiotics and proton pump inhibitors Ans: D Feedback: Currently, the most commonly used therapy for gastric ulcers is a combination of antibiotics and proton pump inhibitors that suppress or eradicate Helicobacter pylori. H2 receptor antagonists are used to treat duodenal ulcers. Bicarbonate salts are not used. ZES is the abbreviation for ZollingerEllison syndrome and not a drug. 27. The nurse questions an order for a proton pump inhibitor when the patient is known to take what other medication? A) Theophylline B) Penicillin C) Digoxin D) Heparin Ans: A Feedback: Decreased levels of ketoconazole and theophylline have been reported when combined with these drugs, leading to loss of effectiveness. There are no drugdrug interactions with penicillin, digoxin, or heparin. 28. What histamine-2 antagonist might the nurse administer to a child? A) Cimetidine B) Famotidine C) Nizatidine D) Ranitidine Ans: B Feedback: Famotidine is the only histamine2 antagonist approved for use in children if a histamine-2 antagonist is needed. Therefore, other options are not correct. 29. The nurse is caring for a 27-year-old female patient who has just been prescribed misoprostol. What is a priority teaching point for this patient? A) You will need to use a barrier-type contraceptive B) Do not take NSAIDs with this drug C) Adverse effects include nausea and diarrhea D) It protects the lining of the stomach Ans: A Feedback: Misoprostol is contraindicated during pregnancy because it is an abortifacient. Women of childbearing age who use misoprostol should be advised to use barrier-type contraceptives. All other options are correct but are not a priority for this patient. 30. The clinic nurse is caring for a 78-year-old male patient who is taking a proton pump inhibitor. What condition is this patient at increased risk for developing? A) Sickle cell anemia B) Megaloblastic anemia C) Pernicious anemia D) Iron deficiency anemia Ans: C Feedback: The use of proton pump inhibitors and H2 blockers in older adults has been associated with decreased absorption of vitamin B12 and the development of pernicious anemia. The use of proton pump inhibitors does not increase the risk of sickle cell, megaloblastic, or iron deficiency anemias. 31. The nurse admits a patient who reports having taken a proton pump inhibitor for more than a decade. What assessment question will the nurse ask this patient? A) Are you experiencing diarrhea? B) Do you experience much abdominal pain? C) Have you noticed any blood in your urine? D) When is the last time you checked your blood pressure? Ans: A Feedback: Research indicates that patients taking proton pump inhibitors or histamine-2 antagonists demonstrate a significant increase in cases of Clostridium difficile infections leading to diarrhea so the nurse should assess this patient for diarrhea. There would be no indications to ask about abdominal pain, blood in the urine, or blood pressure. 32. What symptoms would the nurse find to be consistent with a diagnosis of a peptic ulcer? A) Pain a few hours after eating B) Pain described as sharp and stabbing C) Explosive diarrhea within 1 hour of eating D) Unable to stand comfortably due to abdominal pain Ans: A Feedback: Ulcer patients present with a predictable description of gnawing, burning pain often occurring a few hours after meals. The pain is not described as sharp and stabbing. Diarrhea is not usually associated with peptic ulcer disease and patients do not find it difficult to stand upright. 33. The nurse is caring for a patient with cystic fibrosis who has recently not been taking her pancreatic enzymes recently. What is the primary assessment finding the nurse will see as a result of this noncompliance? A) Constipation B) Diarrhea C) Malnutrition D) Abdominal pain Ans: C Feedback: Patients with cystic fibrosis take pancreatic enzymes to promote digestion and absorption of nutrients so failure to take the drug would result in malnutrition and weight loss. Constipation, diarrhea, and abdominal pain would not be in the primary assessment finding. 34. The nurse, caring for a patient with cystic fibrosis, administers pancreatic enzymes on what schedule? A) 1 hour after meals and snacks B) With meals and snacks C) 2 hours before meals and snacks D) In the morning and before bed Ans: B Feedback: To be sure the pancreatic enzymes are in place when food enters the small bowel the enzymes are administered with every meal and snack. Administering them before or after meals would render them ineffective. 35. The patient prescribed nizatidine (Axid) chooses to take the medication once a day at bedtime. What dosage will the nurse administer? A) 150 to 300 mg B) 100 to 125 mg C) 20 to 40 mg D) 5 to 7.5 mg Ans: A Feedback: The dosage for nizatidine, when taken PO only at bedtime, is 150 to 300 mg. Options B, C, and D are not correct. Part 2: 1. The nurse teaches the patient to best maintain optimal GI function by including what in the daily routine? A) Exercise, adequate sleep, and avoiding caffeine B) Proper diet, fluid intake, and exercise C) Proper diet, avoiding alcohol, and cautious use of laxatives D) Avoiding prescription medications, increased fluid intake, and vigorous exercise Ans: B Feedback: The best way to maintain gastrointestinal (GI) function is through proper diet including optimizing fiber intake, adequate fluid intake, and exercise, which will stimulate GI activity. Drugs should only be used when normal function cannot be maintained. Options A, C, and D are not correct. 2. The nurse admits to the clinic a 7-month-old infant whose mother reports that the baby has not had a bowel movement in 6 days. What drug would be appropriate to treat this patient? A) Cascara (generic) B) Magnesium hydroxide (Milk of Magnesia) C) Polyethylene glycol-electrolyte solution (GoLYTELY) D) Glycerin (Sani-Supp) Ans: D Feedback: Glycerin suppositories are the best choice for infants and young children when constipation is a problem. The other choices are not recommended for infants or children. 3. The nurse provides drug teaching to the patient who will begin taking polycarbophil (FiberCon). What is the nurses priority teaching point? A) Do not eat or drink anything for 2 hours after taking the medication. B) Drink lots of water when taking the drug. C) Take at night before bedtime. D) The drug can be taken up to six times per day. Ans: B Feedback: Patients must take plenty of water with polycarbophil. If only a little water is consumed, the medication may absorb enough fluid in the esophagus to swell the food into a gelatin-like mass that can cause obstruction and other severe problems. The drug should be taken no more than four times a day and should not be taken at night. 4. A new mother required an episiotomy during the birth of her baby. Two days after delivery, the patient is in need of a laxative. What will be the most effective drug for the nurse to administer? A) Bisacodyl (Dulcolax) B) Castor oil (Neolid) C) Magnesium sulfate (epsom salts) D) Docusate (Colace) Ans: D Feedback: Docusate is a stool softener that will make expulsion of stool easier in a traumatized body area following an episiotomy. Care must be taken to choose a mild laxative that will not enter breast milk and not affect the newborn if the mother is nursing. Docusate is the drug of choice from this list because it is mild and will produce a soft stool and decrease the need to strain. The other options would not be appropriate because they do not soften the stool and are harsher laxatives that can enter breast milk. 5. The nurse, teaching a patient to be discharged with an order to use chemical stimulant laxatives PRN, instructs the patient that one of the most common adverse effects of this type of laxative is what? A) Abdominal cramping B) Rectal bleeding C) Confusion D) Iron deficiency anemia Ans: A Feedback: Common adverse effects of laxatives are diarrhea, abdominal cramping, and nausea. Central nervous system (CNS) adverse effects such as dizziness, headache, and weakness can occur. Rectal bleeding, confusion, and iron deficiency anemia are not associated with appropriate use of laxatives but may occur when laxatives are abused. 6. A patient taking a chemical stimulant laxative and medications for heart failure and osteoarthritis calls the clinic and reports to the nurse that she is not feeling right. What is the priority question the nurse should ask this patient? A) Effectiveness of laxatives B) Timing of medication administration C) The amount of fiber in her diet D) Amount of fluid ingested Ans: B Feedback: Because laxatives increase the motility of the gastrointestinal (GI) tract and some laxatives interfere with the timing or process of absorption, it is not advisable to take laxatives with other prescribed medications. The administration of laxatives and other medications should be separated by at least 30 minutes, so the nurse should question when the patient is taking the laxatives and other medication. Other options may be questions the nurse would eventually ask, but the priority is timing of medication administration. 7. A patient who is taking metoclopramide (Reglan) has come to the clinic for a follow-up visit. The nurse will be most concerned about a drug-drug interaction when learning the patient is taking what other medication? A) Albuterol (Proventil) B) Digoxin (Lanoxin) C) Furosemide (Lasix) D) Acetylsalicylic acid (ASA) Ans: B Feedback: Metoclopramide has been associated with decreased absorption of digoxin from the gastrointestinal (GI) tract. The nurse should monitor patients taking this combination carefully. The other options do not pose a concern about drugdrug interactions. 8. What should the nurse tell the patient who will begin taking rifaximin (Xifaxan) for travelers diarrhea? A) Do not start taking the drug until the diarrhea has persisted for at least 24 hours. B) Taking the antibiotic will allow you to enjoy the local water and food without concern. C) Do not take the drug if you have bloody diarrhea. D) Start the drug 3 days before leaving for your trip and continue until you return. Ans: C Feedback: Rifaximin should not be taken if a patient has bloody diarrhea or diarrhea that persists for more than 48 hours. It is started after the signs and symptoms of the disease develop and is taken for 3 days. The traveler should still avoid exposure to foreign bacteria by not drinking tap water and by avoiding fruits and vegetables washed in tap water. Options A, B, and D are not correct. 9. A clinic patient with a history of heart failure requires a laxative for treatment of chronic constipation. What medication would be most appropriate for this patient? A) Senna (Senokot) B) Lactulose (Chronulac) C) Magnesium sulfate (Milk of Magnesia) D) Castor oil (Neoloid) Ans: B Feedback: Lactulose is often the drug of choice when a patient with cardiovascular problems requires a laxative. It is salt free, an important consideration in patients with heart failure; it acts by exerting a gentle osmotic pull of fluid into the intestinal lumen. Senna, magnesium sulfate, and castor oil are more aggressive laxatives and might not be the best choice for a patient with cardiovascular problems. 10. A man with irritable bowel syndrome reports ongoing diarrhea and asks for a prescription for alosetron (Lotronex), which was helpful for his coworker who recently started taking the drug. What is the nurses best response? A) This drug is only approved for use in women. B) This drug is used as a laxative. C) This drug is contraindicated with irritable bowel syndrome. D) This drug is no longer on the market for prescription use. Ans: A Feedback: Alosetron (Lotronex) is approved for use in women with irritable bowel syndrome with diarrhea being the predominant complaint and should be discontinued immediately if the patient develops constipation or symptoms of ischemic colitis. Patients must read and sign a patientphysician agreement before it can be prescribed. Options B, C, and D are not correct. 11. The nurse administers psyllium hydrophilic mucilloid (Metamucil) expecting it to have what action? A) Slows peristalsis B) Promotes reabsorption of water into bowel C) Has antibacterial properties D) Adds bulk to the stool Ans: D Feedback: Metamucil is a natural substance that forms a gelatin-like bulk of the intestinal contents. This agent stimulates local activity. It is considered milder and less irritating than many other bulk stimulants. Patients must use caution and take it with plenty of water because Metamucil absorbs large amounts of water and produces stools of gelatin-like consistency. It does not slow peristalsis, promote water reabsorption, or have antibacterial properties. 12. The nurse administers loperamide (Imodium) to decrease the number and liquidity of stool by what mechanism? A) Decreasing intestinal motility B) Absorbing toxins C) Binding with fecal material to increase bulk D) Blocking the chemoreceptor trigger zone (CTZ) Ans: A Feedback: Systemic antidiarrheal agents slow the motility of the gastrointestinal (GI) tract through direct action on the lining of the GI tract to inhibit local reflexes (bismuth subsalicylate), through direct action on the muscles of the GI tract to slow activity (loperamide), or through action on central nervous system (CNS) centers that cause GI spasm and slowing (opium derivatives). Options B, C, and D are not correct. 13. The nurse teaches the patient that a common adverse effect of loperamide (Imodium) is what? A) Fatigue B) Flatulence C) Disorientation D) Tremors Ans: A Feedback: Adverse effects associated with antidiarrheal drugs, such as constipation, distention, abdominal discomfort, nausea, vomiting, dry mouth, and even toxic megacolon, are related to their effects on the gastrointestinal (GI) tract. Other adverse effects that have been reported include fatigue (option A), weakness, dizziness, and rash. options B, C, and D are not correct. 14. The nurse will question an order for bismuth salts (Pepto-Bismol) in a patient with what condition? A) Rheumatoid arthritis B) Allergy to aspirin C) Hypertension D) Viral gastroenteritis Ans: B Feedback: Pepto-Bismol has aspirin in it and should not be given to a patient with an allergy to aspirin. There is no contraindication for a patient with rheumatoid arthritis, hypertension, or viral gastroenteritis. 15. The home health nurse is caring for a 72-year-old man in his home. He complains about almost daily diarrhea. The nurse assesses for what common cause of diarrhea in older adults? A) Diabetes B) Hypertensive medications C) Laxative overuse D) Glaucoma Ans: C Feedback: Diarrhea in older adults may result from laxative overuse so the nurse should assess what over-the-counter (OTC) products the patient is using. Diabetes, hypertension medications, and glaucoma are not common causes of diarrhea in patients. 16. What is the antidiarrheal of choice the nurse will administer to children older than 2 years of age with diarrhea? A) Bismuth salts (Pepto-Bismol) B) Loperamide (Imodium) C) Paregoric (generic) D) Difenoxin (Motofen) Ans: B Feedback: Loperamide may be the antidiarrheal of choice in children older than 2 years of age if such a drug is needed. Special precautions need to be taken to monitor for electrolyte and fluid disturbances and supportive measures should be taken as needed. Serious fluid volume deficits may rapidly develop in children with diarrhea. Appropriate fluid replacement should include oral rehydration solutions. Although bismuth salts and paregoric may be given to children, they are not the drugs of choice. Difenoxin is not for use in children under 12 years of age. 17. What antidiarrheal would the nurse administer to reduce the volume of discharge from the patients ileostomy? A) Diphenoxylate (Lomotil) B) Octreotide (Sandostatin) C) Psyllium (Metamucil) D) Loperamide (Imodium) Ans: D Feedback: Loperamide (Imodium), bismuth subsalicylate (Pepto-Bismol), and opium derivatives (paregoric) are indicated for the reduction of volume of discharge from ileostomies. The other options would not serve this purpose. 18. The nurse teaches the patient who was prescribed a chemical stimulant laxative that this medication may have either a slow, steady effect or may cause severe cramping and rapid evacuation of the contents of the large intestine. What drug is the nurse teaching this patient about? A) Bismuth salts B) Bisacodyl C) Castor oil D) Cascara Ans: D Feedback: Specifically related to chemical stimulants, cascara, although a reliable agent, may have a either slow, steady action or may cause severe cramping and rapid evacuation of the contents of the large intestine. Bisacodyl and castor oil do not have potential adverse effect. Bismuth salts are an antidiarrheal. 19. What is the priority nursing diagnosis for an 89-year-old patient with frequent liquid stools? A) Imbalanced nutrition: less than body requirements B) Deficient fluid volume C) Deficient knowledge related to medications D) Pain related to abdominal cramping Ans: B Feedback: Fluid volume deficits may develop rapidly in older adults with diarrhea. Older adults are more likely to develop adverse effects associated with the use of these drugs, including sedation, confusion, dizziness, electrolyte disturbances, fluid imbalance, and cardiovascular effects. Although nutrition, pain, and knowledge deficit may be concerns for this patient, they are not the priority concern. 20. What drug does the nurse recognize as being classified as a chemical stimulant? A) Bisacodyl (Dulcolax) B) Polycarbophil (FiberCon) C) Magnesium hydroxide (Milk of Magnesia) D) Docusate (Colace) Ans: A Feedback: Bisacodyl is a chemical stimulant. Polycarbophil and magnesium hydroxide are bulk laxatives. Docusate is a lubricant laxative. 21. The nurse is caring for a patient who has had impacted stools twice in the past month. What is an appropriate laxative for this patient? A) Milk of Magnesia B) Agoral Plain C) Colace D) Dulcolax Ans: B Feedback: Mineral oil (Agoral Plain) is not absorbed and forms a slippery coat on the contents of the intestinal tract. When the intestinal bolus is coated with mineral oil, less water is drawn out of the bolus and the bolus is less likely to become hard or impacted. Other options shown do not have this same effect of reducing the risk of another impaction as well as helping to eliminate stool. 22. The home health nurse is caring for a patient with encopresis who was started on mineral oil therapy. The nurse teaches the patient and family that a common adverse effect is what? A) Nausea B) Vomiting C) Leakage D) Vitamin C deficiency Ans: C Feedback: Leakage and staining may be a problem when mineral oil is used and the stool cannot be retained by the external sphincter. Mineral oil does not cause nausea, vomiting, or vitamin C deficiency. 23. When would it be appropriate for the nurse to administer a cathartic laxative to the patient? (Select all that apply.) A) Partial small-bowel obstruction B) Appendicitis C) After having a baby D) After a myocardial infarction (MI) E) After anthelmintic therapy Ans: C, D, E Feedback: Laxative, or cathartic, drugs are indicated for the short-term relief of constipation; to prevent straining when it is clinically undesirable (such as after surgery, myocardial infarction, or obstetric delivery); to evacuate the bowel for diagnostic procedures; to remove ingested poisons from the lower gastrointestinal (GI) tract; and as an adjunct in anthelmintic therapy when it is desirable to flush helminths from the GI tract. They are not indicated when a patient has an appendicitis or a partial small-bowel obstruction. 24. When would it be appropriate for the nurse to administer castor oil as a laxative? A) To ease the passage of stool in the patient who recently had a baby B) To remove ingested poisons from the lower gastrointestinal (GI) tract C) To evacuate the bowel for diagnostic procedures D) To treat chronic constipation Ans: C Feedback: Indications include evacuating the bowel for diagnostic procedures and for short-term treatment of constipation. Castor oil is not indicated to remove ingested poisons nor to ease the passage of stool after having a baby. This drug should only be used on a short-term basis and is not for the treatment of chronic constipation because repeated use will cause GI tract exhaustion. 25. For what action would the nurse administer magnesium citrate? A) Block absorption of fats B) Directly stimulate the nerve plexus in the intestinal wall C) Form a slippery coat on the contents of intestine D) Increase motility, increase fluid, and enlarge bulk of fecal matter Ans: D Feedback: Magnesium citrate is a rapid-acting, aggressive laxative that causes fecal matter to increase in bulk. It increases the motility of the gastrointestinal (GI) tract by increasing the fluid in the intestinal contents, which enlarges bulk, stimulates local stretch receptors, and activates local activity. It does not block absorption of fats, stimulate the nerve plexus, or form a slippery coat. 26. The nurse administers metoclopramide to the patient with what condition? A) Chronic diabetic gastroparesis B) Impaction C) Encopresis D) Patients requiring diagnostic procedures Ans: A Feedback: Indications for metoclopramide include relief of acute and chronic diabetic gastroparesis, short-term treatment of gastroesophageal reflux disorder in adults who cannot tolerate standard therapy, prevention of postoperative or chemotherapy-induced nausea and vomiting, facilitation of small-bowel intubation, stimulation of gastric emptying, and promotion of intestinal transit of barium. It would not be used for treatment of impaction, encopresis, and in patients requiring diagnostic procedures. 27. What drug does the nurse administer that inhibits intestinal peristalsis through direct effects on the longitudinal and circular muscles of the intestinal wall? A) Bismuth subsalicylate B) Loperamide C) Paregoric D) Magnesium citrate Ans: B Feedback: Actions of loperamide include that it inhibits intestinal peristalsis through direct effects on the longitudinal and circular muscles of the intestinal wall, slowing motility and movement of water and electrolytes. Bismuth subsalicylate inhibits local reflexes. Paregoric works through action on CNS centers that cause GI spasm and slowing. Magnesium citrate is a laxative. 28. The family brings a patient to the emergency department saying he has been hallucinating and falls so deeply asleep he stops breathing when not stimulated. The nurse learns the patient has been self-treating diarrhea and suspects the patient was taking what medication? A) Paregoric B) Bismuth subsalicylate C) Loperamide D) Colace Ans: A Feedback: Opium derivatives, like paregoric, are associated with light-headedness, sedation, euphoria, hallucinations, and respiratory depression related to their effect on opioid receptors. Nonopioids such as bismuth subsalicylate and loperamide would not cause respiratory depression. Colace is a stool softener, not an antidiarrheal. 29. The nurse develops a teaching plan for a 77-year-old patient who has been prescribed loperamide PRN. The nurses priority teaching point is what? A) May cause hallucinations or respiratory depression B) Take drug after each loose stool C) Drug remains in the bowel without being absorbed into the bloodstream D) Avoid pregnancy and breast-feeding while taking drug. Ans: B Feedback: Loperamide is taken repeatedly after each loose stool. Teaching the patient when to take the drug is the priority teaching point. Paregoric, and not loperamide, can cause hallucinations and respiratory depression. The drug is absorbed systemically. It is unlikely a 77-year-old patient will get pregnant or breast-feed so this is not the highest priority. 30. A patient receiving loperamide (Imodium) should be alerted by the nurse to what possible adverse effect? A) Anxiety B) Bradycardia C) Fatigue D) Urinary retention Ans: C Feedback: Patients should be aware that they should not drive or operate machinery while taking loperamide (Imodium) because it can cause fatigue. Anxiety, bradycardia, and urinary retention are not commonly associated with loperamide. 31. The patient had surgery 2 days ago and bowel motility has not returned. What drug might the nurse administer to stimulate the gastrointestinal (GI) tract? A) Dexpanthenol B) Docusate C) Psyllium (Metamucil) D) Senna Ans: A Feedback: Dexpanthenol is indicated for the prevention of intestinal atony or loss of intestinal muscle tone in postoperative adults. Docusate, psyllium, and senna are laxatives that would not be indicated for the postoperative patient with no bowel activity. 32. The hospice nurse is caring for a patient diagnosed with bone cancer who is receiving large doses of opioid medications to relieve pain. The patient has used other laxatives in the past to treat opioid-induced constipation but nothing is working now. What drug would the nurse request the family doctor to order for this patient? A) Methylnaltrexone (Relistor) B) Castor oil C) Paregoric D) Mineral oil Ans: A Feedback: Methylnaltrexone (Relistor) was approved in 2008 for the treatment of opioid-induced constipation in patients with advanced disease who are receiving palliative care and are no longer responsive to traditional laxatives. Castor oil, mineral oil, and paregoric would likely be ineffective in this patient. 33. The nurse, providing patient teaching, explains that difenoxin and diphenoxylate are chemically related to what medication? A) Morphine B) Demerol C) Diphenhydramine D) Diflucan Ans: B Feedback: Difenoxin and diphenoxylate are chemically related to meperidine and are used at doses that decrease gastrointestinal activity without having analgesic or respiratory effects. 34. The nurse collects a stool culture from a patient diagnosed with travelers diarrhea. What bacterium does the nurse expect the culture to grow? A) Escherichia coli B) Staphylococcus aureus C) Streptococcus type B D) Pseudomonas Ans: A Feedback: Escherichia coli is the most common cause of travelers diarrhea. Staphylococcus, Streptococcus, and Pseudomonas would be highly unlikely to grow in the stool of a patient with travelers diarrhea. 35. The nurse administers lubiprostone (Amitiza) to the patient with irritable bowel syndrome and anticipates what therapeutic action from the drug? A) Secretion of chloride-rich intestinal fluid leading to increased motility B) Adding bulk to the fecal matter to ease the process of stooling C) Irritation of the inner lining of the bowel to increase bowel motility D) Stimulate the bowel by increasing innervation Ans: A Feedback: Lubiprostone is a locally acting chloride channel activator that increases the secretion of a chloride-rich intestinal fluid without changing sodium or potassium levels. Increasing the intestinal fluid leads to increased motility. It does not add bulk, irritate the inner lining, or innervate the bowel. Chapter 20. Reproductive and Urinary Systems Medications Female Reproductive: 1. The nurse is caring for a female patient who would like to start taking oral contraceptives. What assessment finding may indicate the patient is not a good candidate for these drugs? A) Decreased appetite B) Dehydration C) Occasional headaches D) History of deep vein thrombosis Ans: D Feedback: Estrogens are contraindicated in the presence of a history of thromboembolic disorders because of the increased risk of thrombus and embolus development. A loss of appetite would not contraindicate oral contraceptives but would require further assessment to determine the cause. Dehydration would require fluid administration to correct but is not a contraindication to oral contraceptives. Occasional headaches are not uncommon and would not contraindicate contraceptive use. 2. The nurse is caring for a patient with an intact uterus who requests hormone replacement for short-term use to reduce menopausal symptoms. What combination drug would be appropriate for this patient? A) Estrace B) Premphase C) Premarin D) Estratab Ans: B Feedback: The patient would be given Premphase because it is a combination of estrogen and progesterone. The combination is important to help avoid risk of endometrial hyperplasia. Estrace, Premarin, and Estratab contain only estrogen so they are not combination drugs. 3. The nurse is caring for a patient who just had subdermal implantation of NuvaRing. As part of drug teaching, the nurse will tell the patient this implant needs to be removed when? A) 3 months B) 6 months C) 1 year D) 3 years Ans: D Feedback: Etonogestrel, in addition to being available as a vaginal ring, NuvaRing, is available as a subdermal implant that may be left in place for up to 3 years and then must be removed. Another implant could be placed at that time. 4. A patient tells the nurse she is taking soy, calcium, and a multivitamin as an alternative to taking hormone replacement pills. What is the nurses priority response? A) Increase her iron supplement. B) Discontinue her calcium supplement. C) Decrease the amount of carbohydrates in her diet. D) Increase calcium supplementation. Ans: B Feedback: The nurse may advise the patient to stop using a calcium supplement because soy is not to be taken with calcium, iron, or zinc. However, the patient may have to decide whether it would be more beneficial for her to continue the calcium and discontinue the soy if osteoporosis is a concern. Decreasing carbohydrates is a healthy choice, especially for a menopausal woman, but would not be the priority concern. 5. A patient with a seizure disorder taking phenytoin (Dilantin) requests a prescription for an oral contraceptive. What is the nurses priority response? A) The effect of oral contraceptives containing progestin is reduced by phenytoin. B) The effect of oral contraceptives containing progestin and estrogen is reduced by phenytoin. C) The effect of oral contraceptives containing estrogen is reduced by phenytoin. D) You will need to increase the dosage of your phenytoin once you start contraceptives. Ans: B Feedback: The effectiveness of oral contraceptives containing estrogen, progestin, or both will be reduced by phenytoin, so contraceptives will not be adequate to prevent pregnancy. There is no reason to change the dosage of phenytoin. 6. A patient is using a progestin vaginal gel. What possible adverse effects should the nurse tell the patient about? A) Diarrhea B) Breast enlargement C) Abdominal pain D) Local skin irritation Ans: B Feedback: The use of a progestin vaginal gel is associated with breast enlargement. Constipation, not diarrhea, is also an adverse effect. Abdominal pain with progestin therapy is associated with the use of an intrauterine device for birth control, not a vaginal gel. Local skin irritation can result from use of a dermal patch contraceptive. 7. The nurse is preparing to administer an infusion of oxytocin (Pitocin) to the pregnant patient. What is the priority assessment before beginning the infusion? A) Cervical dilation B) Cephalopelvic proportions C) Electrocardiogram readings D) Respiratory excursion Ans: B Feedback: Pitocin is used to stimulate labor and often results in intense uterine contractions. It is important that the nurse assess cephalopelvic proportions because a disproportion between the size of the baby and the size of the fetus could result in serious complications. Dilation may be well underway when oxytocin is started or may need to be initiated so this is not a priority assessment, although it would certainly be assessed. Respiratory excursion is expected to be limited in pregnant women because of the enlarged uterus pushing up on the diaphragm. Electrocardiogram readings should not be needed with most pregnant women unless the woman has a preexisting condition. 8. A nurse is taking care of a woman receiving an abortifacient. The nurse is aware that the most serious adverse effect is what? A) Vomiting B) Nausea C) Uterine rupture D) Diarrhea Ans: C Feedback: All these options are adverse effects of abortifacients. However, the most serious adverse effect would be uterine rupture. A perforated uterus or uterine rupture can be life threatening and emergency measures must be taken. 9. A 13-year-old teenage girl is started on oral contraceptives. When following this girl in the clinic, what is the nurses priority assessment? A) Closure of the epiphyses B) Menstrual patterns C) Nutrition D) Cognitive development Ans: B Feedback: A 13-year-old girl is still growing. Estrogens and progestins have undergone limited testing in children. Because of their effects on closure of the epiphyses, they should be used only with great caution in growing children. It is important for the nurse to monitor metabolic and other effects as well. Menstrual patterns, nutrition, and cognitive development are all components of care of the adolescent, but they are not the priority consideration. 10. A patient is in the clinic for a follow-up visit after having been on hormone replacement therapy for 3 months. Which report by the patient would immediately concern the nurse? A) Smoking a pack of cigarettes a day B) Gaining 10 pounds in the last 3 months C) Craving sugar D) Spending less time exercising Ans: A Feedback: All these options are poor health habits and will impact the patients health. However, the immediate concern is smoking. The nurse should stress that women who take estrogen should not smoke because of the increased risk for thrombotic events. A weight gain of 10 pounds, a craving for sugar, and a decrease in exercise would not be as immediate a concern although the nurse should address these issues. 11. The patient asks the nurse, Why cant I smoke when taking estrogen. Is the risk for blood clots really that high? What is the nurses best response? A) The risk is truly that high because estrogen increases serum triglyceride, cholesterol, and glucose levels. B) The risk is high because estrogen stimulates skeletal growth, causing increased production of red blood cells (RBCs). C) The risk is pretty high because estrogen decreases blood levels of several clotting factors. D) There is documented high risk because estrogen combined with nicotine creates a significant drugdrug interaction. Ans: D Feedback: Smoking while taking estrogens should be strongly discouraged, because the combination of therapeutic estrogen with nicotine increases the risk for development of thrombi and emboli and this risk is well documented and researched. The risk is not due to skeletal growth, decreased clotting factors, or an impact on triglycerides, cholesterol, or glucose. 12. The nurse is caring for a 33-year-old mother of two who has a history of asthma and migraine headaches. The patient is on a low-residue diet for colitis. What factor in the patients history may contraindicate the use of birth control pills? A) Migraine headaches B) Age C) Asthma D) Colitis Ans: A Feedback: Progestins should be used with caution in patients with epilepsy, migraine headaches, asthma, or cardiac or renal dysfunction because of the potential exacerbation of these conditions. Age, asthma, and colitis would not be cautions or contraindications for the use of oral contraceptives. 13. The nurse is caring for a postmenopausal patient taking estradiol (Estrace) to reduce signs and symptoms of menopause. What other benefit will result from this medication? A) Reduced risk of endometriosis B) Reduced risk of dysfunctional uterine bleeding C) Reduced risk of osteoporosis D) Reduced risk of uterine cancer Ans: C Feedback: Estrogen slows the bone loss seen with osteoporosis so this will be an added benefit of the drug. Observe for improved bone density tests and absence of fractures. Endometriosis and dysfunctional uterine bleeding do not occur in postmenopausal women who no longer menstruate. Estrogen does not prevent uterine cancer and screening for cancer should be performed before prescribing this drug. 14. The nurse has been conducting patient teaching for a 16-year-old who is starting oral contraception. What statement indicates that she needs additional teaching? A) I will monitor my weight and have my blood pressure checked monthly. B) I will see my womans health provider and have a Pap smear done on a yearly basis. C) If I forget to take my pill for 2 consecutive days I will take three pills to catch up. D) I will take the pill every day at the same time and never miss a pill. Ans: C Feedback: If one tablet is missed, take it as soon as possible or take two tablets the next day. If two consecutive tablets are missed, take two tablets daily for the next 2 days; then resume the regular schedule. If three consecutive tablets are missed, begin a new cycle of tablets 7 days after the last tablet was taken, and use an additional method of birth control until the start of the next menstrual period. The other statements are accurate and denote the patient understood the nurses teaching. 15. The patient comes to the womens health clinic to ask about emergency contraception. The patient is prescribed levonorgestrel (Plan B). How will the nurse instruct the patient to take this medication? A) Take one tablet within 4 days of unprotected intercourse. B) Take one tablet within 72 hours of unprotected intercourse and another 12 hours later. C) Take one tablet within 5 days of unprotected intercourse. D) Take one tablet within 72 hours of unprotected intercourse and another daily for 5 days. Ans: B Feedback: Levonorgestrel (Plan B) is taken within 72 hours of unprotected intercourse with another tablet taken 12 hours after the first. Levonorgestrel (Plan B One-Step) is taken once within 72 hours after unprotected intercourse and is available OTC for patients 17 years and older. Ulipristal (Ella) is taken once within 5 days of unprotected intercourse. 16. The nurse is caring for a patient taking raloxifene. What manifestation reported by the patient would raise the highest level of concern from the nurse? A) Headache B) Weight loss C) Calf pain D) Edema Ans: C Feedback: The highest level of concern would be calf pain because it could indicate a possible venous thrombosis that has the potential to be life threatening. Raloxifene has been associated with GI upset, nausea, and vomiting. Changes in fluid balance may also cause headache, dizziness, visual changes, and mental changes. Hot flashes, skin rash, edema, and vaginal bleeding may occur secondary to specific estrogen receptor stimulation. However, these are not symptoms. 17. An adolescent patient asks the nurse, What should I do if I forget to take my birth control pill? What should the nurse reply? A) Abstain from intercourse for 7 days. B) Its okay to miss a day or two, as long as you dont go over 5 days. C) Just wait until your next dose, then take double the dose. D) Take the dose as soon as you discover your oversight. Ans: D Feedback: A missed pill should be taken as soon as the error is noticed. Telling the patient to abstain from intercourse would be inappropriate but if the patient misses three tablets they should use another form of birth control until the next cycle of pills is started. It is not okay to miss a dose and the highest protection is provided when the pill is taken daily without missing a dose. The sooner the missed dose is taken the better contraceptive protection provided, so patients should not wait until the next dose and then double it. 18. The nurse is caring for a patient who is extremely agitated about finding out she is pregnant and wants to take an abortifacient that will have the fastest possible action. What drug will the nurse expect to be ordered? A) Mifepristone B) Carboprost C) Prostin 15 D) Bepridil Ans: B Feedback: Carboprost is available as an intramuscular injection with an onset of 15 minutes and a 2-hour duration of effect. Mifepristone takes 5 to 7 days to produce the desired effect so would not be appropriate for this patient. Prostin 15 is only available in Europe. Bepridil is a calcium channel blocker and would not be used as an abortifacient. 19. The nurse, working in a womens health center, is reviewing the patients medical record and recognizes the patient with what medical history should not receive an abortifacient? (Select all that apply.) A) Active pelvic inflammatory disease (PID) B) Pulmonary disease C) Cardiovascular disease D) Hypertension E) Adrenal disease Ans: A, B, C Feedback: Abortifacients should not be used with active PID or acute cardiovascular, hepatic, renal, or pulmonary disease. Caution should be used with any history of asthma, hypertension, or adrenal disease. 20. The nurse is developing a nursing plan of care for a patient who will receive a fast-acting abortifacient. What nursing diagnosis would apply to care provided shortly after administering the medication? A) Acute pain related to uterine contractions or headache B) Ineffective coping related to abortion or fetal death C) Risk for fluid volume deficit related to blood loss, diarrhea, and diaphoresis D) Deficient knowledge regarding drug therapy. Ans: A Feedback: The rapid-acting abortifacients work within 10 to 15 minutes , so shortly after administration of the drug, the patient will begin to have acute abdominal pain. Only after uterine contents are evacuated would the risk for fluid volume imbalance occur. Deficient knowledge regarding drug therapy should have been addressed before administering the medication. Ineffective coping usually occurs after acute symptoms subside and the patient begins to cope with the decision. 21. The nurse is preparing to give a patient an injection of carboprost. What is the nurses priority action before administering the drug? A) Explain the ethical indications of the drug. B) Assess for contraindications or cautions. C) Verify that the woman thoroughly understands what will happen. D) Draw up the medication and inject within 5 minutes. Ans: C Feedback: Carboprost is an abortifacient and it is important that the woman understand what will happen after the drug is taken and that she cannot change her mind. Contraindications and cautions should have been checked before the drug was prescribed. It is not the nurses place to share ethical beliefs with the patient. After the patients understanding is verified, the drug can be prepared. 22. The nurse is caring for a woman with a new prescription for oral contraceptives. What outcome would be most important for the nurse to evaluate? A) The patient can demonstrate how to inject the medication. B) The patient can explain how medication will prevent sexually transmitted infections. C) The patient can verbalize how and when to take medication even if a pill is missed. D) The patient makes the necessary appointments for follow-up care. Ans: C Feedback: It is most important for the nurse to evaluate the patients understanding of how to take the medication properly, including how to respond when a pill is missed. The patient need not learn how to inject the medication because the prescription is of an oral medication. Oral contraceptives will not prevent sexually transmitted infections. Although making follow-up appointments is good, it is more important that the patient know how to take the medication. 23. When caring for a patient on estrogen therapy, what is the nurses priority assessment? A) Blood sugar levels B) Bowel sounds C) Weight D) Therapeutic and adverse drug effects Ans: D Feedback: Perform a physical assessment to establish a baseline status before beginning therapy and during therapy to determine the effectiveness of therapy and evaluate for any potential adverse effects. Bowel sounds, weight, and blood sugar may be part of the assessment, but it is most important to assess for therapeutic and adverse effects of the medication. 24. What is the priority assessment for the 23-year-old female patient who is on estrogen therapy? A) Monitor liver function periodically for the patient on long-term therapy. B) Assess for contraindications to drug therapy. C) Help plan a diet rich in calcium and vitamin D. D) Provide patient teaching for diet therapy to prevent osteomyelitis. Ans: A Feedback: Assessing liver function is important for the patient on long-term estrogen therapy. Teaching is an intervention and not an assessment. The patient should be assessed for contraindications before administering the medication initially. 25. The nurse assesses the postpartum patient who has been receiving ergonovine and suspects ergotism when what manifestations are found? (Select all that apply.) A) Weak pulse B) Dyspnea C) Numb cold extremities D) Chest pain E) Postpartum hemorrhage Ans: A, B, C, D Feedback: Ergonovine and methylergonovine can produce ergotism, manifested by nausea, blood pressure changes, weak pulse, dyspnea, chest pain, numbness and coldness in extremities, confusion, excitement, delirium, convulsions, and even coma. Postpartum hemorrhage can occur as an adverse effect of ergonovine but is not a manifestation of ergotism. 26. A couple comes to the fertility clinic for help in getting pregnant and the nurse administers menotropin to the male partner. How does this classification of drug work? A) Stimulates endogenous estrogen B) Stimulates follicle development C) Stimulates spermatogenesis D) Blocks the secretion of testosterone Ans: C Feedback: Menotropins stimulate spermatogenesis in men with low sperm counts and otherwise normally functioning testes. Options A, B, and D are not correct. 27. When the nurse administers an endogenous estrogen, what systemic effects does the nurse expect the drug will have? A) Causes proliferation of endometrial lining B) Provides protection of heart from atherosclerosis C) Retains calcium in the bloodstream D) Inhibits ovulation Ans: B Feedback: Estrogens produce a wide variety of systemic effects, including protecting the heart from atherosclerosis, retaining calcium in the bones, not the bloodstream, and maintaining the secondary female sex characteristics. Proliferation of endometrial lining and inhibiting ovulation are effects of estrogen but are not systemic effects. 28. The nurse is talking with a menopausal woman about the use of hormone replacement therapy (HRT). What statement, if made by the nurse, would be accurate and appropriate to share with the patient? A) Symptoms of menopause are short-term and minor so HRT is not necessary. B) The newer drugs used in HRT cause cardiovascular events even when taken short-term. C) The risk for osteoporosis is much higher in women who take HRT. D) There is a possible increased risk of breast and cervical cancer when taking HRT. Ans: D Feedback: The use of HRT can decrease the discomforts associated with menopause, although various forms of HRT have been associated with increased risks of breast and cervical cancer, heart disease, and stroke. The newer drugs used in HRT have been shown to be associated with only a possible increase in risk of breast and cervical cancer, but with long-term use, they are associated with an increased risk of cardiovascular events. The risk for osteoporosis declines with HRT because of the bone saving effects of the drugs. It would be inappropriate and judgmental for the nurse to say symptoms of menopause are minor because some women experience more severe symptoms that can negatively impact their day-to-day life. 29. The nurse is caring for an infertile couple who will take chorionic gonadotropin to become pregnant. How would the nurse describe the actions of this drug? A) Affecting follicle-stimulating hormone (FSH) and luteinizing hormone (LH) release B) Stimulating follicular development C) Stimulating maturation of ova D) Stimulating multiple follicle development Ans: A Feedback: Chorionic gonadotropin is used to stimulate ovulation by acting like gonadotropin-releasing hormone (GnRH) and affecting FSH and LH release. It does not stimulate follicular development, maturation of the ova, or multiple follicle development. 30. The nurse would question the use of what herbal supplement by a patient taking hormone replacement therapy (HRT) containing progesterone? A) Dong quai B) Devils claw C) Wild yam D) Black cohosh Ans: C Feedback: Wild yam contains progesterone. Do not use with hormone replacement therapy, because it may cause increased blood glucose and other toxic effects, and do not combine with disulfiram or metronidazolesevere reaction may occur. Dong quai, devils claw, and black cohosh are not contraindicated with HRT. 31. When the nurse learns the male patient takes conjugated estrogens, the nurse questions the patient about what disorder? A) Prostate cancer B) Breast cancer C) Osteoporosis D) Andropause Ans: A Feedback: Conjugated estrogen is most commonly taken by men to treat prostate cancer because the estrogen competes with testosterone for binding sites. Although men do get breast cancer and osteoporosis, they would not be treated with estrogen for these disorders. Andropause is caused by a reduction in testosterone so they would get a male hormone replacement and not estrogen. 32. The nurse is preparing to administer clomiphene to the female patient. What dosage would the nurse find is within usual dosage range? A) 100 mg B) 10 mg C) 1 mg D) 0.1 mg Ans: A Feedback: The usual dosage range for clomiphene is 50 to 100 mg/d PO with length of therapy and timing dependent on the particular situation. Options B, C, and D are not correct. 33. The nurse is providing patient teaching for a woman who will begin receiving Depo-Provera injections. When will the nurse schedule the appointment for the next injection? A) 1 month from last injection B) 3 months from last injection C) 6 months from last injection D) 12 months from last injection Ans: B Feedback: Depo-Provera is administered by deep intramuscular (IM) injection every 3 months. Options A, C, and D are not correct. 34. The patient calls the clinic and says she just started oral contraceptives last month and is experiencing breakthrough bleeding, fluid retention resulting in edema, changes in libido, and palpitations. What symptom would the nurse recognize is not an adverse effect of the oral contraceptive and requires an appointment as soon as possible so the patient can be evaluated? A) Breakthrough bleeding B) Fluid retention C) Changes in libido D) Palpitations Ans: D Feedback: Palpitations are not commonly associated with contraceptives and so the patient would need to be seen and evaluated. The most common adverse effects of estrogens include breakthrough bleeding, menstrual irregularities, dysmenorrhea, amenorrhea, and changes in libido. Other adverse effects can result from the systemic effects of estrogens, including fluid retention, electrolyte disturbances, headache, dizziness, mental changes, weight changes, and edema. GI effects also are fairly common and include nausea, vomiting, abdominal cramps and bloating, and colitis. Potentially serious GI effects, including acute pancreatitis, cholestatic jaundice, and hepatic adenoma, have been reported with the use of estrogens. 35. The patient calls to report perineal pain and breast enlargement. What medication does the nurse expect to find the patient is taking on reviewing the medical record? A) Desogestrel B) Drospirenone C) Progesterone D) Norethindrone Ans: C Feedback: Perineal pain and breast enlargement results from vaginal gel formulas, so the nurse would suspect the patient is receiving progesterone because this is the only progestin administered by this method. The other options are all oral medications that would not cause perineal pain or breast enlargement Male : 1. A nurse is caring for an adolescent with hypogonadism who has been prescribed fluoxymesterone (Androxy) for replacement. What adverse effect of this drug will the nurse alert the patient to that could result in self-esteem concerns? A) Acne B) Dizziness C) Insomnia D) Fatigue Ans: A Feedback: All these options are possible adverse effects of the drug. However, for the adolescent, acne is the only one that could cause image concerns, which could lead to low self-esteem. Dizziness, insomnia, and fatigue would not cause self-esteem concerns. 2. A postmenopausal patient has been diagnosed with breast cancer. The patient is being treated with methyltestosterone (Testred). What assessment finding would the nurse suspect is a complication from the medication? A) Increased blood pressure B) Jaundice C) Weight loss D) High-pitched voice Ans: B Feedback: A potentially life-threatening effect of an androgen is hepatocellular cancer. Jaundice would indicate that the liver function has been compromised. Usually weight gain and deepening of the voice are adverse effects of this drug. An increase in blood pressure is not associated with methyltestosterone. 3. The nurse administers an androgen and assesses the patient for what action caused by this drug? A) Decreased production of red blood cells B) Decreased protein anabolism C) Increased protein catabolism D) Increased retention of nitrogen Ans: D Feedback: Androgens act to increase the retention of nitrogen, sodium, potassium, and phosphorous. They increase production of red blood cells and protein anabolism and decrease protein catabolism. 4. An older male patient, who has difficulty swallowing pills and tablets, will begin taking an androgen. What drug would the nurse identify as a good choice for this patient? A) Danazol (Danocrine) B) Fluoxymesterone (Androxy) C) Methyltestosterone (Testred) D) Testosterone (Androderm) Ans: D Feedback: A good choice for this patient would be testosterone because the drug can be administered in long-acting depository forms and by dermatologic patch. These forms would eliminate the need for the patient to swallow a pill or tablet. The other options shown here are available only in oral form. 5. A 28-year-old female patient is taking danazol (Danocrine) as treatment for endometriosis. The patient is upset about increased facial hair, a weight gain of 15 pounds, and a change in her voice; she discusses her concerns with the nurse. What nursing diagnoses would be most appropriate for this patient? A) Acute pain related to need for injections B) Deficient knowledge regarding drug therapy C) Disturbed body image related to drug therapy D) Sexual dysfunction related to androgenic effects Ans: C Feedback: The patient is concerned about the way she looks and the sound of her voice. The most appropriate nursing diagnosis would be disturbed body image. Danazol is not administered by injections; therefore, acute pain would not be applicable. Sexual dysfunction and deficient knowledge would be possible nursing diagnoses for this patient, but the concerns expressed by the patient fail to support these diagnoses. 6. A child in renal failure is taking androgens to promote red blood cell production and is seen in the clinic every other month. What adverse drug effect would the nurse monitor for with this child? A) Epiphyseal closure B) Acne C) Skin color D) Weight gain Ans: A Feedback: Because of the effects of androgens on epiphyseal closure, children should be closely monitored with hand and wrist radiographs before treatment and every 6 months after treatment. The other options are also adverse effects and require monitoring but are not as serious as premature epiphyseal closure. 7. A patient has high blood pressure and penile erectile dysfunction. He asks the nurse if he could try tadalafil (Cialis) after seeing an advertisement on television. What medications, if taken by the patient, would the nurse recognize as increasing the risk associated with taking tadalafil? A) Beta-blockers B) Angiotensin-converting enzyme (ACE) inhibitors C) Alpha-adrenergic blockers D) Calcium channel blockers Ans: C Feedback: Serious drops in blood pressure, leading to potentially fatal myocardial infarction or cerebrovascular event, have been reported when Cialis is combined with alpha-adrenergic blockers. This reaction has not been reported with beta-blockers, calcium channel blockers, or ACE inhibitors. 8. A patient will begin taking sildenafil (Viagra) for penile erectile dysfunction (ED). What is the nurses priority teaching point about this drug? A) He will have an erection exactly 1 hour after taking the drug. B) The drug should not be taken with a penile implant or any anatomic penile obstruction. C) Avoid drinking grapefruit juice for 2 days before and after taking the drug. D) It is important to know the cause of ED because sildenafil does not treat all causes. Ans: C Feedback: Patients who are using sildenafil need to be advised to avoid drinking grapefruit juice while using the drug. Grapefruit juice can cause a decrease in the metabolism of the PDE5 inhibitor, leading to increased serum levels and a risk of toxicity. They need to know that it takes 48 hours for grapefruit juice to be processed by the body, so they need to avoid it for several days before and after taking the drug. The patient should be screened for penile implants, anatomic penile obstruction, and the cause of ED before the drug is prescribed. Sildenafil has a median onset of 27 minutes and duration of 4 hours so the patient may have an erection as early as 27 minutes after taking it. 9. A patient is prescribed oxymetholone (Anadrol-50) for treatment of angioedema. The nurse knows the patient has type 2 diabetes and will instruct the patient that the combination of oxymetholone with antidiabetic agents may lead to what? A) Hyperglycemia B) Hypoglycemia C) Jaundice D) Urinary retention Ans: B Feedback: Because of its effects on the liver, oxymetholone (Anadrol-50) may interact with antidiabetic agents to decrease their metabolism and increase their effectiveness, leading to hypoglycemia. The dosage of the antidiabetic agents should be reduced and the patient monitored closely. It would not lead to hyperglycemia, jaundice, and urinary retention. 10. A student athlete presents at the clinic for his physical checkup. The athlete tells the nurse that he is using large doses of creatine to increase muscle size and strength. What should the nurse instruct the student to do while taking this medication? A) Take nonsteroidal anti-inflammatory drugs (NSAIDs) to stop pain from expanding muscles. B) Drink plenty of fluids while using this drug and watch for swelling. C) Stop eating red meat or other animal products. D) Take cimetidine (Tagamet) to relieve the stomach upset associated with the use of this therapy. Ans: B Feedback: Creatine is a protein by-product that has to be processed through the kidneys. If an athlete is using it to try to increase muscle size, he should be advised to drink plenty of fluids to help flush it through the kidney and to watch for any swelling that could indicate change in renal function. NSAIDs could aggravate renal problems and cause more swelling. Getting protein from food sources is a natural way to provide the body with proteins. If stomach upset occurs, the athlete should stop using the herbal therapy and not add another drug to his regimen, especially cimetidine that can increase risk of kidney damage. 11. A 9-year-old boy is taking testosterone injections for treatment of hypogonadism. What should be measured every 6 months on this child? A) Liver function test (LFT) B) Cholesterol level C) Vision D) Hand and wrist radiographs Ans: D Feedback: Because of the effects of these hormones on epiphyseal closure, children should be closely monitored with hand and wrist radiographs before treatment and every 6 months after treatment. It would not be necessary to measure LFTs, cholesterol levels, or the childs vision. 12. The nurse assesses the patient taking anabolic steroids for what serious adverse effects? A) Elevated blood urea nitrogen (BUN) B) Elevated blood glucose level C) Bradycardia D) Jaundice Ans: D Feedback: Anabolic steroids all have black box warnings as alerts to the potentially serious effects of liver tumors, hepatitis, and blood lipid level changes that might be associated with increased risk of coronary artery disease. As a result, the nurse should assess for jaundice that could be an early indication of liver dysfunction. Elevated BUN, blood glucose levels, and bradycardia are not commonly reported adverse effects. 13. What therapeutic effect will the nurse assess for in the male patient taking anabolic steroids? A) Thinning of the skin B) Body hair loss C) Increase in red blood cells (RBCs) and hemoglobin D) Gynecomastia Ans: C Feedback: Therapeutic effects include promotion of body tissue-building processes, reverse catabolic or tissue-destroying processes, and increased hemoglobin and red blood cell mass. Thinning of the skin and body hair loss are not associated effects of anabolic steroids. Gynecomastia is an adverse effect and would not be a therapeutic effect for which the drug was administered. 14. An woman is prescribed androgens. The nurse teaches the patient to anticipate what effect from the drug? A) Atrophy of breasts B) Rapid bone growth C) Loss of pubic hair D) Nervousness Ans: D Feedback: Antiestrogen effectsflushing, sweating, vaginitis, nervousness, and emotional labilitycan be anticipated when these drugs are used in women. When androgens are administered to women you would not expect the results to include atrophy of the breasts, rapid bone growth, or loss of pubic hair. 15. The nurse is caring for a patient who has been taking androgens and has been admitted to the ICU following a car accident. What lab results will the nurse interpret as more indicative of androgen therapy than actual disease states? (Select all that apply.) A) Decreased thyroid function B) Increased creatinine levels C) Increased creatinine clearance D) Elevated liver enzymes E) Increased white blood cell (WBC) count Ans: A, B, C Feedback: While a patient is taking androgens, thyroid function may be decreased, as well as increased creatinine and creatinine clearance, results that are not associated with disease states. These effects can last up to 2 weeks after the discontinuation of therapy. Elevated liver enzymes may indicate a potentially life-threatening effect that has been documented is hepatocellular cancer. Increased WBC count would indicate an infection. Increased liver enzymes and WBC are not associated with androgen therapy but instead indicate a disease state. 16. The 63-year-old male patient receives a prescription for androgens. The nurse evaluates that the patient understood drug teaching when he makes what statement? A) If I experience acne, I will contact my physician immediately. B) If I experience flushing, sweating, nervousness, or emotional lability Ill know its the drug. C) I will report any difficulty urinating such as trouble starting my flow. D) These pills may make my skin turn yellow but it will go away when the drug is stopped. Ans: B Feedback: Benign prostatic hypertrophy, a common problem in older men, may be aggravated by androgenic effects that may enlarge the prostate further, leading to urinary difficulties and increased risk of prostate cancer. Nurses should teach these men the signs and symptoms of prostatic enlargement and the importance of reporting these manifestations immediately to prevent worsening of symptoms. Acne need not be reported immediately. Flushing, sweating, nervousness, and emotional lability are more usually experienced by women. Yellowing of the skin may be an indication of liver disease, which should be reported immediately. 17. A female athlete is using high doses of anabolic steroids to enhance her performance. The nurse warns this patient that such use of anabolic steroids is likely to result in what? A) Masculinization B) Immobility of joints C) Obesity D) Hypotension Ans: A Feedback: Adverse effects in women include masculinization effects, hirsutism, and deepening of the voice. A woman using anabolic steroids would not expect it to result in joint immobility, obesity, or hypotension. 18. The nurse attributes what assessment finding to the use of androgens by the male patient? A) Testicular atrophy B) Increased fertility C) Increased urination D) Hoarseness Ans: A Feedback: In adult men, adverse effects include inhibition of testicular function, gynecomastia, testicular atrophy, priapism, baldness, and change in libido. Increased fertility, increased urination, and hoarseness would not be expected assessment findings. 19. The nurse administers danazol (Danocrine) be a woman for what purpose? A) Reverse folic acid deficiencies B) Increase hair growth C) Decrease symptoms of endometriosis D) Prevent toxic shock syndrome. Ans: C Feedback: Danazol may be used in women to prevent or treat endometriosis or fibrocystic breast disease. Danazol is not prescribed for increased hair growth, to reverse folic acid deficiencies, or to prevent toxic shock syndrome. 20. The nurse assesses the young adult athlete who has been taking anabolic steroids to enhance his performance. What findings would the nurse associate with this practice? (Select all that apply.) A) Personality changes B) Sexual dysfunction C) Increased serum lipid levels D) Cardiomyopathy E) Weight loss Ans: A, B, C, D Feedback: Cardiomyopathy, hepatic carcinoma, personality changes, and sexual dysfunction are all associated with the excessive and off-label use of anabolic steroids for athletic performance enhancement. Adverse effects associated with prescription use include inhibition of testicular function, gynecomastia, testicular atrophy, priapism, baldness, change in libido, serum electrolyte changes, liver dysfunction, insomnia, and weight gain, not weight loss. 21. The nurse learns a patient receiving a new prescription for oxandrolone takes a lipid-lowering medication daily. What will the nurse tell the patient about this drug interaction? A) Lipid-lowering drug becomes much less effective. B) Increased risk of oxandrolone toxicity with this combination of drugs C) Increased risk of liver damage with this combination of drugs D) Oxandrolone becomes less effective with this combination of drugs. Ans: A Feedback: Anabolic steroids may alter lipid metabolism and cause a lack of effectiveness for lipid-lowering agents. Lipid-lowering medications do not impact anabolic steroids and there is no increased risk for toxicity of either drug. 22. What supplement would be safest for a diabetic female athlete wishing to improve athletic performance? A) Bee pollen B) Damiana C) Wild yam D) Spirulina Ans: C Feedback: Wild yam is an estrogen-like herb used to increase athletic performance, slow the aging process, and improve energy and stamina. It can be toxic to the liver and may cause breast pain but is the safest of these supplements. Bee pollen is associated with serious allergic reactions and random studies have found wide variety of ingredients in each product. Damiana interferes with antidiabetic agents so it would not be appropriate for this patient. Spirulina may contain toxic metals and can cause serious reactions in children and pets. It interferes with vitamin B12absorption and there are no studies to prove it is effective in the claims it makes. 23. When a 5-year-old patient is prescribed oxandrolone (Oxandrin) to promote weight gain, how would the nurse expect the drug to be administered? A) Long-term B) Short-term C) Continuous D) Intermittent Ans: D Feedback: Oxandrin is given intermittently to pediatric patients and should not be used on a daily basis for short, long, or continuous therapy. 24. The nurse gives a class at the local high school on the use of anabolic steroids. The nurse explains that one of the most serious and all too common side effects of using these drugs without a prescription and medical follow- up is what? A) Renal stones B) Death C) Cirrhosis D) Malignant hyperthermia Ans: B Feedback: Prolonged use of high doses may cause potentially life-threatening conditions, placing the patient at risk of death. Conditions such as peliosis hepatis, hepatic neoplasms, and hepatocellular carcinoma are often seen with high-dose street use of these drugs. Renal stones, cirrhosis, and malignant hyperthermia are not associated with the use of anabolic steroids. 25. When fluoxymesterone (Androxy) is administered to a 14-year-old boy for hypogonadism, what is the expected result? A) Enlarged sex organs B) Decreased skin thickness C) Increased protein metabolism D) Increased sperm production Ans: A Feedback: In prepubertal boys, administration of male sex hormone-like drugs will stimulate development of masculine characteristics and cause development of the male sexual organs. Production of sperm will occur when the boy enters puberty. These drugs are not administered to decrease skin thickness or increase protein metabolism. 26. A 69-year-old man is prescribed testosterone. The patient is found to have hypertension and a history of congestive failure after assessment by the nurse. What is this patient at increased risk for? A) Fluid retention B) Impotence C) Liver failure D) Kidney failure Ans: A Feedback: Older adults often have hypertension and other cardiovascular disorders that may be aggravated by sodium and water retention associated with androgens and anabolic steroids. Testosterone would not increase the risk of impotence. Liver and kidney failure could be exacerbated by the drug if they were preexisting conditions but since that is not indicated by the question, this would be a lower risk than fluid retention. 27. The nurse is caring for a patient with cryptorchidism who is 28 years old and taking testosterone to treat his condition. What statement by this patient would lead the nurse to believe that he has understood the teaching provided about the drug? A) My body hair may increase. B) My sexual desire may increase. C) My voice may become higher. D) My skin may become clear and soft. Ans: A Feedback: Androgenic effects include acne, edema, hirsutism (increased hair distribution), deepening of the voice, oily skin and hair, weight gain, decrease in breast size, and testicular atrophy. Testosterone does not make the skin clear and soft, it does not make the voice higher, and the testicular atrophy is more likely to decrease rather than increase libido. 28. A 16-year-old boy is diagnosed with delayed onset of puberty and the physician has ordered testosterone, intramuscular (IM), once every 2 weeks. What nursing intervention would be important to the patient? A) Discuss changes that will occur in his body. B) Have patient fast before injection. C) Have patient reduce protein intake. D) Decrease exercise while on this hormonal treatment. Ans: A Feedback: It is important for this patient to understand what will happen and the changes he will see to reduce the anxiety that could occur if he didnt understand. Discuss the development of masculine characteristics as well as common adverse effects such as acne. Having the patient fast before the injection would not be necessary, nor would decreasing exercise or reducing protein intake. 29. A patient has been prescribed sildenafil citrate. What should the nurse teach the patient about this medication? A) Take the medication with a glass of grapefruit juice. B) The drug should be taken 1 hour before attempting intercourse. C) Facial flushing or headache should be reported to the physician immediately. D) A dose exceeding 80 mg will result in a change of vision, making everything appear blue. Ans: B Feedback: The drug should be taken approximately 1 hour before intercourse to allow adequate time for absorption and therapeutic effects to occur. Facial flushing, mild headache, indigestion, and running nose are common side effects of sildenafil citrate and do not need to be reported unless they become acute. The blue haze that occurs with the 100-mg dosage is transient (it lasts about 1 hour). Grapefruit juice should be avoided 2 days before until 2 days after taking the medication because it prolongs the drugs metabolism and excretion. 30. A 55-year-old man presents at the clinic complaining of erectile dysfunction. The patient has a history of diabetes mellitus. The physician orders tadalafil (Cialis) to be taken 1 hour before sexual intercourse. The nurse reviews the patients history before instructing the patient on the use of this medication. What disorder (or condition) would contraindicate the use of tadalafil (Cialis)? A) Cataracts B) Penile implant C) Hypotension D) Lung cancer Ans: B Feedback: Patients with a penile implant should not take tadalafil. Patients with cataracts, hypotension, or lung cancer may take tadalafil if needed but should do so with caution and should be carefully monitored for adverse effects. 31. For what reason might the nurse administer sildenafil to a woman? A) Pulmonary arterial hypertension B) Sexual dysfunction C) Breast cancer D) Endometriosis Ans: A Feedback: Sildenafil is used to treat erectile dysfunction in the presence of sexual stimulation in men and to treat pulmonary arterial hypertension in women. It is not used for sexual dysfunction, breast cancer, or endometriosis in women. 32. A 68-year-old male patient tells the nurse that he has been unable to get an erection for the past 6 months and he guesses his sex life is over. The provider orders diagnostic testing to determine whether sildenafil (Viagra) is appropriate for the patient. What is the most correct nursing diagnosis for this patient? A) Sexual dysfunction B) Disturbed body image C) Ineffective sexuality pattern D) Disturbed tactile sensory perception Ans: A Feedback: The patient is experiencing sexual dysfunction so that would be the most appropriate nursing diagnosis. The patient did not discuss feeling let down by or unhappy with his body so disturbed body image is incorrect. The effectiveness of sexual pattern is not known. No problem with sensory perception is indicated by this question. 33. The 22-year-old patient tells the nurse he doesnt have trouble obtaining and maintaining an erection but wonders if taking Viagra would improve the sexual experience anyway. What is the nurses best response? A) The only thing Viagra does is improve blood flow to the penis to make it erect. B) Viagra improves stamina and sensation, making the sexual experience better. C) Viagra has its greatest effect if both the man and woman take it at the same time. D) Viagra does nothing to improve the sexual experience. Ans: A Feedback: Viagra improves blood flow into the penis and that is its only effect. It does not improve stamina or sensation. Research has indicated it has no effect on womens sexual response. To say only that Viagra does nothing would not provide adequate information for the patient. 34. The patient is prescribed oxandrolone 2.5 mg twice a day and is told to increase the dosage to gain weight to a maximum of 20 mg/d. If each tablet contains 2.5 mg, how many tablets would the nurse tell the patient he may take per day to avoid exceeding the 20 mg/d maximum? A) 4 B) 6 C) 8 D) 10 Ans: C Feedback: Calculate the number of tablets required to administer 20 mg by dividing 20 mg by 2.5 mg. 20/2.5 = 8 tablets 35. The patient is prescribed sildenafil 25 mg PO one hour before sexual intercourse is planned. The patient returns for follow-up care and says that 25 mg did not produce an erection so he increased the dosage. How much of an increase would be enough to concern a nurse? A) 50 mg B) 75 mg C) 100 mg D) 25 mg Ans: C Feedback: Normal dosage range is 25 to 100 mg so the nurse should not be concerned unless the dosage exceeded 100 mg. Urinary Tract : 1. The clinic nurse is admitting a 39-year-old woman who has come to the clinic complaining of left-sided tenderness, fever, chills, and flank pain. What does the nurse suspect the patient has? A) Cystitis B) Kidney stones C) Neurogenic bladder D) Pyelonephritis Ans: D Feedback: The fever and chills indicate an inflammatory process. Flank pain and left- sided tenderness indicate kidney swelling within the capsule. These symptoms indicate pyelonephritis. Kidney stones cause intense pain; fever and chills would not be present. Cystitis and neurogenic bladder present with bladder-related symptoms such as frequency, urgency, burning, and bloating. 2. The nurse is caring for four patients. Which patient would flavoxate (Urispas), a urinary tract medication, be indicated for? A) A 1-year-old girl B) A 6-year-old boy C) A 10-year-old boy D) A 14-year-old girl Ans: D Feedback: Flavoxate prevents smooth muscle spasm in the urinary tract and can be given to children older than 12 years of age. Oxybutynin and phenazopyridine may be given to children 6 years old and older. Guidelines for use of an antispasmodic for a child younger than 6 have not been established. 3. The nurse is discussing the effects of doxazosin (Cardura) with a 65-year- old man who has just been diagnosed with benign prostatic hyperplasia (BPH). The patient asks the nurse whether the drug will make him impotent. After the discussion with the patient, the nurse determines that a potential priority nursing diagnosis could be what? A) Sexual dysfunction related to adverse effects B) Deficient knowledge regarding drug therapy C) Noncompliance with drug therapy related to adverse effects D) Acute pain related to adverse effects Ans: C Feedback: The patient is concerned about his ability to perform sexually. Sexual dysfunction is a possible adverse effect and would be a concern for this patient. However, he has not started taking the drug. The nurse is concerned about noncompliance because of the possibility of this adverse effect. This is an important part of a mans life and most men would not want to take medication that would cause sexual dysfunction. Headache is an adverse effect of drugs used for BPH, but it can be tolerated and treated with an analgesic. Deficient knowledge about the drug is a concern, but usually men who know that sexual dysfunction is a possible adverse effect of a drug will find out all they can about the drug. Acute pain is not related to this drug. 4. A businesswoman who is leaving on a business trip the next day tells the nurse she knows she has cystitis and does not want to have to mess with medicine while she is gone. What drug would be a good choice for this patient? A) Fosfomycin (Monurol) B) Methenamine (Hiprex) C) Nitrofurantoin (Furadantin) D) Norfloxacin (Noroxin) Ans: A Feedback: Fosfomycin would be a good choice for this patient because it has the convenience of a single dose. Methenamine is taken either twice a day or up to four times a day. This drug could interfere with the patients busy schedule. Nitrofurantoin is also prescribed four times a day and would also be inconvenient for the patient. Norfloxacin is taken every 12 hours and could be inconvenient as well. 5. The nurse is performing patient teaching about the urinary anti-infective methenamine (Hiprex). What information is most important for the nurse to share with this patient? A) Limit fluid intake. B) Drink orange juice once a day. C) Take the medication with food. D) Take the medication at night before going to bed. Ans: C Feedback: The patient should take the medication with food to decrease GI adverse effects. The nurse would encourage the patient to increase fluid intake to flush the bladder and urinary tract frequently and decrease the opportunity for bacteria growth. Orange juice would be contraindicated because it could cause an alkaline rash and produce alkaline urine, which encourages bacterial growth. This drug is taken two to three times a day and not in a single dose at night. 6. The nurse is assessing a patient who is taking oxybutynin (Ditropan). What would be the priority nursing assessment for this patient? A) Skin condition B) Cardiac arrhythmia C) Vision changes D) Mental status Ans: C Feedback: The nurse should assess for vision changes and recommend an ophthalmologic examination during treatment to evaluate drug effects on intraocular pressure so that the drug can be stopped if intraocular pressure increases. A rash and changes in cardiac rhythm and rate are possible adverse effects. Also, disorientation (mental status) could be a concern. However, these effects can be treated and may not necessitate stopping the medication. 7. A patient is taking phenazopyridine (Azo-Standard) and ciprofloxacin (Cipro) for a urinary tract infection. What is the most important instruction the nurse needs to provide to the patient concerning this drug combination? A) Do not be alarmed if your urine is a reddish-brown color. B) Be sure to take your medication with food if you have GI irritation. C) Increase your fluid intake. Drink lots of water. D) If you notice yellowing of your eyes or skin, contact your health care provider immediately. Ans: D Feedback: Yellowing of the sclera and skin is a sign of drug accumulation in the body and a possible sign of hepatic (liver) toxicity. Phenazopyridine should not be used more than 2 days, especially if taken, as here, with an antibacterial agent (ciprofloxacin). The other suggested options are important and should be included in the instructions given the patient. However, the possibility of toxicity is the most important. 8. A patient is taking pentosan polysulfate sodium (Elmiron) to decrease pain and discomfort associated with interstitial cystitis. What medication class may cause an adverse reaction when used with Elmiron? A) Anticoagulants B) Antihypertensives C) Diuretics D) Cardiac glycosides Ans: A Feedback: Anticoagulants may react with pentosan polysulfate sodium (Elmiron). This drug has anticoagulant and fibrinolytic effects, which could lead to potential or increased bleeding risks. Antihypertensives, diuretics, and cardiac glycosides do not cause drugdrug interactions with pentosan polysulfate sodium (Elmiron). 9. A pregnant woman is helping her elderly father with taking his medications. He is taking dutasteride (Avodart). The nurse will instruct the daughter to do what? A) Crush the tablets to help facilitate swallowing. B) Avoid touching any crushed or broken tablets. C) Avoid direct contact with her father while he is on the medication. D) Use a barrier contraceptive while helping her father prepare the drug. Ans: B Feedback: Dutasteride contains androgenic hormone blockers that could be absorbed through the skin if the tablets are crushed and broken. These hormone blockers could have negative effects on a fetus. The woman should be cautioned not to touch any crushed or broken tablets. Her father will not pose a threat to her because of this drug therapy. She should not need barrier contraceptives if she is pregnant. 10. A patient is being treated for benign prostatic hyperplasia (BPH). The patient asks the nurse how the medicine used to treat BPH is supposed to work. The nurse explains that the drug therapy is designed to relieve the symptoms associated with this condition by doing what? A) Shrinking the gland and/or relaxing the sphincter of the bladder B) Increasing testosterone levels to improve sexual functioning C) Increasing blood pressure, which will increase blood flow to the area D) Activate nitric acid, which will dilate blood vessels in the area to relieve pressure Ans: A Feedback: Drugs given to treat BPH will block sympathetic activity to allow relaxation of the sphincter of the bladder or will decrease testosterone effects to shrink the gland and relieve symptoms. They do not increase testosterone levels or blood pressure. Dilating blood vessels would further congest the gland and increase symptoms. 11. The pharmacology students are learning about medications used to treat urinary tract problems. What symptoms does phenazopyridine treat? A) Urinary retention B) Hematuria C) Pain and urgency D) Hesitancy Ans: C Feedback: Phenazopyridine is an azo dye that acts as a urinary analgesic and relieves symptoms of dysuria, burning, and frequency and urgency of urination. Phenazopyridine does not treat urinary retention, hematuria, or hesitancy. 12. The nurse is providing health teaching to a 62-year-old female patient who has been started on norfloxacin (Noroxin). This patient should be taught to contact her physician if she experiences what adverse effect? A) Polydipsia B) Tachycardia C) Confusion D) Hypertension Ans: C Feedback: Urinary tract anti-infectives infrequently cause pruritus, urticaria, headache, dizziness, nervousness, and confusion. Norfloxacin is not known to cause polydipsia, tachycardia, or hypertension. 13. A 50-year-old man calls the clinic nurse and complains of gastrointestinal upset after taking nitrofurantoin (Furadantin) on an empty stomach. What recommendation should the nurse make? A) Stopping the medication B) Taking vitamin C along with the medication C) Taking the medicine with or after meals D) Continuing to take the medicine on an empty stomach Ans: C Feedback: These adverse effects may result from GI irritation caused by the agent, which may be somewhat alleviated if the drug is taken with food, or from a systemic reaction to the urinary tract irritation. The nurse would not tell the patient to stop taking the medication without a physicians order to do so nor would the nurse tell the patient to continue taking the medication on an empty stomach. Advising the patient to take the medication along with vitamin C would be of no benefit to the patient described in the question. 14. A 72-year-old female clinic patient is started on cinoxacin (Cinobac) for a urinary tract infection. Before administering this drug, the nurse should assess the patient for what condition? A) Asthma B) Hypertension C) Diabetes mellitus D) Renal insufficiency Ans: D Feedback: Cinoxacin should be used with caution in the presence of renal dysfunction, which could interfere with the excretion and action of this drug. The patients having asthma, hypertension, or diabetes mellitus would not be a contraindication to the use of this drug. 15. The nurse is caring for a 79-year-old male patient who was admitted through the emergency room for mental status changes determined to be caused by a urinary tract infection. The patient is started on norfloxacin (Noroxin) before his discharge home. What adverse effect should the nurse observe this patient for? A) Liver toxicity B) Photosensitivity C) Excess saliva D) Congestive heart failure Ans: B Feedback: Adverse effects include photosensitivity, headache, dizziness, nausea, vomiting, dry mouth, and fever. Generally, adverse effects of norfloxacin do not include liver toxicity, bone marrow depression, or congestive heart failure. 16. A clinic patient has been prescribed phenazopyridine (Pyridium) for aid in treating a UTI. This patient should be informed that Pyridium will turn urine what color? A) Bluish-green B) Reddish-orange C) Brown D) Black Ans: B Feedback: Phenazopyridine turns urine reddish-orange, which may be mistaken for blood. It does not cause the urine to appear bluish-green, brown, or black. 17. A small group of nursing students are giving an oral presentation to their classmates about urinary tract infections (UTIs). What is a measure that can be used to encourage patients to use to reduce the risk of recurrent urinary tract infections? A) Increase alkaline foods in your diet. B) Take tub baths, soaking 15 minutes daily. C) Use sterile gauze pads to cleanse after urinating. D) Drink 2,000 to 3,000 mL of fluid daily. Ans: D Feedback: Many activities are necessary to help decrease bacteria in the urinary tract (e.g., hygiene measures, proper diet, forcing fluids), to facilitate the treatment of UTIs, and help the urinary tract anti-infectives be more effective. Forcing fluids increases the amount of urine that is excreted and prevents urine from sitting in the bladder. It is helpful to keep the urine acidic, not alkaline and avoid sitting in water. The importance of cleansing is to cleanse from front to back. The use of sterile wipes is not necessary. 18. What is the drug of choice in a patient with renal impairment who is being treated for a urinary tract infection (UTI)? A) Salazopyrin B) Silver sulfadiazine C) Declomycin D) Fosfomycin Ans: D Feedback: The dosage of fosfomycin, given orally, does not need to be changed in cases of renal impairment. It is the only medication listed here that is used in the treatment of urinary tract infections. Declomycin may be used to inhibit antidiuretic hormone in the treatment of chronic inappropriate antidiuretic hormone secretion. Salazopyrin is used in ulcerative colitis. Silver sulfadiazine is used topically to treat Pseudomonas infections. 19. Urinary anti-infectives are used only to treat urinary tract infections (UTIs). What causes urinary anti-infectives to be so effective in treating UTIs? A) They sterilize feces. B) They act specifically within the urinary tract. C) They reach high plasma levels in a short period of time. D) They are excreted through the liver. Ans: B Feedback: Urinary tract anti-infectives act specifically within the urinary tract to destroy bacteria, either through a direct antibiotic effect or through acidification of the urine. They are not used in systemic infections because they do not attain therapeutic plasma levels. These drugs are usually excreted through the kidneys not through the liver, and they do not sterilize feces. 20. The nurse is caring for a patient who is taking a urinary anti-infective. What would the nurse need to assess this patient for? A) Discolored urine B) Jaundice C) Signs and symptoms of continuing urinary tract infection (UTI) D) Flank pain Ans: C Feedback: Monitor patient response to the drug (i.e., resolution of UTI and relief of signs and symptoms) and repeat culture and sensitivity tests as recommended for evaluation of the effectiveness of all of these drugs. It would not be necessary to assess for discolored urine, jaundice, or flank pain. 21. The nurse is writing a plan of care for an 85-year-old male patient admitted through the emergency room with a severe urinary tract infection. What intervention, if noted on the care plan, would be an inappropriate for this patient? A) Encourage the patient to drink cranberry juice as part of his daily fluid intake. B) Avoid urinary catheterization when possible. C) Force fluids unless contraindicated. D) Administer antacids to decrease GI irritation caused by the medication. Ans: D Feedback: It would not be appropriate to administer antacids because it will cause the urine to be alkaline and provide more opportunity for bacterial growth. The patients should be encouraged to force fluids and to include cranberry juice in those fluids to help acidify the urine. Avoiding urinary catheterization is an important nursing intervention in all patients because catheterization can allow introduction of bacteria in the bladder. 22. A patient with benign prostatic hypertrophy (BPH) has been prescribed terazosin (Hytrin). How do alpha-adrenergic blockers, such as terazosin, assist in treating the symptoms of BPH? A) They increase gastric motility. B) They increase skeletal muscle contraction. C) They inhibit contraction of the urinary bladder. D) They decrease blood pressure. Ans: C Feedback: Alpha1-adrenergic blockers block postsynaptic alpha1-adrenergic receptors, which results in a dilation of arterioles and veins and a relaxation of sympathetic effects on the bladder and urinary tract. This action makes these drugs useful in the treatment of BPH. BPH is characterized by obstructed urine flow as the enlarged prostate gland presses on the urethra. Alpha1- blocking agents can decrease urinary retention and improve urine flow by relaxing muscles in the prostate and urinary bladder. Options A, B, and D are not correct. 23. A 72-year-old man presents at a blood donor drive. The patient tells the nurse he donates blood on a regular basis. While reviewing the patients medication history, the nurse notes he is taking dutasteride (Avodart) for his benign prostatic hyperplasia (BPH). What would the nurse know is a contraindication to this patient giving blood? A) The testosterone blocking effects will be passed to those receiving the blood. B) Blood donation may cause malignant hyperthermia to occur during the donation. C) Severe hypotension may occur during blood donation. D) Malignant hypertension may occur during blood donation. Ans: A Feedback: Patients using either finasteride or dutasteride cannot donate blood for 6 months after the last dose to protect potential blood recipients from exposure to the testosterone blocking effects. The use of the drug dutasteride will not cause this patient to develop malignant hyperthermia, malignant hypertension, or severe hypotension during blood donation. 24. The clinic nurse is providing care for a patient with a urinary tract infection (UTI). Which drug would the nurse expect to administer to this patient? A) Solifenacin B) Pentosan polysulfate sodium C) Norfloxacin D) Alfuzosin Ans: C Feedback: Norfloxacin is a urinary anti-infective that is used in the treatment of adults with UTIs caused by susceptible strains of bacteria, uncomplicated urethral and cervical gonorrhea, and prostatitis caused by Escherichia coli. Solifenacin is an antispasmodic, pentosan polysulfate sodium is a bladder protectant, and alfuzosin is used in the treatment of benign prostatic hyperplasia (BPH). 25. Urinary tract infections (UTIs) in children do occur. If a child has repeated UTIs, what would be important to assess for? (Select all that apply.) A) Obstruction B) Sexual abuse C) Drinking apple juice D) Bubble baths E) Drinking too much water Ans: A, B, D Feedback: Some children, because of congenital problems or in-dwelling catheters, require other urinary tract agents such as urinary tract analgesics or antispasmodics. A child with repeated UTIs should be evaluated for potential sexual abuse. Children need to be instructed in proper hygiene and should not be given bubble baths if UTIs occur. Repeated UTIs can be related to drinking alkaline juices such as orange or grapefruit, but not apple. Children should be encouraged to drink a lot of water. Drinking too much water would not cause repeated infections. 26. A 77-year-old man has been placed on alfuzosin (Uroxatral) for his benign prostatic hyperplasia (BPH). The nurse explains to the patient that because of the medication he is taking, it will be necessary to monitor for what? (Select all that apply.) A) Pulse pressure B) Intraocular pressure C) Fluid intake D) Blood pressure E) Bladder emptying Ans: B, D, E Feedback: Special precautions to monitor cardiac function, intraocular pressure, blood pressure, and bladder emptying need to be taken when using alpha- adrenergic blockers with these patients. It would not be necessary to monitor pulse pressure or fluid intake in this patient. 27. A 72-year-old man is being treated with doxazosin (Cardura) for his BPH. What nursing diagnosis would be important to include in this patients plan of care? A) Sexual dysfunction B) Chronic pain C) Disturbed sensory perception D) Risk of impaired urinary elimination Ans: A Feedback: Nursing diagnoses related to drug therapy might include sexual dysfunction related to drug effects, acute pain related to headache, central nervous system (CNS) effects, and GI effects of the drug, risk for injury related to blockage of alpha receptors, and deficient knowledge regarding drug therapy. The nursing diagnosis of risk of impaired urinary elimination would not be appropriate because the effect of the drug is to improve urinary elimination issues. 28. You are preparing a plan of care for a 78-year-old female patient who has been hospitalized with a recurrent urinary tract infection (UTI). What would be important to assess for before administering a urinary tract antispasmodic? (Select all that apply.) A) Allergy to eggs B) Glaucoma C) Pyloric obstruction D) Dumping syndrome E) Duodenal obstruction Ans: B, C, E Feedback: Assess for contraindications or cautions: any history of allergy to these drugs to prevent hypersensitivity reactions; pyloric or duodenal obstruction; or other GI lesions or obstructions of the lower urinary tract, which could be dangerously exacerbated by these drugs; glaucoma, which could increase intraocular pressure due to blockage of the parasympathetic nervous system; and current status of pregnancy or lactation, which would require cautious use. It would not be necessary to assess this patient for an allergy to eggs or dumping syndrome. 29. What would be an appropriate nursing intervention for a patient on a urinary tract antispasmodic? A) Monitor for patient use of hot showers. B) Advise patient about change in color of sclera. C) Offer sugarless hard candy. D) Teach proper personal hygiene. Ans: C Feedback: Offer frequent sips of water or use of sugarless hard candy to alleviate dry mouth because antispasmodics have anticholinergic effects that cause dry mouth. The use of hot water for showers will not cause the patient any danger. Urinary antispasmodics do not cause changes in the sclera. This patient has an issue with bladder spasms and not a urinary tract infection. 30. The nurse is providing health teaching to a patient who is taking methenamine (Hiprex). What instruction would be most important to include for the patient taking Hiprex? A) Drink citrus juice with the medication to acidify the urine. B) Take sodium bicarbonate with the medication to make the urine alkaline. C) Limit your fluid intake to 8 ounce per day. D) While you are taking this drug, limit your intake of foods high in sodium. Ans: A Feedback: Urinary tract anti-infectives act specifically within the urinary tract to destroy bacteria, either through a direct antibiotic effect or through acidification of the urine. Therefore, the nurse would not instruct the patient to take sodium bicarbonate, limit fluid intake, or limit their intake of foods high in sodium. 31. The nurse is caring for a patient who is beginning treatment for benign prostatic hyperplasia and knows that this patient may be treated with which classification of drugs? (Select all that apply.) A) Urinary anti-infectives B) Urinary antispasmodics C) Alpha-adrenergic blockers D) Testosterone production blockers E) Urinary analgesics Ans: C, D Feedback: Alpha-adrenergic blockers and testosterone production blockers are drugs used in the treatment of benign prostatic hyperplasia. Urinary anti-infectives are used to treat urinary tract infections. Urinary antispasmodics are used to treat bladder spasms and urinary analgesics are used in the treatment of pain associated with urinary tract infections. 32. The nurse is caring for a patient with a bladder infection. What symptoms are most common with this type of infection? (Select all that apply.) A) Frequency B) Urgency C) Dysuria D) Flank pain E) Temperature elevation over 102F Ans: A, B, C Feedback: Patients with bladder infection most commonly experience urinary frequency, urgency, and burning on urination (dysuria). Patients with pyelonephritis also experience flank pain and temperature elevation. 33. A patient with interstitial cystitis has just begun to take pentosan polysulfate sodium (Elmiron). The nurse would notify the physician if the patient exhibited which symptom(s). (Select all that apply.) A) Petechiae B) Anorexia C) Decreased blood pressure D) Blood in the urine E) Headache Ans: A, C, D Feedback: Adverse effects associated with pentosan use include bleeding that may progress to hemorrhage (related to the drugs heparin effects), headache, alopecia, and GI disturbances. It would be necessary to notify the physician if the patient showed any signs of bleeding. Anorexia and headaches are not associated with this drug. 34. The nurse is providing discharge instructions to a patient who is taking pentosan polysulfate sodium (Elmiron). Which statement by the patient indicates a need for further instruction? A) I will make sure and take this medication with my breakfast. B) I will call the doctor if I start to have any unusual bruises. C) This drug I am taking may cause me to lose hair. D) I will take acetaminophen if I develop a headache. Ans: A Feedback: Pentosan polysulfate sodium should be taken on an empty stomach, either 1 hour before or 2 hours after meals. The patient should call the physician at the first sign of bleeding. Alopecia and headache may occur with this medication. 35. A patient with benign prostatic hyperplasia has been self-treating with an herbal called saw palmetto. The nurse would know which drug is contraindicated in this patient? A) Tamsulosin (Flomax) B) Finasteride (Proscar) C) Alfuzosin (Uroxatral) D) Terazosin (Hytrin) Ans: B Feedback: Saw palmetto is an herbal therapy that has been used very successfully for the relief of symptoms associated with benign prostatic hyperplasia (BPH). Patients with BPH should be cautioned not to combine saw palmetto with finasteride because serious toxicity can occur. There is no contraindication in the use of tamsulosin, alfuzosin, and terazosin, which are alpha- adrenergic blockers. Finasteride is a testosterone blocking agent. Chapter 21. Herbs, Vitamins, and Minerals MULTIPLE CHOICE 1. What is black cohosh used for? a . Preventing miscarriage during the first trimester b . Reducing symptoms of premenstrual syndrome c . Providing antispasmodic activity of the gastrointestinal (GI) system d . Controlling migraine headaches ANS: B Black cohosh is used to reduce symptoms of premenstrual syndrome, dysmenorrhea, and menopause. Therapy is not recommended for longer than 6 months. Because of its effect of relaxing the uterus, black cohosh is not used in pregnant women. Black cohosh does not affect the GI system or relieve migraines. DIF: Cognitive Level: Comprehension REF: p. 814 OBJ: 1 TOP: Nursing Process Step: Implementation MSC: NCLEX Client Needs Category: Physiological Integrity 2. Which herbal medicine may improve a patients short term memory loss and cognitive function? a Ginger . b . Green tea c . Feverfew d . Ginkgo biloba extract ANS: D Ginkgo biloba extract is used primarily for increasing cerebral blood flow, particularly in geriatric patients. Conditions treated are short term memory loss, headache, dizziness, tinnitus, and emotional instability with anxiety. Ginger is used to relieve nausea. Green tea has antioxidant effects. Feverfew is used to relieve migraines. DIF: Cognitive Level: Knowledge REF: p. 818 OBJ: 1 TOP: Nursing Process Step: Implementation MSC: NCLEX Client Needs Category: Physiological Integrity 3. Which statement about dietary supplements is true? a Dietary supplements are considered safe and effective. . b Dietary supplements have not been tested for safety or efficacy. . c There are no serious adverse effects to taking dietary supplements. . d Dietary supplements have full FDA and USP approval. . ANS: B Labels and advertisements of dietary supplements must contain a statement that the product has not yet been evaluated by the FDA for the treatment, cure, or prevention of disease. Legally, dietary supplements are not required to be safe and effective, and unfounded claims of therapeutic benefits abound. There can be serious adverse effects when taking dietary supplements. Dietary supplements are not fully approved by the FDA or the United States Pharmacopeia (USP). DIF: Cognitive Level: Comprehension REF: p. 812 OBJ: 1 TOP: Nursing Process Step: Evaluation MSC: NCLEX Client Needs Category: Safe, Effective Care Environment 4. Which dietary supplement should be used with extreme caution if the patient is on a platelet inhibitor? a Aloe . b Ephedra . c Green tea . d Garlic . ANS: D Garlic reduces platelet aggregation and should be used with extreme caution in patients receiving platelet inhibitors (e.g., aspirin, ticlopidine, dipyridamole, clopidogrel). DIF: Cognitive Level: Comprehension REF: p. 817 OBJ: 2 TOP: Nursing Process Step: Evaluation MSC: NCLEX Client Needs Category: Health Promotion and Maintenance 5. What is St. Johns wort used to treat? a Rheumatoid arthritis . b Asthma . c Depression . d Viral infections . ANS: C St. Johns wort is used to treat mild depression and heal wounds. The action of St. Johns wort is unknown. Studies have shown a prolonged effect of serotonin, dopamine, and norepinephrine as a result of decreased reuptake. Aloe is used to treat arthritis. Ephedra is used to treat asthma. Goldenseal is used to treat viral infections. [Show More]

Last updated: 1 year ago

Preview 1 out of 691 pages

Reviews( 0 )

$25.00

Add to cart

Instant download

Can't find what you want? Try our AI powered Search

OR

GET ASSIGNMENT HELP
49
0

Document information


Connected school, study & course


About the document


Uploaded On

Sep 21, 2022

Number of pages

691

Written in

Seller


seller-icon
Nolan19

Member since 2 years

10 Documents Sold


Additional information

This document has been written for:

Uploaded

Sep 21, 2022

Downloads

 0

Views

 49

Document Keyword Tags

Recommended For You


$25.00
What is Browsegrades

In Browsegrades, a student can earn by offering help to other student. Students can help other students with materials by upploading their notes and earn money.

We are here to help

We're available through e-mail, Twitter, Facebook, and live chat.
 FAQ
 Questions? Leave a message!

Follow us on
 Twitter

Copyright © Browsegrades · High quality services·